You are on page 1of 194

VARI ATION \VITH CROB S--AEC TION

27

and the ahear 8tre1111 on the


Theee are called , respe ctive ly, the norm al 6tren
ed by '1,. We see that when
obliq ue sectio n pq, the a.spec t of whic h is defin mn, eqs.
2 .; = O and the secti on pq coinc ides with the norm al sectio n
(2.1) give

Tension, Compression, an d Sh ea r: n
(b)

incre ased, the norm al stre88


and T = 0, as they shou ld . How ever, as t/, is
there is seen to be no
u,. dimin ishes , until when q, = 1'/2, u,. =
0. Thus
al fibers of a. prism atic bar in
norm al latera l stres s betw een the longi tudin
the angle cf,, the shear stress r
ction tensi on. On the other hand , with increase. in
2. l Variation of Stres s with Aspe ct of Cros s-Se
incre ases to a maxi mum value
2. la, the stres s on a IP
In the case of axial tension of a prismatic bar, Fig. = 2A (c)
itude u = p / A as Tmax
normal cross-section mn is uniform and has the magn
discussed _in Art. 1.2: Let us consider now the state
of stres s on an obliq ue
when ti> = 1'/ 2.
norm al cross -sect ion when q, =r 1'/4 and then dimi nishe s to r = 0
cro•section pq cuttmg the bar at an angle q, with the Thes e obse rvati ons lead us to consi der more caref ully the quest ion of the
P\ is made of a mate rial that is
1m stren gth of a bar in simp le tensi on . If the bar
m&y happ en that failur e will
p-+c------..+i.-----+--~
\ I
much weak er in shear than it is in cohes ion, it
p parts of the bar along a. 45° -
take place due to relat ive slipp ing betw een two
r than due to direc t ruptu re
I'✓
"--"'1• \fl plane wher e the shea r stres s is a maxi mum , rathe
s is a m&x imum . For exam ple,
acros s a norm al secti on wher e the norm al stres
(o) fl

N,. . . ,.....°\" a short wood post loade d in axial comp ressi on,
as show n in Fig. 2.2&, may

f __ __j✓~~rJ
P-- __ p a-= PIA
V
(b )

Fm. 2.l
Q

! tttf
mn. First, we isolate tt.- PQrt1o .
ti ·ua.t n Of th b
left of the obliq ue
eec.1.~~ fpq as a free body and repre sen t thee aart · to the
f
on WU11 ree bod Y bY the streas resulta t c ion o the remo ved porti on
equilib . 8
coUi nea~ ~;m: tion, this internal f:rc ~ show n
in Fig. 2.1 b. From the
components N ~:d external force P, 8.8 s~ownmust
be ~qua], oppo site, and
Q, 00 nnal and tang t· · Reso lvmg the force S into
·we find en ial res .
. N =p ' pecti vely, to the plane pq,
Then since the area of the b _cos it, ; Q = p sin~
-
0 lique seer
(a)
A, - ion pq is
the correspond · - A/ cos ~
mg stresses are '

(b)
} (2.1)
Fm. 2.2
jS
-~""''ON coMPRF-SSION, AND sBEAR: II VARIATION WITH CRoe&-ellCTJON
28 T...,.,;;u ' '29
. ·agge<l plane inclined roughly by 45° to t he
actually fail br sbeanng along a J . till specify the value of PI A nt which clockwise with respect to a cente r inJJide t he elemen t, t hP, .!hear st.resc
w of the post.. I n such c.ase. ' we may s th of the wood m. cornprcss1on. , even is negative. Stated in a d iffere n t w ay, the sheo.r stress on any surface of a
this failure occurs as the ultimate gtreng -c::ion failure of the material. body will be consid ered to be of po'!itive sign if it pom l<l in a.. d1rection cor-
·1 · ot a true compre...<,:, responding to clock wiRe rotAtion about a cent.er l.lli!irie the body. otherw;a..
though the f.u ure 15 0 · . _ f fiat bar of low-carbon s teel with
• • · a teI1S1le test O a •
~imilarly , ounn~ . · ible to observe a very interest.mg phenomenon .
lished surf aces, it is poss . t-""' == P/ A visible shp bands approxi-
p
. o

. s.l
Ate. cert,am \' ue O
r the tRnSl
h
1e s '..,.,., (1 '
. Of the bar will appear on the fl.at sides of
t.ely inclined by 45° t-0 t e a.·us ued . , z· · ( o) ~ b)
ma. . • · r· 2 ?b The.'-e lines, called L ers ines, mdi-
tbe specimen as stio~lrn inf . ~gg. ; shear even though the bar is being loaded
cate that the m f\.tens IS a11m , •
• Simp
m · ·
1e t-ens1on. ThLS . relati,·e sliding along 45. 0 -planes causes. the spec1-
. .
t axiallv and after unloading it will not return to its original
men to el onga e ., , h. 1· h ·
length. Such apparent stretching of ~-he bar_due tot 1s sip P enomeno~ 1s ( C) (d)
called pla.s1,u; yielding. Again , the 9.Xla.l tensile s~ress u: ·P· = PI A at which
F m. 2.4
this occu ri mav be designated as the yield stre~ rn tension, even though the
failure is not ~ true tension failure of the material. These matters will be of negative sign. Several examples of both positive a:id negati,e gies.r
discus.'!ed fa rth er in the next article. stress are shown in Fig. 2 .4. These sign convent.ions, while arbmary , must
Fonnulas (2. l ), deri,red for the ca.....c::e of axial tension, can be used also for nonetheless be carefully observed to avoid confusion .
a.xi.al corn presfilOn, simply by changing the sign of P/ A. We then obtain Returning to the case of a bar in axial tensio n, let ll." cooode.r now the
negative \"Rlues for both the normal stress u" and the shear stress r. The stresses on an oblique section p 'c/ at righ t angles to t.he seruon pq , a.s
compiet.€ ::i"Ul.Le of stress on a thin element between t wo parallel oblique shown in Fig . 2 .5 . To obtain the stresses u' ~ and r ' on th.IB ::;ect ioo, we oeed
8eeti.ons for axial tension and axial compression are compared in Fig. 2.3.
The directions of these strP.sse~ associated with axial tension (Fig. 2.3a) will
be con£idered as -pom.tive; those associated with axial compression (Fig.
2.3b}, _a..., TI.Pflalwe. Thus o" is posit ive when it is a tensile stress and negative
w~n it rs a r.,ompres.<-tve stress. By referen ce to Fig. 2.3, the rule for sign of

p
p

(bl

Frn. 2.5
p - - . . ,J only to replace <t, by 90o + q, 1· n. eqs. ('>
" . t ) . T l1en n-ruembt•riog
. · tbs :. ~11
I p
(90° + q,) = cos q, , wh ile cns(ro0 + (/, )
= - ~ 11 ~ . thts gi\--e:i
=~
( b)
u,.'
,., cost(900 + q,) = !:_A sin' ip,, )
1''10. 2.3
l (2 1' )
sbea.r stresa r will l,t> ai; followi; Tl t
·11 b . . i ~ llnf:ar F;tr~
1P
.,, = - - s.in ( l80" + 2~) l f' . •),. I
w, fl con.,,df'rNi ~ tii r wht-n it h ., on any fo.r,c of the eleme nt 2A
'5 - .; •

• ·"
sm ...,,. ,
to a cen{R.r z'n.wi.f: thf' ,-lern,:,,t ( J," iL4 ~ ~ kWise momeut with respect
ig. 2 Ja; If th These stresse~ on the plane p'q' act IL'! slw wn Ill }:'\g. :?°.Sb
· e moment is counter-
l(PRESSION , .um SHEAR : II VARIA TION WlTB CR06S- BECTIO N 31
30 TENSlO N, CO
. hy eqs. (2.1) aliO (2.1') are called S imila rl y, from Pqs. (2.1')
The comple te ~t oi stresse s gibven cur on mutua lly p<' rpe n<iiculn.r 1
(1 " - 25,000 X (O .&o, ' = - r,2r,o i.g/cm •,
be use t ey oc
compl.emenlar y sire.~ .cs ff ula.s we observ e that
, r' = - ( - 12,500) X 0 866 = +10 ,820 ltg/cm ' .
planes. Comparing the t.wo sets o orm
u. u' . = P / A,+ L
(d) Pflu& L E M BET 2 .1
-r' = -r. J t.ion of di.a.mew
I. A mild stee.J tensilErtest specim er: ha\ring a circul.&r c~sec
and 1/ on any two mutua lly perpen - d =- I ·25 cm. shov.:s an elonga tion r~ ing of 0·005 rm over a gµige length of 5 cm..
of normal st resses tr• . " d J t P/ A , the stress m the matena J. M.,,7lming th.at E •2 · ro1, kg.cm•.
Thus ihe sum . . · I ten sjo n 18 consta nt an cqua ho Calcul ate the maXImum shear
.
dicula r sectJon s of a bar m runa ion ·
mn . Also compl ementa ry s ear stresses Ana. 'T"mu: = l ,000 kg /cm•.
2. A brass wire of diame ter d =1. 6 mm &nd length l = 60 an
t is tightly ft.nk.h ed
norma l stress on t be norma I sec . . ,.
a temi.on of ts -~!J- lf the
are al.wa:ys equal in magni"tude but opposite m sign . between fixed points A and Ba.tit s ends so that it is under um !hea r~
r' The equali ty of compl ement ary shear stresses tempe rature of the wire subseq uently drops 25°G, wb&t ia tile
rwui:n::
l exp&D.S ion for brass is a.= 19 (J O'- •
such as rand r ' on the faces of a rectan gular elemen t, in the materi al? The coeffic ient of therma
1
.A.ru. TOI&£ = 6!2·j
cm/crn /°C and the modul us of elastic ity is E• =1 0 ( JO)' kg/ an

T1-dsd• T
(Fig. 2.6) also can be establ ished from the equilib -
rium condit ions of the eleme nt itself, as follow s:
Let dz denote the tbickuess of the eleme nt normal
to the plane of the paper and ds, ds', the length s of
kg/cm• . •
3. A prisma tic bar carryin g an axial tensile stress <r, is cut
1><J a.s shown in Fig. A. If the norma
are"• = 825 kg/cm• e.nd r =275kg /cm•,
l and
6nd
shear
the
stresse
value
s,
of
respect
rr. and
by an oblique !leCtion
ively, OD thu! section
the s.ogje ~ defining
its edges. Then the areas on which r and T
1
ad will the aspect of the section pg. Am. u. = 917 ?cg /cm •; ~ ,. 1a~ 2r:·.
bar s.a shown in Pie. Z.5
be, respec tively, dsdz and ds'dz. Multi plying the 4. Referr ing to the case of axial tension of a prisma tic
ical data: A = 5 sq cm, P = 5.,000 ~ . ,:, - 20°'.
we £\re given the following numer
FIG. 2. 6 shear stresse s by the areas on which they act, we the stresse s u., u' ., r, r.', for section s pq and p't/ . A.:w. u. = 8lH
Calcul ate
obtain two counte racting couple s, the momP nb; of leg/cm•, tr• = 117 kg / cm , r = -r' = 321 ·5 k.g/cm•
1 .
which must balanc e each other. Thus
r(dsdz ) X ds' = r'(ds'dz) X ds,
in
from which T = r' , where r' has alread y been repres ented as negati ve
Fig. 2.6 .
I. A short s~l bar ha ving a I-cm X 1-cmsq uarP cross-s
ection is sub-
. EXAMPL E
P = 25,000 kg axially appli!.'d as shown in Fig. 2.7. Fto. B
jected to com pressive forces FIG. A

S. A steel rod of circula r cros.i- ~tion is to carry 11 teruile


load P "" 15 t . The
for the rod is .,., - .550 kg/cm' Finri me required
allowa ble workin g stress in shear
diame ter d for the rod. Ana. d = i · 17 cm.
sngle ¢ which
6. A prisma tic bar is subjec ted to a.-ru.1 tension . Find tbt1 &SJ)Kt eqo,.l• ..:1 11..a.
on which the norma l and shearin -g s ~ .t.re
defines an obliqu e section
"'= 45°.
7. Referr ing to Fig. A, assume t-hat the angle t/> .,,, 30• d.tld 1
tha t <r . ... -oo k&Jcm•.
In such case, what is the shear stress r '.:I .4 n&. r = 4lH l.s 'nn
in
.
Fig. 2.7 are gjven L'I
8. The norma l stres.5e s on the edges of the elemc:i t A.
FrG. 2.7 as~t. angfo ¢ da.finin g
f.b~
follows : <1. - - J,UOOkg/ cm•, <1'. "" - 655 kg/cm• . What is
t and what is t.he a..xil\J stress Pl A t.u w.hich the bar rs
Compu te the corn plt"l.e !>Ct of corn n]f'
~ ffil-'lltar y st t the orienta tion of the elemen
· •- (J-• 8.8 eh,,"'n
e Iement ,.,• (IT!Clll<C ' r f'S.<;es t hl' ·s: l,Jn~ I
' ' , of tiw r ec nng11 a r .
subjec ted? Ana . ,p = 39°, I' / .4 = -J ,655 1-. .gJrn• '·
(,n
~ ·
having length l ... .30 cm and di&met er d -1 5 cm LS
T aking P f A = _ 25 000 9. A concre te tegt cylind er
SOLUTI ON. If t.h e ma.•dmu.m ah~
' kg /cn1• and tf> =- o . subjec ted to axial compres.5ive forces .P in a ~lwg m.aohiue.
u " =- -25,00 0 X (OJlGfl)'.,. l 30 iu eqs. (2 .1) HOka/c fn'.wha t i.s r.ho safe value for the ax.iiu
stress in the concre te is not to excood
r = -12,50 0 X () - 8,750 kg/<. n ,a load P? An4. P .,.. •l9, ~00 kg.
-~6 ~ - l O•820 lrll1r. 111 .'
ION AND 8Hll,\R : II
:!2 TE,'iSIO ~, coMPRr.88 • DI.AOR .Uffi, W OBXIN G lffllDl8

. .
5
b 5 cm !QUAl"8
l,
crOS8-SeCtion bAs a gluod joint at
the iulowable working stresses for thehglue in I 6 -!
_.... .r
2. 1
. 10: Aw~ stickh· haVL?gFia. cBm
its mid-sec

tion
d _..
&S s own ID
a.re,·-, ,--
g.
.-..cn,,rtively
. <J - 70kg/c m!,a.nd r.= 42k g/ cm 2, wh a t 18
' " th
teD8lon &n :wear for the joint? What is e correspon w d · g safe teruiile l
. t e Opti-
Oad p for __... -- "' E
..
tv
mum aogle tJ, < 45°

-
r.-~
~
fl
the stick ? Am. 4> = 31°; P. =- 2,380 kg. I
Structurut 9'HI
7 I C.,-'-'
I
2.2 Stress -Strai n Diallr ams; Work ing Stress 0
00 OJ 02 Q.3 0 o..ooe 00!
atic bar,
Jn the preced ing discussions of tension or compr ession of a prism Strain Strn.,

and obeye d
it was always assume d that t.he material wo.s perfec tly elastic ( 0) (bl
iuls so long as
Hooke 's Jaw. This assum ption is justifiable for many matcr F IG. 2.8 F m. 2.9
d a certai n
they a.re not too highly stressed . Howev er, when loaded beyon
als begin to exhib it some a decrease in the
stress, called the proportw nal limit, most. muteri
's point D, furthe r stretch ing of the bar is acco rnparri ed by
i.e. , they no longer obey Hooke E . The fact that the
nonlineari ty between stress a nd strain, load and fractu re takes place sudde nly a t point
m of calcul ating deform ations streng th at D is l!Omewhat
law. Wi th t his nonlinear beha vior. the proble nomin al stress at E is less than the ult imate
nes becom es bar gradua lly
and distrib utions _of i1,t~mal forces in &tructu res and machi mislea ding. As failure develo ps betwe en D and E, the test
stress- strain behav ior. length as sh.own in Fig.
much more compli cated than for a linear necks down in a short region somew here along its
beyon d the along 45°-pl anes
In i~vestig~t~g the me~hanica.l proper ties of mater ials 2.9 . This pheno menon again is the result of shear slip
corres pondi ng strain is ection al area &t the
propor_t.iona.l hill t, t he rela.tJon betwee n stress and which causes a prono unced decrea se in the cross-s
m obtain ed ex- be.."lare en C
usually repres ented graph1 ca lly by a stress- strain diagra
strain
narrow est part of the neck . If for each value of the
dii:igra m o:na l
r nment ally from ~ standard tensil.G test.• A typica l stress- ·
strain
and E, the tensile load P is divide d by t he reduce d cr~ti
are&,

.or structu ral steel 1s sho\\n m Fig 2 8 •h • h · the dotted line


a.s ab .· . . . a, w e1e t e axw.l stram s are plotte d it will be found that the true stress- strain curve will foUow
curve ~~;;t th~r:o; espond mg stresses are given by ordina tes of the
• . h d. . . Of to A the stress isb propo rtiona l to the
strain
CE'. Howe ver, it is establ ished practi ce to calcul ate the ultima
on the basis of the origin al or nomin al cross- section al area
te> streng th
A of the gpecim ~
Beyon d .-i , 1. e eviauo n rom H k , , ·
stress at A represe nts the pr too el sl _-a\~ ecome s marke d i hence the witho ut regard to the reduc t ion of area.
p ronounce-d yicld
BOmewhere between 2 100 k / cm~pordw M imil. This is usuall y found to lie Struct ural steel is the only mat.eris.I that exhibi ts a
an 2 S30 k / 2 f or a 1.o~-ca rbon steel. Upon traos1t1on from the
• g point. Other mat erials show a more or less gradua l
loadin g of the specimen bevond th . g cm _
tional l.nnit is
creases more rapidly and th~ dia e ptpor twnal limit, the elonga tion in- linear range to the nonlin ear range , 9.nd even the propor
of a bnt~e ms.teri&l.
of t.be specimen begins to take ~m ~. comes nonlinear. At B, elonga tion rather indefi nite . As an exa mple of th e tensile behavior
on is shown in Fig. Z.Sb. This nateriAI
load and the ma terial is said to Pb ce \\ 1thout . a ny apprec iable iucrea se in a. stress- strain diagra m for cast-ir
Tb 18· shows no yield point.
· ~ · contin .
ues until the test b
ecome pla~tu: " · p henom enon, called has a very low propo rtiona l limit and
yie tll{l , s materi als <=All
as much as ten Stress -strain diagra ms for axial compr ession of variou
tu fif~e n times th£ elastic stretch : may strN ch plll.btic:alJy s s.s the propor uonal um.it,
The stress also be obtain ed and such charac teristic stresse
a t ~wh~eb thui yiPJdin g begins il'i ca~e~o tthe propor tional limit. the ~
c&.. o f~~
be11avwr rc:>.sul t..-, from th . r . he yield pofrit of th t Th. yield point , and ultima te streng t h ca n be found . ln
th . e s ee 1. IS same ~lue,. a:; for
lines as discuss ed . l e s ip assoeia t.ed wit h
, . c appea ra n('e of Luede r charac teristi cs for compression a rc fo und to have the
. . m t 1e preceding a rt icl
ofy1rldm g is rea Uy an indir e. 1 h1.5 mdi ca tes that the
phe-

r~;~a:r=~~r
nomen o.1

>eg11.s
~ l, J ~g 4.5c. ~planes of
..o strain h-0 d
::::nifc um
sta tion of failure of the
shear st ~e ,
: ·. ss:
A
t
.
pomt
tensio n.
Worki ng S irfss . A ten1;ilc test d iagram gi vt>~ rn!U2.h1t>
the mec hanica l prope r! ies of a matni al. Kno wing the
info rms tio~ on
I.unit of PCl.' J.k)r t,Lt.n i~
ul, it i.s ~bl( '
a nd with fu rther elt ,ngatio n r en, reeove rs some of . ality, tht> yield point , a nd thf' ult imate st n'ngt h of the m.ster
repres enting I.h e maxinitUn te th.e ;o;t reg,'l-stra; n c1 I ~-; elasti c prope rty ' magm c-udo 01 th<"
stn-ss o irve climbs to )oint D to e.s t.abli sh for f' fit•h pa rticul a r c- nginee ring pn.ihlt· m the
• . nsiJe is u:;un.11:-, called
. For more deu.de of the et.ar r the u.llt1n<JL f' I .
d
' s tress whi1.:h may be consid ered u sofc strrs-5 fl11 s st n>SS
. I , p . 2Y . ida, d -Zl~iJ,i
t,, !.<:at lln d . 1Strenyl l1. Beyon
d1agram , sec Art. JI 4 v11nou11 Wlpecui the worki ng slre11s.
of th e s ttc!11s-s 1rnin In choos ing t.ho magn it.udt' of tht' \\ (Wklllg str~ss for stool tt m ust bl,
• 3,4 TE?,,1810l'f I OQJOJlll8810J'I' f

.d t 1 0 that. at strcg..qes
AND 8 euR: II
below the propor tio na l limi t the
d h'
OJAORAM8 , WOllKflilG fl'1'RDi3

fn the r a.c;e of n !rt.llt1rally detl'rmtna.tl' srt,,m, t h.- stl't'.:9.'!l!l! ui 1t., ~~


taken _into oons1 era ~ . as .rfoctly elast,ic, e.nd be!on t, 1s limit a
mat eris.I may be oonsid~N'd . p(' ft r un loading the bar ' 1.e .' permarumt eet will nit in r reru:e in pro portion I.I> thf- applied loed.11 ev"n thi:m,;l: ~ pm-
8
part of the stn.in usually remains e •n an elastic conditio n nnd r-0 remove portiono.1 lim it is e xC'eerled . Thu", in surb s~ t P.J1l.'II I.he Ulle o£ a wnrllmg
< hA ,·e the strur.ture I h strt'M Ill! defined hy one of cqs (2.2 ) glVl!:3 1 hf" ck;,r,o,d nbj~ttve ~ 5Ut.ed
occurs. I n ord er (I t 1•t •_ usual practice to keep t. e working
ihT of permanent se , is Rbove . HowPver , in thP- ca.-e of a st...Lit'.&!Jy 1r;,:i,1PrJm.ste r ~ LkP
mit In t he experime n tal d eterminatio n
t.he po5Sl , ity & .
1r those discussed in Art. 1.4 , t hP tret;St'!'I in rr.~ m'!mi:lf'r; dPrpt!M on t.hcrr
0

s t.res, weU_b«>low -~ e proporu~nsin~tru~ en ts (e:-<tensom et«'rs) are necessary


of this limit, senSJt l\--e measun ng - th deformations . Thus, t he rnethO'is of annlys:a U3ed u:i .\r I t a.nd b&...ccd on
•• f b 1- ·t depends to some ext.ent upon e accuracy H ooke's law will no t be o.pplira bl~ beyor: d th,, pr~r.:on..u lam• In
and th~ position o t e tm1 . . h. d'ffi
• • ,1.. • b•'-
v.'1tb ~ ·111c u,e tnf'.&.SUre.me ...,
nt"sre matie. I norder
. t,oehmma tet 1s I cul ty . , such cases, the m;e of working 8tl'C'ISC-. as definl"d by •q3 1. 2 •;inll oe mo::-e
--·-'l y ,_1. '-- yiau ·. u ,n,r,nt or the uUtmale strength of the m aterial
one ·~ =es t w:: ,--• . _ or less meaningle!'IS.
- . _ __ . c - d te ining the magnitude of the workmg. ld
a.s a ..,_<:l.l! o, e rm
stress. D enottng
. d
yie pomt, an the
b J o-.., 11 1 . p • <Toll ,.....,.,.,ctivelv
the working stress, t he 2.3 Lim it De8ig n
• v-,y~ .J • h k'
uttim&te strength of the material, the magi:utude of .t e wor rng str;ess We shall here diso.:uss bne fly a method of analyw d ~:....ou}y ,n-
will be determined by one of the two following equat.10ns: determinate structu res whjcb will enable ll5 t-0 predict tl.~ ,-:::\.~ u=der
e1..,
C1 y . p .
= --, or e1., =
O"ult
(2.2) wh ic h the structure will col..l.apse d ue to SJmultd.Il.eow ~d,.ng cf ~e: ;JI
n n1 all of its members. Such an analysis, of course. IB auoonut ~Ly um.t.ed to
Be.re n &nci rv1 are usually called Jae/hrs of safely, which determine t he steel structu res where there is a pronounced y!.eld polL~ oi ~e t:::.!ttt-rJ.I
~ itude of the working stress. In t he case of st ructural steel, it is The d iscussion will also be confi.ne<l to structures ma.ce up oi ..un.;,e te.ns.icn
}ogmal to take the yield point as t he ba Pis for calcul ating t he working or compressio n members under uniform str'P.:>3. \\-"hen we b 'll! ~ ~ -
6b'es!!, because hen> a consid<>.rable permanent set m ay occur which is t udes of such colla pse loads nnd specify the working !:osd3 ti.3 I ~ :.rad t:::~
not permisible in engineering struct ures . In such a case a factor of safety values, we have realized a true fac tor of safety n ~ \.~ ::i~te iLl..uu
n = 2 will gj ve s conservative value for the working st ress provided that of t he structure . T his philosophy of lim u ~V'l or pl..:-~ .r'lo:Ugo..i tS
only eons.tant loads &re act.ing upon t he structure. In the cases of suddenly gaining wide fn vor a m ong structural engmeers reoiuse r..s \L~ ;-esL.'.t.:s
&pplied loacib r;r ,·ariable loads (and t hese occur very often in machine gener a lly in more efficient and economical design~ •
p&n5 ). larger fsctors of safoty may be necessa ry . For bri t t.le materials T o begin with , it is customary to idealize the poruon OA. .8C L'-l ~ t' ~~~
Bl.I.ch a.a ~ t iron , concrete . and vari ous ki nds of stone a.nd for s uch material strain diagram for steel as shown in Fig 2.8a to tha t i:l r--:g :l.ll\ "&".'lr.n- 1t ~
& wood , the ultimate strength is ui,ually ta ken as a basis for determining assumed t hat proportionality holds up to t he ;vield 3i.re5S :tnd •h!it ~
the working streS!eS . after the m ater i&.l y ields indefinite ly Such un iJ calu.e<l c!l!\!<'ml ~ !3.->d to
The ma~i t ud~ of the £actor of safety depends very much upon t he
r.ccuracy w1Lb which the externa l iorces acting upon a structure are known
upon . the accuracy wi th which tbe stresses in the members of a structur~ 4

maThy oe cakulakd , a. nd tuliiO upou the homogeneity of the materials used .


e c-o mm<m p mt·I ice of spea king of k. 0-
fet . biw!d . . wor mg stresses and factors of
m Y on some <: haract.ensuc et.ress such as t h . . 0-yp

c,r the ultima t.e til rt· ngt.h of r-a.:it-iron i _ e yield pomt of steel
leading lf p p p . s JiOmewhat da ngerous a nd mis-
. , • ure u set of external I :J
. i. t,
we rek.l.ly meun by a factor of hilfot n
18
°~ s for a structure , wh at
o'---- - - - - - - -
<: retu,ed 1,c, ri l' ,. nP,. nP, . L,.. ty l hnf if nil these loa ds a re in-
. ' · • w J.._ E l"\J elu n • ·11 b ·
failure, \\l,ere ''111.ilure" of C<,ursc L. wi e Just on t he vc rgP of F1u. '3. tO
. t.h
in
I Ill U/i!t f)f_: dearly dt f
e CB.bf' v a .,u,ef •t1w·ture: that . 11 . · • rne d . ,t
r
may nwa n,
. ' tu npt;e due IA1 1. Id .
-iern c.an or-c:ur, v r in t~w '!.'lbe, f .0 1 Y <' 111iz of Home mem -
.' ~1- ' e. '- H 'rr•t <) iJtru ·• 11
t& on i.al? \ 'Nfl!' o f fnv:t..u ri- . '• rc, I.hat Hume rncmbe r
~---10N AND 8HIIAB: II LIMIT DESIGN
,r, TENSION, co...-- •
_ _ _ -'-_ _ _ _ _ _ p
37
. perfectly plastic depending on whether the 8 s, 1 5, A
B,_.-=---- ---.A p
be eitJ_ier perfectJy elasUth c_elodr . t It will be further &SSwned here that ~
stress 18 below or at e Y1 value pom ·in both tension an d compressi· on. For s. I
· . . h h I
t.he _yield pomt __ ~ : ~ - ~ value may be taken as 2,800 kg/cm2. I
ordinary structunu t111tXU, ·d h · I
·u t te w t.he idea of limit design, let us conS1
To I us ra no d ·
er t e slAt1ca.lly
ssed · A
I
I
indetermin ate system shown in Fig. 2.11 and already iscu 1D rt. 1.4,
assuming now that all three members are made ?f steel and ~ave the same 0

aros.s-sectiooal area A. As the external load P 1s gradually mcrea.sed, the (a ) ( bl


axial forces X and Yin the members increase and are in the constant ratio
Fro. 2.12
shown by eq. (c), p. 17. This continues until at a certain v~lue P1 of
the external load, the force in the vertical bar reaches the magnitud e
a statically determina te system . D urin g this phai;e of loading, the axml
X = o-y.p.A , i.e., the stress in this ba.r reaches the yield point. At this
forces in the bars that are sti ll elastic contin ue to increa...~ until finall y a
t ime, the vertical bar becomes plastic and will stretch thereafter without
second bar reaches the yield stress. W hen this happens, the trus.5 becomes
further increase in tension , but the inclined bars having the tensions
non-rigid and can collapse' m ore o r less freely d ue to stead y yielding oi the
Y = X cost a are still elastic and capable of carrying further load. Thus two bars that have reached the plast ic conditio n. Thus the I.unit load hll.3
as the load P is increased beyond the value P 1, the tensile force X in the
been reached .
vertical bar remains congtant (o-,. .p.A) and the tensile forces Y in the To disoover which two bars will first reach the yiel d stre.51,, we corunder
inclined bars continue to increase. However during this phase of the in-
the equilibriu m conditions of join ts A and B a nd concl ude t hat at a.! l time.-.
crease in external load the relationship (c) on p. 17 is no longer valid. We we must have
have only to satisfy the equilibrium condition St
"
S 1 = ...-.6 = S,
,/2 1:1.nd S , = 'J'if (c)
X + 2Y cos a = P. (a)

Finally , the in_clin~d bars also reach the .}ield point and we have X = y = Thus if each bar has the same cross-sect iono.l area A . it is d ear that the
u. _,,,A . Subst1tutm g these values into eq. (a), we ohtain diagonals 2 and 4 will be the first two bars to re:i.ch t he yield !'tress. Wn en
this happens, t he system can collapse as shown in Fig . 2.12b a nd fro m
PL = cr,..p.A (1 + 2 cos a). (b) equilibriu m coQSidera tions of joint A , we may write
This represent.B the so-called r-ollapse load z· .
P =St+ S, _ (d )
A working load p = p I ill, ha or tmit wad of the structure. •
L n w now ve a true f to o,. 88 f ety n against '1'i
complete colla,_. of t.he system W . ac r
,-- · e see from thi d'1 · ·
analysis of a statically indete . s scuss1on that a plastic Expressin g 81 in terms of 8, from eqs. (c), this becomes
rmmate system is t U .
el&Btic a.nalysis as given in Art. 1 4_ ac uo. Y simpler than an
f
p = s. + s, _
AB another e:-tample of plastic anal .
a square frame with pin joints load dYBlBho a s~ructure, consider. the case of '12
. be . e ass own m F ig. . 2 12
18 seen to statically indetermin ate h . · a . This structure Finally, se1,ting S , = S2 = u >' .PA , we o btain fo r t he t' oUap...-oe kiad
we make no at tempt here to cal ul te' having one redundant member but
d . . . ca t e . If ' 2uy.p.A ( )
u_n ng elastic behavior. A-; the external I I\X1a . orces induced in the bars PL= ~ = '15, o-1 .p. A - e
\ ''..!
axml forces in the bars all increase l ~ad pis gradually increased the
reached a t whjcb some bar lie1nr18 t . e a.stically until finally a load p' I JS ·
The abov e result is based 0 11 th<' assumptio n t hat ~ ch bar llAS the same
l d n ,· h f urt he r m . e· o y1e 1d a nd t h t 3 5
oa . n it crea.se in p be ereaftrr will 1!lk" f
cross-secti onal are.a A . Thus even in the limit ('tmdition, the bars l.! ,
yond the vaJ p . -- no urt 1er . nl ' , !·>· see
~ ughou_t t he 11.hove diaoul!llion i . ue i , the tr111,s behaves as are not working to full c~pac ity , their unit stre.55l's be mg o Y u,_. _ , ~,
the st.ructure I.I! oot ee!leutialJy differ t Ill ll&'!u rned that th 1
eqa . (c). Clearly, we m 11.y now reduco t he cross-aecti ooa.l ~as -"'· , Ai , A,
change in the a.ngJ ,. a d ue to dcfo•••--eot frorn theb undeformed e cdorrned r.onfi g, 1ration of
· -lion or the co nfi . to A/~ without reducing the yaJue of the limit load PL given by eq. (e}.
· &rs ia neglected. gll tb!ton , i.~ ., small
~ TEN SION ) COMPRESSf
ON
I
AND SHE AR : II LIMJ T DES IGN 39
38 ia binged t.o 11 •-1crt' 1 ll
2. A hor ison t.al rigi d bar AB aod a stru t. EC as ~ wa_ ~ A aod sup port ed by
= As = A~ = A / rea
A . 14 y2, we obt ain
c h t he y ie ld two steel guy wire s BD and CD lg. B. Eac h guy wire
1tmss, \,'. her e a.II bar s will cro• and & yiel d 8tr wo tn 3
The n>Jore, tak1n~ Ai = A, = load p = y'l<Iy .A . Usin , has a crOSH1ectio ns.l area of o=6 11 ~,..,.. k · ~ <Ty . ~. "" -000icg/ cm'
r . . Tbe stru t
t th£ uru t lo~ PL
m lim it des1gn of t,he t,nk J"ed /:> g ,L h.aB a buc klin g stre ngt h of Q g tn eom pre&<!Jou • C&kula e
the <'ptimu I k. d P - 11 b e
,.JVV
.,. 2,24~ i.:~
con diti on sim ult. s neou:-Jy whf
!n r'1e .ll PP
g safe wor mg 1oa w1
_
,. - for t he 11y etem . AN . PL ,,_ .
oor rl'.sp on d m " 3. A Z-rn leng th of rail road rail wei ghin g 225 k ~ per unear Tu< nd ed
tr r: IS ~1-_r--.
fact or of :-;nfety n., thP • ·- II f ·
gir d er by thre e stee l wire s each 3 · d. . - -

mem ber co nsis ts of h on 1on l,Wj y rom &


e.r an~ ~paced 1_m
, fio-,, ,., A 'n. EXA M PLE 1. A tRnsion Wh at is the mar imu m ~:: nc~ :~t ro:n
apart ; t ~e wire s are ~er tica l.
::es :~~,th~
30 cm _ wid e by 18 can be hua g with t t ne_,aud -pom t
two pieces of flat stee l plat e o( th e ro.il tha t a vert ical load
p ,. 15') li:g
mg
11 ri vete d join t ~u;
inm thic k conn ert« i toge ther by ding ? The yiel d rues i, for the
ts a re 22 mm in sup por tiag wirea due to yiel ession . ,1.iu . ~ = 24 rm. 7 P
- · , cm
as shown 111 Fig . 2.13. T he rive and t hey can not carr y co mpr
,,,..; 1
a l ha.s a y ield s t ress in are give n the fo llow mg nw,., ,l_
" ""'' ca t.U1 ~ . a =. 1dm
diam eter and t he ma t<>ri 1 4. R efer ri ng to Fig . 1.15, we . dLimeter d == 3
e t h e safe
shr.a r, 1',- . p. = l,400 kg(c m . Cal
ber bas
r ulat
ed on a fact or
su p pcr ung etee l wire s hM
b = 2·5m. l = 4 m. Eac h of the wei ght of t he rigi d b9.r AB i:s W ,,. 200 k11 C~ &
t~nsi le load P.. fo r the mem yiel d.st ress of 2,5()(Jkg jcm '. The B . AM . p 1, = ,;-; !.g · ate
e fail ure du e to
a:~-
plet
the lim it valu e of the~ oad _P
of s.'licty n = 2 aga inst com at
between fixed mt 4
shea r yielding of t he ri vets . rn diam eter and ",{) cm lo ng stre tche d
5. A stee l rod I -'.lcm cm' Suh..~ quent.l~
SOL UTION . Thi s syst
em is hig hly stat ical ly tens ile ~tr ~ <I -, P- = 2. :..00 ~
and B at its end_s is und er a. hed ai an 1nt~: -ned ~ :>Oint
dur ing elas ti c beh avio r of the g the a xi.s c f the rod , 13 spp
inde term inate, and tem al for: eP , d~r~ ted alon .!r.a. p!. "" 'i,6~ i:g.
true dist ribu tion of this fo rce P based o n )'iel din 6
?
m.a.terial a calc ulat ion of the C. Wh at 1s t he hm 1t va lue of ·.-e M.d , he bolt .1re ~ e of
of rive ts rep rese nts lrne tha t bot h che s.lee
load amo ng the vari ous row s
6. Ref erri ng to F ig . 1.16 , B..'>.51. n 1.5 .-t, = 1 i.1 en
eve r, as the load p e cros s-se ctio nal1area of th.~ :;lee
'lil impossible prob lem . How !teel (a- y . p. = 2,500 ~g/c m') . Th cm. The nut a t the end oi the bull. LS li.m tu rned
ease d one row
on the mem ber is grad uall y
incr
yiel d,co ndi tion
and tha t of the bol t 15 Ab = 3-0 sqt is reac hed . Sub ...~u enc ly, e.ne :ral tt>n:sik fort'e!
the bol
of rive ts afte r a nc,th er reac hes up unt il t he yiel d stre ss in the nlae uf tlrese ~ l e
smi ts a con st a nt end s of the bolL \Yh at is the !.i cm
io shea r and ther eaft er tran P are app lied at the two
i.:g
fina lly all rive ts will hav: e forces bas ed on yiel d ing? Ana
. P L= 7,50 0
amo unt of load unti l
At suc h tim e tile
p
reached the yield con diti on. '
C!ln tran smi t is
tot.al load that the splice and Co m prCS-'-tOD
·F10 . 2. !3
2.4 Str ain En erg y in Te ns ion
PL = IO X
2(T7.p.A),
of a p rism atic bar in s.unple
ten aon . .F1g .
h . Let us con sid er aga in t he cas e the IO&ri~leooction
--a....<>re A = 11' (2·2 J' / 4 = 3·80 cm . ~ t e cross-sectional &re a of one rive t. Thu
· a
beh avi or of the materia:I.
2.1 4a. Ass um ing elas tic
the limi-t. load for th~ 5 liced m~m
i:&, and the saf.e wor ~
load (,a:;
ana v
l. .,;~
I B ~ •=
. . mesd
10 ~ 2 (l,40 0 x 3.8) .= 106,
P_.. _- 53,200 kg. Thl S exa mpl
400
e dia gra m will be a s t raig h t line
0.1 a.s sho wn in Fig Z 1-tb, an-d
respond ing elo ngs tion oi me b:.u-
for ~y v~ ue
lS denoted by
of the ten sile loa d P ' t he cor
sholwE the ad van tage of plas tic ts.
'J.,.., m nve te Jom el,m gat ioo 5' will
loa d d.P' is add ed to P' , the
8' . No w if an inc re me nt of P'.:J,r' . T~
the load P' due5 ~ t.i w wN k.
inc rea se by the am oun t do' and
PJIO BLE MS !: T 2.3
1-i b, i! stor ed in
8Uppofted b fi . .
rese nte d by the are a of the s had ed st np in Fig 2
j • A vert ical 108.d p ii, l WJres sym met rica lly arra nge d as wo rk, rep fg'.J , as it i3 more com-
ruwing c ~ ? sr
8?DWD rn Fig. A and each r:, ~ ooo1.~ '0 ~ ardea A = O·GS 5 q cm. Cal cula te the en t ial ene rgy or stroi "ll t'11f
the bar in the for m of pot remo ved , the
furut !O&d for t he system if 7 . g . 3• ~ 1era • an a = 450 , ,.,~ -- 30.
o
Ans. PL = if t he inc,reroent 0i losd dP' 13
8.100 ~ mo nly call ed. Sub seq uen tly, d.i' 11nd the soo red
· up thro ugh the d1!'-tru1<.'lt'
low er end of the ba r mo ves g the exr. errutl I~
bac k into thd Wllr k oi ·rnk;m
ene rgy P'd8' is t ran sfo rme d b:it a .!pT t~
s . ..-r m~.y n.~ d the ela.s-Ut'
a3
P' thr oug h the d ist..l\nce do'. Thu t he load P' t.s
red or l"t'leL.<.ed imw din giy
t\3
in wh ich ene rgy can be sto rb a~ rerea.se
inc rea sed or d ecr eas ed . Thi
s pro~)\'rt .v of au eta.sue btu to a~ e-
~, th iuu
ene rgy wit h cha nges in load
ing is , ery imJlOl"t~ot io dCA!mg
rt urt\'., a ud n.li3dUnL,"
A va.rying or dynanu c loads in stru
p

Fw. A
I, l'ra. B
ND SHEAR: II
TENSION, COMPRESSIO?'J, A . l:!TB.AIN ENEMT
. 2 143 under a tens ile load P is
41
40
Th total energy stored in the bar of~~- ·P'df>' between
e h I ntsl stnp::; as .
O a nd Band is
·
equal to the sum of all sue e ~me . ? Hb Denoting this st.rn m e nergy by
, a bso r b with~u t ex~.ee?ing its _p roportional limit ca n be found by sub8tituting
t h io proportional l1m1 t for <1 m t he first of eqs. (2.5). Thj_g quantity is then
called the modulus of resilience of the material in tension . Taking the C8.!e of
represented by the area OAB m Fig. - · .
Pli (2.3) steel, fo r example, with a proportional limit <1.,.1• = 2.000 kr,/ cm•. we obtain
U, we have for the modul us of resilie nce uR = l kg--c m per cm3 •
l' =2 ·
The foregoing d iscussion of st rain ene rgy of a bar in tenaion can be used
. . .f H kc 's la w applies, we also have, also in the case of axial compression . I n such case both o- and, are negathe ,
Since expression ('.?.3) is va.hd ~nly 1 oo
but since they appear to the square in t he expresa:i.oos for !train energy ,
betv,een p and 0, the rela tionship : Pl
this has no effect on t he final result. In short, strain energy is alwa~·s a
0 = AE. positive scalar quan t ity .
AB an example of a n applicat ion of eqs. (2.4 } to a problem of dynamic
.
U81Ilg this, .
. expf'e$lon ~- 3 ) can be written in either of the following tw o
(?
loading, consider t he simple arrangemen t for producing tension by i...rnpact ,
forms :
AEfr Fig. 2. 15. A weight W, starting from rest, fal ls through a height. h and
P1l
T'
v = 2AE or = 2 u -z-· (2.4) strikes a flange at the lower end B of a prismatic bar of length l, the upper
end A of the bar being fixed. It is desired t-0 fiud the maximum elo~ tion
and stress that will be induced in the bar by such &n impact-. In discussing
p' oi'the b&r i.s
the problem for elastic deformation, v.a assume tha t the ~
A t there is no loss
negligible compared with t hat of t he the weight W , &11d tha
dP ' of energy due to impact bet ween t he weight and the flange a.t B
J_ After striking t he flange, the weight W continues to _,,,
'---., .--- --T,-
movP d ownwards, stre tching t he ba.r as it goes. How-
'! .
T
I

I 'e
bar 1 it decelerates rapidly a nd soon comes to reSl ha v-
ing stretched the bar an a.mount o. At t his ~stan t th~
'
ever, due t o t he resis ting force o ffer ed by t he stre t en ed
d~
w :a \.J-
t?°'"., ;
1i 1 1
0 tension in the bar is a maximum and the stram e.uergy . - - I
stored is given by the second of eqs. (2.4 ) · Neglecting 11 I
(b) minor losses, this stored s t rain energy in the ba.r must 1

2.14 be equal to the work done by W in falling


thro ugh tbe _·': j _ _ ' _j_l
il
total vertical distance h + c5 . Thus, we write
F IG.

In the first form , the strain energy is represented as a func tion of the W (h + c5) = A.2lE1;1· (a) s
tension P in the bar ; in the second form, it is represented a.s a func t ion of the 8

elongation l> of t he bar. Botb forms will be foun d useful in further discus- Introducing the notation c5u = Wl/ AE for th e s~s.ti~
sions.
elonga tion of the bar under t he action of ~ t> ca FrG. 2.15
_ Io some applications, the strain euergy of te nsion per unit volume is of weight W , and solving equa tio n (a) , we obt.a m_ _ -:-_
importance. In lhe case of uniform fA:n 81·01 0f , .· · • ·
_~ u. pn ;:;nui.~1c bar a s m Fig. (h)
2.1 4a , th is may be obtained sim 1
volume Al oi the ba r Thus u .· p ) hby di\ idin g expre~ious (2.4 ) by the
c5 = c5., + ✓c5.,2 + '2 o, Ji = &., + " J0, 1'+~·",
(), <-

. . , smg t e n'>tal!on u = U/ .1(, . - "' bodJ· a t t hEl moment of striking


E i where v = J2gh is the veloc .ity of che falhng
u =-.
o-2
or u == ......!. 1
(2.5 ) the flange at B . . _h eq (b ) redures to
2E ?
where <1 == PI A is the tenai le stre!,6 and _ . If the height h ,s large in comp!ll'ISl.)ll wit 6••, ·
6111·'! th e te nsi)(' strain in t he bar.
The grea~ st a.mount of strain en.erg t - 5 = ~·
y J)'-r u,11 t volur.ne th .
at a. n:.ttew e1.l cau
(l N" A.ND sn£AN :
n STRA Uf Dflm OT

42 co&!PRLSSl '
TENS ION ,
OAD /? g:ivea the wor k ~
• . tlle bar is then rn\UD elong a tion 6, t he corre spon ding uea
· t le !It ress 111
T he oorrespono mg .enSJ • ~
oE - !i_
[C
zE H' 1r
~ = ,r A-l . 2n. (c) to p rodu ce su ch an elong ation , th is mu<rt
duce d by t he we ight W . Whe n W (h +
equaJ t he work W (h ...L &
6) 111 equal tn or large r ~n~
u = y- 1 g"' :,
. . d . t.ly p ropo rt.io na l to t he kinet ic p
ra.d ie&I hJS . • f h .
· ' th od ulus of e last1c 1ty o t e m a teri a l
)Pe{'
8
Tb e expression unde r e 8
and to_t elmt t he ,,0 \umP. Al o f t h e ba r . H e nce
energy of the falling body , propo rt1on11. o . h ·
of the bsr, &n d JO\-er
.
se 1} tl ·b v nn incre ase rn t e cross -sect iona l A
. · · hed not 0 1 Y t h. u .- the ba r o r by a d ecrea se in the
t he stress can be d mu.nJ S . h 01
, rea..<:e in t e 1engm static tension of a ba r , whe re t he
area but al.,;o bY s.n inc . . f .__---~ !
d l E
rnod llius E This LS q uiteh chffer ent ro
I gth l a.nd t he mo u us · l to zero , t ha t is,
h h is equa
01.--8-- t C 0

stress i£ indep ende nt of t e en · case ' in whic . .. l l . Fm. 2.1 6 r l G. 2. 17


Cons ider no w ,moth er ext rP.m<' 1111 t 1a ve o e1ty.
. . 1,he fl ange B witho u t an . . f .
t he body lV 1s sudd enly p·-ut onkinetic energ y at t he begm nm g o exten s10n
• • th . I d' am , t he fallin g body wi1 fnct u."'e
\ltho ugh JD tlus ca..~ ere b no m g of total are.11 OA B C of t he tensi le test diagr
•of the bar , th e pro bl em 1s · q u1·te di fferen t from t hat of a stati c oa . .
- the bar.
dua l a p p h cat10 n. of
f c:: •n t i·c te nsion , we assum . . e .a gra From t his it follo ws t hat any chan ge in the
fo rm of the bsr wh:ch resul t3
o:..r. I n th
L - ,
tLJK . e case o 8 ~....,. ee n ~he a ctmg
the load and conse quen tly t here is a lway s eq
m hbnu m betw the diagr am d1mll llibe3 a_:30 Ult'
t10n of t he in dimi nishi ng t he total area OAB C of
loa.d and the resisting fo r ces of ela.st icity in t he
bar . T he ques the groov ed spec imem Mt.own in
at a ll unde r s~ch resis ting powe r of the bar to impa ct. In
kinet ic energ y of the load does not enter int o
t he p.rob lem m etal will be cont:fflt r.ued iU
, t he e lo ngat1 0n Fig. 2.18, for ins tance , the plast ic flow of the
n of t he load t.o produ ce
cond itions. I n thf' ca..c::e of a sudd en a ppli catio the groo ve and the t otal elong ation and
the work n~
case of the pri:smso.e w show n in
ro at t he begin nin g, and t he
of the bar a.nd the ~l.ress in the bar are zf' we igh t. fract ure will be much smal ler t ha n in the
the actio n of its own very wea k in unps ct ; a. shgh t
sudd enly appli ed load begins to fall unde r unt il it the same figur e . Such groo ved specimens are
bar gra dua lly incre a ses mat~ rial it.self ;s duc uk>. Y em-
Duri ~ t!lis motio n the resist ing force of t he Bu t a t shoc k may prod uce fract ure al thou gh the
emen t of t he weig ht is ci,t- in cros.1H1ecw n &..--e ~ y
ju.st eqllals W when the verti cal d isplac bers havi ng rivet holes or any sharp varia t ion
y acqu ired durin g the dis-
t his mom ent t he load has a certa in kinet ic energ weak agai nst impa ct.
;i. cont inues to move down ward until brou ght t o rest nds upon lhl' dut·tili Ly of the
place ment o.i; henc.,- e
elong ation for thi s co ndi- The resis tance of a bar to impa ct a lso depe
by the resist ing fo rce in the bar . T he maximum le test d t a ~ of Fig_ ~ 17 .
mate rial. This may be seen from t he tensi
hon is otta..in.ed £rum eq. (b ) by settin g v = 0. cune fo r ~ w.su-ruu of
Then matt on
The curv e OAC repre.sen ts th e load- defor
(d ) 0.4 8 i:, for a m~~ ri~ o.f
high stren gt h but low duct ility, wh ile t he curve
ore ducti le mate risl ~ ~ap~b le of
na mic cond i t ion s, prod uces a lowe r stren gth but high ducti lity. The m
that i1:, a sudde nly applied load , d ue to dy abso rbing much more e ner gy bt>fo re rupt
u re than the l~ l.h.1ctilt' msten .'\l.
ned wh en the load is appli ed ares.s unrll' r 0.{ B \'\nd OAC Fo.r
dcfle eti<m wb1ch is t wice a.s great li.6 tha.t obtai as is evid ent by compa ring the shad ed
gradu ally. ..atcr I't\.'>-lSt an('(' w fn rtun-' nnd-er
th e assum p t ion that t he this reaso n , a ductile ma t eria l has n gre
T he _above diBCUSSi?n of_ir~pa. rt is based on
10 th bar remru ns w:thi n t he propo rtion a l lim it . Bey ond t his
limi t shoc k loadi ng.
e
!-rt.reBS r is no $ ~ WU in .!-'lg. :?.is
the p roble m b<:~m es mvre mvolved becau se thf> elong atio n of the ba
E XAM PU: J. T hree wnsio n nwm bl.'r:l h1w mg th\! J1mc:> nsn.lll
]o~n pror,o rt1ona.l to the te ·1 f · tht> ~ vl stn\.l n t>µc>~y ~c."t'd i.n
each <'arry the sa me ten. si lt' load P. Cump u'k • •
tHU01u n
nsi e orce. Assu rrun g t hat t he tens il e test I \• Lj i,;t( 1"b u tt-<l l')',"{' i \'.ll Ch 1•.l'('l'.S.."'"
. d d . . . h un11., rm
diagram oes not epend upo n t he 8 ed. Of st ram1 ng t he bur , elo ngat1 on the thret> cases, a.ssum1ug tht1t thP :1 t rl'.~
·
bcvo nd the elasti c lirn ~ dur· . pe
· ·
mg lln par,t can be d eterrmned fro m a n o rdma ry sect ion . th(' i\tr:.in 1'1ierg_ v 111 Uk pn;uu.oUt.' bow-,
SOLU T IO~ . F rom t h<1 fi rs t <•f .-qs
_j
. ..
· ·h . ' . . t~A),
tensi le test diagr am sueh t1.11 lt! s own in Fig · 2 ·16 • J,•or any a88 ume d m a xi-
·
• . case (a), iB
It JM here 11.Hum etl · t ha1. tJ,e ver.1::ra.l ., .. .
•" - ·b 11 t10n
"' 8Cu!lrn e of 11tr<:&! w,,tn un der impa ct is the
eame as t h a t un d "'' " t.a tic 1oo..-J.
.om s11mAR ; II
8TILUN 'ENIUIOT 45
TENSION, COMJ>RJ188TON,
44
E XAWPL fi 2. A WPight W - 5,000 .le~ attarl,,,..d ~ thf' nd r 8 j
For <". ul' (b) , thr 11tr&ID t>ncrgy i8
(Fig . 2.20) rnovC11 downw&rd with a r.oMt.ant velocity • a e 1 M°p...,. •t« ,.ll'e T'OJJP
~-O'ld. What
' , , ~~P""i' TI
8tr098CS are pr od ucc d ID the rope wh en 1t,J up pn r od II srudd,n,l·,
_ P'(} )+ p•(D - i6i 2:tP'lE ., !_16 Uo. length of the ro~ at the moment of im psct LI! t ,. 20 m 1~ r,,:f. 'v
iB A - IO sq c m, and B • 10 x IQ$ kg/c m• . ' · e~ .t.xical
G'! fr'l'1'
IU1!!t
l I = 2t 4-.-t )E 2.-tE
N cglec ting tho m!!.811 of the rof)f' th" l,:ital en .. ru ,• ,~ . .. , .
SOUTTION
· h
. ' -.. ... ... ,e srsv-m ~t 11n y
·
met.ant 18 t e sumd of the elast1 r strain rnergy in the ro r- ...,.. u,... kll'~•·· ~ _, ,.__
. · h ·•ft .
f a II mg we1~ t, an 1..., potentia l energy relative to Mme dat,1m plane (/wre ~
- ur en~ w .. 14"

to be a horizonta l plane through thP lowe,t pc.-1tion of t.hP ,r,,e1,,nt ,- •


. I . f ·h ., .,~- "P~:it
thl" static e o_n gat_1on o_ t e TOJ.>" caused by W , an d.; , th,, tota l "lonu tlr.A 'If t1P r-,pt.
.- when the weight IS at 11.~ lowest pos ition . If 1t is &Mu med th&t th,.r- ~ 00 ~ m

1 the over-all energy of the system , the energy Just prio r to ~top;lL'lg .r~n a~ !Lil
elc~a_t10n c5 - 6,t above the datum plane) may bP equatt>d to the t'r:£t'l.Y 10 ti:t !ow~t
pos1t1on. Hence

} I 0
-,-
,,. AE6,t' W 2
""zJ" + 2g 11 +
W (• _ • ) = A E0
a
1

"•• 2J
I l
-1..
I Introduci ng W vbtained from the known relation 6., = WI AE mto tbs J.bo,.e

l_ equation, one gets


{Wvti
o = o" + '\JAffg
Upon sudden stopping of point A , the tensi le streS!I JD tr..e rope r n ~ t:i ci:,e n t:o

Fro. 2.18
-r v
C',ompariaon of th~ l.hree exµression11 shows that the strain energy becomes
~ = !+ ~
o.. c5,, 'V/Wl=l-
AEg -ygo..
mi&!ler a.nd llm&iler a.s the cros.s-~ctionaJ area of the bar is incre&Bed over more and
Thus using the given numerical data, we ha ve
rnore of rl.l length . ThUE, &11 a.Jr,;Miy not.ed above, a grooved bar is very ineffective
i:n aba:Jrmng eoerg}" under dynamic loading. It has l:w.eo found, for ex11.mplc, that
the life oJ engiM-head stud bolt.ii 68 shown in Fig. 2.1 9 can be incre&&ed by turning O' = !(1 + _-vuc5,,
A
;.) = ~00( 1 .!. -· · 1- = ..' .-1:h ki: r m'

A We see that the dynamic stress in th i11 ca...c.e 1s mcrf' than fi n• t1r:1t-S tnf' ror ~ ~
static stress WI A .

1 PROBL E.MSET 2.4

6. -f~f1 , ~'
It
I. A prismatic steel bar, ZS crn long, 1::1 subJectN tv tl.."r.W ,•~ re /,:;.f\.~
P = 2,000 kl(. Compute the 11.mount oi stra.in m t>rg_v ·1 (a l 1! th,• i::~~u..,iuJ• ~
A = 25 c m•, (b) if A = 12·~> c m •; E "':?1!O J• leg u11 A ru. \ll \ l "' l ~ - t )

>
.
y
3
) w
l under 1t.8
lg-crn .
rismatic steel rod of lt•ngth l and rro.o;..,s-..;;ec t1\).!1.'\I
. P own weight. How mu r h strnin
unit. \·olumo is -y? A ns._ C • 1'' -! l-1 lil:' .
. ~nt, r~u i,;
, ,
.
:t.t'l"tl A , ~-
·. ,,.t,)l"'t'<l 1·n t !t.>
·
hrn ., ..

l\.-u- if 1\.i wttght pn

w k~ " hi<:h l'"Mt 00 st,11'.-<l


\-erh .."l'LI,"
-

Fw 2. 19 3. Con1putf' tht• s!ram 1• 11.· r1,.} p.-r uni t \iilu nu t1i..d ~ - -"> hml ~
. , •h ut 1 ., ,,,·,'(/iru? tl• ·1r t1J·,. r1<>rtw1=
Fw. 2.20 .
in thf" foll ow in~ 11
.
,atMwl., t11 ti: nsiu n 111 · ' ' "\) , _ · r'' w,11
3 ,.. , , !() •k.: . .. ,•.J , h -..it ''" ...
down the ah.a.nk o< thP l.,o! t t.<, the rv1,t rf1 . ' I

= U ,tlt ll ~g ' 11 1 " l ru t.1'-o,-r-• 1' .,.


th (I ) strn, l' t11rnl stf't• l · )' tl ra~• ~ '1 I r 1"

( ,J
crea.Bing the amount of st.r1ur, i-nergy lhat ; 1: e~~t: 1111 e t hreaded portion, thus i11-
.. I , -
steel ; 'Y .., O·O(JU l- .: ', ,n3 J:. •:! , Ill k,t, I[\. Cl e I
6 • ,
}0A<l. &btiorl; under a given tcneile 0 ·001 k~ .:, 11~ , E ,..',l(lk l( , 111•,o·,, , .. .cO I.I{ 1 111 '·
D saEAR: II STRESS CO!l'CEN TRVC'IO N 47
co)(PBESSION, .AN
4G 'fYN'RION, . the height h throuih _which t he · ld '
ne
1
-~"" ... J 40()kg/c in rn the rod factor of safety in stress analysi s and to the benefjc ial effect of loca1 y1e mg
/ F ' 2 15' determi
· JO strE'""" J ·5 m ' A "' 3 sq cm, E :::i· . t ·b · . ' d" ·
/ _. -L. wn in ig. ·
nduce a ma,amu l upon stress d 1s nh ut1on . ThJS does not justify ' howeve r , our d LSTegar mg it
.A For Lhe systR-m m,o ' · •
. W -= !IJ kg. =- of this
/ : · ht W wust fall in. order
wea,: I
to pr
data are given . m those . cases_ w ere it d oes enter. In this article, the import ance
The following numenc a 3 Iastic string and the other end !actor ~ill be illustra te~ by treatin g briefly stress concen tration 88
it oecurs
7( JO)'kg/cn••· A~- h = ~ ~ched to one end of an. \t W is allowed to fall freely
stress" will be produce d, m tension or
. . h
compre
d
ssion membe rs. This abbrev iated treatm t•
en W1
·11 be
5. A small ~ m~ht TV is: ed support . If t~e we1~ns ile . occurre nce and
of the string lS tied : 0 ~
throu&h the full lengt .
th:
string, what maxi mu~ length l, modulu s of elastici ty
cross-sectional area ,
sufficie nt, 1t IB ope , to acquai nt the studen t with the
effects of s_tress concen tration and to help him recogni ze those cases where
it

E, and can be conB1de


st
in the string? The _ n~ h~gbtlfl S!. Ana.
wc1 ~EA) must be given conside ration.
. A simple examp le of stress concen tratio n occurs in a rectang
ular plate

"-w(
- A
1+ 1+7 v· with a small circula r hole at the ce nter when subject ed to tension
(Fig. 2.21a).

,,,,. addle with 8 small, soft ru~ber ball con-


~ tov consists of 11 small w~odcn p l, d It somewh at provide s a way of
~t.ed- to its foce by a piece of thm rubbe~ ;:: '.30g· and the unstrct ched length of
ping-po ng by oneself. The ball we_,g off the face of the paddle , the ball
P
r:ying
. 0 If after bOU[JCIO g al lengt,h berore re t urnmg,
. w
hat was
the rubber band is 3 rm . , . . t
gravity . The
stretches the rubber band to fo~r times ,tsr°a u~ the paddle? Ignore
cm•, the modulu s of _cla.sticity
the velocity of the ball when it left;~~ ~ce=o 0·0IS
of t~e ba _ A ume that the Ioad-de flect1on curve
nature.I cross-sectiona:I area 05
E =20kg/cm•,and P?1ss~m's rattO µ ~ 0 ·=· 3l ssWhat is the true stress
in the rubber
for the band in tens10n 15 hnear up . ? A~ ,, = I G25c m/scc; <11 = 240 kg /cm•.
band at t he instant of
_ grcat~t . cx~~ns;o ~ a is ~adual iy applied o.t point A, deflec-
3
7. When th~ vertical load ' \big. · k done externa lly is P6/ 2, where l, is the
tion is. proport1 ?nal to to~d ~! fa;~ ;;;t
11
this externa l work mm1t be stored in the
1 ulate the deflecti on 6. Assume that each wire
deflection of pomt A. . sing t E in tension . Am. ( C)
wires in the form of str~n energy, ca c d I f l t"city
1
(a) (b l
has the sa me cross-sectional area A and mo u us o e llS

:~c + 2\0S 1
a)· t
Fm. 2.21

Investi gation shows tnat in this case there is a. high stress


at points m a nd 11 o.t the e dge of t he ho le . The distrib u tio
co nce ntratio n
n o f st ~ -ses o ver
ers t he cross-s ec tion t hrough t h e cen te r of the h ole is sho wn by
the shaded aret\.
2.5 Stress Conce ntratio n in Tensio n or Comp ress ion Men1b
plate, the
been a s- If the size o f the hole is small com pared wi th the wid t h of th~
So far in the di scussion of simple tension and compre ssion, it has ection, a distanc e
ce ntrally applied magni tude of the normal s tress at any p oi nt of t his cross-s
sumed that the bar has a priMma.tical form . Then for
ly di s tribute d r from t he center o f t h e hole , is given by t he equatio nt
forces, the stress at some d istance from the ends is uniform
over the cross-se ction . Abrupt cha nges in cross-se ction give
irregula rities in stress distribu tion . These irregul arities a re
ri
of
se to great
particu lar u = ?(2+ i + /6 ~) (a)

le e xte rnal s to cur~nt


import an ce in the design of machin e parts suhject ed to variab "For 11 rather complet e discussion of this su bjec t , together with reference I, &!l d thll
Chap. ~,II
forces and to reverRal of st resses. Irregula rit y of stress distrib
ution at s u c h litRratu re, see Timosh enko's Strenr;th of Matena l,, 3d ed., P!lrt II , rn t1on, .\ fa;n,iru rol
article by M . M. Frocht, " Photoclastic Studies in Stress Co11cent
places means tha t ~t certain points the stress is far above t,he
a verage va lue
Engmee ring, August l 936 . Also see papers in th e Proc. Soc .
f
Rrp tress A ';" /!I sis by the
and, under the a ction of reversa l of stresses , progre...<;;sive crac
k s a re likely
followin g : Dure lli and !\1urruy , Vol. J, )lo. I , p. 19 ; Berkey , \ ol. f. ;'\ o. H . P· :>6·
t f h 'ne tion.s of fa ilu res ongma.t:uig
to s tart ll y from such poi nts The maJ·or,·t ff
Y o rac u res o mar 1 Ht>!Rnd Vol. I No . I p . 147. For 11umcru us excellen t· illudtra
. gradua . · ·
k - ' of stress '
' concent. · II .
see Ba ttelle Memon a M t.llute, er;
Pr •cnJioq of faJ11J'.1. t
parts 10 scr n ce ca n .b e att ributed to sttch p rogrcss 1ve ·
c.rar s. at points rat1on,
. d b of Mef.n/,$ John Wiley and Son!l, Ne w York, 1941 .
·. f
Stress concen t ration 1s a matter which 18 reque n tly o ve rlooke Y tSce T imoshenko, Strength of l',/ateria l,; , 3d ed ., Pa rt lJ ,l\lAr: . 56 • ~ ; ~~k Co l g34_,
· Th th ncgler t of t his facto r • h f . . . ty, cGrl\W-
1
d es1gner s . at e O o ur engine ering d esign lFor de rivation , see Timosh enko'B The<rr!I of EkutJCJ "

bas n ot Jed t o m ore freq 11 ent disaste 19 . m muc


r d ue to the employ ment o f a la rge p . 75 .
T}lNSJON, CO
JLPRiJSSIOl'i,
,AND sHEAR : JI
- BTRE8 8 CONCE ;NTR.\T ION

and n and t.hat this stress is a bout 1 ¾tim es the averag e stress
over the cross--
49

48 d of the plate. It can bed seen that


t the en s A
t m ~n n , where sectio n mn.
. .18 the uniform sti:e58 a 1 calized in this case_. , the maxi-
in wbioh 110
•ration is b1gbly ~ ·th increa se m the distan ce fro- In the case of a plate havin g two wrtio ns of different .,.idths
.... in Fig. 2 .23 . This stress may be
' """"" concen • p1d1Y w1 hole eq ual to mum stress occurs at points m a nd n
the 8 "'....,..,.~ tre&'-85 decrease rs ta.nee from the edge ofh t he be d ·
s
11 - 30'o i.ue essed point.5; 8
t a dis only. Due to t e. d n mg action repres ented by the equa tio n:
-
0
these verstr
'
d) " == l
1
IT CTo .
is set up O'ma.x = Kero
. f the hole (r == ' ssive stress of magm t u e uo
the ra d ius o f F. ') 21.a a com pre end of the narrow
. in whi ch uo is the unifor m tensile stress applie d t1t the
around the hole o ig. - . of the hole. he radius r of fille t.a
t the top and the bottom . I h le (fig. 2.2 1b), t he maxim am stress is portio n and K is a numer ical fa ctor which depen ds on t
or of &treu concen -
& In the cs$C of a small elliptica t~I a.xis
of the hole and is given by the and on the ratio b/ a . This factor is usuall y called thefad
2.2-3. • It will
likewise at the ends of the horizon traJ,ion . Value s of this factor are given by t he cur ves rn Fig.
be seen that the stress conce ntratio n facto r increa ses
with red uction of the
eqWltion (b) ution a t the weak
O'mu == CTo( 1 + 2~} radius of the fillet. A more sati.sf actory stress distrib
Th.is fact is no w
sectio n can be obtain ed by using a larger radius of fillet.
I t is the cus tom to
. . ·th the ratio b/ c so that a very narrow hole perpcn·
- well recogn ized in machi ne design and in shop practi ce.
· l of prog.res&ive
This stre'"':' increases
. th di t '
WI
f tension produces a very. hig 1. stress conce ntrat ion . specif y ii. minim um radius of fillet to preven t the develo pment
dicul&r to e rec -JOO O 0n of forces tend to cracks at reentr ant corner s.
· h c-"ks perpendicular to the. direct1
. u tion assum e
Thi s explarns-w y '°"' d by dnllm g holes at the ends of the All the conclu sions reache d above conce rning stress futrib
1.. ,, i;;preading can be stoppe h· h
spread- T ~ ~ of t he materi &l.
crack t.o elimina~ t.he small radii at the euds of the crack w 1c produ ce that t.he maxim um stresse s are within the propo rtiona l limit
distrib ution depen ds on the ductiltiy
Beyon d the propo rtiona l limit, stress
high st,re.<;s concentration. be subjec ted to consid erable s tretch -
SmaU sem.icircul.ar grootJeS in a plate subjec ted to tensio
n (Fig. 2.21c) of the mater ial. A ductile material can
increa se in s tress. Due to this
also produce high stress concentration. Exper iment s show that at points ing beyon d the yield poin t witho ut great
the yield poin t becom es more and
fa.ct, the stress distrib ution beyon d
hes. This explai ns why, wir.h ductil e
3.0 more unifor m as the mater ial stretc
a . /2
o .
II mater ials, holes and notche s do .not lower the ultima te strengt h when the
of a brilU,, matui ai,
~ 2 .6 j-,.5 notch ed piece is tested statica lly. Howe ver, in the ease
~u a I such as glass, the high stress conce ntratio n rems.i ns up to too point of break-
•2.o as demon strate d by the
ing. This causes a substa ntial weake ning effect,
_i..--- 0
~ 2.2
5 V I decrea se in ultima te streng th of any notche d bs.r uf brit tle m ateria l .
:g no tches and re-
The above discus sion shows , theref ore, that t he use of
/)
~ 1.8 \" ""_ 0 .3.0
ductil e
the ca_~ oi
&,

g r\ ~ h./ entran t corner s in design is a matte r of judgm ent. I n


' ....
struct ural steel, high stress conce ntratio n is not dange rous
provid the.re
ed
m --- n

a
~
.
~
14
f'....
"'
---,._
~
is no altern ating stress. In the case of brittle mater ial , poioui
of stre&1con-
b places should be
cii I centra tion muy have a great weake ning effect and suc
10 ....__....._
I ntrati on reduce d by using genero us fillets .
elimin ated or the stress conce
I I I I I
of stress, th e effect of sl ro.ss concen tratio n
In memb ers subjec ted to revers al
I I I
0 .8 1.0
0 02 0 .4 0.6
cracks a.re likely to l:>t.art &t such
Rot,o & must alway s be consid ered, a..s progre ssive
F 10 . 2.22 Points even if the mater ial mduct.He .
FIG. 2.23 •" · • P· 47.
wr'heae curve• are taken from article by M . M. Frocht-, loc.
•·-
m and 11 the stresses are auuut
ds three t'
~n oft-he plate . The standa rd te ./mes th e average stress applie d at the
is a~other example of a tension ns1 e ~~stBJ>ecime n for cemen t (Fig. 2.22)
ts
81:!ctJOn. E xperim en IUIOW that th mem~ r w'tb -L
_1..
. 1
cuiarp variat ion in cross-
e rnaxunum stress occurs at points m
r
THIN-W ALLE.D PRJ'.88URE VEsSEI.5
51
Then the stress resul tants actin g on t he ed ges f •h
. F. o ~ e e1ement are <T ·d&:l and
~,ds 1t a.~ s h own m 1g. 3.2b. The two stress ro°' •lt , _ · h ' . .
= u an..., in t e meridional

3

Biaxial Tension and Compression

3.1 . Thin WalJed Pre 99 ure Vessels


S tr~ses 1n -

In Preceding chapters we have· I discussed


. d"
simple
.
tension or compres.5ion of a materia m one irection. (a ) (b )
Very often we may encounter th_e case of a p~te ?r
sheet of material subj ected to tension or compression m Fro . 3.2
t wo perpendicular directions at the same time. To see
how such biaxial stress may arise, let us consider, in direction h a ve a resulta nt in the direction of the normal to t.l:e element.
Fig. 3.1, a closed thin-walled container having the form equal to
of a surface of revolution and subjected to internal u1ds1dS:t
2t IWJ = - (a)
O'J
.,Jn
ds --.
pressure of intensity p. Such containers as this are of ten r1
F 1G.. 3.1 encountered in stff>..ss analysis problems and are callPd In t he sa me m a nner , th e st ress result ants in the circur.lferen a 1 direction
pressure vessels. If the wall thickness t of such a vessel is have a normal resultan t eq ual to
sm&l1 compa red with its principal radii of curvature, the wall will have very
little bending resistance and a.ets primarily as a membrane in which the uv.d.s1t d{}.z = u-z<isids,z!,
-- r:
-· \b l
stresses are tangential to the middle surface of the wall and unihmnly dis-
t ributed a crOS.'3 its t hickness. Such stresses are called membrane 3tresse3 The sum of these normal fo rces is in equilibrium with t he nurm.3. 1 pro...&UI"c'
and a-re eaEily calculated from equations of statics. Isolating an element A force on t he inside surface of t he element ; thw;
of the wall cut out by t wo parallel circles and two meridians we conclude
from symmetry co nditions that only normal stresses u 1 and' u, act on its u1ds1ds2t + u-ils,dsit = pds1ds!, (d
edges and we obta10 the case of biaxial stress. r1 ri
. T o cal~u late t hese siresses, we refer to Fig. 3.2 and int.vduce the follow- (3.1)
mg notatwns : from which
= tensile
0- 1 stress in meridional direction 0 r _. .,J • _ _ , . l i . thin-w~led pre5.'-U~ \'i'S.•, ds o! vsnuus
0-2= tens1-1e st ress in
. .
ctrcumferentia.l direct'
me, uiivrf.Ul, stress
hoo · Applicntions of tlus formu a 0 1
·11 t
• ·
1
shapes ,,·111 now be I us r ll t.ed b~
.
t = un iform thick ness of shell wall. ion or P stress. .,· -,evcral examp es. --'--'
. d . • for a tbin-wa!!t-d sphn~
r1 = rad! UB of curvature of meridian at A. h branc •t("('s..""(':; Cf r s.n " · ua.! ~urool
E XAMPLE 1. Ca lculate t e_mem_. ·r ·tis ;;:ibJt·ctrd u.- uruform i.nt,e.r P -
r : = radius of curvature perpend itular to m . . vt-sscl of rn di u~ rand we. II t lrn:krw,-:; 11 1 .., t,"6
d81 = angle su btended by meridian a f e rid1 an a t A. interi ., ity p . .
1'h<'n ,--q. (.,.! ) !lt on
_ . gl . re o e 1ement . e CT .., J r = 11 a nd r1 "" r , ; r.
d8: - an e subtended by arc normal to . . SO LUT ION. I n t h ts r as 1
. •
d&1 = r1 d81 = d 1meI1S1on mend1an at A redu ces to
of element i .d . •
dsa = r'id,01 = dimension of element iin ~ eri 1<mal d irection.
• i circumferential direction.
To 6rtd W '71.lu-<>1 ~,!;11' -.h.iar.l:dw, ~ •~ ,
,:nan n,uw
~ to ~ And
,....,. • t u.ie ~ . . '-n,"'
"~.& lit
of u4,, s;,-4." ' ,.,
-- ---- -
d#, ,_ ~ 3 (~ - 2i, ... 0.
dv I c~a

from •blC.b v ... A 2. TMO ""lb Ul.tl' "'ll•_.. '"' 1


eq . {h) ~ <rf"I
fi'.-V taa (.Y
(111) ... •
4 l ~ eI

The ~ua. , o, the m-nd,UOAJ r t ~ 1 t C..' -


in
er, ~~ ..
leveJ ,nn 1s found from t.h,, Mnd ~Jl th.l.t J .
' 'i weigh t o ( the ehad cd volum -- of -wsa r i.D
total
,~~
~ e, Fi1. 3.4 ia supp ort.ed by th¢ '-'tr.J eal C">m
0n on t.be c.r:utrd'""ffl:IIICl!
1\\ of the mttid lonAl tena1
,..,a. Tbue
Cb ) i,} rt' ~ • <111 - o-ey :c> o,
fc l o- 1 • 2ry tan ci • t · eM a - ,v-(4 - >

=
Pio . 3.3
Thi• si vee
t.';;."I::.": cn t~ e-.od a.I th..- wit 1"' p-r,-' and this forc-e must be balan ced
by the
t he cy linde r . 1 hua
i» tan
21 coe a
a(..,· _:_,.)
r
-Jr..~ S::."ff6 /l : li.."OUod th.- r ircum fel"(>nc•" ,if O'~ - " JI'
~
2m - rr2p,
A1ain aetti nc dqJd11 - O~ ,.. obt&Dl
(1 ,

C I • !!:.
~
(D " ' - • '60
d11 11 ix.
Q
a
(i\ - !j# ) - 0.
·
{.., ~~ '-fl!l illJo1 f, we .!iet- that in h,

l.u <'~ oJ irr-ula r c-yltn driea l tank ,iul>- 3,\/ 4. Wii.h thta Villa. ul. , . eq.. rJ twc
l!'"Ctc'.l lu • ~ cuiv....r,J J ~ , : tht
ll. t
from whic h y -
st ress a- i is Just ha lf M la rge a:s
~ bcq- ~ , . ' . kirtg1tudm 11l
Ut,C ,;j
(6V- - ~
~ .6 ;\J., ( . ~ ~!!.~ • ---
utulr.1rmJv eus~ • d ~d a.roun d 1ta uppe r rirn is let ClOlt c,
~ .._ti. ..,u.:i- "-- rt dtit!t ,, 'AW':
t1..c ~.own l.D F •• l"'-n
41 u.,, • ., i.g. ~-4 Calculate th mem brane st.reMes
~=•'4-~ - .._ ~ tY-....., ,. vl:..ii.,,
tJ
o! ,, for 'i ,·ruclJ · e Paoc u...... ... ,. 3..l
~...,, . _ ,,.__,. eacb of tl1e.se stress es will be a
~·-u.'1',,
= • itSt * ~ i.1'i~ . - ~~ .~
•t. Calculat.e t.he w e m.c.erna.1 &U ~--
4
,_ '-<It'·~ of
coo IOI.J ve!J§e] • eq . ( ,,·J I ). g 1v r' S ~ ~ ~~ ~ :, - -.... u
OB dud i.f c.})c ica,;,, J..~
Ii
of t.hin macneaium plau tr..!'\ ~ • ='
0 , '"' ~2 cm and the &llowahltt s1r. . m t.en,a.
.,o. Ii> c'--'\) 'l,g-,- -."' -l1lloil;,. .» ..,
~ ~ ~ ~ ~ ~ .,..._, ~ i:~
•. 2. A virtua.l.ly ~~t. hia.
~ , , J:I tL, ~ 4
~~ ( , IZ. ~ t ~ -' a: ruQC

~~f
t
th I-
(g) tluo ~@!' te!U

I la held h&lf au.bmttpd in wau t b ,.- 4 6.x-t


~~ .-i, E\c, \ • ·-i!l'~
~:~~ ~
_.. ~ ~~... ._ ?'tw
~ ~~
~d •• ..~ ~ .- 111.r..tr ~
•. i.£J . Cll.r\·11.Lu_r., lit 1..11.., ~1
.u:vel mn ill sh own . S ub- prlom pal sttt;dik'\ii 11 1 - .._ ~ Si.
C
- -.. \ ~ ):: T, _,
'{)ecifio wci1 h l of w & hlr 11 "'. _I ,u. ~, -

(b }
r 54 BIA.XIAL TENSION .um co-~
.
3._A thin spherical shell of _me&n rad108 r
tensile forces P acting along a dwneter of ~e ~al
internal pre<mll'e, i.e., p = 0. Find the pnncip
point A on the shell defined by the angle 4' 88
sin' q,.
..n,Jll',BSION

d trail thickneea t iB subj9Cte_l to


":ere
auhown in Fig. B. Th.ere 18 no

:o:
brane stres!tJB 0'1 and cr, at a

.Ana. O's - -cr, - P/2m
F•a.
-e
TIHN-W ALLEJ> ?P E88UltF. VE88 F.:L8

9, A rubber torus inflated to a pressure p ..,. ;:.g,·c.ml Ii.!:! t b d;m,.n clo -1..
· F . Ca Iou Iate t h e membrane
stresses on an element at A ifuu
cm, c - 20 cm., t = 2.5 mm . A M . 0'1 == 50 kg/cm' ; u, = 20 kg /c m•.

3.2 Further Analy s is of Biaxial S tress


a = '"'.:,v
on

r s •-nown 1n
cm, b = 40
55

. . rted on a borisontal baae ae


• 4. A thin-walled cone (wall ~ck:ness t) is suppo Neglecting the weight of
shown in Fig. C and subjected to mtern&l gas pressure
the cone itseU, find the principal membranes~ eri n, 1
,:d "
at the level h below the In the preceding a rt icle, it has been show n how an element of ma.ten.al
may be subjected to tensile or compressi ve 1!tresses in two perpendicular
apex. Th.e ape1: s.ngle of the cone is 2a &8 shown. A · directions simultaneously . Consid ering again such an element (Fig_3.5a),
ph tan a. pla tan a_ we shall now study this sta te of bia xial stress in more detail . Here, we
- =----- er, -
11'1 - 2t coe a' t COi a denote the tw_p given stresses by uz in the x direction and a, in the y-direction
i--1·2m-J and assume that tensile stresses are positive. ,
We may now inquire what stresses exist on a plane whose normal n m&kes
the angle q, with the x-axis, a.s shown . T o answer -this question, we isolate,
as a free body, a triangular portion a.be of t he element as shown in Fig.3.5b.
Let dA. and dA., denote the areas of t he faces ab and be, respect.ive1y , and
dA n, the area of the face ac. Then the st ress resultants on the faces ab 1a.nd
y
1oo-- - -3m - - - ~
a
FIG. C Fm.D
• 5. A truncated conical tank haring the dimeneiona shown in Fig. D is filled with a1 dA 1
wat.er ( 10 = I gm /cm" ) . Calculate the membrane stresses 0'1 and er, for an element
I "'•
A of the wall situated u shown in the figure if t -= 0-03 cm. An,. C11 - JOlkg/cm•; tTz

er1 - 536 kg/cm 1 •


6. Uthe cone in Fig.Chas no internal pressure but instead there is a concent1'ted "',dA,.
C
" "'r
vertical force P acting downw&rds at the apex, find the principal membrane streesee (c)
er, &nd "• at t.he level h below the apex. An.. 0'1 = -P/2rAt sin a; "' = O. (a) ( bl
7. Two vertical steel standpipes each 30 m high have diameters D 1 and D 1 that FIG. 3.5
an i n ~ ~ I>{D, = 2 and are filled with water. What is the ratio tJt, of their
wall thickn~ if they are to have equal maximum hoop stremes "'? Ana. be, respectively, will be crsdA.s and tr,dA,, as shown . T o balance these forces,
e,/e, - DJ D1 - 2. • we shall need both a normal stres.5 resultant o- .dA • and a shear stress
a. An inv~ hemiapherical shell of mean radiUI R - I· 5 m and wall thicknese
t - 0-02a c:m is bolted to a horiaont&l slab and filled with water (specific wei ht resultant TdA,. on the inclined face ac of the element as shown. ~ otmg tha t
tD =- I gm/cm ) to a depth _h ~ l ·~mas shown in Fig. E. There is a small air hol: dA. = dA" cos q, and dA., = d.4.,. sin q, and projec t ing all force,s on then-
1
at
the top of the ahell to equalise inside and outmde air prearure. Calculate the princi- axis, we must have, for equilibrium of the element ,
pal membrane atreeeeB "1 &nd "• for an element A of the shell one foot above the hue
AM. tr1 = 68 kg/ cm1 ; 0"1 • •72 kg/cm 1• • cr ,.dA" = O' .(dA,. cos q,) cos 4> + e1,{dA,. sin ♦) sin 4>,
from which
) U (3.2&)
er,. = cr. coal q, + er11 sin' ♦ = ½(tr.+ u,) + ½(e1. - a, cos ·
j . . f th element on t.be direction at,
I In the same way, proJectmg all orces on e
} A- - + - - -A
we obtain . )
-rd.4. -= a-.(dA. cos ♦) sin ♦ - tr, (c.U,. 510 • cos ♦,
! from which (Ub)
Flo. E . ...,.,_,,"" • t(O'. - O',) ain.2 • .
,,,, ) 81D., ~..,
Fut F T - (c7• -
Al'iD co»rR ESSION
AN Al, J'JHJJ OP IJLUlAJ', ~ !57
J.A,Xl,4J, -r,;l'lSloN
8
56 .. tbe mll>'rnitudcs of no nna.J st ress
. (3 2&) iu,d (3.2b) gi,c. t,at ion of whi c h is d e fin ed bv :" t he s t ~ in th&t d i.rt>ctt.o o mit e.l.!1<> 'Jp!JO the ~t.r"91 1n UM! o r ~ ~ l
Tbus~on.E ' ·n am· pls nc, th c ondcnremember that sin (!)0° + 0·) · directio n becau8e of the P <YU11m1. r<J1. w Pf!PCt dir,,.JS:!!iied in Art l .3. In the
s nd ·sheAT •
!<lire~ -r Cl •
angle~' c: 90
+ ~ ~ an
(1 Zs) and (3 .2b) lx- <'o m "
"" .x dire<:tion, with in the etut.1c llmit , th-- p!ate ,vil} h&Yt! pcau-re lllnm
U we. take an ~) :,= - sin ,fl,, eqs. " · . ,, = u-/ E and nt t he 81lme time 1t will hav ~ IM!ptive , trn.in ~,, = - ,w,JB
COc' ¢- whil e- tC\!l (00" + ;,,t == ½(o-.. + <1r) - ½(a,. - a.,) cos 2iJ> , (3.:lo.) due to la t.era! co n t raction ~iat.ed wilh the ~...sile st.ml!t ,1 _ . The -.me
_ _..1 ¢ .... a 1 c~ ¢ ) · 2 res.saning h o lds for tow! '3tram 1:1 the II d.tJ"l"cuoo. Thu.s rn Uw ca.a,e ot
t1 .' "" t1 c bU• ' . ,._ _ _ 1 (a,. -
1
<1r, s m <P - C:i.3b )
~sm..,, - biaxial tension of a t h in plate , thP. tot.al or ne-r. dra~ will be
r' .., - v,, - <1.,) cos I a.nd shea r stresses on n pl a ne r>e r-
~- flvn--=....,nF repre.sen t
th e nF~nn a c~) As discu ssed in Art. 2 . 1• thC
,uiese =t-•~.,.,..~ - <JC (see · 1g. 3 ·"" · •
pendirolar t <l the plane - d by eqs (3 .3) are called comple mentary
gi,-en b:· eq!'-. (3 .Z) a n ~f hia.xia.l st.ress, t he s um of com ple- '!.z /,UT !I
8
~
•\.. . ore geners.1case d 1 "' =E-E
~•'-'· l o Ul ~ m d , i~ a,<Nlin constant an equa to az + u
me-ots:·y nQI"Jlla
l stre~ <1 sn <1 • ~ r,-
. • -ses Likewise the com ple m e n ta.ry shear
••
In the z-<Jirection uormn.l to t he plane of the pl&!.e the:.~ "7o1lL ~ ~
of the , t wo givento!-tres5 · · d b f · con t raction due to the Poisson ratio effect a.saoca~ wnJi eac-h ~ t.=ie pmx:1.
the sum be equal in ma.gmtu e ut o o ppos1t.e sign .
,.._ _ -rand r are seen . . d. pal stresses u,. and u,, so t ha t the net st.nun 10 '1)13 di.~ :.cn Deeoau!:8
.,...,__. . - bear stress 1 t he sign convention 1scussed in
W~ oonn.'lue t0. l.lle.sh rnr sstress ' 1 · · · when i J-.Sb j
. 00 any face of a n cement 1s posuwe
Ut 2-1· that 15, a esr . . 1 k .
' · · • bout 8 c.enter inside t he element 1s c oc wise . Thu s
ltS sense of rotation a . . . . Very often in experimental stress analysis the princip&l ~ . . . , ~ ~.,
. _ 1.. ,..,,_,,, -r' has been shown m 11:.S proper d1rect1on in
the oeganve Su<".&r "'"'<-= will be measured directly by strain gnges. Then the co~ ~
f rg 3..5c. . . u. and 11
c- can be calculated from eqs. (3.Ss). which, for O w . ~, m.sy be
Fart.be enroin&tion of eq. (3.2a) will show that as the angle q, vanes
put in the form
!mm uro to -r/ 2, t.be norm&l ~ O',. varies between the values u:r:, when (E,, + J.t., )E
t - .o, e.nd <rr, whtn 1/J = r/ 2. Thus the stresse.<1 trz and <r11 represent u. = 1 - ;.'-
)

nsnmum and minimum values of the normal stress. For this reason, (J_a·
u., = (f. + µf .) E
they &re e&lled. principal st.Tesst8 . 1 - Jr
Similuly. it will be Be"..n from eq. (3.2b) that the absolute value of the
If the plate carries compres...;;ion in either principal ~ t.lllm- et;-'- ,~., , .:, r
t!hnr 8"~ r is a maximum when q, = r / 4 and that the magnitude of this
(3.6) can still be used by ta king tht' correspond.mg~ or twit.!: c ~ !.1...-e ~
m• rimum i:,hear BtreSB is
EXAMPLE 1. A thin circular s~l plate o f radill.!I r &.ntl t h ~ ! ?S 1 : 1 ~ ~ t.."l
(3.4) radial stress u uniformly distributed a.round its 1:i.reumfo:'t'nre Jlo3 SOO 'f:l :. ~- l .d
Determine the state of stress on any ekmeot such as .-l a...."'Ci 100 llii! ll!ni. re;-~
~ the difference between the two principal stresses. If the two
i.,~ change of the entire plate.
~~IP ~ a.re equal (11z = a., ), there will be no shear stress on any SOLUTION. Since both the plat~ and the
YJ&r.e w ~h u i:V- in Fig. 3.5b. external loading are symmetrical around the
W-rt.h proper changes in ti\O'n (3 2) and (3.3) can be used also if· one center 0 , it follows that the deformed plate
or both f •L . . • • - · , eqs. · • wiU remain perfectly circular in fonn , This
o i.m: J.m llelpal atresee .od h . .
ti,·t. f~r e.x&mr.l :f . 8 ~ :r: a u'/1 s ould be compreSo1on, 1.e., ncga- means that both thf' radial and ci~umfc.re.nta.l
~ e, I one pn nc1pa.J 8tr . . . strains must be uniform and p.qU&l throughout
COO!pltf.lrion o( th . . ess u ~ 18 terui1on and the other "'JI 1s
the plate. Then it follows from l'<l• - t3. Sa) anJ
th.at the rnUu.nurit. .~e m&gmtude (cr11 = . -a,.), we see from eq . (3.4) reference to tho elt-ment.a A a nd B in Fig. 3.6
So f ; 0Ut:11r stress becomes _
. ar, we h~v ,~ < on::1d(•r•·d l T mu - u •· that u., = rr. = <T . Thu.'> we ha ve tbt' e~ of
Jecwd t-0 rm.x1rtl l.t:rlb,or th
_uri Y c_ttatc of i-;tress for u11 cle me nt :,11b- equul principal stresset1.
t.h IIt . i <>r <
;orr, pr~ (] Before deformutiou, the volu.mt, of Utt' pt.i.,
e ram or dcforma.t,kto ! Hm . We shall no w d i:K·u~s bri<' .Y
Btr~ ~ d . (J 5Uch an L ·1 is Vo = rr't. A fter cieforn1.ation, the rlt'w
3~ . un . v, in ~)(1th prine. , I . e _ernent. When there are tcnBI o
V ..
r&Jiu11 ui r(l +
•J a nd t.hc n~w lhit'k.tlNIII is
.va), the strain in ci tbct of t.~fla d1_rrc t 1or,l:l x a nd !I o f t h e el e m o n t ( Fig t(l + •,). ThlJi. t ht' now voh.un~ i4 ~- .,..
Pw, 3ti
~ d,t,,c·tic . .
·· ,nij WJ 1I drpPml not only upon
t.
i
COII FRJl 8810 N
Tlllffl101"' ~D AXAJ .. TIJU Of' .OIAX JA t. ~ 69

~ &nd .l,a~ ing only t<>rms of first order 111


tt ( l + ,. + ?A~ ) , and thC' tot.al
v ,,... 0 3 anrl l'I 1r,-, 9 ,-:::, !°"l (. ,if r.i, (r A .. a. --
f;xp-Sllding th);'
-

5S 1 tl11, t. p, - I 10, • kg lc m1 , JI (1

The corr espo n~in g unit v~ 1lfllt


.' re(iu(:('6 t.o l ';' .-r+ 2,,).
n lfl Fl;(. .J .l p 53 hu r.a11_.. ,. _ =,. 1 ,-.,.
1
_ + t.,,Y(l -+:- t, ) . 11111 ~ 1.0IJ.'J kg / <: m
1
,,.,,.nut,· t. ' . J' Q .. t:1"'l lt• - ' eqs. (3 .5e.) gi ve 8. Thfi thin- waJIPd cyf ,n<ln ral lll.olc ahr,·o p,.,. ~ . • ,: ..-_r,• °""'n~
~ -"
~ lSIIJlO-'--' (f IS
•; rf \f> 1..- ~ ~
V
i - -

tht
\I -
mes Al ""
1
J\o"· fn r e1• '
wall th1ck ne~ t = 1 mm a nd UI su b1r,- r .., 1.-. "l =•
t cha,'Jgt' t,e,t(l -r S f I + 2.f. !_ 0 - µ) , Calc uln t<' tho mAgn itudf' of s hea r 11tr-
g r a.Jioo 7 u. •Vi- i.PitX A"tit
p1&.:](t ~ t,...,.. ..t. ol
r ... mum •h-,•.u stno q r.,. ... .n ~
[I d>.s-'1gt' is il c
9. Drr nr<' a. for cnulA for t.ol" maiu
p ,-. ,. "• E shm rn in F1~ 3 ◄ p. ~1 t;,u
!i.
thr coni r n.l tonk fill ,,d with WAU-r Ill!
.w h' ta.a a
~: r ,roo ... = 64! ~

t:
a ·t ·olum e chan ge beco mes s1
· the uni \ 3.3 :\>foh r 's Ci rcle for B iaxi1 1 J Stre
W---dh ~ r.l~ for ~ - _
.1 V = ?'.! ( 1 - 2,u) . (a) pla t.> ~bJe<"te<l · o b..:u u! •~._ , 0 u
r E Con side r Bgtlin the cnse of a thin
Art.. 3 2, t be oor.::ia.! :u.G. ~-i. ~ 'tr~
show n in 3.7a . The n as disc usse d rn
0

2(J0\ 3 kg/cm ', and µ = t, this give s an angle r;, ~,r.h the ~-ts:L 3 r?D ;;, ~
_ on any plan e who se no rma l n makes
~ £ - 2 10'" ~ cm·, " - nven ienc e of refe renr e
by eqs. (3 .2) repe ated belo w for co
~ J' _ !. = 2 ( ) ' = 0.001
10
I
E 2( 10)• r. - q,. =
I
2(<F. + {Ti, ) + 2 (0-.. - C7~) c.os u,

P&O BLE KSE T 3.2 sin 2~.


T = ~(CTz - g 11 )

the elem ent in Fig. 3.6 if tr. • 560


L ~ uie rlrm!eS "., ,r'. , T, &nd r' for the equn t 1on.s 0f !1 c::ir-.....e o a.;:-- ~ ~~ -
= 30° . .4 rnr. <F,. = 350 kg /cm• ; u',. = - 70
q ~ ; c-, = - 280 q /cml. and¢ It can easi ly 1:-e s how n that t hese are
□ trod uctn g the a.u t.a.u ~. ..
q 1et::f , "" - r' a: 364 k8 '='- with the angl e </, as a para met €r. l
and r' for the elem ent in Fig. 3.5 if "• •
w 2. ~ the gt.re9Bes fl., rr' ., T , -38J kg/c m•;i r,.= -529
,gfan '. and¢ =22 "30' . An,. <Ta•
A - l50 1.i·= ~, a, - - 50.lli
q =,: T,. _ ,,, - 74 q ,acmJ .
and tr• •2 IOlcg/cm ' for the element in
(1 .

l. I1 the pr..nri p&l. st-re!ll!f.! r:. • :l~ /cml co mp.<3.(· t form


~ for whic h the shea r streJ S will be &
maximum we can wri te e()s. (o.) in the m ore
v, 'F"tg. 3.i, find the "-.Jue of the angle
70 kg /cm 1.
&.ad t!!l'Lluat.e lhi! eh.ear litres. Am. 1"mas ,_
y, z~c tioM , ~
.~ A ~l a: bl~~ ~vin g dimensions a, b, c, in the z, sin ~ -
and "• of such mAg nitud es that the UD1t =
~ ~ -.i~.oo ~ tu_ml stre8ea!
T Ti:nax
gs
Show that if thes e strai ns are small, !W!\.- ODrl ..-.i ~~~- ,c' ~ t
tbei mm ~ r, z-directwru are (a, ts, and t., The n to elim i na te the para me{ er ~.
we note from the
• ch&~ of the bloc.k will be .'1 V
/V = e. + ew + e•.
&. A v <T = 1 400 kg/cm • in one
~l
cted to t.ensi Je 8 -..._ t.h
re pla-te l cm thick i!! !Uhje
✓1 -

-',__,_ aqu&. .., ~ • '

tl'
....,~ 6lld c,-~
~ m von:m,, of . 1.. ~t.e . An,. v:~tresa
a, = - 1,400 lcg /crnz
AV - O
i:u th e othe r dire ction
.
. Find e cos 24> = sio 1 2<ii x: -

, - ~e
u
p
A
· l ili , r, ..;•u~b
1
~
one ia
but

free
z]:
A _ha.rd rubb-.er block com plet ely conf
to ei:p& n.d in both the
ined in the
v- &nd 20,. Subs titu tion of t h is in r he first of eqi:1
(e )

=
1
1 -
gt\'"\ '$

,-1 '°•--<>
· V,
iU ~ec~ to com pres sive stres
a <1• • - O' ., - 17.-. Tro u \
~ tg/ r--m•·
g.A. Calc ulate t.he
j / ~ atn..ee m_ the i,-dir eetJO n u show n inFi
or (u ., - a • .,)' + r 2 ..,. , . ... t
in_tbe ~ t i o n. Wha t ~i is the chan ge in volume
~ L.... [ .C J{~
JO ~lll l t.l.<.U \ tll !l nr0 ~ l:<:
An ~ :t. l"M:UCU¼
½?Ana. This is L.he sta nda rd form for tht·
<T• - -
ig ·m ,•, ~1!_ Eo."'20Jcg /cm• and µ -

LJ
r, th<' ,
..,-..,a.. ~ &t o ..-. ... , , .,._. + ;~) aa.ih own
7• A UliJJ circu1&r b und r ... .,. ""' Hu. - c,., ) a nd cent ('n"t l on
~ d aro a, cv d . f.._,r h1.H Ull s t ~ 40J. it iri V&r f
in Fig. 3. 7b. T hill is ~lll 'd MvAr 's
~ a ri . . _ rue rnem brao e jH supp ort.a
, v, rt. rill' l>y
. ;.; Pf!rature ( :L<J-!, ~
etrcul&r br&aB ring, and at roo m~ £_
e mezn t,rane i8 free from etrM II• UHef ul as & grap hic.a l m ctUh i o.i
~l! ""ing- v.;i~ ta )
e
i'lo. A PMncil)&l 1~
C onsi de r, fo r ex-n rupl o, R O)" po.m t l) ;_,fl the .•1 f't'le &nd de-outc ch£> fl.Ogl
bra11e it it.a ~ "" ~ tr, will be- indu20 ced in th.r. aiefll
"C ! A.-Ulfll
I
t !.Pera t.ure (,Jui to
.i
I I

l: ;
COJO'BJ',881 ON MO B_R1 8 <JtRCLr. FOB BIAXIAL !'J'nll'al
61
SN&f OM Al'f])
sJAX!Al- 'f

B A

,,,,

(b }
(o)

( b) Fro. 3.8
(o)
Fro. 3.7 ZJCAMPLE 2. Conatruct Mohr's circle for the cue of simple ten.ion whue <7 11 - o.
a1 ahown in Fig. 3.9a.
from geometry it can be seen that t he co- r
ACD by 24 as sho1V11. Then ., ,
orcfin.&t;e5 of point Dare
OE = OC + CD cos~ = ½(O's + O'.r) + ½(O's - CT11) cos '24,,

DE= CD sin~ = ½(<rs - 0'11) sin 2q,.

Thus the coordinat.es of point D in Fig. 3.7b represent the values of "•
and r for the plane whose aspect is defined by ti> in Fig. 3. 7a.
for each different aspect of plane in Fig. 3.7a a., defined by ti>, there is a ( 0) ( b)

~ n d i ng point on the circle, the coordinate s of which represent the


normal a.nd shear s ~ on that p lane. For example, when q, = 0, the
Flo. 3.9
normal n coincides with t he z-axis and point D on the circle coincides with
point A , giving v. = <Ta and T = 0. When ti> = 45°, 2q, = 90° and point D SOLUTION. In this cue, CT... ... ½(CT.. + o-• ) ,., ta-. a.ad the circle ia ~ • to iibe """
uia u shown in Fig. 3.9b.
falls at F giving O'• = ½(er.+ 1111 ) and T = Tma" = ½(CT. - CT11 ). When "'U.illPLE 3. Construct Mohr's circle for the cue of biaxial ~ti-ea ahoYU in Fic-
• =: ~ • the _no~ n coincides with the y-axis and the correspon ding 3.10. where"• U! tension and tr11 ia compression. Amume f,.,j - 2 f•.J.
~ of pomt D l.B at B on the circle indicating CT,. = CT11 and T = 0. It F

~ be noted &lso that the coordinate s of point D' diametric ally opposite
pomt Don the circle give the compleme ntary stresses CT,.' and r' as defined "
eqs. (3.3) . Thwi all y~
by_ __
1
nnssible informat'ion a bo u t t h e stresses on variom1
P1AL1t:~ can be found from Mohr's circle.
.

__ l"
\
U either principal stress .is com . .
aign eo that in general th pres&on, It must be taken with negative
the origin in Fig,. 3 7 b beucet n ter C of Mohr's circle may lie to either side of
. · , a1ways on the 11 • Se veral particula .
are illustrated in the followm · -aXIS. r cases
g examples.
(bl
",=::~ ~ Construct
u shown
11
~.ohr'■ circle
"• 3.&1n
for the
Yig.
. .
f .
CUe o blUJAI atreas of a thm plat.e
( 0}

Fto. 3.10
IIOL'fl'Tlo.H. In tbia caee" ·
the circle fa.lla at the · ·" • H + 0'11) • j{ f
11

Thua Mohr', circle • ilJ ~igu), trhiJe ite radiua "• - CTs) - 0 and the center C o IOLUTJOH. Inth.iscaaeo-..., - J (cr. +vw) • -~.,and~.... - t{,. - , .) - "2c,,,.
ae ehown in~- a.ab."••• - HO'. - O'w) =- t(2a.) • "•· Thu, Mohr'• circle will be u ehown in Fil- 3, 10b.
I P U fUl f\IT&AR 6a
Bf~"CB.L T:&....._SION .\.ND COM P R~JO N
62
scrue in Fig. 3 .1 l c S ioce there l\11" n0 n,;,rrtal 8'tTll!&Y'...il 00 It.a ~dgew, ~
P ROBLE M: BET 2.3
ed ~ wilJ re mai n unch.nnged in length du.nng ddo,nation of the e!ffll£tlt
J. CollS'b"Uct !\fohr' s circle for the particular cn.._<:e of binxi i:il tensio n whe, e The ho rizon tal d lo.gonsl ab will simply elnn~ a.nd ~ -;-r rw:sl d ~ l
. VillA t i! the m&Ximum shear strC'SS T.., .. in t.--UCh caso ? An.~. T mu ..,, o
t1 _
0 will contract eo Lhat , after deformation , th"! el--...ment W\11 b.s•~ the form 00 •
• !. ~ c t Mchr 's cirde for t.he cn...<:.e o f biaxia l st.rc.."-'l shown in Fig. 3.7 if
rhombus as sh own by dotted lines Thi.JS, th,, f'Jngum} n~ht acg!a at a
a. -cc,56,' li:g <'m•&nd aw - - 280k!<]cm~. From t h is circle , find t.he str esses u,., er"' r, and
-r' fa: the inner el.eroent where (f, = 30°. Am . a-. =35 0 kg /rm ' ; cr • -= - 70 k>! /( m'
e.nd b becom e r / 2 - -y and t hose at c a n d d beef' m,. "II' 2 ..... .., _-,.,her.., dl.!E.nrs
T"" - T -
1
5"64 lqr <cn> 1 . '
the a.mou n t of angular deformation of t he eltament Thl.3 quantity i.a a pUtt
!. Reterring to I.he conical tank in Fig. 3A. construct Mohr's circle for the state number and is called the 11Maring 3tra1-n . ru1&logoua to ~ :.etwle or com-
of stres., on an element si tuated a t the elevation y = h/ 3 above the apex 0. The pressive strain , as discussed in Art . 1 3. To ·ri5u&liu t.h:.11 is►rl!SS condi:t.ro:n
followmg numerical data &re given : h = Sm a = 22° 30', t = 0.2 5 m m . From this more readily, we rotate the elemen t acbd by 45° aod ~ t h e ~ o',!'
circle, fi:id the maximum shear stress in the pla.ne of the wall a ~ this point. An.,.
and bd in juxtaposition o.s shown m Fig. 3 .12. Then the an~~ 13 ~n t.o
-,, .,._ = 14 .5 i.i;, cm'.
'.'- _R eferring to Flg . 3.10, s.ssume u. = 3~0kg /cm ' and a-, = - 700 kg ;cm •. For these represent the previous!y defuied -y a nd its value iii
pnDCI~ ~..real!eS,_eonstnrct Mohr's circle and find the value of ¢ defining the plane cc' c5
O?I which a .. vanishes. What is the magnit ude of the shear stress on bhis plane? "I = be = l'
.1tu. ti>"" 35° 16'; , = 49 71 lcg 'cm1 .
5. ~erring to Fig . 3.7 , assume that u, = 840 kg'l c m 1 e.nd a-11 = 420 kg lcm• . For where a is the lateral displacemen t of t h e
tbeie pn.n.cipa.l stresses, construct Mohr's circle a nd find the angle ¢ defining the
upper face of the element relative to the lo wer
P ~ ~o which the resultant 6t,ress IIUl,kes the minimum angle with the plane on
wbich it_ ads, or, s~ted ~other wa.y, the plane for which the ratio of normal stress face and l is the distance between these fa.ces.
to mearing lltre9! 1B a mlilllDu m. An&. qi - 54° 44'. Thus the analogy between shearing strain ·r
and extensional strain t is comple te . e.""t Ct!pt
l.4 Pure Shear that in the case of shear the displacement o
takes place at right anglei:1 to the length / so
Let us °?nsider now the partic ular ca.se of biaxial strPss where u., = -a.,, that we see the ratio ~/ l as an angle .
~ shown m Fig. 3. lls.. In such ca...-.;;e, Mohr's circle will be as shown in Within the elastic limit of a material, it is
Fig. 3.1 lb. From pointe F and F1 on the cfrc!e, it is seen that the maximum reasonable to assume that the shearing strain
abe&r- st.Tees on 45"-planes is Tua.a = ±a.. and abo th&i the normal stresses -y is proportional to the shearing stress T t hat
•• and _er' • on these planes vanish. This means that the square element produces it. In fact , experiment...s with
ocbd oriented a.t ~5° tG the directions of principal stress is subjected to shear materials in pure shear show this t-0 be t he ca.....<>e, go t ha t
~ only on ,ts edges and is .said to be in a state of pure shear. This
r-.=:-~~l.a.r~te of stress is of special interest and it will now be examined in "( =
T
a'
un.ot:r detail .
Conmder firat, the deformation of the element acbd shown to a larger where the constant of proportionality , denoted by G. I.:! c:illed t.ha ~
of euuticity in shear or simply the ~or maduLw.. Lib the t.."!!l.Sh'll ,)_f rom-
r: n pression modulus E, it bas the dimension of st.ret,S., k&!cm'
In the case of pure shear , there will be no ch-s.~~ in vdumc cl ~ ei~ ~nt
-.r-.......'-+-'-i...,;~ -\.45• F
during deformation . To show this . we must return to ~ l3.5) de1Ul.lng

I
_\I the straintl e. , t ., , e• t- in the case of bia..~i..sl sue..~ Th,m for .,.., = -~. "" r ,
these express.ions become
I
I. ta = E(1 + .u), 1y • - EO + ,ao), 1
• - o.
., ti
Then the unit volutnt> oh&ngt> ~ V, V - es + ._._+ ~. - O.
l (o l ( b) .· t.l001I
~ disec
( C)
It is seen from Fig. 3 . 1 la. that. t.he line~ st.rwo.:1 c.. &nd -. .m
1 I
F 10. 3.11
l

lL_
owaroN AND COMPRESS ION
64 BIAXJAL T ....~.,

of th~ d=s
. and cd of the element acbd must be geometrically tel4
~ n 'Y· Out of this, comes an importa •~~ relations~ d
~ t h e ~ modulus G and the tension mo~ulus E, "'.hich will now ~
P'UR&SBA .Ul

P ■ OBLaM8&1'

J. In Fig. a.lla, a. - -a._ - 2, 100 kg/cm


3A
1 .C&kul&t.ethe abrarini atnin .., for lb,
established. Using the fU'St two of eq~. (3.8) for linear stm111s and referring elcunent acM if E - 21 ( l0) 1 kg /cm' andµ - 0.25. A N . -, - 0J)026.
to Fig. 3. llc, we ~ that z. Calculate t he modulu.e of e.luticity rn ahear for robbu ( E- 21 q ic:n•,_. _ t)
E(I + µ.) J j
and forconcret e(E-143 ( l0l 1 kg/ cm•,µ - 0 .1) .A,u. G, - 1 k« cmJ , G,-65110}• ~-cm.J
oa' = oa(l + ts) = oa [ 1 + s. A short length of thin-we.Jled steel tube with c.l.08ed fflld.s baa mean dlamnu
D _ 5 cm ; w&ll thicluie:!!8 t - 0.06 c?1, &O~ carriea Lntem&l pa p r ~ o( m~ t y

oc' = oc(l + f.) = oc [ 1 - -iJ (I + µ.) J (a) 1' -35 kg/cm' . What externally applied a.rial compre!111 ve loada P moi ~ applied
at the end11 to put the we.ll of the t.ube in a condition of pure aha:., Aiv. p _ 2 060
kg .

Also from the geometry of the right triangle c'oa', we have Riveted and Welded Joints in PreMure Ve8~h
s.5
I I - .!. (I + µ.) In the actual fabricatio n of vari-
r.&n oa'c' = tan (i - I) = :, = I + ~ (1 + µ./ (b) ous kinds of pressure vessels as dis-
cwised in Art. 3.1, it is usually neces-
E
sary to have one or more joints or
since oa = oc in eqs. (a). Finally, from trigonome try, seams as shown in Fig. 3 .13. Such
joints may be either riveted or welded
tan ! - tan 1'. depending upon the material and the
tan (!4 _!) = 4 2 = 1 - "Y /2,
(c) service condition s to which the vessel
'Y12
2 1 +
1 + tan ! tan !
4 2
will be subjected . The general ob-
jective is to obtain a joint that will be
tight and as strong as possible . Fla. 3. 13
~ ~ r / 4 = I and tan -r/ 2 = -r/ 2, the angle -y being very small.
p&.n:ng expressions (b) and (c), we conclude that Riveted joints for boilers and other
pressure vessels are of two kinds: lap Joints and butt J<n. rla as illusaated
'}' T
in Fig . 3 .14a and 3.14b. The rivets in such joints are driven red ho, so tli&t.
2 = E <1 + µ.). (d)
after cooling, they squeeze the plates tightly together. Thus when mtB'rull
it.s value from eq. (3. 7), this becomes pressure is applied to the shell , relative motion between the pb tl!S LS pre--
Substituti ng for Y
vented by friction as well as by the shearing strength oi the ri~ts- In tact.
T T only after the friction is overcome do the rivets begin to work m :!hear. We
'2G = E (I + µ), see that the behavior of a riveted joint under load is extreme!)" eumfl!et To
simplify the problem, it is usual prort ir e to neglect the fr ic-thm 1..-umpiett"ly
from which
, I
G=-~ and to assume that t he ri\'ets carry t hE> los.d in :; hE>tl r with t he 5-h"'aring i L--ess
2(1 + µ. ). (3.9)

i' Thi.a theoretical relationsh.1

II
te~on modulus E .is in good ~ between t~e shear modulus G and the
Talnng, for mild steel, E - 21 ( IOJ~rlcee.rnent with that found by experime nt.
.lrg/cm•. lf Poiaso , • L · g/crn• and - o3
. • ' we find G - 8(10)'
{ 1° o:: ~ol
(3.9) ·ves G :::r o s w1eoret.icat value of th µ.
gi 8.4{1 0) • k«/cm• It ill ...._ e ratio µ. = 0.25 is used eq.
::a
.I
mod uJus of l t" · w •'1l sho · th '
<~
'
f
I
a eircuJar aa(t •c~~ in ahear can be dete · ~n m e ~ext chapter that the
• DU.Cb ~ •
the value of o to lie . --:-i-tuneots with stru
1Vlthin
rmmed ex,.....run
- en
tall b · ·
Y y tW1Btmg
the above wnits. ctural steel specimen s show ( o ) Lop Joint
.F1v. ~. 1-l
j
• '""' coMPRJl88ION
BIAXIAL ftNSION .-~-
JOINT8 IN PREBBUU VMll&LS 67
66 s-section of each rivet as discussed
. .J:-t ·buted over the croe OI)
rivet pitch /I ._ ♦ c m . Then for this length of undu1url>Ni ahe!I the urength u 3,870 "'
uniformly w,, rt .
O 6 )( 4 _ 9,290 kg . Thf' crou- ■ectional area of on., ri ve t u 2.55 c m• , &nd the cone■ -
p. 5. . . ~ there are other w~ys by which a rive~ . din ■ hearing 11reng1h ii 3,090 / 2 .55 = 7,880 kg The proJKted aru, ~hind on.-
Besides sbe&nng of _the _nve can fail after friction has been overcorn
3 14 ~n ,
rive 1 11g0.6 ,.. J ·
8 ,,... 1.08 cm•, and the corrf'■ponding crushing llrength u 6 .680 . I OB =
boiler joint like those in F•~· · ·uustrated in Fig. 3.15. As already d; k . The net r roas-aectional arra of plate ~twttn rivet boln i-< 0.6 ( 4 - 1.8 ~ 1.12
Theae po.wble modes of failure are I 6 710
cm: 1 an dg the corresponding teuing strength o f the Joint i, 3 ,8 70 / 1.32 - 5 , 110 kg.
, •IS the ,malle11 va.lue and therefore repr~enu the Jlrength of the JOtn t ~

Io 1 J [1~• 1J }I §>! 1
'Jlul
corre1pon dUl . g efficiency is 5 • 110 .,. 9,290 - 0.5, or 55 per cent

J The efficiency of the joint can be unpro ved by mcreuing the rivet pitch 10 t.llat
the tearing 11trength com~s up to ~he s~rengtM in shear a.nd cruah.ing. The equ&tioD

~~ l 4
{al ~~:, for determining the optunum pitch 18

( P-1.8) 0.6 X 3.8 70 = 6 . 71('


Flo. 3.15
.from which p = ♦. 7 cm. The 11reogth of thia length of undisturbed shell 3,870 " 0.6 ,. u
♦ .7 _ 10,900 kg . Hence the new efficiency becomes 6,710 -"- 10 ,900 = 0.60 o r 60 percen t.
U8llled the rivet may fail in shear across the section aa, Fig. 3.15a, aJ.
~ough it may undergo considerable bending _before this h~ppens: The Wel.tkd Joinu. With the present day advances in welding techniques,
resistance of the rivet to shear failure can be mcreased by mcreasmg its welded joints for pressure vessels and structural connections are rapidly re-
diameter. A second possible mode of failure is shown in Fig. 3.15b, where placing riveted joints. Typical lap welda and b-utt ~Ida for the loogitudin.&I
crushing of the plate in compression just behind the rivet allows the joint aeam of a pressure vessel are shown in Fig. 3.16. Io the case of a lap weld ,
to open up. Resistance to this type of failure can also be increased by using
a larger diameter for the rivet or by increasing the thickness of the plate.
If the rivets are too closely spaced, the net section of the plate between
rivet holes will be so much reduced that failure of the joint may take place
throat
I o

: : J,
' ' l ~ I ~
due to tearing of the plate in tension between rivets as shown in Fig. 3.15c.
Resistance to su. i failure can be increased by increasing the spacing or pitch t
poftherivetsalongtheseam (see Fig. 3.13).
The problem of de.sign of a riveted boiler joint consists of adjusting the ( o) Lap Weld ( b ) Burt Weld
plate thickness, the rivet diameter, and the rivet pitch so that the joint is
F10. 3.16
equall,v stro_n~ against ea.ch mode of failure . For such design of steel pres-
llW'e veMel Jomts, the ASME Boiler Code recommends the following ulti-
Fig. 3.1&, the thinnest section through the weld bead is called t h e ~
mate stre8lleB to be UBed with a suitable factor of safety, say n = 5.
of the weld. The length l of the bead is called the kngth of the weld. The
TENSION: tT1 = 3,870 kg/cmz, product of the throat dimension and the length of the bead gives the cro:R!-
SHEAR: T' = 3,090 kg/cm•, sectional area of the weld to be used in calculating its strength . In th" CA.Se
CRUSHING; Cle = 6,680 kg/cml. of a butt weld, Fig. 3 .16b, the thickness t of the plate would be taken a.s t,he
ObvioU!l.ly, the strength of a riv ted boil . . throat dimension.
that of the shell itself Th . e er Joint can never be as great as The calculation of the strength of a welded joint 1s largely e~piri_e al
shell proper is called ~he .!~~ 1
~ _of th e s~r~ngth
e.u...,,,r,r«;y of the JOJn t.
of the joint to that of the because of the difficulties involved in making any rauona.l analysu o~-ing
to· t he presence of stress concentrat10ns. · · · usua11 )• assumed
It 1s . . t.bat the
EXAMPLE 1. The cylindrical con•·'- . L

l 0 f thi lcneaa
· c
OGUJer euown · Fig
t • 6 _mni. The rivet.a are 18 mm . 1!1 • 3.13 18. ma.de of steel plate strength of a weld is the cross-sectional area of the throat m~~uplled by_an
arbitrary working stress. The Code for Fusion Walding spec.tiles a wurkin.g
l C&!culate the ult.un.te strength
Boiler Code apecifi~tio118•
IIOLUTION .
o( th
Referring to Fi .. , a 1.. - 1-
... · ._, ict ua co
. . in diameter fWld the pitch p - 4 cm,
e Joint and it.a efficiency, w:iing .\.S.M.E,
·d
na, er a length of joint equal to the
· f h · tded arc welds · [ f the
llt"'88 of I , 120· kg /et1•' in tension or compreSS1on or s ie
weld iii subjected to shear, the working St.re$S i:'I 9:IO ~ /<m'.

't
JOlNTB IN PB.DMro"&E vr.68~ 69
- o·.. AJf1> collPW,a SION
alAlDAL ·.aan..• "
68
A 15 cm X 15 cm x 1.2 crn
EX.AMPLE 2 . guaaet pl.ate is 6 mm truck . II each dia.gon&I ia !rtJeaood ool , 100 kg/nn' ,hmr many
:.!Zmm-dia meter ri v ets will be requi~ to _fa.et.en it to tu ~ t pla.tet
Aa~ that
angJe«-ct.ion teruiion member is welded
the working et.reM in shear for the nvet.e IA,.._ - :YiO kg /cm•. In such cue, bm, ma.ay
1' 12
to • gmeet plate by two eide fillet welds
which act in direct shear u shown in Fig. rivet.a trill be required _to fas~n t.he &UMet p!At..e to the lo,.-wu chord member? A.11,1_
Oi
t 3.17. The total teruiile force in the mem- 3 and 5 rivet.a, re:spect.1vely.
12 - bt>r i8 P • 47,000 kg and each weld bead
o, - f
"• -. llt.f .aid
baa a throat dimensio n t - 9 mm. If
the working etrem for the weld metal in lo g![g oj 'f ~
1--.L, - shear ii! Tw - 950 kg/cm 2 , what lengths [1
1.2
2.5

Fto. 3.1 7
and l, should the two weld beads have?
80LOTION . The line olaction of the ten-
sile force P act.a through the centroid of
the &ngle aection defined by the distance!! a, - 4. 14 cm. and a, - 10.86 rm.
In
p◄ ~ 1.2
:; ~
order to b&.lance the load P , the resultant of the two weld resistances must have F1a. B
F10. A
the same line of act.ion as P . Then if these two forces are denoted by Q 1 and Q,, it
follows &hat the following oonditiom of equilibriu m muat be fulfilled:
6. ~ steel plate IO cm wide I .2 cm thick is oonnect.ed to a gU8llet pl&te by three
fillet welds a.s shown in Fig. C. Calculate t.h.e required bea.d hmgUl l cf the \,ro aide
welds if P - 20,000 -kg and the working stnsaea for t.he weld m.euJ a.re • • . I 120
'
kg/cm• in tenaiona nd Tw = 950 kg/cm' in shtt.r. A. ,u. l - 6-5 cm.
from which, with the given numerica l data, 7. Solve the preceding problem i! only the two sid e welds &re u.,ed, All oCi,er data
remain the same. Ana. l ... 12.4 cm .
QI - 3-4.000 kg, Q, - 13,000 kg.
Since the ahe&ring resist&nce of each bead percm of length is
q .., 0.9 )c 950 ~ 855 kg/cm,

the oorre.poo ding lengths l I and l, should be p l p

l1 _.
34.000 - 39.8 rm.
lt -
15,000
- 15.2 cm.
-r--+
-1
855 855

PROBLEM S.ST 3.1,

1
~iel::;:n x 25mm •eel •trap ii 1pliced u 1hown in Fig. A. The riveu are FIG. C Fto. D
22 mm •.m r. The allowable work · I t reae, are Tw - 560 kg/cm• in ■hear and "•
- I 120 kg/ant . . mr
-.,/___ - ~ temior1.. Calculate the afe load P for the ■pliud 1trap and the I. One chord member of a tru.91! consista o( two 75mm J< SiOmrn:. JOmmang leaec,.
-2.. W.ha'trith eJo~u. A,u. P - 12.750 kg; eff. - 45.5%, tione (see Table B-6) welded to a 12 m m gusset plate as shown in Fig. D. The member
·,I attain maxim=
What:~ ~et.er d shouJ~ ~ Wied for the eplice ahown in Fig. A in order to
o( ~ Jomt? ~ll other data remain the same aa given in
carries a tot.al tension P - 35,000 kg . Caloulate the prope r~ l1 t.nd l-c al .Ure
l
Problem l. weld.a if the working &trees for the weld metal in shear j., T'. • 9..'lO q ~rn~ AM.
I S2.8%. maxunum efficiency? Am. d - 23.6 mm ; max. elf. _ la - 9 cm ; lt - 17 cm .
3, Rderring to the sp!i · F . A
~an• u~.p.""£.a:k~: m!g~
in abear, ~ nsc:1culate the_l:mit load PL if T,-.p. _
0 ivet.8 are 2 ..'icmindia meter
I .400
What
18 the efficiency of the · t · . n,andther
I
. Jom in thia caae1 Ana ·p .5 . 9. The cylinder of a ~ engine~ outilde
I f, A tank like t.hatabown in Fig 3 13 18 .· · £ • :.1 .ooo kg ; eff. - 50%.
diame-tt>r 0.. - 105 c-,u, s.nd in:ud·e <hamete:r
··
~ p joint with two. row1 ol 25 m m - d.1&me ~ e of 12 mm steel plate and has a riveted hffd baa dia.mell~ I>. -
i ter rive ts Th · 11 · the first row of rivets D-a - :t() .-m. •~
12
~ c-m and that Ul the second ro- · . · e pitc Ill The heMI 1~ aU.ehed to the cylinder
j in t.enai .. J8 6 cm U81Jl gwo!k mg
fi .
'
stresses" • = l , 120kg/cm •
21 cm .
. ~n ~ r ..... 84-0 kg/rm'in shear bv a CU"ru.rofeNUtiAl bead W(!ld L'I ibuwu. Whi\t
!lfmmdlamW?r. What is the effi cie' nd the ~f~
I
mternal pressure p if the tank
ij~the sa.fe 1nllx1tntjffl pres!lir'\', p in th-, c,yiinder
,j p - 10.5 kg/cm•· eff.
r-
I
58.3%. . ncy of the Joint? Am.
iI the "'vfk..tng ;.1re.~ for the wdd metal m ~n•
I

.
5• A det.i) sketch of the lower c . ~ ~ ~"7>11 I sloJ1 ill is.. ... t .I ~O kg k m' and t.hio same for ihe
da gonal members are fiO mm x ,..,,mmx.,, hor~ joint o( a tru11 ie shown in Fig • B • The o, . . , ;
,1 mman 1 . 1"10. E c·ytiuder wall 1t.self. AM. p • 28 q/ cm•
g e BeCtJo~ (see Table B-6) and the
- CIRCULAR 3HAJIT

Since c'c is the small arc of radius r subtended by t he angle ctcp, c'c =
71

r~ -
Thus
4 1'
c'c
= ac' =
rdq,
dz · (a)

Torsion For a shaft twisted by a torque at the end, the angle of twi.rt q, is proportion-
al to the distance x of the cross-section from the fixed end and hence d.4,/ dz
is a constant. This constant represents tM angle of twut -per unit length of the
ahaft and will be called 9. Then, from {a.),
'Y = rfJ . (b)
,.1 Tonion of a Circular Shaft The shearing stresses which a.ct on the sides of the element and produce the
Consider a circular shaft built in at the upper end and twisted by a couple above shearing strain have the directions shown. The magnitude of each,
applied to the lower end (Fig. 4.la). It can be shown by measurements at from eq. (3.7), is
T = (},y = GrfJ. (4.1)
the surface that circular sections of the shaft remain circular during twist,
and that their diameters and the distances between them do not change So much for the state of stress on an element at the surface of the shaft.
provided the angle of twist is small. As for the state of stress within the shaft, the assumption will now be
A diec-like element of the shaft, such as that adjacent to the section mn made that not on:y the circular boundaries of the cross-sections of the ehaft
&nd shown as a free body in Fig. 4.lb, will be in the following state of strain. remain undistorted but also that the cross-sections themselves remain plane
There will be a rotation of its bottom cross-section with reference to its top and rotate as if absolutely rigid; that is, every diameter of the crose-section
through an angle dcp. A thin element abed of the surface of the disc whose remains straight and rotates through the same angle. Tests of cireular shafts
sides were vertical before strain takes the form shown in Fig. 4.lb. The show that the theory developed on this assumption is in very good agree-
ment with experimental results. Such being the case, the discussion for the
element abed at the surface of the shaft (Fig. 4.lb) will hold Also for a
similar element within the shaft, whose radius p replaces r (Fig. 4 . lc). The
·thickness of the element in the radial direction is considered as very mnall.
Such elements are then also in pure shear, and the shearing stress on their
aide is
T = Gp8. (4.2)

This states that the shearing stress varies directly as the distance p Crom
1 the axis of the shaft. Fig.. 4.2 pictures this stress distribution in the plane of
I' d1

.!! (o)
J.
( b)

Fro. 4.1
lengths of the sides remain essentiall th
comers change. Thus we rnay c / d e ;me and only the angles at the
I
! pure shear (see Art. 3.4) and the one u_ e t at the element is in a state of T

I
I by the angle cac', is given very c:::;;i;:e of the shearing strain, measured
i' c'c
1 "Y == - .
ac' Fto. 4.3
FIG. 4.2
70

Nth
,.,...---- .

I . ~ . .. .and.
T0.1118101'

abo the compleme ntary


- 01:acvt.AJl aiurr
73
shearing strosees in l\n rucinl pawer of t.he di&melPt . H ~ ahait. ~ of len«tb l , the total &Of9.e o{ t wu~
the ~ Th '°) ma.x imum stt'eS8 occurs in the outer surface of tho
plane {aee p . 30 . e 1rill be
.haft, where..,: P - r . . h.
· . t p••otic flo w 1--mn~ first, mt 18 outer S\I rf a ce . F' Or Q
ma tenai
1
For a d uc1a1e , ._, ""'"'
~-...;., b . h · ___ ,_ _ in shear lonaitudin ally t han transverse ly - for
m&1e11a1 W lC 18 --...c.r. D" . This equation ia usefu l in the experimen tal v~nfia.tw o o( t h e ~ !!Lnd t.,
instance, & wooden shaft wit..h the fibers pa~llel . to the ~is - ~he first
checked by numerot!8 experimen ta wtud1 just.dy th~ ~pL.on s mw lll.
c.racb will be produced by shearing stresses ad.mg m ~he ~xial.~t1 ~ns and
its deri vation . l t should be noted that expen~t a i:n to~o ltt"! eornm,.,rdy
~ will appear on the surface of the shaft ~n the l?ng1tudm~l direction . In
the cue of a mat.eri&l which is weaker m tensmn than m shear - for
wied for de termining the mod ulus of m!\tena l3 rn gh,,..11r U t.h.e :u,~
of h rwt
produced in o. given shaf t by a give n to-rque be meuur'!!'d tbto- ,u'P-'lrt.,Jd~ of
inl.t,ancle, a circular shaft of cast iron or a cylindrica l piece of chalk -
a G can be easily calculated from ef\. (4.4 )
cmck along a helix inclined at 45° to the axis of the shaft of ten occurs
Subetituti ng 8 from eq. (4.3) in eq (-t l ), we obta.tn an eqnat.ioo fo r
(Fig, 4..3). The expla.nation is sim,ple.. The state of p~re ~hear is equiva~en t
calculatin g the maximum shearing stress in t.w ~t c i & ,a/.:,d ro-:ular lhaf:
to one of tension in one direction and equal compressi on m the perpendic u-
lar d'ire:tian (see Art. 3.4}. A rectangula r element cut from the outer Tr 16T
T• n = J = rd'
}a.yer o! a twisted abaft with sides at 4-5° to the a.xis will be subjected to such
~ u shown in Fig. 4..3. The tensile stresses shown produce the helical We see t hat this stress is proportion al to the e.pphed tnf{l~ T acd w-.e.~ !r
erackme:n tioned. proportion al to the cube of the dia~eter oi the sh.a.ft . . .
The relationsh ip bet ween the applied torque T and the stresses which it If we substitute 8 from eq. (4.3) m eq. (4..2), we oh~ m an ~ 3 S l l 0 , to:
produoea will now be found. From the equilibriu m of that portion of the the shearing stress r at any poin t in a aolid circular m3.ft .
lihah between the bottom .a nd the section m.n, it can be concluded that the Tp
sbMring atrbiMll6 distribute d over the cross-sect ion mn are statically T = y · -t.6)
equivalent ~ a couple equal and opposite to the external torque T. For
In practical applicatio ns, t he diameter of the .:ih3lt mu:s: ,,.~mt'..ba hf'
ea.ch ~ t of area dA (Fig. 4. le ). the shearing force is rdA. The mo-
calculated from the horsepowe r which it is req uired to tnusn::..t z C h~ -~
ment of thlfJ force .about the &Xis of the shaft is (rdA)p = G8p2dA, from eq.
horsepower hp, the speed n in rpm, and t-he to rque i in ~~ 01'· ll
(4.2). The total resmti:ng torque T about the axis of the shaft is the summa-·
formula connectin g these quantities is deri ved as follow3.. 3-i.nc-e the ·iro.Tk
tton, take.n over the entire crostH1eCtional a.rea, of these moments of the done by the applied torque T per re volution of we sh.JJ~ rs r :!~ tte ~·r,ts1t
indi vidual elemenu ; that is,
done per minute at n rpm will be T 2rn. Then with t hp= ·.~c ~
T = !A G8p' dA ""' G8 J,. p2 dA = GBJ, (c) per second we have
2~T
h p = - - - --
J = IA p2 dA
or
7, 500 X 60

iai defined & the polar moment of inertia of t he circular cross-sect ion .• For a 7 500 X 60 X hp 71.600 (llpl • I
T = . =- - - ...-,..on. \ t_-;')
circle of dwnet.€1' d, J = rd•/ 32; 80 that 21111 >I

When the horsepowe r and rpm &re gi· ven, .l.. ,.....,nd.tn,,_ lctque f C'i.."l
T=G8~ wt> C\~•-' "' .
32 . .1 d . 1 :i
be computed from t hJs fornnu a au u \t'I\ U.'<" ,•11 6.U..J" . ui t!h' lorei\,\UlS
and equations to compu te tbt> shear :i l res.i or :Lil'g\t.l oi l \~ 1.>t 10 t.he- sh~~'-
,

,-1
e = .:!__ = T.. 32 In this way it ma,y be sho wn t hKt l b\, NqW...~ i \ l1.."\!ll~~ .J vt n 11sui.ru
Gj G ..-d• . . ,
(4 .3) 01rcular shaf t , to transmit hµ ht'~ pG\\l' r t\t rt rt ,ru " lt.h;; nusunum :1uo "" -
Th u.s B, the aJ1glc of twjst per unit of lengt h of t he shaft . d. able working stresd r., in slwM, " ill be>
the applied torque and invtN;P.I ae th od , varies 1rectly a.s
y cm ulus of shear G an d t he fourth 1,_d}
•&e- Apper,dix 15, J>. 3.46. d
l ~
-a '.' 1.t , ,1
~ .
...,,,.
F 74
TORSION

Holl,Qw Shaft. From the pre~ing discussion of ~rsion of a 80lid shaft of


CIRCULA R SHAFI'

SOLUTION . The greater torque is in the left-hand portion of the shaft. It!! value
75

circular cro85-6CCtion, it is seen that only the material at the outer surface Crom eq. (4.7) is
of the shalt can be st.re~ to the limit assigned as an allowabl e working T = 71 ,000 X
50
-== 6,820 kg-cm .
stress. All of the material within the shaft will work at a lower stress and is 525
not being used to full capacity . Thus in those cases where weight reduction The correspon ding maximum shear stress from eq. (4.5) is
is importan t, it is advantag eous to use hollow shafts. In discussin g the
torsion of a hollow shaft, the same assumpti ons will be made as in the case = 16 X
X (4)1
6,820 = 5,• 2.5 kg/ cm ' ·
T ff"

of a solid shaft. The general expression for shearing stress, eq. (4 .2), will
apply. However, in calculati ng the internal resisting torque Tin this case, Similarly, for the right-han d portion of the shaft which transmits 20 hp, T "" 2,730
we sum the moments of the elementa l forces TdA only over the region from kg-cm and T = 217 kg/cm.•
The total angle of twist ia the sum of the angles of twist ~1 a.nd <, 1 in the two
P1 = fd; to Po = ½do as shown in Fig. 4.4. In this way, we again
obtain, for
portions of the shaft. Using eq. (4.4), this becomes
the internal resisting torque,
• 6,820 X 300 + 2,7 30 X 600 Q
1465 rad =
a • "" '
-~ .
T = G8 /,._P' dA = GOJ, tl>-tl> 1 +t/> 2 = l(I0)'x25 .14 l ( IO)'x25. t4"".

where now
The value of J used in the above calculatio n is
(e) 1rd' ,,,.
J = =
32 ( ◄ ) ' = 25.
14cm'
32
is the polar moment of inertia of the hollow cross-section. Thus, with the
proper value of J, the basic equation s (4.2), (4.4), and (4.6) apply also too. EXAMPLE 2. A stepped solid circular shaft is built-in at ita ends a.nd subjected
circular shaft of hollow cross-sect.ion. to an externally applied torque To 11.t the shoulder as shown in Fig. 4.6. Determine
the angle of rotation ¢o of the shoulder section where To is applied .
Taking, for example, the case where d; = ½do, the angle of twist ¢, eq. SOLUTION . This is a statically indetermi nate system because
the slui.ft is built-in
(4.4), and the maximum shear stress Tm.,., eq. (4.6), will be found to be about at both ends. All that we can fi11d from statics is that the sum of the t wo reactive
6 per cent larger than in the case of a solid shaft having the same out.side torques TA and T n at the built-in ends of the shaft must be equal to the applied
diameter -do. But the reduction in weight will be 25 per cent. torque To- Thus

d,
From considera tion of consisten t deformati on, we see that the a.ngle of twi8t
in
each portion of the shaft must be the same, i.e .,
(g)
"'• ='Pb= 'Po-
Uaing eq. (4.4) for angle of twist, expression (g) becomes

50 hp on (h)
T,ta = Tab ""'ti,,.

~ ~~~-
GJA GI.
zo~m•
Thia defines the ratio ootween TA and T n as
TA J ,1b (i)
~ 300-J ,-.l----jG ()()---.. ,j Ts .. Jaa·
F1a. 4.4 ,,,.,-,-- F
IO. 4.5
/
o--j
sh✓. Fi
l, :f -
0
.
l. The solid line shaft in 4 .

~1A:
EXAMPLE
d = 40 mrn and runs at 525 rpm It . g . . ·5 111 ~de of steel. has diameter
of ~he shaft will be negligi ble. Ad~~,f~P~~~1 ~ bearmgs so placed that bending JA J.
while 30 hp and 20 hp respec ti\-cl g , ee s 50 hp to the left hand pulley
and right hand pulleys'. Compute fh
are ta_ke n off by belts O\'errunni ng the middle
st ress T induced in the shaft
d, 0 01 ,

and the tot.al angle of twist ¢ <\&u e m&GJOmum shear


· · me = 1( 10)1 kg/nn' . F10. 4.6

l'
76
TORSION
- Cl'.RCULAR 8HArr' 77
From tJii9 expreuion, we eee that in the particular C&t1C where J A • _J s, i.e., ~ hu dialDl!tu d. tLt one end
for a sh&ft of uniform c~tion, t he reacti,·e t-0rques T,-. and Ts &re 111mply in 9 • A aligbtly tapered eh.alt. "'lt.b circular e r
and diameter d./2 at. t.he ot.ber; the length of the tdw\ ia l. Det.mn:ine w t.ota.l
the mvnne ratio or the lengths a and b. F rom eq8 . (f) and (i), we obtam . for the more angle of tv,i.st ♦ between the t wo imdll of lhe s_ha! t if it UJ subJect.ed to omionn
general cue, 1arque T . Ana. ♦"" 14Tl/ '30J., where/. • .-,1.•/ 32.
To A t;:i the g!'.al
T - Te ' Ts = - - - (j ) 10. For the 11ystem in Fig. C, power ill tf'I.Mlllrtt.ed from the ~
A l + Jsa 1 + J_..b D. Uthe pitch diameters or the ~ rs Band C are in the ra.uo l ;z -what ia the prope-
J .. b Jsa ratio of shaft diameters d,:dt for both shalt.a to have the aame muimum sbnz stns
,.T .Ana. d 1 :dt - 0.794.
Uaing either of ~ ,,&Jues in eqs. (h ), we have for the angle of rotation tl>o of the
junction

t/'i'. A that l!IUch


abaft 6 mm in diameter
sn,el
(J ,.b
Tr,ah
(/)o = - - - - -·

PB.OBLEK8ET
+
J sa )G

4.1
at 10,000 rpm . What is the maximum
twn8
(le)

.,-r cl~f' =lo


WWW
~
'i
...,,
~v
1/'
0

PCJftr a abaft may develop if the 888igned working stress in 11hear


■_,,.. -!50 l(g/cm 1 ? .Atu.
2.07 hp.
/ J. ~ the .proper ~eter d for a solid steel shaft to transmit 200 hp at Flo. C F1 a_ D
106 rpm if the working 8tre1!s in ehe&r is 420 ltg/cm 1 .A n.,, d - 11.8 cm.
S. ~ the ~roper ~ t e r d ~or a l!Olid steel shaft to transmit 300 hp at ,..rC A priamatie shaft of diameter d has built-in ends and is subjected t.o the achco
3600 rpm if the working stre!s in shear 18'420 Jt·g /cm 1 • A,u. d = 4· 16 cm.
-'· A hollow st.eel sh&ft is to have outside diameter d and inside diameter d/2.
Mexternally applied twisting momen~ T1 and. T: as shown in Ftg. D. Fmd the
~ t e lh:e proper value of d for the 11haft if it is to transmit 200 hp at 105 rpm
internal torques T., T., T., in the three portions a, b, c, of t he i!ufi. The i~
,nth a working stress in shear of.t20 kg/cm 1 .Am. d -12.1 cm. numerical data are given : a =- 75 cm , b -125 cm , e. ,.. JOO=· 1\ = l'lCXlO q-<m ,
?' and T 1 - 24-,000.kg-cm . Ana. T. = 17,000 kg..cm ; T. ,.. 5000 q-=. T• .., lQ,tD'.>
_5. A ~ r ~ t diameter d -=15 cm is 11pliced as ehown in Fig. A. The two
kg-cm.
Plm are~ 2..:;cm 1D duuneter and the working stress in shear is 420 kg /cm'. What is
12. A solid brass rod of diameter d ,.. 63 mm has a steel ~ve with wall thidc!llS
the ~ u m sale ~o~wer that the shaft can transmit at 120 rpm? What is the
efficimcy of the spliced Jomt? A,u. 52 hp; eff. = 11.1%. e -6 mm solidly fused onto it for reinforcement . What is th~ sa!e ~ for the
compound ehaft if G, =-84 (10)'1.:g/cm•, G.= 35( 10)• kg/cmJ. (,.. ), ~~ 1S·cm, {- . ).-
1.30 525 kg/cm•. What is the ratio of this torque to t.ha, which lhe h..TS...• rod.alone cou!d

I~
., r &&fely carry? Ana. T - 49,350-kg-cm; ratio =- 1.91.
' '
.J L
0
6 / A C B
d
/
4.2 Close-Coiled HelicaJ Spring
Frn . A
An interesting application of the t heory of tonuun l\rt.."IFS m t ....K .:A..--e d a
~ FIG. 0 close-coiled helical spring.• Assume tll!lt suc h a spring. irnund fr<'m ~ w·ire
ea.rn:
~
:e
~ ~ s h a f t of diameter d •12mm is built in at
700 ~ /~1,.,.,~ is show~ in Flg. B . If the wor~ str~
it8
in
ends ,4 and Band
shear
is ,;.,;di t 1.~mum u!e angle of rotation that can be given to the disk
for the shaft ia
of solid circular cross-section on a circular core, tS subj~ted k' · ht' .1.c tit" D oi
axial forces P (Fig. 4 .7a), and tha t. a n~• o ne coil u,-s ne.1rl :w in 3 pbn'"' pt-r-
tA1. 1 .... fat&cw:utothe sh&ft? Ana. 1°35, pendicular to the axis of the helix . Con:,idNing t hr tX1ttilibnum (.>t dw uw r
For the at.epped shaft in Fi 4 6 the f 11 ·. portion of the s pring bounded by a n a..x in l ,-ec tion :.u,·h s..s '!I '! , Fig -l :'h1. it
a •20 cm, b .., 40 cm d -=- 2.5 dg. .3 ' o owmg numerical data are given .
.. _, ' l cm. , • cm The aJJo bl k' . ce.n be concluded from equatin ns of $!.a t ic.:, tba.r. the str&sS rt:tiulunt rm the
f or the 11uait, it ,.._ - 56() ltg /cm-• CaJcula · h wa e wor tng stress in ehear
T. - 3,378 kg~. · re t e eafe value of the torque To, Ana. cross-section mn of the coil reduce~ to !\ ~ ear1ug ion--u P thn•ugh t.he
center of the cross-section and a co upltl f\ tt 1n g w 8 l-Ountcrch>el.." ~ dt~ ~
8. A hollow •ted tube (outside dia-m
~ a torque dyn.sunometer. It ia deeired e:• . .
d. ~d inside diameter d,) ia to be uaed
of length per 100.11,~ m of torque witho t attain .an angle of twist of 10 pt;r 30 cm
tion in the plane of the cross-€ectioo of magn itude JJR, "·hcf\.' R is Llw mdht.s
of the cylindrical surface containiug tbt> ~n ter hoe of th.c spru\g. The
= 420 kg/cm•. What AN the req uired u I " ~ g an allowab. lA! shear Rtress 1'•
J .606 cm. va ues o d. and d, ? A n11. d• .. 1.7 I 9 cm; •For a complet~ treatiae on 11pt inge, 8i'C.' .\ f«/w,,u""1 Sr,,>¥ b~ ,\ M. Wahl, ~ nton
Puhliabin1 Co., Cleveland, 19-a-i.
I
I t

1
iB
TORSION HELICAL SPRING 79

couple PR twists the coil and causes a maximum shearing stre~ given by There is another reason for expecting higher
eq. (4..5), which becomes here stresses at the inner side of the coil. In calculatin g
, I6PR th~ stresses due to twist, we use eq . (a) , which was
r = fli' ' (a)
derived for cylindrica l bars. In reality each element
of the spring will be in t he condition shown in Fig.
where dis the diameter of the cross-secti on mn of the wire. Upon this stre&,
4.8. It is seen t hat if the cross-sect io n bf rotates
due to twist, that due to the shearing force P is superpose d. For a rough with respect to ac , due to twist , t he displacem en t of
F ro. 4.8

p Plan View the point b with respect to a will be t he same as t hat


of the point f with respect to c. Due t o the fac t t hat t he d istance ab is
smaller than the distance cf, the shearing strain at the inner side ab w ill be
larger than that at the outer side cf, and t herefo re t he shearing stresses
produced by the couple PR will be larger at b t han a t f. Taking this into
considera tion, together with the effect of the shearing force, we replace eq.
(4.8) by the following equation• for calculat ing the maximum shearing
stress :
= I6PR (4m - 1 .615) , + (4.9)
( C) Tmu rd' 4m - 4 m

in which m = 2R/d. It can be seen that the cOTTedi on factor in the paren-
theses increases with a decrease of m; for instance, in case m = 4 thls
factor is about 1.40 and form = IO it is equal to 1.14.
In calculatin g the deflection of the spring, usually only the effect of the
twist of the coils is taken into considera tion. For the angle of twist of one
FIG. 4.7 element between the two adjacent cross-sect ions mn and m'n' (Fig. 4.7c),
using eq. (4.4), in which Rda replaces l, we obtain
• t 'b ted
approxima tion, this shearing force is assumed to be uniformly d 18 n u P - R·Rda.
'
over the cross-sec t ion; h
t e correspon ding shearing stress will be dq, = JG .

,, 4P Due to this twist, the lower portion of the spring rotates with respect to the
T =-• (b)
rd' point O (Fig. 4.7a), and the point of applicatio n B of the force P describes
' t' , ,, the small arc BB' = a dq, . This movemen t of B is easily pictured by
At the point m the d irec .
wns of r and r coincide so that th e maximum
. . e imagining all of the spring as rigid except the element between mn and
s heanng stress occurs here a n d h as th e magmtud
m'11.' . The vertical componen t of this displacem ent is

Tmar-= r' + r" = I6PR


rd'
(i + 4Rd ) · (4.8) do= B'B" = BB'
R PWda
a= Ra..= 7 o·
(c)

It can be seen that the second te . h The complete deflection of the spring is obtained by summatio n of the _de-
t
effect of the shearing force • rm mt e_ par:en beses, which represent s the ftections B' B" due to ea.eh element mnm'n', over t.he length of the spnng.
practical importanc e in hea~;n;:;;:;::i5wi_th the ratio d/ R. It becomes of
Then
eani. Due to this term, points such sprmgs, s~ch as are u.sed on railway (~" PR' 64nPR' (4.10)
less favorable condition than po· tsas m on the mner side. of a coil are in a. o = j JG da = d'(J '
m such as n f t
shows t;
stresses , a d • .
t n rk act in opposite directions E ' or_a po~t n the shearing
a crac s usually start on th . . ~per1ence with heavy springs
•F
O

...I Wahl "Btreuee in Heavy Cloecly Coiled


or the derivation of thil formula, ■ee A. "' ·
.E., 1929, Vol.

51, Pa.per No. A
PM 51
- -
e inner &de of the coil. Helical Sprinp, "Trana. A.S.M
.
r
TORSIO N HELICA L SPRI NG
80 81
cemen t in maximu m sh~a r stresses in the two sprin~ are found to be
== •
in which n denote s the numbe r of coi'ls. The net horizo ntal displa
T 1
1
225 ~ fem and
r, .-406 kg ,'cm 1•

one complete turn is JSero. d by k. 2. A conic11.l spring M shown in Fig. 4. 10 is subjected t


.
The ratio P / 6 for a given spring is called the 8'pring conatant, denote
EXAMPL E: 0
load P. Each coil is a.'lsumed to lie e8Sentially in a horizon tal I.an'-0"
~ ~ 1'/t>
p t> an shape
of the spiral in pla n view is defin ed by thP, eq uation
£
Thus, from eq. (4.10), we have
p Gd•
(4.II) R = Ri + (Ri - R 1)a ,
k- j == 64R'n. 2:ni (d)
measure1 u
nts are where R is _the ru~ius a.t any point A on the spiral flild a i.s the angle
Two springs are said to be of the same stiffne ss if their spring consta shown. It 1s reqwre d ~ -develop a form ula for the spring eonst&n t k .
materi al, the e.t A of length ~ wiU be
equal. The spring consta nt k can be varied by changi ng the SOLUTIO N . The cond1t1ons _of a~ elemen t of the spring
the same as for the elemen t m F ig. 4.7c. Hence, using eq. (c),
wire diamet er, the core radius, or the numbe r of coils.
from the same wire bul with { -rr" PR 1
.ll.AJOL I: I. Two c.loae-ooiled helical springs wound c5 = }o JG da . (~)
different oore radii are aaaemb led as shown in Fig. 4.9 and compre88ed between
induced in each
rigid plates at their ends. Ciilculat.e the maxim um shear stress SubBtit uting now the value of R from eq. (d), we obtain
P - 50 kg. The core radii are as shown
apring if the wire diamet er d •12 mm and
in the figure. 32P {JT"[ R1 + (Rz 2m
- R 1)a] J 16Pn
da = Gd4 (R 12 + R:$) (R1 + R,),
o= -,,d4G }o
p
where J has been replaced by 1rd4/ 32. Thl18 the spring constan, is
k=~= Gd4 (0
c5 l6n(R 12 + Rt 1)(R 1 + R r)
PROBL EMSET 4.2
ameter wire,
I. Two clo~o iled helical springs are made from the same &n&ll-di
-diareet et core. II esch
one wound on 2.5 cm-dia meter core and the other on 1.25-ou
spring has n coils, find the ratio of their spring constan ts. A,u. i.1 / k-r = i-
close-co iled helical spring, the followin g nu merical dai.a a.re gi ven:
2. For a
d G 10)' k~ ,' cm'. If the allowab le working
; R =2 .5 cm,d =3 rnm, n =30 coils,an =84(
/cm 1 , what is the safe load P for t he spring~ 'W'hst is the
stress in shear is 1,400kg
1
corresp onding elongat ion c5? Ana. P = 2.88 kg; ~ = 12.7 cm.
&Dd. that
1 3. Assum ing that the two springs shown in Fig. 4.9 are made oi suid
. .J ,u. A: =- 9J.8
each one has 8 coils, calcula te the spring constan t k for the ~ mbly
j kg/cm.
4. A variabl e-radiu s coil spring is similar to t.hAi sho1n1 _in Fig. 4.10
excepi ~ l
by Ule equation
l Flo. 4.9 Fm. 4.10
its plan view represe nts a spiral de.fined in polar coordina t.ee
R = Ried. Numeri cal data are given as
! !OLUTIO N. Since the two 8 . uat ha
. . . ve the u.me over-all shorten ing &, it
follows: R1 = 2.5cm, a "" 0.01, d "" 0.25
cm, 0 -84(10) •kg/cm1 , and n ""' 10 coils.
follows frQm eq. (4 IO) that pnnp m 19 ratio
of the cubes of the~ 00 ~he load p ~vided between them~
the inverae Calcula te the spring constan t k. 11na.
.11pring and by p 1 that ~a-:_.J· bThushe ~en?t.mg ~y P1 the load earned by the outside k - I .12 kg/cm . , , 3::
...... ,.-.a Y t LDB1de spring, we have 5. The base of an electric motor is ~
moun ~ on four heavy- 0uty coil • , ? --. -•~
Pa R,• 422
springs . For each spring, R -7 .5 cm, f- - 50 _ ...,....,__-
P, - R 1• - 1000· d • 2.5 cni, G-84 (10)' kg/c01 and n - 6
1 FI "
Also
II . coils. If the motor weigh., 500 kg o. ·
·mum shear
Pi+ P 1 • p - 50 kg. what is its maximu m ,illowab le
amplitu de of vertical vibn.t.io n withous danger of ~ceedln g " ~
From theee two equationa we obt.am p 1
1
with
t.beee valuea, togetbP.r the other gi v: i:...a:,.
&nd P' - 35.2 kg. Substit uting
'mto eq. (4.9), the COl'l'e8J)Onding
•treeaof 5251ig/cm• in the sprinpT A.u . I - 0.23:Z c;1u.
-,
! .
TOMION
82 8TIU 1N EN lraOY 83
6. Using formula (4.9), find the safe load P for the ~i':'-1 spring in Fig. 4.lQ i(
the working strttiS in shear is r,. ...s ,:200kg/cm•. The spnng ts wound from n 2.5cni_ The first of these equation, exprP.35el!I t be st ra.in energy as a function oi the
diameter !t.eel rod -.;th R - 5 cm and R 1 -20 cm. An&. P., - 450 kg. shear stress -r ; t he second, ll.8 a f u n d i.()n of t he shl!ar st.rai n 7 _ The limit oi
1. A rigid bar AB weighing 10 lc.f{ &nd <'.&rrying a load P - 40 k~ is supported by elastic strain energy per un i t, vol 11 me ,,nil be obtained by 5cl'iog ~ 10 t.he
three ~rings havi ng spring const.Mts ki - 20 kg 'cm, k1 "" 12 kg /c m, k, =. 8 kg /c n1 first equation t>qunl to t he elastic h m it in shP.ar
(Fig. A) . II the unloaded springs were all of the Mme length , find the distan ce z Having expression s (4.12) for st rain energy CJf shear per unit volinne, Ule
mch that AB will be horisont&I . Am. z - 18. 75 cm .
total strnin energy in a 11olid circu la r 5ha.ft of radma r a nd 1=ngtb ! ;subjf!Cted
to twisting mom e n ts Tat its end s ( Fig. 4 .121 can easily be found . Denot.mg
4.3 Strain Energy in Shear and Torsion by -rmu: the maximu m Rhear stress a t the sun.ace of t.be ah.af-:.. t.h.e shear
CollSider, in Fig. 4.11 , an element of elastic material in a state of pure stress a t any intermedi ate rad ius p wit! be -rr:a.a fp / T) Then . from the fi:r!r, of
she&r. The strain energy stored in such an element may be calculated in eqs. (4 . 12), the strain energy per unit volume at the rndi:w! ~ wt1l be
the same was s.s for the case of simple tension (see Art. 2.4). During de- (b)
form&tion of the element, the top face cd moves horizonta lly through the
disiance -ydy relative to the bottom face ab as the shear stress is gradually The energy in t he elemen t.al tube of length l, radius "· imd t.b : e ~ do
will be
dU = u dV = •;{: - l · '2 '11'pdp. (c)

Summatio n of expression (c), from p = 0 top = r , gives for the k>w strain
ene1·gy in the twisted shaft,
--lrd.Y t-- I

L_/c 1 - ~

1
I
r T ! p"-
-rdldr I
U = ~ · l · ?- r-,Jp = ~- \rrl1' ~'::£ - {4.13)

/1
I
'2G rt /JU

if.;-
I o

'
I
I This is seen to be ju~t half the value that wo uld be obmw.ed ii ~n the

1
I
I
material were stressed Lo th e m a."Ci.Inum val ue • ~ · );oung W t r .,._ =
'' Tr/J , where J = 1rT4 / 2, expre."'Sion (4_13) can be w-rinen al.,;o m roe form

0
I

I
r.:· _-_iI' 1"£.
l ., -_ '2GJ
(·U3'1.

t ,-dx=--1 r
I F IG. 4.11

increaa:i ng from .zero to the final I


Frn. 4.12
This strain energy U fo r a shs..ft in to~on
may be obtained in imother ws.y by using a~-
pression (4.4) for the angle of twist in t he :::haft. "
This shows that the relatior. betwet-n torque T
..1
j
I
force Tdx dz. on the top face . : ~ T . Then _the work done by the shear
eide5 ac and bd do no work, t~~ r · -ydy. Smee the ~hear forces on the
and angle of tw is t, q, is linear within tht> ela~ti~
limit of the mat e ritll as sho wn b•, the t.()r-qui'--
the element. Dividin b the p esents th e total stram energy stored in For
uny :.< tn:ill
. g Y volume dz dy dz of the element, we obtain twist diagram 0.4. B in Fig . -l . l J .
increment dq, of t.bP unf!:l<' of twist, th\' \\ v.rk.
u = I: ~) done by the a cti ng t.orque is repre..~'ntt 'd b y t ht'
2
for the strain energy r ...... ·t area of t he shaded strip in th i~ Ji~ra.m . Thu~
N0 t · h -• uru volume a& the torque is gradually im' n '!i.st-d ftllfU :g~•ro f1a_ 4. tJ
~ IDg t at, within the e la,gf,·1 li • ·
(a ) ma.y be • •~ c nut of the mate · I
= T /G, expreBBion to anv finaJ vah11.i T tht' total wo.rk , t•qLW to tht'
written in either of th.e f Jl . MA , -Y
o owing two forms: enellU' stored , jg ro~refft1J1 t.oo by t.bt' •rt>•
0 ~1B in F1..g
Ll3, 1.e.,

u = ..!!. or u - - G-yl r. (d)


'JI)
2
.
(4.12) u :;%
..
.,·

'
'
:. I
85
roMt Ol'f
8-4 't'l:wo tm:m u;a ea.Ju (0
,-.d;o. R ,rarw with a, a.coordm ~ t.o l"C\ (ti> on p,. 81.
where • _ r v 0..1, from ~ - (•~•>-
With tlus rela tio~ ip bet ,r.•ee n T &nd "• abon , , ~ -.o«lc! of ,,,,lat l,)ef on1t ~ f'lf ~nu lit ,( i,t

~ {d) nay be writt-en in eitht-r of t be followrng two forms :


11 ts. PR
. 'N GJfJ ( dz .. OJ
l - @ or
T1
1.. = 2.1· 4. 14 )
t.be fm::a.t ol
5ub8\i tu t.ing t.bJ.8 into Pq. (4.16) , ~ h ..,. R tJa -.cad
:ha~
Tb, finrt .of ~ cqu.atioM exp ~ the B:tnuo
e nergy in terms of the i:ot,ep tion accord ingly. we obt&m
t,eyrqm- T : t h e ~- m terms oJ the"&ngle of t1'-i ♦- 4 Expre
st ssione (4 . 14 ) U_GJ {,,,,.(PR )• HJ,r,
may be a.~ ~ i« a !IO'l id cin:-ular 8haft with J - rd / 32 or for a hoU 0 .., 2 J. f'}J
o.rcuJs.r shalt ~.th J = -r (de - d: )/ 32.
4

U the in ~ernal :Ql"Qut> ,:-a.nes a.long die length of a shalt as in


t he caae of a Equati ng th~ to t.b.e work P &fl of~ load P •~ oc:rtam
di8C of l'!ngt.h dx &nd
ooruca l ~ ~ - 4 IO), we c:-.0116ider one eleme ntal [ ~• Nl•
undn t~~ IaOme nt T,.. Thtm the angle of twist in th.is eleme ntal disc i_.13
&- I, a:r»·
4
di; - (Udz:, (e) ,...hicb agrees with eq . (e) , p. ~I.
i!XAMP ~ 3. A 90Jjd steel 1ba.f t "'"1th a li;-whtt
.l &'- a:.t: Qd :"01ak;I , ..i, "XliM!b!!\
f~ • ~ eax:::rw m
,peed n - 120 rpm , Fig. 4. H . lf t.he bearing ,t ~ ' . " d!~~ ~
(f) produc ed in the ah.ait d~ co ~
ahear at.retie r_,. will be
&ad . t a ~ ol t:,":'S'bo o
l -1.5 m.d - 5 cm, the weight o! the flywht ti !V • 50 ~ - .

;~ ~ o!dcwi:st per uni t thfolrness of t he elernen tal d isc . Subst itution


·
i - 25 cm.
aot.OTI0N. The mu:im um shear atn!a m the- aha.ft will
~..L"' • :.;a t.;:g ',.oaJ
«4' dz for I HI the second 0 ( eqs. (4 · J4 ), gives,
or ~t...~r..ndis(', for the strain at.orb ed by the ab&Il. Th,:s ~ - •
~ in kinetic energy of the flywbeeJ h.aa been

dU = ~(::ydx. Wi\.i 1
~ -
50 " 23' x .f-r l '
2 X 98()
> "-1 - \ . -
_. - ,.J :, "'!I~ -

T'ber, ._ .summ ation over the full len&th l of h .


litRstll energy t e shaft gives, for the tot.al

u .. GJ (
2 J.
1
(4)'
dx dz . (4.15) A ~6=--- -r--- ---~ .._,. ....~
~
n....tl' U 1 ~ the . ' 1~-- ----- ~-- --- ~
P-1g. it-7a., ½ ~~
erpre•i~ ~ for the cl~il t'd helical apring in Flo. 4.1 4
Sc.t_L{."':'10.W_ T~ tr.un« momen t ~n., enev of to1'8j()n _
m Fl(,. O
· r ~ PR H~ b th ~and every elemen t of the ooil like that ~ g11>~n ~-"' ~ ,-U3 ~ •.ik ~ fo!-
~ o Setting this equal to t he str&in [
-'-Al . energy t' ll'-"J){.v
. • y , e nst of t-qs. (4.14) - • straw
the to ..wu
m tA::' ~ of J.€" .. s- ., R u ,
I
·I
J ~r ,i u; T.,aa , W'P. find
1 ' t6'.
U = (PR)I 2.-k n FiG'f: {~_" ln I .. .,. ' - I.ti ~ rm '

'KJJ
T" m at - -- • '
'J~ it ' ~-!>l • -.., t \iJ
t '- .
'-"{!Ul.l•"' • , 1..4=1
.
lo !,~ 11i'W;; ~
Pt l ? vf lnP
. -~ ...- .. load p d • obtain
- urwg deflection, we

'Zrrir
~ .... -
kI "'~
U"lt.~R • 1. Two ttl~ft:; of S<)lt<l eir,·uL"\ r l'"'-~-~ ~t.•u Mt- kt.-oti.
.~ r i, rpt !or cilit'll' ~ -
. (JJ - ;;,- - ~ , r ,..,t,~t ,,.·JI t>..:- tfu, n1.t1<." u1 the ,unuunt.s...J
e teni d , a nd d 1• l iu~for ihtt ~me t~)J qu,·
'IJ;h,ah ~ ~ rth r 8\rlllin ent!rgy st ol'\ld I n uad~ ~(t~ t"" { , l ,
... ,J, ,t:•
l lAM J"U'. 2 \'cri:P ~i!li:, {4 J{t) (I{) p. ']{.,
2. A &0lid ,• in·u~r l!lh.ft. and" th_m--..-.U..'-1 '11-K'. Ul:r..r tub,il ~~,
.:li th~ ame- matcru d
spr l.t}g Ul&d ed u ..ti 'Y ii,rt'iifl.,J f.1 (., J oo )
l>wn in f'1g. '4 W I I . /SJ' ti,r
.
the dd ttef . •nd ha vlni tho sumt- Wt>16h l t' ~ ~u-.-.--J m c~•o. •..i1 to ~ ~ Utt' m~.,ttnum ish.e5!
60Ltrr
,~ . At &ny f)t) mt A C4l , 9' i~ i ,~ , (U o; f to~ d of the con iC&J
li't reti.11 r . Wl1a~ l:I tht• r•llV vl uw atnoun u vi i rnrn 1 .-tll.' l"K.V ~--•reu
rn th.t> 1w1.>ah.f bt .
Ulrcu.,jj (Y~ 4, Ju; thli vr str1un t•llf! ll(y of twist.
' t(m~ u~ 1',, • Pll, where t ho 1t llf. ( iJ V , ...
1 t-
;

5
r
86 TIUN-WALLED TUB~
87
s. For hard drawn aluminum &nd for coJd rolled steel, th e phroportionaJ limit.a in
· el. , •and:i 5W 1tg/cm•.~'hat are t e amounts of st.rain where r is the radi us of t he mean center line
ena-gy per tg
_!~..
ahea:r are, ra,pecthstiv 700
~~1 ..... ~b-,orb in pure shear witbout exceeding thn
t ea<;.u ma.,.,,,zu c..... f h
and tis the wall t hickness . T hen assuming for
· te . I
proportional limit stress'.' See Table A.I for the densities o t e ma na s. Ans." such a thin-walled tu be that t he shear stress T
"- - 327 l.g-c..-n,'kg; u, - 475 k.g-cm/ ltg. . . . is uniform acro8s the wall and equal to the
,. Two steel shafts eaoh of Jength z and out.side d111mc_ter dare s~hJ~C~ to_uni.
value at th e mean radius r aa shown in Fig.
form torsion. The ~t shAit is l!Olid while the second one 1s hollow with ms1de_d1arn-
eter d/ 2. What is the ratio of the strain energies that they ~n :bsorb without 4. J5b, we obta in from eq . (4.6)
1
exceeding a maximum allowable shear stress ..,..? Ans. U,/U, - , 5"·
5. Referring to Fig. 4.9 and using the T
. ,. = 2,rr2t ' (b) {o l
numerical data given there in Example I
calculate the total strain energy in th~
two springs when P = 75 kg. Assume Likewise, from eq . (4.4), the a ngle of twist of
that each spring has 8 coils. Am. 30.7 the tube becomes
kg-cm.
6. If t.he shaft in Fig. 4.14 is hollow Tl Tl ( r, )
with d. -5 cm and d,= 2.5cm,what max- 4> = GJ = 2-rrtG.
imum shear stress will be produced by the
Fro. A sudden freesing of the bearing A? Am. In Art. 3.4, it was shown that the state of T (0 i (C l
Tmas - 1,750 lrg/cm 1 • pure shear such as exists on the element A in
7. A flywheel of weight W - 18 kg and radius of gyration i - 25 cm is mounted Fig. 4 .15a is equivalent to biaxial tension and F m . 4. !.1
at the r_niddJe of a 90lid steel shaft of diameter d - 5 cm and length l. The shaft
rota~ m bearings A and Bat its ends with an angular speed n =- 120 rpm. U both compression on an element oriented at 45 ° to
been.op suddenly freese so that the ends of the shalt become locked the shaft will the axis of the tube, like the element Bin Fig. 4.15a . From this, we see tha t
have t.o ah!orb t~ kin~ic energy of the flywheel. Calculate the shortest length l of a long narrow strip of the wall coinciding wi t h t he 45°-heli.~ s ho wn in ~ ig
the shaft for which thui can be done without exceeding a maximum shear streae 4. 15a is subjected to axial compression, and if t he wall of t he tube is very
-r. -840 kg/cm• in the shaft. Ana. l.J.D = 2.2 m. thin, such a helical strip may buckle . This phenomenon can be de m0n-
~ ~ BOli_d ateel shaft of diameter d = 6 mm fits loosely in.side a hollow steel shaft
strated by rolling a sheet of paper in to a tu be and then subjei:t.ing it to
of ~de diameter d - 6 mm and out.aide diameter d1 - 9 mm as shown in Fig. A.
A pm AA prevents relative rot.ation between the ends of the shafts at the left. Pin- torsion . Analysis of this problem• s hows t hat fo r a long 5teel tubE.> under
holes at the right a.re initially at right angJee t.o each other as shown. The two torsion, we should have th e ratio l/ r > %0 to avoid the danger of buc kling
~afts ~ now ~wisted in opposite directions until the pinholes at B line up and a at normal working stresses .
pm BB ia th.en 1IU1erted. How much strain energy will be locked in the system if Introducing, in eqs. (b) and (cJ, the notat ions
l - 250 cm? Ana. U - 42·2 kg-cm.
Ao = 7rT 2 = area enclosed by mean center line .
4.4 Touh>tr of Thin-Walled Tubes
s = 2rr = length of mean center line ,
. In the ~ of a hollow shaft of circular cro&HeCtion for which the inside we obtain
diame~r 18 very nearly equal to the outside diamet.er, we speak of the shaft T = _T_ , (d )
u a th • ~J~- Fpp-such a tube in torsion (Fig. 4.15), the polar 2Aot ...
moment of _we.ma of ~ Cl'088-8eetion cannot be calculated with ~ and
accuracy from
l
tire Jormula
.
avvu rsl (e)
4>- - - ·
- 2A oG
J =;,i i (d.• - d,•);
The above formulas can be used fo r cti.kulat ing the tiht':ir st ress <rn~ augl~
and it is p.referab~ to U8e the appro:J.mat.e expffilllion of twis t in a thin-wa lled t.uhe of a rbitrary c ros:;-~ction as shown m Fig.
..See L. H . Donn ell , Staln'lity of T li in- U!allcd T 1d>e~ w1dtr Tors,011 , Nat Adv, Comm .
J =J,,.{ p'd,d - ,. { d.A
).6. = 2ff'i• (a)
Aeron11utic11, T ec h . R ept. 479, I9:l3.
88
TOBSJON

ti
4.16. When euch a thin tube of noncircular cross-eee on ia twisted, the
- THIN-WA LL:ED TUBES

ThU8 the integral of this quan tity over the full length a represents double
89

. •Ate _1: btly one with re8pect to another , but they do not the area enclosed by the mean center line of the wall. Denoting thia area
croe-1eet10ns ro- Ill.lg , 8
light' ..
..., warped , and
remain plane. After twist, each croee-eection becomes by Ao, we have
T = Tt · 2Ao,
from which
T
r=-• (4.16)
2Aot
which coincides with eq. (d) above .
To calculat e the angle of twist 4> of the tube, we. use t he method of strain
energy as discussed in Art. 4.3 . From the first of eqs. (4.12) the total strain
energy in the tube is
r
U = ZG X slt,
J (b)
where slt is the volume of materia l in the t ube and r is the shear st ress,
r
uniform through out the volume. Equatin g this strain energy t-0 the- work
Tq,/2 of the applied torque during twist, we obtain
Tr/> rslt 1

2= 2G
from which
r
r/> = TGsU.
Fla. 4.16 Since T = 2AotT from eq. (4.16), this becomes
q,= ~ . (4.17)
if no restrain t against this warping exists at the ends of the tube it takes 2AoG
place in 8Uch a way that the shear strain 'Y of every element o/ the wall
which agrees with eq. (e) above .
BUch as A has the same magnitu de regardle ss of the radial distance p of that
In practica l problem s we often encount er thin-walled t ubular members
I, I
elemen~ from the centroid of the crOBS-section. Thus the shear stress T,
proportional to the shear strain 'Y, is unifonn through out the wall of the
of other than circular cross-section and eqs. (4. 16) and (4 17) are very
useful in the analysis of their behavio r under torsion . The fo llowin~
I
tube.
examples will serve to illustrat e several such problem s.
To rela_te this shear stress T to the external torque T, we conside r any the ssme material have
EXAMPLE 1. Two thin-walled tubular members made of
Cl'088-1leetion ?f the tube as shown in Fig. 4.16b. For an element da of the the same length, the same wall thickness , and the same t-0t.al ',\'f'ight &nd are sub-

:n :
mean center lme, the shear force is rku as shown where tis the thicknes s of jected t.-0 the same torque T. If their cro~sect ions are circular and ~qua.re, respt."C-
the wall, assumed col18tant. The moment of this force about point O is
dT = Ttd8 . r, I I

where r is the distance from O to th


Summing such elemental m ts e tangent to the mean center line. <l : - I
l
omen over the entire length B of the mean.
' V

center line, we obtain I

T = Tti' rds. ,/ ~-~


\D)
We see that the quantity rd& under the inte . . . ( 0)

of the small shaded triangle of base 1 h gral &go 18 Just double the area F10. 4.17
engt <U aocl altitude r in Fig. 4.16b.
roRMON
-
r
90 TJTTN - W,~LLED TU B f..S
91
··- • _1. • r. .. J 7 •·hat~ tht ratiu@ ot tlwir t<ll~r !!L,~~ r u.nd of tbn·
as eiuOWll ID rig . ,.,
t.lve.ty, , ' " ll-
_. th the ultio a - a/1:i if lht' t-01.AI IPUgth of mun <.'~te:rline, a. 2(1l ~ •i d h
&ngtes of twi!t ,;? is .. 1 T ·
sppHed t.orque remain cons.tan t
"A 4 •
n., . rJ • r ~ a, (l + a )' ta•
, <l an te
I soLtrMON. From e-q. (t.16). the rat.io of shears~
_ _ _A"G ,
r' (ft .ff prov" b
, . Us.ing t he rCBult of . ProbI ._.3, that. the r~t.An "..._. o.u.1.r2rn m ~
""• '.a.r tul,.e -1._
u " 1g A
1
will have maximum ton11on a ,nr neae w en a .. I , 1.e wbf"[! a • b ·
,,-" .4 0 'I
5, A tubular shaft with circular rrou sec-
wh~ A ' 0 and .t "c are the areas encl08ed by the_ mea.n center line11 the Baille ~! t.ion bB,S a mean diameter d = 10 cm and a wall
thickness t - 3 mm. 'Inis sho.ft must t ra nmnit
length ! (Fig. 4 .17) . F or the circle A'0 ,. a'/ 4,,.- "'·h1 le fo r the square, A o == a1; 16 _
Substituting the!e \"Blues into eq . (f) , we find 50 hp without excooo ing 11.n aasi p-ned worltiRg
stress in shear of -r. = 420 kg /cm• . Wbnt L~ the
-r'
- Sl
4r
- .--
r
- -
O ·ss
./ •
required rpm of the shaft? Ans. 181 rpm .
,,-" 16 4 6. The cross-section of a thin-wal led steel
From eq. t4.17\ the ratio of angles of twist~ tube.I ·8 m long ha.<3 t he form of a hexagon wit,b o----
mean centerline length a - 6a = 12 cm and
¢' ,,-'A "o
¢ " -= T"A 'o =
(r)4 1

= 0.616. wall thickness t = 1.5 m m . If ,,.. =560kg /cm•,


what is the safe t wist ing momen t T for the tube? 'l\~n.t rs w COl'Telf,(lndmg
F10. A

We conclud~ from this that the circular cross-section represents a more efficient angle of twist 'P between the ends of the tube ? A,a. T = l,7i J kg-o:i; ¢ ,.. 3":;.a·
the m&ierial.
uae of 7. A slightly tapered th in-walled tube of length / baa t. 9QUL-e ~ t . w n with
EXillPLE 2. A stainleM steel tube having an mean centerline dimension a at one end and 2a a t the other. Tne -nil thk~ i5
elliptical cross-section 8.11 shown in Fig. 4.18 t and the applied twisting moment T . Derive an expression. for the J.ngi.e of t r.nst 0
is subjected to torsion. If the allowable stress between the two ends of the t ube. A m . rp = 3Tl/8Gta' . ·
in shear is ,,.,. = 700 kg /cm•, what is the 8. The cross-section of a thin-walled tube of length l is a regular poino.n w:th "
corresponding allowable angle of twi11t per sides. The mean centerline length is s = na and t-he w&.ll thi.ckn~ i3 t. F::i.d the
unit length of the tube? The following data angle of twist 'P due to uniform torque T. Ana. ti> = 4Tl,'n.Gt.a'rot.=(T 11 .
b
are given: G = 84 ( 10)• kg /cm•, a= 7.5 cm,. 4.5 Shaft of Rectangular o r Profile Seetion
b = 5 cm , t = 3 mm.
The problem of twist of a shaft of rectangular cros.-.-
SOLUTION. From eq. (4.17) , the angle of
t wist per unit length of tube will be section is complica t ed , due to t he warping of cross-
F ro. 4.18
sections during twist . This war ping can be shown
(j = 'P. = ~ - experimentally with a rec tangular bar of rubber on
l 2A ~
whose faces a system of small squares has been traced.
~ e &re8 enclOlled_by the mean center line is A o = rah=37.5 n cm•. The length a of It is seen from Fig. 4 .19 • that d uring twist t.be line~
::m~n center line may be calculated with good accuracy from the approximat e
0 originally perpendicular t o the axis of th e bar become

« == 1r[ 1ca+ b) - 'Jab J curved . This indicatP,s t hat the dist-0r tion of the small
squares, ment ioned above , varies along the sides of this
cross-section , rf'ac b e:; :1 maximum value at the midd le? .
which g ivee z =- 39 7 cm. The n tor
I
a bo ve expr~sion for 8 becomes
·l and disappe ars a t the corne rs . We therefore e..\ ~ t't that
8-= 700 X 39.7
I
1 2 x 37.5 r. x 84 (!0)' = 0.000 14 radian/cm . the s hearing s tress will va ry us this distort ion : munt>l~·.
I
I
it, is a maximum at t.he m iddle of the ;:;ide~ ~nd 1t>ru at
i P ROB L E M 8 ET 4.4 the corne rs of the cross-sec tion. Tm e:-:tigi:l ti,in ()I° tht:•
. 1. A tu hulor alu minum i.haft ha . , . problem t indi<'u tes th at the maximum sht':m ng st.rt.-&~ 'f'tG, 4. 1£>
BIOn r.i c 2.5 cm m ust- safelv v,n~ _a square cross-scctwn with outside dimen-
. ctury ak tw1St1n~ mo ' t T occurs a t t h~ mid dle of t he longt' r sides nf tlw l'{'d.:rngu-
propn wa lj t I11ck rlf'SS 1 1f lhc. . men = li95 kg-cm . Calculate the
.
2• For t he alum inum . " wc.,r , •LTJKd st
tuL. ,j ,...
r"ss l . .
. · s r.,, = ~LO kg /crn 1. 'Ins . t == O. 15 c m . l11r croHs-scction a ud is giv('n by thr t•q ut\t.ilm
ft .
,0
. UC <!otn,* in the
P!.'r unit 1.-ngth of tube u.ndn tli<·
ced ' . .
pre mg problC'm, culcula te the angle
r (-U8)
Wll<t
modulus~ = 281W1• kg/cm•, Arn . 0 == /1i,pl '. ed torq ue T .z:: GY 5 kg-cm. T he slwar
r,Qu = obfl'
3. A thm-wullerl ~tam le..., ~•- 1 • - .) .(J{IJ :.!8 rad ,an/i:rn. •Thie figuro is reproduced from B.-ch's E!~tmlO! i.nd Pc:llti;A:a !.
F' A H
ig. ·
"" "...:" tulJC has th
ow does the a.ngfo oi twist per ·t
{1'Ct.:J.ngu l&r ctoss-scr·tion shown in
trni engU, of tu~x, due to torsion vary
fl'he compll• t.u 110luti~11 i11 due to ck &.in l \ enlillt-, JJ~m ,in &vo71 tJ e-l ran~rt, t.
14

(1 855).
9'J
in whlch bi! the l~nger and
TORSIOJlf

th shorter side of the rectangu lar cro~


~c :nding upon the ratio b/c. Several va~tio~
- n.EC'IANG ULAR SHAIT 93

&nd a is a numerical f ~ctor p Uee of


4
a are given in Ta~le · 1· .t I ngth in the case of a rectangu lar cross-.
The angJe of twist ~r uni . e section
is given by t.he equation T
8 == ~ - (4.19) (b) (o J ( bl
(o l
FIG. 4.20 F 1G. 4.21
numerical factor p are given m the third colu Inn of
The va Iues of the
Table 4.l.
Tabl 4 1 DA TA FOR THE TWIST OF A stress is at the reentran t corner.• This maximum strebS is obtained by
SIL~ OF RECTANGULAR CROSS-SECTION multiply ing the stress given by eq . (4.20) by a stre<>....s concentra tion fact.or.
This factor has been found to vary with the ratio of fill~t radius r to thick-
b/ c a fJ
ness c: values of this factor are given in Table 4 .2 .
1.00 .208 .141
1.50 .231 .196 Table 4.2 STRESS CONCEN TRATIO ~ FACTOR
l.i5 .239 .214
.229 r/ c Factor
2.00 .246
2.50 .258 .249 i 2!
3.00 .267 .263 l 2¼
'! 2
4.00 .282 .281 1 l¾
6.00 .299 .299
8.00 .'J07 .307 identicil except
EXAMPLE l . Two thin-walled tubes of cir cuJar cross-secti on &N
10.00 .313 .313 that one is seamless and the other is split as shown in Fig. -1..2 L If r is the nuliw
a, .333 .333 of the mean center line and t is the wall thickness, find the rst.io of their angles of
I twist 8 per unit length of tube when they &re subjected t-0 the same torque T .
I
BOLUTivN . For the seamless tube, we obtain from eqs. (4.16) and
(4.17) ol Art.•·• •
I
It is seen that in the case of a very narrow rectangu lar cross-section such
l as that f~r a thin ~p of sheet metal, a and Pequal ½and the equatio~ for T, T
!' tbe ma:umum sheanng stress and the angle of twist per unit of length be-
91
= 4Ao'Gt = 2rrGt
(a)

come
Using cq. (4.21) for the split tube where b = 21l'r and c = ; tre obtain
Tmu:
3T
= _, (4.20)
bc2 3T (b)
8,=--
2n-Gt'
' 3T
'1
8 = bcJG' (4.21)
Thus (c)
l1
These equations are of practical .
only for a narrow rectan 1 b unportance because they can be used not '
Taking, for example, t/ r = 1/ 10, this gives 8J 8i • l / 300, i.e., the seamless tube
j in which the width of thg e ut also for approxim ate solution s in other cases of is 300 times as stiff in wrsion as the corresponding spl~ tu.be. .
th ·
e cross-sectio 18 I EXAMl'LE 2. For the structural angle S(>Ction shown in Fig. 4.2:(>b,
the foU~wi~
j n sma I. For instance, in the case
e c~o.s~sections of uniform t .
of tw1St is obtained from e h1ckness shown in Figs. 4.20a and b, the angle dimensions are given : a = JO cm, c = I. .!cm. A steel b&r ha \·ing ~ cross-see~=
4 21 th l.2m long and is subJ·eoted t-0 twisting momt>nts T-.s,000 kg.cm at its eJ1ds. Compu
developed length of the ce ~ - ~ · ) by putting in this equation for b e · · th ·
the maximum she,ar stress r and the angle of twist ,; l:w>t~·een e two en
ds of the
represented in Fig. 4.20 n rdlme, namely, b = tf,r in the case of the section
f'g1 bar if the fillet radius r -o.6cm .
a, an b ::. 2
tr
4 20b Tl ·
. ·. · le maxim inn streRe f a - c m the case represen ted in · •The above methoJ1 apply where the ~Uoo a ol the atnlotural abapea are free
tamed from eq. (4.20) . F~r the first of these two sections will be ob· lo 'Warp .
e angle section (Fig. 4.20b) the maxirnum
I
M -
2a _ c =' 18.8cm in eq. (4.20) ,
SOLO"TlOfl . T&lciog b '"'
S X S.(l()(J ..,. 33 3 kg •c 111 1 .
r.,,n .. !8.8 >- ( l. :!) 1

12 t the ~ ooncent.ratio n factor is 2 fr 0 111 5


~ wjth r/c =°:6+ · ~~rstress st the rt'entran t corner becomes T Tab1~ •
2 )( 1.2
&nd the m&XUUUm ' ..,
&00\.1'
- 666 i(,'an'. 333 • Bearns: I
Stresse s In
The total &Dgli' of twist from eq . (4.2 0 becomes
3 x S,000 X 120 = 0.0:JgB
, .,. !tl = 18 _8 x (1.2)i x 8+ ( 10, 1 rad ian.

PROBLEIISJ: T 4.5

1. The aluminum channel section shown in Fig. A has the followin d" 5.1 Shearing Force and Bending Moment
" • 2 5 cm..1-i - ~.5 cm, t .. J.5 mm: 1:1e fillet radius at the two reentra~t unensio 0 :
k~orners i8 A structural m ember t hat is rea.30 nably long compa:rad with its l&t.e?&l
r -o 7.5mm.Cslcu-late the safe twisting moment for the channel if the
in she&:- is 560~g_=•. What is the corresponding angle of twist() per wo~ Ing str~ d imensions when suitably supported , and subj~tro io transverse fo-reee 90
chaanel ii ~::15, l0}41.:gJcm 0 A,u. T = 25.6 kg-cm; 8 = 0.00667 rad /cm~nit length of applied as to induce bending of the member in a.n uial p.13.ne. is c.alled a
beam. Several examples are shown in Fig.
5.1. The beam in Fig. o. la , supported by

rI a pin at A and a roller at B is called a rimple A,(L


beam. The one in Fig. 5. lb, built into a ~
wall at B and free at A, is called a can.tile»e-r
z;,; z ;;,;; ,; CZ½77! L beam. Since, in l,oth of t hese cases, the
!c '
8 ,;,
b-- ---.Jf conditions of support a.re such that the re- A - - - - - - - - - - -(.
f w. A actions can be found from equations oi ( t> l
Fm. B
static8, these beams are said to be statically l c t
1. ~ "'11.C'tural 8ieel angle section sho
~cm, b • ·71. cm, t •12 nun.The fillet
· Fi
r:i
m g. B hM the following dimensiom: determinate. On the other hand , the beam .:
JS:1~,----,....-----...;.,(
st in Fig. 5.lc, supported by a pin a t A. and -
· t.e -t~ tale twisting moment for the us . t~e reentrant corner is r ==l2mm. ( cl
840 &ection if the working stress in shear is rollers at both C and B, is swtically inde-
G q l:ll'.I'. What i! tJie cotrespond·
terminate, s ince the reactions caunot be J 10, S l
1

-lfi,o,~ 10JJ., A"'· T. 7, JOU .kg~c~~': of twist fJ per unit length of bar if
' 0.0004 76 rad/cm. found from equations of s tatics alone. For
!.Ap~ the present we confine our attention t o static!ll.tv d~term..ina.t.i be3.ms. •
~ ~· . IUt,jec~ ~be.rhavin g a2s x Li, crn square
1,500 to t . . . / Consider ~ow a cantilever beam AB subj ect.t!d to ~terna l l~ds Pi and Ps
kc-on at LtA -.A
a,_ T. Wh&t 18
ttnit I
.

the ~ d i n
Wl8ting momen.•·
. "'1'll.la, C&lculate the . w
T •
tn&XJmum shear
"' j
as shown in Fig. 5.2a. If we imsgine th is l'<'d.111 tu ~ c1.H by a ~tion MJ~
we see that the .1pplied forces tend to d 1spl iiN th.e lt!f t-h!A.txd P')Ct.1on .of the

~
tht b&r-1 A~ g angle of twist 8 per
beam relative to the right-hand portioo , wtu_._•h ~ a11ehored in the. wsll .
0.JY.Al~~ of
•• A ) 0 f"tn . • T-.. -no 1'g/crn' · 8 -
"' Ill !, ..._ Q o r, '
This tendency is resisted by internul f v~s l'lt't"-"l~ ll th~ two par~ of the
I:Srno, thick ;;:, . .o% e&rbao ~I
C. Caklllaki th ae<I lot a duo-, 8 • It.rap 2.5 crn wide beam . Thus if we isolate that. 1xJrt11)n u.f t-he bt>::UJ.1 to the leit oi the ~tioo

□g
bald the d~ e r,uJJ P ~ the d Pmlg aa shown. in F'1g.' ~n i's a free body, wo reprt-sent thti t\~tiou t>f th€J bu.ilt- i~ ~ rti~o. t heron by
'K>r band•
handlevur daa.tributed foroes wi show•\ in Fig. [\ .~b, Th\} true J1Ji~.l'l b~uo~ ol ~
•"- Qperi t r--
ie required to
"'~ is 80 a ftT to tht ,A
. cni from ti. - . P1&tie ol th
internal fore~ on the IIC<'t1on mu ~ tomph~!\ted , but to- rnam tain ~~wlib,-
was !me e OJ>ening if
Pretwtet.«) .d . . · . iu: h1nge
Urtng Ulilt.. 11~,:
torque 1'0 • 2,.) kg -crn • The •'=I
Jnaxim wb . """"l(ion so ·- strap riuau of the froo body, thoy must be st-tl.tieaUy ~qu.ivale.rit to the. equihbmnt
fully urn •hear et.re. _en the dQor ie Ill to 8UB1.4jn a
ope11? An, p Ill the 8fir1 shut. Wh.e.t ;,. •h
· • 0.57 k . P \Vhen th "' 11 e
l •Metbode oI dealhtl w{t,h ,iati"6lly iJlclt)tiv-tulnat.e boama will ~ d:ituu.ed in Chap-
\4tr 0.
g, T-. • 2 e .door i1t
,43$ kg/crn•.
J.1'10. C

~ F . 8 IN B~ ; A-

l 96 d p T his stre33 result~nt on th D J;NO U~O M OJO:NT


l f roes Pi an 97
i i- .
f the applied externa o ~ by a force applied a t t he ce n t roid e l!ect·
of lo~
~ can always be represen both in the a xial pla ne of t he a pplied lo lh~ M 8 re positive when t hey tend t.-0 bend t he elf!ment coru::a~ up-wtLtde.
ne-eection and a couple, . turn be resolved i n to rec tan g ula llds ~egotive d irections of N , V , and M nre 8how n m Fig. 5_:n,
~rthennor e. the forre cspln , in _,
nnal t-0 the ane Vi
th:
section , and V ., lyi ng in t he prl cor-n,
ane
Usi ng t he t h ree equationB of equ ilibn wn
pooen~ Ns, no . F.
the aection. Thus, 10 ig. 5 · '
Zc we represent t he stress res ultant on 1
&.ny
° l;X ,= 0 , ~Y i= o,• ZM ,=0 (a )
for the free body in F ig. 5.2c, t he norma.l for ce N . , ~ ahea.r f0,1U V a• and
P, the be nd ing moment M . a t any cr oes--eectio n mn ca.n no.a.chJy be ealculated .
,,___, Using t he centroid of the section mn &a a moment ~ n f.e:r , the9e e:qt- u:>ns
I p m
give
,. B N • = P i sin a , )
V. = P1 - P 1 cos a , t fb l
q
" M. = Pi(x - a) - P 1 coe a · :r. j
,I

P. (a)
These expression s, of course, hold onl y for a < :r < l. For O < :t < a, we
P, consider t he equilibriu m conditions of tha t portion of the bea.m to Ule leh ai
i.---o
I a sect ion pq (Fig. 5.2a) and obtain

N . == P i sin a ,

-~
}
V. = -Pi cos a ,
( b)
,I M • = - P t cos a · :r.
Pc
1---o pl It should be noted from expression (b) and (c) that . in~ral . _v __ v.,. a:.:f
I M . vary with the distance x defining the c?Oll!Heetion at whieh they oa:m
In most practical problems, we
I
I - - -- --- N11
encounter beams which are hori-

l
zontal and are subjected only to
r vertical loads. In such cases,
~
Vz there will be no normal force N. on
( C) any crol!B-eeetion and we have to
F10. 5.2 deal only with shear force V. and
bending moment M •. The rela-

\
eection fM by the th~ c uanf .
~ f <1r~ , the Mt.Gr }orce ltleS N •• V ., &nd M ., called, respectively, _the
Tbeee quantitiee _;11,.. _

.' and the bnuling moment at that eectwn.
.. u. ue COflffidered
shown m F ig, 5.2e. Such si
·· ' • ·
P<>8t~1ve when they have the d1rect1on.e
tionship between these two qua.n-
tities is of special importanc e &nd
will be establiahe d in the d iscu.ssion
that follows.
A

!l
Ill,
( ~I

... , I' } ,
: -c
i e&refully obeerved to avoid ~ .. eo~ventions, although arbitrary , must be
T0 PW"lue .wue
,..:_ quest," COm us1on
·
, v V
i 1-¥ ~
! bet-w •
een two adJatent
100
further let
cl'088-aec .' •
U8 consider an element of the be&ID '
,. ' M

i
l
forces N, ~ea.r forces V, &lld betv.,?JJ 118 shown in F ig. 5 .3a . Then n°~1
eie~n~ will be eoniidered a, ~ ~Jng moments M on t he two faces of thi9
N N

& J)Of!ltive normal force ia d' ttve wh~.n directed h . We see that
'' !' which it a.ct.$ Aao U'ecte(t awa . a.a e own. V V
n
~ P<>.sit~ve •hear for y m that face of the free bodY O f
( cl
I
rotation a.bo~t 10
!
a p01nt ina1de the free-ce l.8 one ~ha.t h88 o clockwise sense o
ta I (bI
Fao. 6,3
i 'I body· FtnaUy the bending rnomenti
,,.._. ~ • '--'_,m . l >
.:
,..- _~
. -'e beam .4. 8 csrry,nR n un l·rOil-,
t,bi, ~mI·" .
,_i.-, ('('13"'" ,._ 'ty 1t' &.s Fhown m Fig f> 1a :.I)
M ~ .J mtt-.o.• · f1. • dM. _ V .
"snt ~?-,r ~ d 1 • • ..J , tihe reactions s t A &nd B lltt> fou '-'
rrom 'l"h•.rh riL (!> 1
d~ ~ t,,,.1 ,~ . ~• f~c:w."-') 'ld~ •
- t.V t'~ b1' ~ .
~f 1
'- , ~ ~ If«. -'---"'"" fo,,...~ and b(>ndmg, moment
..._ "' -.:i f
at a c , 'fhut1 at nny r ~.."'r t ton of tb,. t,e.am bt-t.ON"t'fl porT'tt nl &ppl,catol}r ;,l
or 8 • ~ 10-w t.ht ~ ~il-..~..; ... .... of 1b&t portion o t he bcarn 1 _ concentr11t('(! lo&de, lh!' rnte nf chan~ of oenr:lmg mm:.rnt -..,,t. ~ ~ ~-<> .:
i'tite 1JJ fl"JIW \,he eqUl 11.11 ,u,,. bod ~ IL
~ ~r A ting on this free Y, we have lb ~ iB equAI to t be shettr f nm·
~ all. , - - {fti, ~,~ ) c d:......ibuted load between .. t ~
~ d WI - -- part <i t.ht "'"' · . .. n. and ~
• , !ld .nti Iha\ .i.- Portion of the dIBtn buted load wi}} ~-
d-+.&J'IC(' t t'li from a.s s own . On the fa,..ut,
~ ~ - ~ -tt&Dt of vUD , A b 1"/1.I
"
~ , . . tu,: ~"'
~ f. JOl ~~ arU.t'"
, ___..,..
t,1,t.,;-UrJ(l ~-11
~ »l tht' I&'sh,e£r force V• an d a be ndi ng moment- l,"
,r(' hafC a h h .
of the extnnal fo rces 8.8 any orizont.at co•
t-,,.zt cs th1.'S CS!lf ,_$Jl(lt' nont rm,.} force N,. Now using eqs. (a ), we ob~
ii,
v,

. I'
V,

"'
·-~ 1t ~

~ u.,·,.- ",11 ~ no 00

t.
""2
r . .. - - WX ,
-,c 0
) *( 0 )N
,c ~ ·)
M,-=
wl
2x - 2 ·
wx'
(d

" q
.f/1 , dM,
n q
.IJl, • d~ I "
-.--. I 4

il
. • ---1
we .-,
'l!'t' Cllz:IS'JCI'
.1... • ..,.,.uilibriom of that portion of the beam to the righ t ol
~ d.1-
~ ~ d.1
V, "d\', Lh-1
I t ~.

~ ~JDC 11mfFig .'.i.k ), we obtain (0 ) ( bl cl


r.::. 11J(l - x) -
2 , Fro. 5.5
wl ) (e)
M. a: w(I - I ) - i(1 - x)' · U there is some distributed load of in tensity w be,ween ,v. ud Pf, tl,,e
free-body diagram of the element will be as shown m Fig 5.Sb. ff.ere a ~
we equate to zero the algebraic sum of moments of aJJ foret3 ,nth ~ t
1l:-.M! ~ oan readilJ be reduced to coincide with expressions (d) t.o point O and obtain
~ Smr.t- ~ t ~ beam ii in equil ibrium, we must obtain the eame
n:u:ic Sat~ foroe and bending moment at a chosen section, by using tht -M. + (M. + di¥!. ) - V, d.z + wdz (~) = 0
~.2~ cmirlrtwns for eith!'f portion of the be&m.
A ~ R!l.ttlbrll>bip between !bearing force and bending moment at Neglecting the laat term as a small quao uty of ~-ond urd~!' to U"11:1 ~s~
~ ~ d ·& Lrin!verse·ly loaded beam will now be shown. Io preasion , it reduces to
F-1€. !ik, ~ •n tlfflnent uf the beam cut out by two adjacent er~· d.\f . - l' . dx = 0,
. -. u.d r- • d.is!~net cu "- apart. Vo the left-hand face of this
ileCtJi.r.,,, . element,
d . which yields the sa.me relation:sh1p (5. 1) obL~lfu.'d al>..w~.
. , .,,.~ _ . :ar tv..rnr ar, oeod1og mome nt by l'. and M. (assum ed·
"-" ~ )!'t:il!i) l t!it, ~ '
p!'.lli!ll\i~ ' , uc:n CLI. 1:Xten,J losd ,
Equating to zero t.he algobralt sum of veru-:al inrces un th.ti el{'m<1m i u
•~ °' ,1.. .+..,, 1 ~ · · 1.8 applied between the cross-sectionB '"" Fig. 5.5b, we fiud
'"• · WI.' -..;.,at: <Jnif. 11.lld btndi
~ a ..,.i!J Lit v. 11.r.d M . og moment on the right-hand face of tht. I' . - o·. + d V #\ - U'<U "" 0,
~ RVRMnl lr..-1 • .,.. dM • aa ah.own , where dM, is the change 10
...we. il.!gtr,.ic ~ t,1 s· .
-•"tien ~ 1rid THJ. ll lOO the element lB 111 ..
cyu1h brlUll
· 1
'
.·· from wh teh d\'.
dr = - Ill, ~5.J)
t.od • .e fu1d tli!JmimtJJ ,,f th1-.ae forctl$ a.bout point O must be r,erv
Thlll when there kl dlStrth lll<i<I l~cl Qf mw.nst,s III bet.ween th\1 1..'1'08&-
- !( + ('I
• ,I>J, +dM , ) - V~.t a. Q
8e<!tiona mn and pq, tho atw~r forco 1.00 ob&n~ alvng the bco.m !Uld lt.5 ra~
I

;
>
JN B ~ : I 9 E NDLNO M OMEJl,'"I' 101
8 ·
100 ~
. the intensity of load bu t with OPPosi It will be not.ed that thew> exp reMirms aa t idy the relatwn11.hip expre9ll6d by eq .
. t to x 1-s equal to te (5.1), i.e .,
of change w1 th respe<'
• the beam between the cross-scc tio dM. wJ toGZ 1
sign . · -A load Pon ·11 b l\s cu - 6 - 21 - v,.
f the eleme11t. w1 e a s shown •
If there is a concentrst.eu .
m.n and pq, t he free-body dJagr&hm ~ear force on t.he left-hand face of
. denot.e. t ht-hand
es . toe
V' s • -Th e n equating
t.t Setting thi8 eitprcfl8ion for dM,/dz "" 0, we firu.l z ... l; \[a, defining the !ocation
Fig. 5.5c . In this c,a._,qe, we face, by
th n1
c-lemen! by V. and thn t on ~ f rces en the element, we obtain
of that nection for which M. ha..~ it& maximu m v&lue. F".wa.lly , au beutut.ing trull
value of x into the second o! eq8. (f) , we obta in
~ ro the algehraic sum

.
of ver11ca
. .o
V., -- 11 - P.
b t ch.s.nge in the shear force over the length
M..,...., ,. -
T;Jr}'
9,(3
·
15
Thus in this_CA.."C t~ere n~ ru~luded from eq. (5.1) that there will be a U,UlPLE 2. A simply supported beam AB carriffl a co11tinu omly di:nr..ooiarl
dz. Accod~J.D~ldy: i t n~nyuit"' ~: the derivative dM ,jdx at the point of load the intensity of which at any point z a.long tJie bea;n is w~ - tre no l T%/i}, u
correspon :ng 1sc0 w J
th shown in Fig. 5.7. Find the maximum bending moment in the be&.n.
application of a concentrated load P on e beam. . . SOLOTJON. In & ca.,e of th.is kind, the dire!t method of writing ~ f-OT V •
.
a triangular distribution of transverse and M., as done in Example 1, becomes c.umbenome becawe ol the d!ffiru.lty I.Il
A
ia.UIPL£ ! . .~ !UillP e
-
1 be&m AB carries
· Of 1oad a t B be'mg Wo, A t what
·mtens1ty
' --~ ~- "· t";n 5 6 the m&Xiinum d' t
K..U u. 1!11'.IU•'!l Ul • 'I!, • . '
~ t i o n (defined b-y z ) does the maximum ben mg momen occur an~ what is
iumapl.iwde? . . . .
SOLU'F!OS The resultant load on the beam 18 w,,l/ 2 acting at a point C d1Stance
l/ 3 to the lfft Gf B. Thus, by !MB = 0, for the entire beam as a free body,
RAl - (wJ/ 2)(l/ 3) = 0,
wJ, Fm. 5.7
from "'h.ich R,. - - 6 ·
n ae frtt-bo:!y diagram for a portion of the beam to the left of a section mn is shown defining the re.suit.Ant load to the left of any c~tion z . Instead. ~ csn proceed
in F'ig. 5..!}b. The result.ant oft-hat. part of the distributed load which acts on this by using eqs. (!U) and (5.2). Beginning with eq. (5.2), we b:&v&
free bod-'' i! w.z:121 applied at point D, dist.a.nee .:r/3 to the left of the section mn. dV. . n
Poaitive Ehar force V, and bending moment M. act on the section boundary a.s ~ = -tDtsm , ·
ehown. From the equilibrium conditiona for this free body, we find
Separating variables and integrating this, we ob.~
V. =z wal _ Wo,%2 }
6 2l I w,I. r.s (i)
(f) v. - -:;:- C08 T + Ct,
M. _ w,k _ 1DoX'
6 6l where C'1 ia a constant of integration. Now using eq . l5,l ), •e ha"
dlJ.. w.l -rr
-- -
d:i
-
T
008 -
l
+ c,.
Again separa.ting variabl~ an<l integrating, we obtAin

. n:
wr:IJ si-n T + C1:r -4- <.,...'> UJ
M • ., -;,

~t~ :=;p (o) where C, is a !lOcond constallt o( inkgmt.ioo. . .

·---,
1
~~ To evaluate the con!ltan~ C, and C., we m., t.:, ilit1 fol lo-..it\g l)Cuntlary oondilltm,1

at the ends of the beam , name.I) ,


' 11• wh~w .: - 0, (lrj
( bl
wb\\n .r ,... I.
Fw . 5.o
103
DEN DJNO M OM.E N T
- •N JJE.4 "" . J
sr.tr~, s .
c• .. r&t.i on of eq,J . (5.2) and (5.1) . Sot~
. (j) we find in this case that xpre Mil)D8 for V,. and J.,fz hy dire<-t ioteg
102 ' cond it.io~ mto eq. , 1 Ana . See eqs. (J), page 100.
gen: r~:lo &d inte nsity will be W&X/l . n
ting t,heee boun dar) porte d beam carri ~ a !in.e arly V&r'/Lnfli t.ra nsv ~ IO&d u !!hew
.
(j) becomes tba A simp ly 8\Jp Usin g ~ (5..2) and
Sube tltu end of the beam u, 111o.
u,J' . n . ~ E The inte nsity of load at ea.ch
0) and M. s.t a cr088 !leeti ro diat42lce . z
C, - 0. ThUB eq. from
M. • -;:,- 810 T . (6 ) gde~ elop gene ral expr eMio ns for V,.
eut be a max nnw n ? Wha .t i&
1 the bend ing mom
rnaximuro valu e whe n z "" l/2, &nd · ~rt A. At wha t valu e of z wi ll . ,,.
. •ts etion of the beam ? A n.1 . max Jf . at
...... . L.;a •n>re sPOD ha.e I
'l\>e
: : shear force at the midd le crO!l&-Se
ItiBff/1/JY to see. ...-~ ..--. - 0.21 1.i; v. - -Wo l/12 .
obt&iD

. t at the midd le of the beam .


·:= i
•. ~ ~
for the m.aximwn bending momen
~ /~...
5.1 {- -
~- _j_ - _ , _ _____
-
PBOB L.EM SET
and bend ing moment FIO. F
. be&m . Fig e\ evalu ate the shea r force . Ana
Fw . E
m
1 p' oint of appl icati on of the 2 ,000 kg load
th; rly vary ing tran sver ae load a s ~
I. For_ tbe_~pto ethe lef: of •
9. A simp ly supp orte d beam carri es linea r force V z vani sh ? Wha t is ~ V&1ue
at • secti on ] woo the shea
1,500 kg Fig. F. At wha t dista nce x from A will - i/
2'
000 k
g r force at the midd le cros s-sec tion of the be:am ? An8 . V. - 0 &t z
of the shea
1,000 kg ( to1al ) t-+,..., ...-r-r', -,,..·~ ~
/ +
-t ,SOO kg ( COtal )
1
,.:_~,.:;.c,...c.,.rr.,.,. 8
(2-y3 ); V. = ±wo l/6. lr~
:£1-/ /I80·--
' I Ii 'jI I A'-- --- --- ,-- ---- IO. Deri ve gene ral express.ions for norm
al force N . , shea r force V., snd berd
.V. - "T"P ;
I
n i.n Fig . G . .A.iv .
---- 90- /- 90--. l---120,----- mom ent M,. at the cross -tiee tion mn of
the beam show
1-- J
FIG. B V. = -0.2 P; M. = -0.2 Pz.
FlG. A
n
G~rrP
5 00 lq,!
I :,00 q cou..l
V ""'81'.?.5 lr.g · M =i l6,250 ltg-.m .
%. Calc ulate the she.ar force Vs and
the bend ing mom ent MB at the secti on kg;
B. Ans. VB = -4,100
just C
:JI'
500 -., ( tow )
I I II IO li !>() '
1 6

~
lever beam in Fig.
to the left of the w&il at B, for the canti
.'t!11 '"' --436 .000 lc.g-cm .
-sections C and D of the beam shown
t6 0_ j~L 120-
1205 00 ¥8 1 --.I
S. Calc ulate the bend ing moments at cross _l - I 20:.5 df.
,000 kg-cm .
iD Fig. C. AlM. Mc= O; .MD = -225
Ip p FIG. G
Fm. H
,,. percm
as show n in Fig. H , e&lculate the ~e& r'
j!l l II l lcl l 1111£t J ll. For the beam supp orte d and load ed ~ t i o n D . ..tliv. V O = - : 1 :.; !l.g
La . - ,t . a~ force VD a.nd the bend ing mom ent MD
MD - -+ i ,500 kg-c m .
at the

beam in Fig. 5.7, 5.8."rufile w iths inren


.sity
F10. C 12. Refe rring to the simp ly supp orte d h~ wui is the
D g to a. para bolic law. In suc
• A · l .
F10. of th~ tran sver se load vari es acco rdin - 5cc.l 'r 48.
... sunp y SUIJported beam wi_th over ~ngm g ends carri es tran sver se loads as ing mom ent at. the midd le of the be&m
? A n., . .Mc
octe d to a unifo rmly d~ bu t«i ~
ehown in Fig, D. If wl = p wha 19th bend
M, 13. A simp ly supp orte d beam is subj
~ ratio a/ l for which the bend ing mom ent
t>! iog
at the midd le or t he beam ~ill
AM. a/ l = t. load of inte nsity w over one- half of
its span l . Calc ulate the m&.:rim um b.md
5. Referring to the aim I 8 zero.
beam shown in Fig . A ' aasu me that the mom ent in the beam . Ans. Mm,u =
2/ 128.
9wl
load i8 rep'ft .. ~ Yb
0
rted ~f_P
clistributed
·u hj ~ Y ,..,
;:: ) · • have on the reactions a.t A and
resul t.a
;.,t
a., a co~c entra ted load . Wha t
· What will be the shea r force
effect
and the
d-~ w~ moment at a aectio n just to th I f new con· She ar and Ben din g Mo1 11en t Dia
gra m s
'J 0
~ A~. V =- -68i .5 lr.g, M • ,.:3 e7so ~ of e 2,000
th -kg load unde r thes e S.2
, kg- cm
• · emn g to .the ai.mpJ t hat the :-ihco.r fo.n:e V. and ben ding
shown ·in Fig D . Y IIUPJ>Orted bea ·
1
ing ends loaded as We hav e seen in the prec edin g artic.le
a conc entra ted 1 ':;_8pUin~
that the diatributed with over hang
. ~ wl ally replacedh 'byb mom ent M. in a tran sver sely load ('d
bt'..iun will. w gene ral , \"ru"Y with the
°
C. Under th
the bend.in mom ent atatth .d
"" P
.
eee cond1t1on.a , wh at 1B
is actu
the ratio a/l for w ic di.st ance x defi ning t he loca tion of
, ~tf on on whic h t hey occu r
the cn.'l..._~
g bea
7. Refe rring to the . e mi die of thebe.a. Am. a/ l = i- ,<1 to use u grap hica l ~pr esen tatio n of
sirnply eupP<>rted m will be zero ?
e For this ,eas on, it is ofte n adv anta geou
m load ed ll8 show n in Fig . 5.6, deriv
i I
I
D'&N ulNO M OM.ENT DlAOR.AMB
104 10 5
. . I g t he fl.xis of t.h c heam . To do t hi
b uantttJe!! a on . . d h . s, ~e
w t.hP. nght of tho load P . 1 e., fo r < < {, we
the vari&tion oft ese q . . of the scc t10n, an t e ord1nn +.,. h fo r a c rCIB!HJCCLion P<J '1 2
· · the po..."'1tton ~ t
1et tihe abscissa mdfo&te f • or t>t-nding moment . Such graph· " ob tai n in t hf> same way
· 1· f shear or<.e - . 1ca1
correspondmg ,·a ue o . fori-e a nd bending moment diagra m~ V,. = Plb - p = P O, - l1 = - Pa,
. . --'led
rep.re.sent.awons are uu s/lMrinQ .
·
.
l l
p I b)

}--- o- - -H""- - IJ M ,, = 9--x - P ( x - a )= .._pa(J - 7)·


L- . ~ m
uq p
Thus again , t he shear fo rce 1s rorutton t but of nf'ga tne 51 ~ . -.,,,hrle ~

.
'II
bending mome nt v ariee linearly wit h z . At z ::::o L, 1t.s t,-..11..• L'l i.er0 while cu .
-~
t,.----1- , tI- - --+t x = a , it becomes P ah/ l. The corre~pondmg dU\glUm.a of ~heti.r forre 1tld.
'- - - --
~ ,- (a) bending mome nt for t his part of thf> bPam al"! ~ hy the !;"_,, ugh!
., lines cb and c'b' in F igs . 5 .8b and r
0
1
c
Viewing the complete d iagrams of ahear fo rce and bending m.omen.t. vre
q.{1 11 ! l ($1/ , 111 1 £E,_ see that the point of appl ica tio n of t he load P t.here 1S an 11bmpL chn..gl? 1D

JIlflIII~~
1

the shear force from +Pb/ l to - Pa l l. Corre'3pondrngfy , ~ 13 s ~


(b l discontinuity in the slope of the bending momeo t. curve a'c-t/

.~ -+
If several transverse forces act on a simp le beam as ~ o Fl~ 3.98 .
the same reasoning as above shows t hat between tranave..-se loads. tbe ~
force remains constant and the be nd ing moment varii's hne-ar;:; ~ tn£
shear force diagram will hnve t he gene ral fo rm :1hown in F ig 5 ~~ wh1:e !.:-:~
o·~ ~ __l
( C) 1:/
1
Fm. 5.8
A
ra P, P,

a- - #
To_illustr~te, let ue consider as an example the case of a simple beam AB
~mg a _m1gle_ concentrated t.raDBverse load P (Fig. 5.8a) . From equilib- b~
rium corum:iera.tloru! of the entire beam, the reactions are found to be I??
'rI· {a}

RA = ~b, Ra = ~a- ~
y

Tb~n for a ny Ct'Ol!la-flection mn Lo the left of p . . be


·i
1
concluded from Lt
m,i that
.,.h . · , 1.e., for O < x < a, it can
te equiu rium of that Portion of the beam between A and R,-{ll~lll,}1:1,:, ,I. . ._~: :· ei
I
.__..._,
(bl
Vz = +Pb Pb
l , M« = +-x-
l
(a.)
From th~ exprf-fs.Sions we M J
and the poin t of appli~,.,,.; ,e t lat the shea r force is ~onstant betwee n A
. l .. w.ot1 of the load ,
varies · inearly with .t. Fo P , while t he be nding moment
.c = a it .iR Pab/ l. The cor;esx ~•
th e. lxinding moment is zero and for
. l
I lines ac a nd a'c' in Figs . .5.8b : d tg
d1agr11.ms are oh own by the :-;tra igl1 t

k,··v•

l 106
bending moment d iag
. ram w1U
.
8'1'R,E88J"J! ll'l sEAll<S :

o
ponding to eo.c
. th
b abrupt change m
M /d f
I

. be f the fonn shown m 1g. 5 .9c . I t sh


e magnitud
·
ouid
e of
F'
Bz.NDINO MOMENT DIAGRAMS

I: 11 1~•
~l'.1111~1Ii Ii Ii 11 ~
107

be noted that co.rres . , . it in the slope d :r x o t he bendj


shear force t here is a dtscon .,inu y Ilg
moment diagram. . f t h diagrams in Fig. 5 .9, for given nurneri 2 I I I l
The a ctual const ruCtJon ° ~ dist.a.uces hetween their points of a.pplj cal t
11
I (Q}
I
I
I
,·alues oft.he loads p .,
. . . .
f!r,
P, a.n t · 8 R and R 2 will be calculakd Thea.
le First the reac .1on • en
I
ol I
I
I
~
i
tion, is quite simpl . of shear force and bending moment at only th~
on comp~tmg va ues_ t to t he left of ea.ch applied load - the dia.gra ·: ~ : :
I I

cros.s-sect10ns - say Jus


LlL?l be cons tr uc ted Sl
.
h
k
·mply by connectmg these ey pom
·
· ts b . •

as shown. Finally it should be noted that t e maximum or mate of the


d'
tn.s
Y straight lin...
""' l ~·i'
:
I
(b)
l e' Ib
1
bending moment diagram (un der the load P2) oc~urs at that section where
the shear force changes sigu, since here also the sign of dM /dx changes ac-
:~
I
I
r
ill
I I

oordingly . If, going a.long t he x-axis, t.he shear force changes from a ~' ~
positive to a negative value at a certain cro_ss-sect_ion (as in Fig. 5.9L), (cl
we have a maximum bending moment at this section. A change in the Fm. 5_10
shear force from a negative to a positive value, conversely, will indicate a
minimum bending momen t. In general, several che.nges in sign of shear bending moment occurs at the section for which V,. = dY ,/ dz = 0, t o
force may occur alung t.he length of the beam. If so, there will be several accordance with eq. (5.1) .
m.B.Xlma or minima in the bending moment and each one must be in- In Fig. 5 .lla, wc consider the case of & can tilever be&O ca.rryu:.g a.
vestigated to ascertain the numerically largest bending moment in the uniformly distributed load between A a.nd C as shown. F ollowing the sigµ.
beam. conventions of Fig. 5 .3, we see that in this CMe both the ilies.r forre ~ t.ne
Let us consider now a simple beam AB carrying a uniform distribution of bending moment are negative throughout the besm . Specm~, fo::- &
transverse load of intensity was shown in Fig. 5.10a. In this case the re- section mn between A and C,
actions are each equal to wl/2. Then at any section distance x from the
left end A, we have v. = - : } (d )
M.= _ _ ,
V~ = wl
2 - wx ' i while for a section -pq between C and B ,
2

(c)
M"' = wlx _ wx
2
__,
wl
2 2 J 2
(e}
(see eqs. l d), p 98) F th fi
shear fore~ V h th rom_ e rst of t hese expressions, we see that the M:a=
_ u:l/ 2 at x :._ las f ~ !:'°sitive value wl/ 2 at x = 0 and the negative value
- J

constru ct the shear fo


U1 w1crmore V va. r .
d' ' z . ries mearly with x . Hence we may From the first of expressio ns (d ), we construct ilie s ~ l lt hne ac m Fig.
rce . 1agram 10 F' 5 b . . 5 . llb and from the first of expressions (e}, the str!ligflt line rt> This
straight lixic ab B.." sho wn. ig . .10 simply by drawmg the
completes the shear force diagram a.s shown . From the se<'ond of e.~pres--
Ij ' From
.
vanes qua<lra.ti~ally with x
. . ,
.
thP- scc<>nd 0 f expressions \
~)
} (c 'we see that the bending moment M.
so t 1at the . .
sions (d), we const,rut· t t he parabola <J'c' io f'i&. !>-1 k and fN.m the ~,oad
of expressions (e) , the straight line c'b'. T his rompletos th~ bending mo-
I ta ordnu,te 1s zero for . _ corresponding curve 1s a parabola.
. . z - 0 and a l f ment diagiam a.s shown . Siuc.e tht>re is no :1,bruµL ,.hango in t.hc sht1M force
0 rd1 nat<• at z = l/ 2, which i so or x .= l . It has its maximum
2 tlt section C, t,he re is correspond ingl_v no d~s.t.-vntiuuity in slo~ dMJd.:& Qi
through the poin1.s a' c' b' · s Flt~ a,~ .. = wl /8. Thus drawing a parabola
. J ' in • ,g. 5 10
' . . the bending m oment c urve at this ~ xtfo.n. 1'bll8 tn 1-'i.g. 5- l l c , ilie ~ ght
moment d .iwam a& i,hown A . ·. c, we obtain the complete bendwg
· gam 1t ·t1 line c' b' is tangent to the parabc,ll~ o'c' a~ c' .
i ' wi be noted that the maximum
1'
l J

~
.....-c:c~"'S I N Bf, <\. l'l{S :

H 108
STR,:,c.., "'
BENDI NG MOM:mn D L\OllAMI!

I t will be sh own in t he ne-xt artirle that we may expect the highest


100

bendi ng stresses in _n beam at lhe !¥.!Ct ion wb.e~e the bend.i~g m oment 11'
. oally a. mrun.mum : t he rt>fore , the
nuroeri . loc'lt1cn
. of suc.h sections I.B of
pract ical importan ce . They a re mo~ eas1 1
y iden tified: by a change in @ign of
t he shear force wh ic h can be CMily de tected from a study of the sht>ar force
diagram . H owever , ther e are ~ any c,tbe r 1™:ful and eignificnnt appli.e&ttons
of both shear fo r ce an d beniliog momen t d ia grams , a.s will be seen l.a~r

EXAMPLE I . Construct shear force and bending moment a~ f.ar the cantile-
ver beam loaded as ahown in Fig. 5.13&.
I soLtJTION. In the region O < z < a, we
p
",J~ -:-rr.'-;7"7_'7"TI111-"TT)
, have
(h)
V. - -P; M. - -Pz.
In the region a < % < l, we have p
I

( C)
b' V ~ - - P + P - O, } (i) TI
I
t
M. - -P% + P(:t - a) - -Pa. I I
Frn. 5.11 FIG. 5.12 The corresponding diagrams are llhown in
Figs. 5.13b and c. (b )
ZXAKPL& 2. Conatruct ahN.r fon,e and
We no t.e in Fig. 5.llc, t hat the numerically largest bending moment bending moment diagrama for the ai.mply
Jf 11 = - 3u:f!/8 occ urs a t the built-in end of the beam, but that there aupported beam with overhang, loaded u
ap~srs to be no corresponding change of sign in the shear force at this abown in Fig. 6.14&.
section . T o clarify this situation, we must consider that portion of the eoL'OTION. The reactions a.re
beam which is encased in the wa.11 (see Fig. 5.1 2) . Assuming that the beam R,. - 0, R• - tol.
bears on the wall only at points Band D , we will have reactions Between A &nd C, we have
Fm, 5..1 3
Rs = w/,
2
+ 38awl2 and Rn= 3wl2
__ , V. - 0, M. - 0, (j)
8a where :t ia meuured to the right from A. Besween C and B, • • ha~
directed as shown in Fig. 5.12a. Theo for any cross-section mn between D
and B, we ha.v~ v. - -to:,

where :t ia measured to t.ne right from C. Bet ween B &.od D, ~ haTe

=+Ro= +3wl2, ]

.
Vz

Al, - -Rox~ -~:•, (g)


V. - w(~ - z} M. - -{G -~)'·
where z i8 measured to the right from 8 . The com!t!pood.ing ~ ~ mom~nt
where x 11:l meas ured to t he le! f . diagrams are shown in F igs. 5.14b and c.
, · t rom D b
WP. may now construct d' .' as 8 own . From these expressions, EUMPLE 3 Construct sh~r fo.rC(' and bendi°' ruoruen, dta«ra.m& for the. wnpi.,..
· ·· iagrams show1 8h eupported ~m with overhanging end.~ 10&4ied aa sbo..-.n i?
l1g, 5...lS&. ,
vanation aln ng the 1•ncased . ng ear force and bending moment
Rho; 1 .. Rt • .P(l +
80LUT10N . From equilibrium of the t'n~,ro b~•m• Y ~. fiad R._ 'lo.If}
These diagrams
from n f .
that thPoerrt,_on BD of the beam (FigR. 5.12b and c) .
e 1s a cha ·
nge In the sign of the shear force
directed u shown. The ahe.a.r force in ~ch ovt"r ~ " CODJN,anl ! ;;,'•,0
"t'1;t f>~ii6b..
.
1c
ega IVe to positiv~ at B • . h
a negat · · wnic con.fir h The ahear force in the middle portio.u ia con.a~'- &nd ~ to ~
· ·, ive maximum of Mz at B. r.rui t e condition of a minimum, - +2PQ/ l. The corrC'lponding ahear f0tte d ~ 111 sl»n Ill •· 16·
I
110 Of':~(OINO MO~l!l'T OlAOllAKS
11 I

hi ~ :l·d
1I
I
I
I
I
, .
(a) :
I
/

I
I
I
6 • CoMt,ruot shear force and bending mOlJ14!.nt diapa,ma rm- tu
beam AB loaded M sho-.rn in Fig. E cm ~ lOJ .

t,eam .AB loaded M shown in F1g. F on page 103.


(I . CoMtruot, t.o ~ Ale, the com.pl~ &bear !om, and ~ fDOlll;1!Dt ~
for the beam shown in Fig. Hon page 103.
i
.,i.mp Y ~
7. CoMt.ruct shear force ~d hen.ding mo~t d ~ fat the ~m~ m .......-....i
Y p.,.... ....,...

9, A canklever_beam ?ui\t,..in at B ~ries a bracket a.t. A. , to the me md d " bich

~ IIII@ii II /Yi" : 6 vertical load P 1.1 applied &8


shown in Fig. C . Couatruc.t lbeat- force &Dd ~ o

00• moment diagJ'alDJI for the beam A 8 , if ll • l / 3.


p : {/
,~e,
I ~ J
,
I
-Om[I (bl ---- ~---- A
I I A l >'- ~
I T I I I I
18 :; t

I Ji M~
' ,, ~
l (b) I I I I clPa I iJ
I I J 1·
I J I l
Id
i~~ Pl=o~ L l "' j_

ry
1 cl J l } l
0 o d
Fro. C ~ a. D
bPa ( c) IO. A simply supported beam 1B is acted upon hr two exte:nany app&d ~
(C) of moments Mand 2M as shown an Fig. D . Neglecting the diZJ:ribated wagb;i <1 t,lM
Fm. 5.15 beam itaelf, construct a complete bending momen t di.agn.m far t:-.e ~
Flo. 5.14
U. A lumberjack weighing I 00 kg stands at Lhe middle oi a bn:1.1 ~ ~ m
Since the beam is subjected only to concentrated forces, the bending moment diameter and 480cm long. The specific gravity of the 101 i50..50. Whst ra. oe be:n:fi:rg
must vary line&rly between points of application of tranaverse loada. The bending moment at the mid-point of the log? A.111. M - 6 .000 q-at'~
+ 12. Referring to the preceding problem, LB1me that ~wo mm each ~ - ltJU
~ t ; ave the. Jett . support is - Pa, while that over the right support ia Pa.
lr.!st.&nd at the third-points of the floating IOI(, i.e., 160 cm from the ~wo ecd5. 'C'cdu-
~ ~ pomui m Ytg. 5.15c and drawing the straight lines ob be and cd we
obta.m 'the complete bending moment diagr&m for the beam u sho,;.n. ' ' theee conditions, construct a bending moment diagr&m for the q ~ bd c.he
bending moment at the middle. Ana . .tll. - + -4 ,000 kg-an.
IS. Conatruct shear force and bendinc moment. diagnm& fur thil m:::,?}y mp-
r•oBLl:1181:T 5.2 ported beam canying a t.riangu]arly d.urtributed 106d u shown i:n F~. S..tk.
1
r,- :L.~~ F~ IC&le
A ' t.be complete shear force and bending moment ........,......,
..,..1 .-. ~ m lg. OD P8ge 102.
~;·--·
5.3 Bending Stre&se& in Beams
for t h e : ~ ~ ~~
I.~
fort.be ~ . ~~~
on pace 102. .a,..
=~
comp.lew, shear force and bending moment diagram,

ehear force and bending moment diagraim


Let us consider the beam AB traru;ve.rse!y loa.dcl ~ 5h.o '-'"'D lO F"1g 5 t6
together with its shear force and ben<ii ng momt>n t diagr.-.nli. We ooh, Wtit
the middle portion CD of t he beam i.:; fret} from ::.hear ( ~ :i..."hl uu:. 1U

3,600 k,! ( L0Ul 1


..£:i i irl( J
1 kg
2,000 kg (total) i
2,500 kg
oonding moment M. = Pa is uniform bet wl"en C and l> Tbb C\"lliUt:.'11!. is
ca.lled pure bending.
To investigate the s lat.e of intRrnal stress pn...xfu~ b~ pLre t-.mding, w-e
1· I
,..._,eo--L-1BO-L 180
j Jk
t 1111£;; must examine the deforma tio n whil·h takes µl"1 Ct" w1thm ,he:- rr.::.i~--n:i l. In
doing, we shall assume, tha t tht> bea m i.s pn.:1.mJLW ~J d \.l\t it ~ :rn ~®
"'i
plane of symmetr y whi.c h we 1..ake ~s th~ .ry-pLt.0•' \\' ~ ('ll :~.- :\pp.11~'1 !,.ad.s
F10. A alSt> act in s uch n plane of symme t ry . bcnd.101l '" U 1••k <' fl,.:ll-~ \.,n !) 10 t..~ t
'· Con.i.ruci, to aclle . Fm. B PIH-ue. We 1is::;ume furtht'r thut rhe tn11o t1:n,11 ~-t h.~'tn•' ~nt,H,i> .;.ml th tt it
for the be&rn abow • ' the complete lhear f
s. For t-be beam . tn Fie- B above.
0
orce and bending moment dia,ralJlll obey:,; Hoo k<' \, law , tht' ll\ odu lus <'f ,·l:t~th·I!., t1 1 !,· 1~-tlt 'ft t,., rn !-{ rile -..' itnt- ~
11
P - 500 k 1.11 Fi&, D on pap 102 t.h8. Lhat in co01prt•.ssion . T)w11 si1we th,- l-.~Lidt n~ flh m~ nt ts \., 11ii, mi IJ...t\~~ • (
to scale, :;mp~e: ~~;111· l '"'°2'°cJt1, ~ • ~llowi.ng _numerical data arc, liven: · reusonublc w lls::lUt\\tl t ht\ l tnc
· 1~
an d D , 1t
. _ L, tt)ll<:l mg u'1 •tll•m~twn
• ·
\\ !.It~...., O('
. ~orce Mid bendinc . C lrl. ulllll& thette data conatrUIIL,
.moment diaara,na for the beam.
j I

t.!. -.;:.__ ,
11111 ►
BTJlS881'B JN sllAUS : I
B ENOINO 8 T'l.E88 E8 IN BEAMS
112 113
uniform; i.e., the portion CD of
p beam will take the form of n cir the before bcndin~. From trus com1tniction , we 11ee thn.t t.he segment c.d 'or 11.ny
arc as shown m . F 1g.
" 5 . 17 . In thiCt(l,Q fiber at the distan ce Y from t he neutral :surf ace elongates b-., tbe amo unt
. s d~r d'd = yd8. Sin ce it'l original length was cd' = dx , t b P ~co rresponding
!!...-x formed con fi gurat1on, each c
Af~::::~:;::::=~::==-::-~i5-~7-'-::
: -=~
01
section, originally plane, is assuroSS- strain is
JC --l i.-~
A I' to remam . p lane an d nonnal to llle<j th ydO
e,. = dz = y/ p. (a )
" 1
1
longitudinal fibers of the bearn. • e
1 , As a result of the defonnat·
: l If 8 fiber on the co ncave side of t he neutra l surface iB coruride~, the
shown in Fig. 5.17, fibers on the cton
JTIDl
p,1$?)) , vex
h b
side of _t e earn are elongated
on. distance y will be negative and the strain is a lso nega tive . Th U3 ail fibers
1 1 on the convex side of the neu t ral surface are in tension while those on the
ILm} p slightly while those on the conca concave side arc in compression . Ex periments indicate that the lateral
1 : LJ2y side are shortened slightly. Sorn: deformation of fibers is the same as in simple tension an d compresmon.
' I I where in between the top and hot- Thus the stress in each fiber will be proportio nal to its longitJ d.inal strn.in,
±_L6]1 1 111 rn [hJ_± ~i:'rs :~~;::;a:~•:,~h:~~r:
1111
01

length. This is called the neutral


<rz = Ez E
E
= -y.
p
Fm. 5.16
&Urface. The intersection of this This shows that the fiber stresses rrz due to pure bending vary linearly wiili
neu tral surface with the axial plane of symmetry is called the neutral a.ti, distance y from t.he neutral surface, so long as the material follows H ook.e's
of tk beam. Its intersection with the plane of any cross-section is called law. Such stress distribution over the depth of the beam is shown in Fig.
the neutral axis of that section. After deformation, the planeM of two adjacent 5.18. The position of the neutral axis Oz of the cross-section mav now be
cross-eections mn and pq intersect at 0 . We denote the angle between these fou.n d from the condition that these stresses distribu ted over the section
planes by d8 and note that d8 = dx/ p where 1/p is the curvature of the must give rise t.o a resisting couple M .
neutral axis of the beam.
In Fig. 5.17, we now draw through point b on the neutral axis, a line p'q'
parallel to mn and indicating the original orientation of the cross-section pq

M y J'
( 0) (b)

C F10. 5.18
'
Let dA denote an element of area of t he cross-sect ion at the distanee ti
from the neutral axis (Fig. 5.18) . Then the element of fan~ on this a.reA
is uadA . Using eq. (5.3}, t.his becomes

"• dA = -Ep 11 dA . (b)


F10. 5.17
•careful •tnuo mell.lllre!nen .
of pure bending ta 10 t.be laboratory Now since there mUBt be no resultan t nonnal fo re€ N. on t.be section (pure
· confirm thia aaeumption for the caall
»EAJ'S= I BEN Dl :-JG STR ESSES IN B EA.MS 11 5
STRESSES J?i
· area of t he section •ttUst
rn
as
114 he entire me fihf>rs in tC'n sion and com pression, respectively , 1,y c1 and C1
,dA over t to ex tre .
jnt,egral of " shown in Fig . 5.18, we ob t am from eq . (5 .5)
bending), tne
vanish, i.e.,
§.! p A
ydA == O.
(c) U m i n=
M<>J
1 - (5.5a )

from this that If the cross-sec tio n is symme trical with respect to its cen t roide.1 a.xis ,
0 w e conclu de _ ,.. = c and the extreme fiber stresses in tension a nd com pression
. El ....,
Smee P .,... '
f y dA == Ay, == 0, (d)
C1 - v-,
are equal.
Introducing the notations
" . and y. is the distance frorn th I
ct1onal area d e I
A is the total cross-se . e A .,,t. O we conclu e that Ye - 0 - = Z1 Z2 (g)
w here 'd Finally sine ' . . - • C1 ' C2
utral ax.is t.o its centroi · ' . -nn..'lses through its centroid.
ne . ,J the cro"s-section ,-- .
1 axis called the section moduli, eqs. (5.5a) can a lso be expressed in the fonn
Thus the neutral a.xis O ~ f ,1 A about the neu tra
O
of the
f h lemental ore~ "~ the
The moment o t e e f these elemental moments over M
. . dM _ ~A The sum o . M on that =-· 11m i n = (5 .5b )
section is - Y" . · oduce the bending moment
q
· ma..x Z1
total area of the section must pr
I n the case of a rectangular cross-section of width band dep th h (F ig . 5. 19a),
section. Thus
M = !" y11,. dA =; i y2 dA. (e)
C1 = C2 = h / 2 ,
I = bha, Z1 = Z~ = -bh ·
12 - 6
2

......, • h' SSJ·on in which a summation is made of each ele-


The inte~-1 m t 1s expre ' . .
.plied by the square of its distance from the neutral
. . For a cir~ular secti on of rliameter d (Fig. 5. 19b), c 1 = •~ = d l2,
ment of area dA mu )ti,
• ca11 ed the moment of
· ) , ts
· (z-axis inertia of. the cross-scct1onal area with ~J• rrr:P
axis .
respect to that ax.is. • Introducing the notnt.10n I = 64 I z I = z 2 = 32 .

I= i 2
y dA
(f)
f---t> --j
t

I
for this quantity, eq. (e) may be written in the form /1~2
Cl
I M (5.4)
I

P = E( 1112 c,
..,__
This shows us that the curvature 1/ p of t:1e a.xis of the beam is proportional * ( Cl
<al (bl
to the bending moment M and inversely proportional to the quantity El,
called the flexural riguiity of the beam. We sec that this flexural rigidity FIG. 5. 19
r~flects both t he stiffness of the material as measured by E and the propor-
t10,!1s of _the .cros.,;;:-sectional area as measured Ly / . In the case of the trapezoidal sec tion shown in Pig. 5. 19:1.!, c1 < c, which
Mbst1 tutmg the value of l / p from eq . (5 .4 ) into eq. (5 _3 ) above, we obtain makes Z1 > Z2. Thus if the beam is be nt concave upws rds, the m ax im.um
compressive stress in the fibers of the top face will be greater tha n the max~
My (5.5) mum tens ile stress in the fibers of the bo ttom face In the cnst> of a beam
"~= -I ·
made of cas t-iron this may be advan tageous, sfa ~ c~~t-iron is ~t ronge r in
It is see n that this bending stress wil1 . 5t compression than it is in tension . Va r ious ~ha pes of l'ross-sect ion5 for beams
removed from the neutral su f b~ a maximum in those fibers furthe
bearn an d compressiun ou th r ace, tension on tile convex lower face o f the will be disous8ed in more d e tail in the next ~rtid e.
e concave up f . es All of the foregoing theory hos been de vt>lopcd fo r t h~ ca!-P of pure
•See Appendix B, p. 346 _ per ace . D e noting the d1stanc
bending, i .e ., constant bending m umen t a long the le ngth of tb c beam . In
p
I 8Tllaa8- 1K •:LUIS 117
-no~ TN s&AMB: :BENDI NG
sTR ~
I 16 ss-sect ion is zero and the n
I 2. Calcul ate the IIlA~ ~~ hendin g streu that ~ be induced
m•
t each cro produc eu..J . I n t h c ~ase ofortn II a
.A)IJ'Lfl
0.8 mm ,f it U1 wound o n a drum of diamet er D - ~
= cm..
the shear force a nl, ones ~ wire of diamet er d
!luch cased, to bending arc the o ~he bendin g momC' hnt . varies
frolll ori. For this
11 .01,VTJ OH- calculs tioo, we wie eq . (5.3). ThWI
ue be m where . Goe
st.resses
be ling of a a . hear torce at cac cross-s ection and E
.
uniform m h there ed
.
-section to snot er, induc
is a s . . I • Th d f
·n the mate na• . e e orrnati ot1 11&- q• = ,;v 21 ( 101'
= 25 _04 X 0 .04 - 3,.33.5 k.g/cm 1 .

cross 1 f th .
I
sheari.ng stresses a.re a s~ sses cs.uses warpm g o e variou s crass meel, 80 tha-t the
st remain Pl · I ust be 8,88umed that this does not exceed the yield pomt o( t.he
80Ciated with these shearin g re. s before bendin g do not I b llne . .
cross-s ection . t~rial behave s ela.stic ally.
l
sections so that pane . - th problE>m but more e da orate anal Ysrs JD £XA.MP LE 3. A simple beam 6 '."° long 18
to carry 11 un ifo rm load of 700 q pcr_m
. . omphco .tes
Th 18 e ' 1 (S e Dough a fir ~ 1r1th
after bending . c -1 to bendin g as calcu ate from eq . .5) includi ng the weight of the be~m 1tsel~. (a ) Select a suitabl
a.I stre§eS oue ' . ratio h/ b = 1.2 and an al lowable .-orkm g stre:!111,
h
shows that t e norm h
~ sence of these shearm g stresse s. Thus it .' tangula r section of depth-W 1dth
I ~ n to carry the
. rec_ 8 ,. kg/cm•. (b) Select a suitabl e ~el beam of standa rd
1~
·1 altered by t e pre pure bendin g for c alcula ting the norm
are not great-Y h
·ustifisble to use thef t eory
. _
O
f
niform bendin
.
g an
d • ·
it 1s a ccepte
d , a1
practic e to
::me load if the allowa ble stress is '7'• = 1.260 kg / cm•
soLUTI~N- Referri ng to Fig. 5.10, we see that, for a urufarm
.
ly loaded aimpJ,e
J
stresses m the case o non u t at the middle is
bMm, the maxim um bendin g momen
do so. 2 x 6 x 6 -
EX.UlPL E I. A locomotive axle with outboa rd ~ant.nhgs A a.~d B wbeill bed_
subjected
n ing str"""o Mmax = gwl = 700 S X 100 = 31.:>,000 li:g<m- (i)
" in
~sho~n
to transverse Ioad.s a., . me
Fig · 5 ·20 · Determ , um
ed maxun """
m:diam e.ter = 25 cm , and P = 13,00<i
induced in the axle ii a =33.75 cm, l =14?.Sc
A/ 1.2, t.he aedion
q . Find, also, the deflection oat the nud-po mt of the axle. Alllo, for a rectang ular cross-s ection of depth h and widt.h b =
modulu s is
p p bJ,,? h.J
Z = - = -·
6 7.2

Then with u. = u.,, =84 kg/cm•, eq. (5.5b) become s


.J,
2 -·
I
84
= 315,000 X 7.2
8- - i p bl
- I
I

p from which

Fm. 5.20 h =3 ✓ 315,000 X 7.2 = 3 ✓ ·> 7 UOO = 30 cm


a• - .
_The bending momen t between whet>! loads at D and Eis M =Pa=
SOLUTIO N .
~~ t · mod u1us JS
13.000 x 33.7.J ltg--cm · The ....... . .
z = 1rd1/32. Thus eq. (5.5) gives Thus the wood be.am should h11.ve a 25-<:m x 30-cm section .
c ion
x 32 For the steel I-beam , eq. (5.5b) become s
u _ M 13.000 x 33 .7.j
m u: - Z = re X 25J = 286 kg/cm~,
1,2602 = 3 , 15,000
t . f h .
From cq. (5.4) , the radiu.s of cu rv11. ure o t e c-1rcular arc DCE I in pure bending, 13 t.urning f.l) Ta.b:i! B .3 o.J
from which the require d section modulu s Z = 250 en,, No w
p = _El = 21 ( 10 16 X ~ X (25)4 Append ix B, p. 362 we fin<l that an ISWB 200 st.and& rd r ~ h:U l & aet't.Nll.
.11 D,000 x 33 . 75 x 64=91, 600 cm . modulu s Z = 262.5 cml ao<l will be satilO!!a ctory .
Now, referring to Fig 5 20, we see that t' d fi .
. . ne e ection CoC = o is
6 = P(1 (h) PllOBL l:M8J:'T ~.3
where 8 = l/ 2p =
OOOBJ
0
. - cos 8),
~
J - fJ1/ 2. Then .(h 5 radian. For such
eq. ) IJccom<•s a small nngle, we may take cos O- cm ls ~nt by eouph>s
I. A thin steel rule hav~ a croSS-S t'cti,m U.l"l6~5 c m , 2. 50
0 - p/)1 p/2 /2 ends so that a length l = ..!3 cm of the c-1rcul-Ar ~ rub tends & ~ tr.Ll
= 8- = 147 applied at its
- 2 = o;,-:;
p• IJ -'i ){ 117 ..',
angle 8 = 60°. Cnlcul ste the ma.ximum H~ss mduce.. i in the rule and the ma,gn.itu<ie
•These sbearii,.g st P >< !iJ ,bUtJ = 0.01'16 cm.
g mome nt.\! . .A,i-,, a -..2.7~ ~ cm• . .\J - ~-4<> k:g-<"m
rel!Sts 1.<re d" of the bendin
. l<>elttte,•d in A
- rt . 5.5.
►i sTRESS"""
"'" IN ouMB : I
JJ.1:: NOINO ~'TIU:..~ES lN BF-AMS

I
I 118

., A wood beam 1.8111 Jo_ng is lliinP t


rted at it., ends, h&a a cross-Se<!tio .
1 SUPJ>OJu distributed load of in tensity w , , n, 1-l\-,~
• t •> {l r,., UP• The aUowable bending strecJ.~ in ten11on aol; compr~;;r. rc.;r...etil"e.! Y, a,e
I llJ

,.. n. ·~ • ullu<ltnl J
th """nding stress at a pom _Q - c m abov e "'\J\J • d =- I . 120 '·a:g,·cm ' • Fi10 d .uie
1.. ' r-
Mfe v&lue c:,f lhe mt.eruuty w of I.he
tride hv 60 cm deep and cMTl - th "' •1.400 ks/cm &n "'
· 1"'-!culs.t.e the ~ left support,
A 10
· 1 kg / cn1 !_ e d~tributed load on the overhang. Ana. w • l ,'4a2 lr~ •m.
'm over the full span . va e
n.,. " -
·
it« , d "" m from span l = J.li m is. to. carry a unifo truJ
bottom oI the hesm an vv c
~ J beam b.avLDg a
.
/m. The cross-section 1s to be rectan Y
l. A simply suppo~
distributed load of int.enSJty 1D •
2
t 600 k
th ~towable bending stress in tension or gular
with -depth .h 1.nd tridt.b 0."' hl : t.b ~uired depth h for the cross-section ? ~tn- y ;1111I11g
~ kg/ m

pressio.D is 11 • = S-½ kg_,c m·. what L!I e 71.!,

,\ = 33. ·, <;-"°· • • 2 4 long and bas a 30 cm x 2.'i


4. A railroad t te (F'ig. Mis : mtal The maximum loads transmitted to th ~-
cm rectangular er 0 L120-l-,2o_J .--90-....60 - - - 1ao- 120--

section with the .«J cm faces homo: an.d the ballast is ~urned to exert a uuifore i1 e FIG, E F1c.. F
by the raib; s re P_ : 1:i..dooo kthg eacbottom of the tie as shown. Calculate the maximmu rnY 9. What is the required section modulus for t he beam s.bown in Fig. Fathe &11~-
distributed reactiYe 1OA. on e
, 1 'i cm e.nd a = 48 .75 cm ,4 ns. O'mu ==90 kg/ crn ,
bending . st.re!iS in •'-. ue 1•r l = 1.....
. tJJe able bending stress is <1,. = 84 kg/cm 1 . Ans , Z = 130 ems .
10. A temporary dam in a water channel is formed by setting Vt>.rtial 7.'iJ( 30-<m
111 per un,r length planks between guide r~ils AA and BB as shown tn Fig. G. ~Ililllg no auyport
at D, calculate the maJOmum bending stres.5 induced m each pl.3.ni ~l-wl ~be ;,;ate
AJd I I I I ' £1} 8
I IIII l,.~rilr-rl-.-1,-,11..,. . 1-~ depth on the left side of the darn is IJ1 mas s how n. An-,, ac.u = 208 q =a
~300-,.. ._300I3 oo___j {)_
120
Fie;. A FIG. D I
~s----± -=P qo
5. A com pound beam made of two bars AC and CD hinged together at C is sup- · --r
ported and loaded as shown in Fig. B. Each portion of the beam is a standard 180 7.5

1S Mi3 ,: , ;ecuon for which the section modulus Z = 7 1.R c m


3
What is the safe I 0
value of the mtensity w of the uniformly distributed load if the allowable working
st.res.s in bE:ndmg i..s r,• = UOOkg 1cm'? An.,. w =112 kg/ 111 .
6. A simply s uppor.ted beam with overhang is loaded as shown in Fig. C. The FIG. G FIG. H
beam is made of ca.st iron &od ha.<! a T-section the flange of which is 1n rn1wide and
t.he stem of which is - ..:;, m deep . Flange and stem a.re ea.ch 2.5c,n thick . Find the safe ll. A retaining wall for an earth emban kment is made •ith !(km-diamc:-.= wood
~atue of I.be load P if the allowable working stresses for cast iron are <11 =4 20kg /cm ' piles backed by 7.5-cm-thick planks as shown in Fig, H. Tht earth fill behind. t.he
JO tens.ton sud <1. = ,ou kg /, m' in compression. Ana. P = 432 kg .
retaining wall is UNmed to exert the pressure diagram show.'?!. whm1 Pt .. 500
kg/1q. m and p 1 - 2,000 kg/aq . m. The plan ks a.re s.ssumed to &el ~ .mupJy
supported beams between piles and the working stress in bendtng for but.h,pa.nb
171 cm.
p 2,500 kg and pileis is a. - 84 kg/cm• What is the proper ~;,acing of the piles" .{n.t.
or tv.·o parallel ~ u ebo7
i - - -J8U-..L-9rl
I
I I
T
2,000 kg ( 10 1a l )

~~ ! :l
12. A 6-toone tractor is to cross a bridge consisting
in Fig. I. What 1s the required section modulus z_for eAeh beam if ~a
1
working stress in bending is u • .. 1,l20k1' cm • HiAi: De6n-a the J)Odlll~n .
&llo;'"~
8

ui:
tractor on the span by ibe distance :r and then nh~ .r ~ u to ms:om.1.te ·
L120 - 180 I. 180__.j bending moment under the rear wheel. Ana. Z • 2~ c.m•.
Fw. C
F1a. D
7. A simply supported bte.m wi b .
beam has a heavy T ~tion the t overha~g ~ loaded a.s shown in Fig. D.
. The
!±-
"~
It
whieb ia I:, u1, deep .F'lange "ndflange of wh ich ta 22.5 c 11 1 wide and t h ehiltcin
.
·1muro tensile ~
stre.ss and the tn" .
stern are eac h ·
10 nn t hick . C alculate t c
• stress due to ben d.mg. An;.
a.XImutn comp ressrve
(u,).._.-= 132 kg ,cm·' ; (o-.)_,, ,.. Ul: ,.
of
rnax..-- r-•C2"
,A, Ji
8 A ·... ..1 ""• kl• ,,,r be - - - -- 480- --· - ~
• . e1:,.1~•Y supported bearn with ov . . . .
Fto, J
cross-sect1v1.1 of the beam it1 an 1 ~ M ~rhaog 1s loaded ag shown 10 Fig. E. T·de
81 F'w , I
2 C 00 channel section with the open
--"' J:l:a,\ll8: l
S'fll£'JS'IS ,..,
VARIOU S 8HAPB8 OP OR<>e&-ftJ'.CTIOR S
120 tilever bee,m of rectang ular c ~ 121
t. t.be dept.b ,. of :" can of constant strength under Uttifo lio11
JS. Prove _th& wit.h ~ to 11t.ta1D 11 ~ Jo compar ing various shapes . of c r ~ctiona, conmder fim the
lhould vary linear1Y J) A,18UDle that. e 6exure formula for PM8Illatic h..l'tli.l) ~~
ree--
t,.angular BeCtion of depth h. and width b ( r 1g. 6.21a) . The BeCtion modulu s i.8
distribut ed load (Fig. ·
applies.
Z _, b: 1
cz ~ Ah, (.a)
crosa-S ections of Beams where A d enotes the cross-se ctional a.rea.. It jg !Ileen t hat the rectang
5.-6 Variou" Shapett of . ula.r
_ . . oss-sect ion become s more and more economie&l wi th in-creaae in
5 _3 it follows that the ~axim um tensile it.a
From the disc~o n 10
~ . proport ional to the distanc e of the.rn or ~:pth h. Howe~~ r there is a. certai~ limit to th is i~c-rea.ae &nd the questio
n
comp~ ve stress m a hen\ t:xis Hence if the materia l has the sa.OSt
remote fibers f ~m th ed neu r&_.,.,; ~ it will be logicAl to choose sha""'
of lateral instab1 hty of the beam a.nses a.s the section become s too narro
The collapse of a beam of very narrow -rectang ular section may be due not
w.
h · tension an comp•.,...,.0 ' d
me vercom ing the strengt h of the materia l but to eidewis ! buckling. •
to
strengt 1:11 . t"-S of 0
croes-ee ction m w c
hi h th centroid is at the middle of the epth of the hea
e be . . IJ\, In the case of a circula r cross-se ction of diamete r d (Fig. 5.21 b),
. h e factor of sa.fety for fi rs m tension and for thn....
In
.
this manner
.
t e sam d l ·
.-.:n be obtained. This is the un er ymg i ea m 'd · t h e choice·- f TtJI 1
1D compt'eSSIOn WU!
. • f . 0
z = 32 = 8 Ad. (b)
sections symme trie&l with respect to the neutral axis or matena .ls such Do
. ld . . . ..., Compa ring circula r and square cross.-sectiong of the sa.me area, the side
structur al steel, which have about the s&m~ yie. pomt m tension and corn. A
ression.. Such crose-sections are shown m Fig. 5.21a, b, and e. If of the square will be h = d~/2, for which eq. (a) gives
the
~ tion is not symmetrical wit.h respect to the neutral axis, for example Z = 0.147
a Ad.
rail section, the material i! frequen tly distribu ted betwee n the head and the
Compa rison of this with (b) shows a square cr088-Section to be mo.re
base eo u to have the centroid near the middle of its height.
econom ical than a circular one .
.for a materia l of low strength in tension and high strengt h in compression
Consid eration of the stress distribu tion along the depth of the CTO!!lfi-
- for example, cast-iron or concrete - the best cross-s ection for a beam is
section (Fig. 5.18) leads to the conclu::iion that for economie&l design mogt
not symmet rical with respect to the neutral axis but is such that the dis- of
the materia l of the beam should be put as fa.r &s possible from the neutral
axis. The theoret ically ideal case for a given cross-se ctional ~ A
a.nd
depth h would be to distribu te each half of the area at a dist.a.nee A/ 2
from
the neutral axis as shown in Fig. 5.22. Then

I =2 X ~2 X (~)'
2
= !4 Ah1
and the section modulu s become s
Flo. 5.21 1
Z -
tandcee ei and .es from_ the neutral axis to the most remote fibers in tension 2 Ah. (c)
an COinpreslon a.re m the same •
;
....... •-... ·
! wey._. .z.z.z.....n. .~.. ,.,...///-✓ /_//_a-r
d. ...:ft,
vr:._wn h'18 ion as the strengt hs of the materuwd
an in eompJ"ellio 1 proport ...
/

ft:
compreesion ia obtained / · n t ma~er equal strengt h in tension an
position or the centroid · exa~ple , with a T-Beetion (Fig. 5.21d), the
the aection by properly ;ro~ct ~on ~Y ~ adjuste d along the height of
I t\-2

In deaigning a beam to :nmg it.a ~nge and web.


strength should be •tisfied in rgo bending, not only the conditi ons
II '
~ I

of the beam. Of two e-~~ t .alao the conditio n of econom y in the weight
of 4 rzzaammvzm_[_
·
181 :J . -~t1o ns ha · h
•t1111 YlDI
'th
t.he conditio n of • ving t e 88.me section modulus, t bat Flo. 5.22 Fla. 5.28
· a .. rength · h the
<>ne WI the mnaller area ia wit same factor of safety, t be
more economical. •For a diacu..io n °of lat.errJ buck.liq of t>e.ma, 80CI Timoebe uko a.nd
B1abuit11, McGraw -Bill Book Co., [nc., N•w York, l\lGt.
~ lfkllN
µ
12'i
. . h unattainable 1ma~ be approached in p .
This ide&l liml~ &lt.ho~de-flange eection with m?Bt of the mate
puttm g part of the
~t~ee b
l'lll 1n ,/
- VARJOU8 81L\.P2 8 or

f.,quating tnis t.o the valur of c, from (~J , Wf' find


l°raAa6 -
~~ f!C'l'I ON, 123

um.DI an J-,ecuon orOut WJ . the necessity of . "lit 61 2


t-0 . mate
lli'... e. 21e) tta1 ·
8&nge., \i-re• " · . . ( ) CAil never be rea 11zed · bu t f b, - 11
·
, or standa ,~
the web, the limiting cond1t1on c
wide-flange 8ettion11, we have
t11 t:XAMP LE2. A beam of sq uare cross--!leetion
1 )( a is bent in the vertica l p lane of
one of its
Z = Ah. 4
als shown in Fi~. 5.24. Show that, for a
3 (d)
diagOD &B
giveft mom ent, the rnu:1m um fiber stre~ s can
be reduce d by cutting off the shaded corners to a

C.omparwn of (d) with (a) shows that &n I-section is more econotornicat t
its ~
depth aa/ ../2 and calcula te the optimu m value of
com-
a.
due SOLUT ION. The rnom<'nt of inertia of the
a recl&QIUlar section of the same depth._ Furth ermo re,. I ~ a'/ and the cor-
respe ct to sidew ise b ~de plete cross-section is 12
flanges, an I-be&m ,rill be more stable with modul us is Z = {'i aJ/ J2 (i,ee
and sectio n rn:kh~ responding llt'ction
than a rectangular section having the same depth Appen dix 8) . The mome nt of inertia of the
In the design and selection of be&~ sections, reference is usually
rnatua.
reduced section will be obt4ined by adding t.o the
~ to
f

a handbook giving various properties such as mome nt of


inertia , ser.t1on moment of inertia of the square ocde about its
h Th s such . diagonal bd that of the two parallelograms cfdh FfG. 5.2-t
modulUSi etc., of standard structural s apes. ese shape 1
arc spec~ · and ged.h about their commoo base dh . Thus
section:s (Fig. 5.21e) and chaM el sections (Fig. 5.2lf)
folio ed a)]'=
accord i'\g to the Indian Stand ard Specif ication s by a
l~tter symbo l I = a•(l - a)'+ 2,{2.aa[a(I - a•( [ - a )' "
beam in millim etres. The letter s ~cd 12 J {2 12 (} T ,,a)
by the depth dimension of the
IS.TB, ISLB, ISMB, ISWB and ISHB , in ascendingym ord er
Is
. m are .
fOT' l -scctio
depth of sectio n The I and the corresponding section modulus becona.es
o f wetght per !meal metre for the same • ettcr
for channe l sections are ISJC, ISLC and ISMC . An I-section of
= -~h/2- = -{2 a'(l - a)I (1 ..z.. 3a) It
symboh Z
a(l - a ) 12 ' ·
20 cm dep~ in t~e ISMB series would be design ated
ISMB 200, th~ weight
Thia section modulus is a maximum for that ,·alue of a which
in Appen dix B-1. maka dZ d.:z • (}_
and .other d:imeru1ons for the section being those listed muimlJ 1!l we_!!On
Making thia differentiation, we find a = % and tt,e corres pon~
1· A prwna · g has a ~rapezoidal croSHection u
. tic beam ·in pu,:e be~din modulu s becomes, from (g),
a.um
mown in .t
stre.ea in ~;3• the top fi~ being m comprea~on. If aJlowable workioi
560 kg/cm• calculalt = VZa' ib 1
t.be ruio ( buea ~ eomp ~oDa.re ,,, • 350kg cm• and t1 c =- (Z)mn l.053
12
0 J b, for maxunum economy.
and c1 den 0 t,e d'11ta nce8 to extreme fibers from the neutral
.&OLl'TtOJf. Let c1 Ibo n c ~ by £-lx.-.,:it 5 per
'centroidal) • . ThUI, by cutting off the corners, the section modulus is i
\ a:na u wn. Then from eq. (5.5&) we have cent and the maxim um bendin g stresses will be reduct> d by th!! pt>n.-eutlg\'
materi al sctuall y in~ ~ the strength \lf ~
To understand how this removal of
e that remo\'al of the small l·orner are&$ !imply reduca w
eection, we must obeerv
M
560 - -c diatance to extreme fibers in a greater propor tion than 1,
rt"<iu~ ~ mOffk•oi of
/ 11
. the section modulu s Z - ! , •. ~ acc:ualty inere.as 1.>d.
inertia of the eection Thus

35() - -c,
M
I ' 5.4
PftO.Bt ..EMSt: T
from which c 1/ cs • 8/ S. Alao c, + h• Hence
Cs • d T -sti:Wn snoa.,.n m
1. A 11imply supported cast-iron ht-am is to have thv inwru 1-0 an.d rompre :tililn _ are
stresse s for cast-iro n in tensi.<
8
c,. -h 5 (e) Fi1. A. If the allowable sectlQ.ll .
!i:i I of the
"' •280ks/crn• and"• •560kg.lc m•, calculatt>thr pn.>~r .stem
c, - -h. th.idtOO
13 '
The diltan
ce 11. t.o the centroid oft.._
'"' trapeaoid ia
13 Ana. t • 5 cm.
2. For the channel beam shown in Fig. 8, it~ de.ired to b&ve
er, - J · 7. Whal .IS ~he p,oper wall
the ratiO oi
thidrncllll ' CO re
tu~:~
IMI
fiber bending atrellllt'S 11 1 :
t, - ~(~ + ~)
i ·
thia condition? A n,. t • 5 cm.
3 b,+b
(111111
12t
~
s. . _Jer e,blte
Fil· C an ow
th pper ft&nge of the bea.tn
per ,ridtb b o ~ in bending will be in th IIOct.i_on 11b
tt t,be ~~e fiber e e tat10 4.3<1~ ~
.. ~
- In 0 ~c,o,
VAIH OUA fJ ilAPl:8 OF C1WSS-S~O NS

a sheet of U mm thick C£1m.1glltedaheet metal bu !.hed j~ons


125

b • ~.6'cff'.l- r-
lit
l~1"D
1
in Fig. G . ALlsum iog tb&t. t.be corrugat.loruJ have a lmi:'rwa.e form, caJ~
the i,ect.ion wodulua per foot of width of ahcet. The 1.4-an chmenmoo u, from out.aide
,1 r- b::-i --._Jts .ur{ace to outlJid e surface. A na . Z =- O.Cr433 cm•Jan.
·125 f IQ s. 5 Shear Stre Hell in Be ndi ng

~~-~_1.JJ ~
.. fn-l I 0--.J
In Art. 5.1 we have see n t hat in the case of a beam bent by t t'ME:e~
loads there is, in gene ral , bot h a bending mo ment Mr and a ahear fo ?"ce V . a t.
2.s each cross-section . In Art . 5.3 we have seen how the bendi::ig mo ment
Fto. B F10. C
fll'l, A represents the resu~tan t o~ a _certain linear distribution of normal su-esees a.
...,.hon moduli Z1 and Z, for the T-section shown . over the cross-section . Similarly, the shear force V, on any CT"JS'Hlect,ion
,.._ i-·'•$e the t;wo """"' Ill '1;1;_
'- ~ .A Z - 625cm•;Z1 • 1,250 cm. 1
•ig.4 must be the resultant of a certain d istri bution of shear s ~ T ove.r w t
W'hea t • ! cm.the ~ ~tion moduli Z1 and Z, for the channel section h section. The question to be considered now is ju.st what tha.i dist.ribuuon
5. ~ , ... z1 cl +88 cml; Zs= 3,472 c:rn.1 . 8 0
11'n ~
~ B when t • 5 an. _.,_,__ '. · · · must be to satisfy equilibrium cond!tio ns of the vviol.18 elements of the-
..-l Calt,uWe the moment of inert,Ja of th_e section shown m Fig. C v.;th r~Jletl
beam.
toe troKhl am parallel to the Ba.nges if b = 5.6 cm. Am. I = 1,5 78 1 lo
We begin with the simplest case of a beam of rec:tangub.r C'r~tion
?.~p,-ismAti.c 1-1' having a eemicircular_eross-section is s~bjected to pu: ~nd.
me in iu i:oogiwdin&l plane of symmetry. Calculate the ratio of the extreme 6~ (Fig. 5.25) and assume that V. is the resultant shear focce on !¼ chosen
. - in bending. Am. t1J t11 = 0.736. cross-section. Dividing the cross-section in to infinitesimal strips p&rallitl ~
L A red&ngl:l!&r wood beam is to be cut from a circular log as shown in Fig D the %-axis, it is reasonable to assume that on any one such striµ. t.he ~
Olkulaie the ratio b/ h i,equired to att&in a beam of maximum strength in ben(lin · stress Twill be uniform across the width b of t-he beam and pa.1·~ 1el co the
Au. b/ A • 1/ ~. g. y-axis as shown. We also know from the discu.ssio n oi Art 2.1 tha1 ~ cll
shear stress on one side of the prismatic ele ment mn mus1, be &ee0m~
by equal shear stresses on ea.ch of the other three sides of lhc eie.ir. ent {;;ee

(i}j
Fig. 5.25). Thus, we must observe at the out.set that t here will be b.?r-..wn£al
,hear atru~ between layers of the beam parallel to the neutrsl phn.e ~ w-eU
as tra~ae aMar atreuu between Cl'O&HleCtioos and tha t a t a.ny P->i.nt in

Fia. D
F10. E F10. F
9
·. The top and bottom ofa lo Of A=- _
aect.ioo ahQIPD in F1g E C g uuwneter d = 30 cm are adzed to form the crost
Ana. Z • 1,875 czn1: · a1culate t.he.aection modulua for the case where a• t5'
lO. Calculate th
Yic. p if 1 e two-eectioo modurI f . -L lfD in
" • 30 an, bi • 2s r.rn or the trapezoidal cross-section ,wo
- 'b, •t 5 cm. Ana. Z 1 -2,714 cm•; Za =- 3,20 7 crnl·

Fto. G

ti r
F ,.., sPta8 : l
81'1lps.S ·" BHU.R STRP'.88E.8 Ul DENUINO 127
126 ·--" are equ&l in magni t ude ..,..
~SU.._,..- . .a.b'
the beam t;he9e complerDentarY t the shear streS! -r mu~t var~~ we go frol8 <1dA =- ~y dA .
· , __ .a. to conclude tha . on.sider the pnI1mat1c element ,tn
at OOOll ,-u.a, U8 for 1f we C f h be ?I\ I\' The euro of theM! forces ove-r the face pn of the bloek wil l be
top to bottom of the l,e&m- . b the free top surface ~ t e

1.,
arn , we
1
--L upper fa~ coincidea. • • t ,_._ The sa,rne conclusion holds for an ~l
wuoae ust vanuw- rf f e ~- My dA .
that Lhe shear gt.re:18 f' rn ·. . .-th
1
the bottom su ace o the i.. I
.•
ment one f,a,ce of w:hie co b mCJdes "' • h uearn ir,
•th " and must vams at y == ...L l/ ·
Wit var)' 'NI • . f . ~,, 2. In the aame manner, the rum of the normal forces over t he face J>in1 of
Thwi the ahear st.reSS ,. in . . t,ion of " with y comes rom examining ~
Tbe by to the true la• o(bvlanka flflpini of the beam between two adJ.a~ the block becomes
.
Cl"OM-RCWlDS """
ol -•--oial oc ,,.-
equilibn~ an e,ffllft-
an~ '"'i:k
h
dist&Me d.t apalrt as s own m r 1g. 5.26a. ""nt
• ......
coincides with the bottom surface of the be e
Tb •J, ,,.
(M + dM)y
I dA . (b )
bottom race bd.i oi uus bl r ra.ce is parallel to the neutral plane andarn
The shear force acting on the upper face of the block is
and i.& free from~ u uppe d b h at
the &rbitrary dist&n~ Yi therefrom and is acte upon Y t e horizontal
. •. a•.. .1,u.~
... :. level . The end faces of the block are rep- Tb dx , (c )
abe&r auea ., exist.mg . ..,.
led b t.bt. shaded portion of the cross-section of the beam as shown in where T is the shear stress at the arbitrary level Yi- T'h£ for~ (a.;, (b ).
~ - 5 _26{ _ These end fares ure acted upon by the normal bending stn--SSell (c), must be in equilibrium; hence

-r:b dx = J'•
.,.,
(M +IdM )y dA - J°'., MyI d.A '
from which

T = dM
dx lb 11" '"
yd.A ,

or, by using eq. (5.1),


, (o l I ( b) ( C)
V {.,
Fto. 5.26 T = lb}.,,, y dA . (5..6)

~. Toe26coomplete element.al block., isolated as a free body, is shown in The integral in this expression is seen to represent the 1tahcal mcJMnl,
11· 5- · There are of course also 8h about the neutral axis of the cross-section, of the s.hsded portion PP"'"'
Pfni but ~·e wifl be . ' ' . ear stresses on the end faces rm and
d~tion . hen thm~reshted only 10 the equilibrium of this block in the x- of the cross-section, i.e., of that portion of the cross~t.1on below (or
' ~ . ese 8 ear stresse11 will t · · f above) the arbitrary lEwel y 1 at which t he shear st~ ~ is required. De-
·equilibrium -.nd are not ·sh no enter m the equat10n o
Before go.i nir 1.. ... r.._ own on the free body . noting this statical moment by Q, eq. (5.6) takes th~ fonn
. - ' un,ger, we may obaerv: t
m the beam has the ~ . . . ea once that if the bending moment VQ (5.7)
':"w-wu.t iona on themagnitude
W'eal d l-..:.\.. at sect Lons -Jb·
mn and m 1n 1, the norm&1
f'. • • ende · T

I ·' · oc) will be 1dl'ntiral. Tur f pn ~~ Ptn, o( the elemental block (Fig.
2
To aee how this shear stress varies with ttw dist.'Ul<'t' lil from the neu\till
Wf' conclude , n or equ1 lt hriu f ·
,,': rn o the block in th" ..r dirrcnon,
. t,illt r • 0. Th · .. · axis, we must now examine tho variat.i<,n of Q wit h 11, .
/ can mdu~ no he l!i Mmply v<'r 1 ·i.;_ h d.
c0 nm·d. er now
· JI .ar !!t..-r~ in th L
the
"t'i'! t e fact that pur" bcn mg Referring to Fig. 5.26b , we see that for !i rt~· t.ang\1lar cross-eoot.ion
. e ~am
more gen• I dA - bdy and c1 = h/ 2, so that
noting by M and M .....tll C&le of· v ·
m1n1, respectively. + d,M the inornent,s a.rying bendmg moment, de·
area dA of the end~n the norm&! force ~t the croas-sections mn and (d )
&oe P'l of the bloeL ~ting to the left on an elernent.61
.. Will be , {rom eq. (5.5),
128
.
This ~
sta
-
ticAI mome nt w1
ST~
---ss J1'l IJ~AMS : I
. be obt.ain ed ns th(' prod uct of 1h
11 , <' s)ill(J
~d
> S H E AR WJ'RE88.E8 lN BllNDI NO

ual to the shear strai n f' mu/G . As long as the ahear force V.
are e:!nt a.long the beam, the warpi ng of all cr~t
ion s is the
remAins
same ;
·~
con-
area
bG-v) eorUI . mm,_ pp', nn' = qq' , etc. Th ug the shear
th.;t : to the longit udina l "Strains in the fi bers and
streffleS r do not
the d.i8tri bution of
trl u al streS&Cs u is the same as in the ca.se
of pure bendi ng.
and the d~nN '
00
_:1 more elabo rate theore tical in vestig at ion of thiB proble m shows that
I(~+
Z2
y,) ,,arping due to shear strain d~s not 5Ubst antial ly
along the beam ,
a.ff~t t he l?ngi t.~d~
provid ed this var1& t.lon
fiber strains, even when V. vanes
the neu tral axis of t.he cross-section . unifor mly distrib uted
. . ontinu ous as in the case of a beam carryi ng a
of rentroid from . 18 C
lo~ s on t he beam , however, the streM
1t~
(5 7) we obtain load- In the case of _conce nt rate~
. a t1on of extern al loads becom es more
Su~tu tjng (d) mto eq . . , distrib ution near pomts of apphc
ud and do not appre ci-
T:: 2~(f - yi2} (5.8)
compl icated , but these irregu laritie
ably affect the over-a ll stress d istribu
s a.re
tion
very
in t
locali
he beam . Thu.s it is wrual ly
d for pure bendi ng in the caae ol
y as shown . ·ustifi able to use flexure formu las derive
This sbo,r;s t.ha.t the shear str~ T varies parabolically with 1 ~on-u niform bendi ng, as we have alread y done in t he prect'< iing articles.
Fig. 5.27b. When y, = ±h/ 2, T = 0, and when Yi
= 0, •ma,. ::::: Vh21st st ress in the web or an I -beam ,
• In discus sing the distri bution of shear
Noting th&t I = bh1/ IZ, we have as for a beam of rectan gular
we can proce ed in about the same mann er
denot e the level at, whieh
3V cros&-section. Refer ring to Fig. 5.29a, let Yi
Tmu = 2 A
1
(5.9) t he diroen!!ions of r.he
,re wish to evalu ate the shear stress r . Then with
d portio n of the er~
section 8.8 shown , the static al mome nt of t he shade
where A is the total area of the cross-section. Thus the maxi
= bh
section about the neutr al axis is
mear st~ (horisont&l or vertical) occurs at the neutralA.axis (y, :~rn)
&ud is 50 per cent larger than the average shear stress V
/

1---,.....
r-b ,.. .....

T.
s
t
---
" ·r- T

•L
wat ~
...1. __
(a)

Flo. 527

Since the shear ow


shear
~~-
etrain ..
,
( b)

=
r
/G
. f
CC!IS r vanes rom to
P_
to bo
must vary 10 a Sllm
F10. 5.28

ttom of the beam, it follows


· ·1
ar way. Thus originally
1 ( 0)
"v2

"lz

Flo. 6.29
( b)

~ C?O!J&.f!ections of the beam


;~mg ca.n be demonstrated b :co~ e warped after bending.
This
5·28 · I! straigh t lines mn ~ nd mg a rubbe r beam as shown in
(h 2 + t (h,2 - Yi ) 2I 2 + Yi
h,) 21 (h2 + i\1) ( "• . )
.:,· Q- b 2- 2 p

~~ be~ding, they will beco : pq areline _scribed on the side of the beam
48 curved d"
8 m ' n ' and p'q' after ben mg
auo-..'tl ln the- fii\ll'e. At th • .
iero ao that the e P<>mta m' , , and eq. (6.7) becomes
~ strain 1s

e~, -¥1•) J
lower surface of u:urve s in
1
11' and p'q' P '. n, q', the shear ing

Tt ri(~ - "l) + ~
between th(. ~ beam alter bend.i mam nonnal to the upper and
(5.7')
nts to these c••-· ng. At the neutral surface I the angles T -
'" re,s &nd th
e normal aections mn and P'1
1111 131
8.BLU.l IITJl-aft r..8 1N BL~IN O
130 t.be ina,ximum shear stres, r.t tbt .
this give! for , e\it
[O&XilDWD shear force V and muimu m bendm,r momen~ M
are
0, 1\1
Taking ~l -
Pl
~ I ![b /'! _ hi') + ~
".,.. • It .i\ 4 4
1
•] ·
(
'fhe p
- · M
V • 2' . - -• • 30 p1.--an
~ .
. th e)
.. Ulll shear stress m e web at t he . then from eqe. (5.9) and (5.5}, ,ve have
JUnct·

r~ (~ _,...)J·
"'J2 the JllllllDl 'on 3V P M 12P
Taking y1 ,=, " '
2 A.., 500 u • .,. - Z• 625 .
aft ... - t:,eoome ! T mu -
,nth Ult u.<'6~ _ !
tting the!!e m&Ximum ~retmel l equal to the given working meaea
,.,.,a - It 2 4 4 &nd 110lving for
(D
:e load p in each ca.ee give11
. . seen that between Yi = 0 and Yi = hi/2 th P 1 ...500r • ,.. 5,500 kg, baaed OD shear 1Jtrea1,
7 1
From eq. _(s, '), ~ca.11y 85
represented by the horizontal 0 ~- e shear
1
m multipliedlllab~ of P 1 . = 52 . lu. = 3,647 .q,baaed on bend ing~
~ -,-snes ~Fig 5 Z9b. The &re& of thish diagra f y th
the dilpm IDth . b. represent:5 the total s ear orce V.. carried b e Thus the bending etreM goveffl8 and the safe load is P - 3,,647
lq.
.. of e we . y t~- l • l.2m&n.d c.arriea &
~ t EXAJIPLE 2. A beam AB supported ~ it.II endA hu a span
i"'. l--•' DtJ. ns show that for the
usual proportions . of stand &rd ueI ction of the
web. \.tll,lCWA o . h fla niform load of intensity w = 30 kg/cm over the full span.. The croa-e
. d WP-sections.' v. = V, 1.e.1 t e nges ·contn bute verY littl · ~ ill a T.«ctio n having the dimensions shown in F'lg. 5.31.
Calculate t.he
!l!COODS an
to the t.ot&I shear force V carried by the cross-section, and
need not ~
muiJDum shear etre88 r, induced in the beam.
consdered. not appli b
In this regard it will only• beh pointed out that eq. (5.7). is ca le
a_ . . __L 12.5
in &tndying shear stresses m t e IJ,l1.nges anyway,,smc e 1t 1s not justifi bl
:f:
to l6mlIIle that the shear stress will be uniform across the
flange. For example , we see at once that for Yi = hi/2, the shear
over the frf£ surfaces ab and cd of the flange must be zero
width b

while
stress T
2.5 T
,,
L:
• 1.. _ th . . , across
e value given by eq. (f) . This non-uniformit
. 1.. · · L
wt: JU!lctIO~ (I(;, 1t LIM

~ tbe width _b of the flange prevails to a considerable extent throughou~ 2.5


flange, and is too complex to be analyzed by eleme ntary methods.
p Fm. 5.31 Flo. 5.3'J

~ - fi07 715 ,- SOLUTION. The maximum shear force occun, on a sect.ion j U!t inside one of
the

=r~ ,supports and baa the value equal to the reaction, i.e.,

l'J- - - --~ B
v.u = wl
2- 1.oookg .

( o) The centroid of the section i'I located by the distance


{bl
25 ( 1.25) + 50 x 10 OS
Pio, 5.30 Yo ,.. 75 = 7. cm.
!:.XJ..vJ>r..t l. A sun I .
aron~ •ted load / Y ~P~~ Wood bea ular cross-section carries The moment of inertia of the cros&-.ec.tion i.s
st l"e86es 111 tension At >te l'tlld-eection m of r
_ ectang
follows:"" .. 70kg/~r ;°mpr% n and i;8
~hown In Fig. 5.30. Allowable working I • 12!i (25 )1 +2.511 7..5)• - 75 (~.58) ' ""2,958 cm(
3
~I I kg/crn'. W~ t ~ar parallel to the grain are givew!IB
80t D1'I or-.. I 3
The er~
· ctional area A and t~e safe value of the load P?
18
(·tio.n ; the
The InAXimum shear stress occurs at the neutral uis o( the cr0i9Hl!
A ""b section modulus Z are with reapeot t.o this u:is is
Bt&tiC&) moment of the shaded area of t.he 8teru,
h ""375 on 2.• z ... bh2
_ Q ... 12.92 x 2.5 x t (12.92) "'"208.66 ems.
6 "'° 1,562.5 ems
r 132

Su~Wung
• . ~quan
sTaSSSJra

,
t_itiea 1nto eq.
,
in &1--
(5.7) we

VQ I , ~ ~ ·66 = 51 kg/cm'.
• - 2958 X 2. 5
-
I
obtain
► SHEAR 8TRF.SSE8 1N BENDING 133

d what is the safe load P th.at can be carried at mid-span? Wba.t is the oor-
r - ll , ita en d'ing maximum flexu re stress? Ana. P = 787.5 kg; <Tm.% ,_94.5 kg/cm'.
~~ simply supported beam of span l -2_. 1m h&.8 the T~tion shown in Fig. B.
·rnum shear stress induced at the n • beam carries a load P :a 500 kg at a point 90 cm from the right-band support..
3_ Ca}culate th~ m~. 5.32) if the total ~he~r force at th: utra! •~
EXA u:·rcular ~t~ rt· of the cross-eect1on 18
111
llf'ct1on i 01
1'!i8 late the maximum shear etreM in the beam. Ana. r-., = 19.6 kg/cm•.
1
I btslJl 0 Cl e mom ent of me II I~ C / \ wood beam 5. 4 mlong i8 simply ~uppo~ with a apan of 3,6 mand an over-
901,(1TJOH. Tb .-r' . . at one end o~ 1.8 rn_The beam c&rnes a uniformly distributed load of interulity
I,• 7 · b&DIS ltg /cm over its entire length. The rectangulac cr068-scction i8 15 cm wide by
~o:in deep. Calculate the maximum shear stre88 in the beam. An.a . 1 _ - 2.25
bout the neutral axis of the shaded semicircular a . q~cin~e cros.wection of a wood beam is an equilater.J tri.a.ngle with sidea of
The st&tical moment • f'l'a 18
~h a s:
rr• .fr 2
- • -r'.
cm. Assuming that the horuontal shear s ~ at t he neutnl surla,c,e
1_en oiformly25distributed
Q • -2 X 3r 3 a.crOl!IB the width of the beam, calculate its magnitude. The
~~ force at the cross-section is V = 2,250 kg. Ana. r_,. -11.1 kg lcm•.
8 A beam of extruded magnesium baa the cruciform crOM-t1eetion 11hown in Fig.
With thetie values. eq · (5·7) becomes C. Calculate the shear stress at the junction ah if the total she&r forte on the aoo-
VQ 4V X 2r' -= ~ ~, aection is V :s 22,500 kg. An•. r = 180 kg/cm'.
r.u • lb = .-r' X 3 X 2r 3A
(g)
where A ., Tr' is the a.rt'& of the circular cross-8t'Ction. 5,6 Stresses in Built-up Beams
Fabricated or built-up beams and girders a.re frequently found in
P.ROBLEMSET 5.5
engineering practice. Several examples of such beams a.re shown in Fig.
rrJ/If the load p on the beam in ~- 5.30 is ~laced by uniformly distributed IOld 5.33. Figure 5.33a represents the cross-section of a simple box beam
ol intmsity wkgfcm and the working s t ~ m shear and bending are as give . built up of four wood planks held together by nails or screws spaced at
KD,mple 1, what is the sale value of w? .-lna. w = 46 kg/cm. DIll intervals along the length of the beam. Figure 5.33b represents the croe&-
~ Referring again to the beam in Fig. 5.30, assume that the load P is replaced section of a wood girder formed simply by bolting t wo rectangular timbers
bJ a uniformly distributed Joad of intensity w kg/an. Then using the same data
pwn in Example_ I, calcula.t.e the critical span length l of the beam below which11
~ ~ stms will govmi &nd above which the bending stress will govern the we
mt.msty ci load "'·· An,. l = 159.1 cm.
• 3. A limply sup~rted beam of span l = 3 m carries a uniferm load of intenaity
"· '!1'e ~ I O n of the beam is a rectangle 20 cm wide X 30 cm deep. The
:::,mu; xv.re ltni.sa due to bending ia 84 lcg/cm 1 • Calculate the maximum shear
-~ • Ill. Tau e 8.4 kg/cm!
'• A la.minat.ed wood bea 18· mad
form I aolid C10el-seef m e up of three 5 x 10-cm planks glued together to
in the glued joint.a is ton;ox 15 cm as shown in Fig. A. The allowable shear stres.'!
7
" z .!ig/cm•.Jf the beam isJ ·Bm long and simpiy supported at
(bl ( C)
(o)
r-1
.
.s-, -1. ----~, 10
Flo. 5.33

2.5
t -tJO together n.nd Fig. 5.33c represents a typical cross..section of a SlW
f girtl~r

-~
·
formed by riveting angle sections and co,•er P\ates to the edges o a weu-
5
7.5 plate.
JO al 1 ted 0 n the as-
5
' _J
--, 2..'>:.. -
The stresses in such built-up beams are usually c cuha. 1...-~ behaves
• · .1
sumption th1\t the parts are r1g1u 1Y comlected so that t e ~m
F 10 . .\
7,5+5+7.
F1<,. n
FIG. C
.I
tih-
ut.ations will then involv e · ( )
,............-- STRE88 E8 IN BUMS (BUlLT ~fJP)
135
134 . d . · a. de .
. fbe coDlP sign•
of lid sectwn- d (b) designing an spacin g the nr stress a.cross the junctioruJ ab and cd of the section , from eq
. (5. 7),
like one : • eolid ~ •;ts
of (a) . In the first step,
of
the
rivet
r:;nnec
h rnu1~
ti~~@
,~
'fhe 8be.. .
the ~ulfi ll the requi ~ allowan ce for the effect ,
Th ole 8 01 is _ VQ _ , ,~05 x 562.5 =
~ . __ ,1 01,Jc.mg
90
to
.d belJJlS are 1Jlll'U• ~tio nal areas. e caIculatio n8 • ala~ T - lb - 19,378 X IO J ...S kg/cm ·

BOli b .~ re(iuoed can ts


best be discus sed in conn ~e%ir._;
ect1on :'ll by each screw is
,ik., Y necting e)eJDeD
. "'% The corresp onding horizo ntal shear force to be transm i tted
c.o desi«D the con e F = i(IO. n )
particular cd!B- the case of a box beam having the croSS--s
u t us consider ~F. . 5.34a. We assume tha_t ~heto beam carry
is c~ion,1
8
ltnp1~ froJD which F 280
dimensions sh.own in ~h _n z ssJ.8 m and that 1t 1s
a V e = -~T =- -= 16 cm.
, ends M t a r - 0
1
erticai 5 X 3.5
8Upported at, its
Built-u p I-beam s are more frequen t ly used in practice
than built-u p
tr:20-i wood beams. Such a beam (F ig.
section, making suitabl e deduct ions
5.35)
in
is
crOBS-
first design
sectiona l
ed
area
as
to
one of solid
allow for the
rJO weakening effect of the rivet holes, a nd using a suitabl e workin g streF.S (1 • •

X
be selecte d so as to develo p the
The size and spacin g of rivets will t hen
us consid er t.he
nomina l.flexu re streng th of the beam. T o illustra te, let
required spacin g of the rivets which connec t t he flange angles :.a the web.

2.5 ~ 5 /.- _./ 5 j....


(b)
(ol
Flo. 5.34

== k
load pat mid-Bpan. 'The allowable working stress in bendin g is "• 70
cm 1• It is required to find the proper spacin g
e of lag screws, each of whic~
wi transmit a shear foroe F = 280 kg.

'.fhe ~oment of inertia of the cross-section with respect to its neutral


Ill! ZZ 18

I_ JO x (20)• 2 r·20 x (2.5)•


- 12 + 12 + 20 X 2.5 X (I 1.25)2] = 19,378 cm4.
FIG. 5.35

and the corresponding section modulus Referr ing to Fig. 5.35, let e denote the spacin g of t hese ri\·ets
and consid er
n at which the bt- nding momen ts are
corresp onding cross-s ections mn and m 1 1
z= £ = !9,378
_ = 1,550 cm3 . + d iffe rence ~JJ in these beadin g
c 12 5 Mand .IU t,.M, respec tively. Due to the
s mn and m1n1 will be un-
Then from eq. (S.5b) momen ts, the norma l stresse s u" on the section
by the shsded
balanc ed and there is a tenden cy for t he fla nge, repre~ented
portion of the cross-section in Fig. 5.35a, to slide with respect. to the web.
rr,, = ~ == _P2_180 streng th of the
Z 4 x 1550 = 70 kg/cmt, This sliding is preven ted by friction a.nd by the :;he1.u-ing
to the differ-
I
rivet. Neglec t ing friction , the fo rce on the ri\-ct beC"onws equal
I
I from which p = 2,410 k
beam·becomes g. Thus the shear f orce at each cross-s ection of thc ence in norma l stress resulta n ts in the fia ngt' a t st"<.'tion
~ mri and m1n1,

This Jifference is
AM [ (a)
2 -_ 12
V:::!: , OH g. .F = - J- jY d.4. '
r-
1
136 ent ol tJie shaded area of the fla
► 8'1'1taea Df BIWim (auwr,.w ) 137
{ . th st,aticS,l molD . . ng~
~ , tM II e
1tnl &Xis of the . Now using the relat1onsh1p dM ; ~
the oeu . dN by ~ l
(5J), and replsCIDi
sectw~·i:
and~ by e, we have
AM • Ve.
""'
aho~\
1', ~
q_ .
No•~ eq. (6.5a) , and not.ma that the muimu.m bendinc moment iii JI • Pl/4,
~

C1aas ..
Mc
T • P x 600 x 20
~ .T,sr - 1,120 tg/cm1 ,
. . (a) and also denoting / y dA by Q ,., We lb)
L.,;;wtiog tJi11 in t-Q• obta· f,olD which the ale load P • H ,%40 ti,. The 0011'aponding lbear !oroe "MlY
Su uew Ve '11 ~tioo in V • tP • 1, 120 q .
F == T Q,. To calculate the avera«e ahear It.re. in the aide plat.ea betnen t, ud e. " m•
(S.10)
6,_ calculate the at.atical moment Q of h&U the croeHeCtionaJ ana in Ytc, 6.3ea,
total shear force that must be carried by each . with reapect to the neutral axis of t.he cro.-eect.ion. Thia becomes
'Ibis repft!!!lell18 ~ . t is in double shear and assuming an ti.vet, au Q • H .Tl x 17.29+ 2.4 x 10 x 15 - l,.130cm•.
Obeen'iog wt ·f nvtehe rivet the force F is readily compu•~.J fowab~
.shell' str'e55 or · ' . . --s::u or The total abear force F to be carried between ~ t.ioll.l through a a.od r.l ran
•~ . . The corresponding spacing e 1s then calculated , a11, no• be found from eq. (5.10) by taking ll • 36 cm, the diat&nc:e behrem cmius ol
pen rivet ~ . frolll . , - bole.. Tbua
eq. (5.10).
~
L- - is made up of two ISLC 250 channels ,.,., 7.120 X 36
1. A boI F • 3B, 134
t;o 40 >( 11-cm -
...:.i. -i.•- with 2-cm-diam
IJCa' "

.--, d .
et.er nve"" llpaK,CU at l 2 ct1 &8""nnec1.,
• • .. _ ~·,.,.,.1
sh ~
x I ,I ~ 6,750 ks.
5.38. Al!o to allow for riveting an pam ting ' ~ccess holes are cut in thOWn_IQ.
"

~ - abo ' The bet.m ha!! a span l = 6 m and 1s to carry a conccntrateJ Bide The web area available to tranamit thie force ia2.4X8• 19.l cm•. Bence the • Yf!nll!
~~ ~ To allowfor the weakening effect of rivet holes, the working ~Clllj _,. ltrell in the aide plate. along be ia
for t:ndm& • eholen 111 ~. c 1,120 kg/cm'. Calculate the safe load P, the ave6 ea. F 6.750
_, svea T in the aide pl&tel along be between access holes, and the shear st':: T.., • A • 19.2 • 560 kg/cm' .
in tile rivda. To calculate the ahear •tress in each rivet, we must fint find the statical IlXlmll!Bl
of one channel eection about the neutral axia of the croa-eecti on. Thia becomm
Q, • .5.12 X 17.2{>- 790cm'.
Then, apin Wling eq. (5.10) and taking 1i • 12 cm, we find
11 12
F - • ~~.;. x 790 - 1,675 kg .

There are two rivet.a each of cro11HeCtional area A • S. 142 cm' to vanamii t.hil
force. Hence the average ahear at.reel in one rivet becoma

1,675 .. 261 kg/an'.


2 X 3.142

8
(o l 5.6
PROBLU U!!T
(bl
l. A built-up beam having the croaHeetion ahown in fil. A couista of two
Fro. 5.36 ISMB 100 steel be&IDtl connected by %--cm-di&met.er ri~ia .pM)ll!'d OD 10-cm
IOtffloW. Befming cent.era along the length of the beam. The beam ia 1.5 ~ lone, limply I l l ~;
moa.eut of men.a of 0:
T&ble B. 4 of _Appendix B, we find that the amaUer
~ n a l are& of cha.nnel ,_bout ,te own centroid&! axi.e is 298.8 cm• and
and uniformly loaded 10 u to develop a maximum bending .,_ ol l,l20q/ttn ·
Calculat.e the average ahear at.reu induced in the rivet. ne&rNt the enda of ~
entire Cl'OIHeeti-on irn.~ ~ 11 • 5.72 an•.Then the moment of inertia of the beam. An.,. T • 347 qjcm•. . i.- b bolta
-awurtng nvt-t holet, ii
I 2. A cantilever beam ia compoeed of two 15)(1.xm tuDben held •~..-d
"~· Y and
and connect.or ringa aa lhown in Fig. B. The bolt holN are 7cm 10 d~!1 loaa
I ,• 2 [ 298.8 +U.12(17,29)--;J + 2.4 X 4-0' 2.4 X 20' eo 1"4 cin••
each connector ring can 111fely tranamit. a foroe of S,000 ks m ahe&r,
'J 12 - 12 • JIU , .,
P • 2,500 kg, what ia t.he required &pacin& of the bolt.? Ana, • • 24 cm.
11

6
Stresse s in Bea ms: II
FIG, B
F,o. c
~a A fth
. ding streSS ~ in the beam o e preceding ProbJ
ulat.e me JDWIDUm hen is 5 cm from the wall. A ns. u == 128 k em,
1eft
~ w•
3. ~ .L., the ~ t bolt on the . Fig 5.33a is made
•'-•t shown m . I
of four 15 x 2.5 g/crn•.
. -crn Wnr.,i -6.l PlaBtic Bendi ng of Beam s
ch of which can safe y transmit a shear for -VI.I assum ptions
._ A box t,eam like """'
c;"g of screws along the length of the beam ce of The preceding theory of bending hrui been develo ped on the
..1. .. 1ra connected by sere~, ea ·1 bendin g remain plane a.fteI
v- ,1.,, mirumum spa .....
V = 'iOO kg. An&. e = 10.95 cm. I that plane cross-sections of a beam before
t25 q . Calculate we; 's law. These a.sum pt ioDB
the m&rlmum sh~.sr foi:r,emad by riveti 11g four ISA 150 JI 5 x 12 mm angles to bending and that the materi al follow s Hooke
the depth of the
s. A bwlt--uP girder JS le te using 2.4-cm~iameter rivets, as •shown in F~he 1g lead to the condition of a linear distrib ution of stresses over
40 x2 xm P & , supported at t he ends , and cames a concenr, t· for many materi als, long
beam. This theory applies with good accura cy
90
edges of a_ ~ 6m ·long, .,.... .;"'ply . . to d I h
..:..Ao,.is
midi™ Calculate the nvet spacing e eve op t e full ft limit. Howev er, fo r the ,
C. '1'L...
rn• g u=< as the deformations remain within the elastic
~ : = ;ha~/ : girder.if rr,. = 1,120
2
and the allowa ble average sh:~- point, it is no longer
kg/cm
bending of a steel beam stressed beyond the yield
. •. JS. T • -- -~20 1.n/cm•. An.,. e = 19.55 cm. stra in in the
St.re!S lD
. t he TIV!'•S · "5 .
300 channel sections connec•i,cu -.J b permissible to assume propor tionali ty between stress and
- box girder consists of two· ISLC t he elBStic range
6' A b
. u: upilt Fi D Th' . d
er
1..__
~ a ~pan of 4-Bn,
Y
longitudinal fibers. Such bending of a steel beam beyond
wo »<Zl1 X l.2-cm flange plates as shown_ in g. . IS gir
and e&rries a concentrawd load P at midspan. The allowable workin
g str~ . of strain is called plastic bendi ng.
To develo p a theory of plastic bending
kxure is r1,. =J,120,kg/cm t &nd the allowable shear stress for the 2 cm-<iiameter
rive:
i! r. ~ 2D kg/cm•.C&lculate the we value of the load P and the proper rivet spacing tT
of steel beams , we begin with a consid era-
t. Nec:Ject rivet holes. Am. P = 17,IH kg; e =-16.3
cm. tion of the stress-strain diagra m for
trTP
structu ral steel as shuwn in Fig. 2.8&,
p. 33. Since t he elastic limit stress a.nd the
i.L r-:-30 r-- ·--1 20 yield stress are close togeth er and since
the plastic strain during yielding may be

r
30 IO
f25 many times greate r than the ehistic str~
before yielding, it is custom ary to ide&fue
this stress-strain diagra m by three straigh t.
L f 25
FIG. 6.1 lines as shown in Fig. 6.1. This dia gl'l}.m
assumes that propor tionali ty between
I.Zr stress and strain holds up to the yield stress CT y . p . and that
for any strain
FIG. D heyond this, the stress remain s consta n t and equal to a,._11 I t is also
FIG. E in tt'nsion and
assumed that the mateir al has the same yield poit1t
7. A wood ..ird 18 · b. tw· · • . . compression .
and ,:,- er u,lt up out ofTh oi_o•cmX 2J-cm timbers connec.ted by bolts
oak blocks ae shown in Fig E al cross-
and is to carry a Now let us consid er a prisma tic beam of arbit rary $ym meuic
: nceotrated load P "' 8 tonn~ at· . e girder has a span of 3 6 m as shown in
st ~id-spa n. For
a~rillnelg ress pa.ro.llel to the """ ;n 1S rl w :c ]O k / · the oak blocks the allowable s~ctio11 subj ected to pure hending in the plane of symme try
will be elastic,
Fig. 6.2. If the bendin g mome nt M is not too lnrgt>, th(' besm
:,S
P... a to the gra· . ...... I
ble shear _stre
and e. Neglect thin Ill ,,. -=16 i,g/cmt. Caicula g cm' and the allowa will be liMar
2J.IJ cm. e shear resistance offe ed bte the proper values for dimenswns a the distrib ution of bending st.rcsses over the depth of the beam
r Y the bolts. Ans. e _, 61 cm; a •
~
M in the most remo te fiber (
HO . ..... ~ 1'h" PLAf flO BmKOUfO CW l l ~
"-~ rn~(l luu- . ld tress- t s repreRCnt-<; thel'Qlt\ ll ~
(Fi&· 6..2a), ~d be t.-~
~ tb•n
•"- v ie 8
1tJ-•e ; reced ing chap t er . c~
1 l)f t.o the yi.-ld poin t f1 ., . p , t.b at1e abovi , the rwutn
neutral axie_••ill disc ~ in_t~o ~--d beyo nd the elasti c range J U ll' ~ icg m ,;ump tt91DOn
. "'--..ding as and tho11e below the neut'_"! ax i,r h,-u~ 1n ~ runcm
east,10 UP' • if the beAD1 111 IJlO t,efore ben d .mg con tin . u sh.9w n an 1 ,~. f!, ~b
•~ . , e:11.h..
.... t,endrog o ~~u .
ons . . uc to ~ti.. T he n since t he sum o f t he in te r nal i'orl".e..<\ ov~r
th- t>ntln • c tc w :,y-tro n mu.st
U ~e h t plane c~ - bave the stram m ea ch fiber ptern. ,
rectue e to I\ coupl e equal tn t he be<nd in~ mome
·~tn
men ui sbo-Wbet din,;
& .
- 'fh\18 .,.
·e a)v.-a)"S f h Iii..-
ual axis. Some o t. e 1.X::ts rno1-e I'{) t. toPo n t .'4 •• ,.,. cc:m:rf od-! t h.at
plane af~r · d~••nce f'r()ltl the ~eut m u., p.A I - u., I> A t -o,
:zn
· pl to 1ts .,,._ _11
tbe neut~ ! ax.is ·1r1J.1 ac:
wre S r& 8
· beyo nd the elasti c limit . H-- ttlott
diagr&Dl, Fig. 6.1, that _th e streSSes in vWev
th er, and
(a ,

we eee from the s ~ nst t and equal t-0 the yield stress "
---•oed
OVOJ..,.. -
fiberS rem~
· co .an
.
tea ID Fig. 6 . Zb The centr al unsh
~ -P • 1h'

h adedb Portio n of t~'


"r,~
(b ,
ronditron is r e ~ b'l the shade. d outer regio .
ns ave ecorne Pl&sti
..,her e A 1, A 1 are the areas below a nd o.bove t he
neutr al a.:ca~ , ':!'.liP,t d r n ly .
be&m is still e!a.sUC Vil l e and y 1 and y, are the dista n ces to the ir centr oids.
c. For lor.au ng U-l! rurutn J
axi8 , we obtai n from eq . (a )

A, =A 1 = 1 A (8.1 1
2
where A = A 1 + A1 is the total area of the CT'09f
Hlect.lon- Thus the ~ UM
(0) ( C)
axie di;id es the total area of the cross -secti on in
to equal
noted that the locat ion of the neutr al axis for pla.eti
h will be .-.-u.
Flo. 6.2 c bend ing ia.. in ~
differ ent from that for elasti c bend ing (seep . 114)
Repl.&cing A 1 and _-t.. by
With further increase in bending, more ard more of the A/2 in eq. (b), we obtai n
inner fibers reach
~yie ld condition until fin&lly the entire beam , with
the excep tion of a very
thin layer &t the neutra l &xiB, becom es plasti c. This
cond ition is represented
in Fig. 6..2c. The corresponding bendi ng mom ent
is calle d the pla,tic For the parti cular case of a recta ngula r Cros&-....4f4?<:
~fo rt.h e section and is denot oo by M ,,. Negl ectin
g strain hardening h./4 and eq. (6.2) becom es
tian , A = bA, !it = ~ -
in the outer fibers, no furthe r increa ae in the bend
ing mom ent can be at-
t&lned . Th.118 the plastic moment M,, repre sents the limit
ing alrength of the
bea.m in bendi ng.

,.,,_, The maxi mum elasti c mom ent M. for the same ~
on is

M, .,,, Az M. =: CT,- . p . z= '7 y . p . ( ~ ) - td)


Y,
( -- )., Thu8 , for the recta ngula r cross -secti on , we have

.v~
the -ratio
,,,,., 3
M. =2
(o l and concl ude that the plasti c mom ent is 50 per cent
( b) g_n,ilt et' th4n e.be mA..u-
mum eluti o mom ent for the same rectang1.1l».r
Fro. 6.3 ~ tn..·•n.
For a circu lar OJ'OS&-80Ct.ion of r&diu.s r, .,t ,. n-1,
To-eva.lu&te lh • .-, • 4r "3..- -...oo
....,,_ •L- . the pla,at,1c moment M r -
eq. 6.2 becom es
-"""" e wmt ill the fully l>la,,tic ." . or a pven cro88 -8eeti on, . ,,A
Fig. 6,.:J111 1 •e
oond.ition every fiber of the beam ~ B ~
(e l
F
1~
8"l"Jll8P8 IN BJU)m: II PLASTIC B END lNO OF B UMS
143
. is If t he load P is no t too la rge, the
The maxi.mum elastic moment for the S&IDe 88Ct.i0D
entire beam will be elastic and we
M. = Uy . p . z= Uy . p . ( - ; } (f) ha vc t he d e flee tio n c urve AC B us
Qhown in Fig. 6 .4n . Increasing the
load gradually , a value will soon b<'
-------- ------
-f - 1- II
Thus, in this case, the rat io
reached for which t h e max imum C
M,, = 16 - l.70, xtreme fiber stress at C will be equal (a l
M. 31r et0 vy
- . p . although the beam is s till
• t · - n to be 70 per cent greater tha.n the completely elastic . This load , de- A B
from which the pl.&stic momen 1s """'
noted by P K, re presents t he elastic __._.__ .......
maximum el&stic moment.
limit load for t he beam . As P is
If we write eq . (6.2) in the form C (c I
increased beyond this value , the
M,, I
(g) fibE'rs in the neighborhood of point C F10. 6.-l
U y .p . = I
2 .A.(y1 + Ys) (maximum bending mome nt ) begin
to yield and this yielding will progress until , when the bend ing morut::n t at C
reaches the value MP• a plastic hinge will be formed a t C . :\l ost of the beam
we see then the quantity ½A (y1 + ys) may be_ consi~ered as. a section
will still be elastic but with the plastic hinge at C, 1t continues to deflect
modulus for the case of plastic bending. Denoting this quantity by z,, without furt.her increase in the load . The value of P which produces t his
we have
condition is called the limit load and i:; denoted hy P L· T o calculate this
z. = 2l A(y1 + y,). (6.3) hmit load, we have
PL l
2·2= M,.,
For lh.e cue of a rolled atcel I-section ISWB 300 conforming to Indian
from which P L = 4M ,./ l. A safe working load taken ll5 P., = P • / n
Standard Specincations, the plastic section modulus is will have a factor of safety n based on the elastic limit load . .'\ safe wor-.k.ing
load taken as P,. = P d n 1 will have a factor of safety nt bs.sed on t.he
plastic limit load. Using this second procedure for selecting t he allowable
z~ = 731.2 cm•, working load represents the idea of limit design.
EXAMPLE 1. A stepped steel shaft of circular cross-sectio n is bu.Lltrin at C and
loaded at the free end A as shown in Fig. 6 .5. Calculate the ratio of duune\ef'S
while the elastic seetion modulus is Z = 654.8 cm•. For such a section the di : di in order that plastic hinge conditions will develop sunu lt.sneou.sly at B and. C.
ratio

M. Z,, 731.2 C
M. = Z = 651.8 ,- 1.1 2 8 JJ

and _the plastic moment is only 12 per cent greater than the maximum 1 - ----.;_ __ J __...,
elastic moment.
2 2
In the practical design of be th p
selected on th b . ~ms, e a 11owable loads are sometimes
t" h e hasIS of t he plastic moment that can be developed by the F10 . 6.5
sec ion rat er t an t he maximum elastic . .
analogous to that of Art 9 f . moment. This procedure, · B 18 - Pl ·> ·hill' th,n llt C Iii - Pl. tbwi
· - -3, o ten results m 11. ffi . f h SOLUTION . The bf' ndmg moment at - " J c t bd in
mat.erial and is called l ·mit " •-' T . more e c1ent use o t c they are·in the ratio J : 2, 11nd the plv-stic .,el't1on mcxiuli 11 t Ban mug
i = gn . o illustrate l t .d h
of a simply support.ed beam 1 <led . , e us cons1 er t e case the flame ratio. From thi11 requirement., wo havtl (~ l'll ,•l
oa a.l th e nudd!e ll8 shown in Fig. 6 .4.
p
h 1\,\,1,U flf TIV O k,._ff'IUA!Ji

JH 6,2
0,•'"" of Two Matf'ri•I•
rJ / ,'dr' • t f3t'&tn8 mAde of f.wo or m (1n' d1nn... n, mi:H••t"lal;i ~"' ,,~~r;
t(y ,.nc:rmo-
0~ ·~ r, l'i -
tcff'(J In eng1t1{' 1'rl Ug pr-art f("{' Cc, mmr,n ,.°(Jtr ; ' I ~ 11,;r. ~ori,,S
,-
f ,.hich c~•i, .., 1.
V
.,il"" ~tic
·- .
rnomrntM ,.
forcr<l by strrl plAl"" nnd to nr rl"t" h--1m,1 r.-info,rrf)ff Ly ,nir·,c!.J
,i,.,j .,..,.,
roro '1 (',JNJ!stt' ,he P in F'lg 6.6
&Vvru : • t.ion •'11 . Id
ro<Ja. The thr ory o~ ~r,tlm ~ of iruch ~am, ;inttw• • f-,. ,. ~t rr r..u tt, t!-r
~
tor £hr U l l ~ ~ ~u!:$1 11 t,t,el "i th a )"If.
... p; r~ ! ~ t'4
jf tl\r ro.ik'TIIU ~ \ . rnatdisls 1-' <Jlll ff' 1>1I11plo J /Jr A 1.;i('nm r.! t w·o ma,,n ~, lli
IL.~ cf :!.~ kl•e10 • Japed pl!IBtic b(>nduig , to transfo rm thr rompo 411lf' 'rlf>am into an ,..11,m•nlrrr• ,.,,.~M
r.f ,m., - ~1r-.al
i;('U'TW"' fr,r full•· ac~(' total &In or the
tM 11rt1L,-gl .a: df, ~ rts Henoe we ha,·e
fl"('tl".O into t 1"\1 e-qi.&J pll .
.. ..51 • \ JO ); Z~J + .. ,2.5) , \U
F10. 6.6 r b
dr dt r.- -
T"
75
( i ~>.fj t ~•l t "-
. d -6..25 cm, as •3 cm. -Ye may now find th ...t...._ 1
snoe 0 1 "'1" Ot • JOclll. we tin dlJi per centroids as follows:
T'bf;n.
d ~ ),, 1.11d v, to t.he lower &.n up
t _l_
-r -;=-r,
5 ... 1.75 1 + 6.25 X 2.5 3.)25 = 5.5 1 cm,
f
/ f I

l
l< ♦

• _sx25 lC (6·2 25
Jl .5 .)( 2.5) + (b.L5 X • ) _j_
E ' \ - I
·, i , '
10 >! 2.5.,, (3.7.5 +
1.25) +~3.75 X 2.5 X 1.875 - 4. 15 cm . (a)
// '
r '
)'r • - 10 X 2,5) + (3.15 X 2.5)
Fro 6. ;-
FwUJ, W!IIII BG· (6.!!), "'cmwn
M,. • Llt\J x S4.!75(.5 .51 + 4.15) = 996,000 kg-<:m.
Consid er, fo r exa mple, the ca~ of a wood be.:lm oi ,;·, -,:t:1
.,~ r - :Go--
shown I .; ~. -
seclion rei nforced on t he bottom by n steel place lb 1,-i
paQJLE ll!IET 6.J
tht> wr,10 ~nd ,t .,.. :.... :-:-.(
s tri&ngle aa As..1:uming thtn no slip takes pl ucP bf'twt"t' n
l. A li«i ht.am L1,ving a cro!!-5eetion in the for~ of _an isoscele Tha L ~.~ •"7 ~
bPnd ing, the theory of solid beams wi U st d! apply
t.-.
drnal plane of symmetry.
COW"C mf tg. A ~ rubjecte d to pure bending in its longitu
g a nd t r:t :i t.ht'
section s before be ndi ng remain µliw e af tcr benJm
J1;a.!::.::,
Locat,e ~ ~t.ral am NN for fully plastic bending and calcula
te the plastic section
'es irom :c,,: ec-tt. ~
JDOduha. 14.IU . h1 ,. 0.707h, 0J)977M•. z,. -
in Fig. B. Locate
longi t udinal fi bers are p ro port ional to their di.st.anl
Lhe ~ oJ t.hft
2. A prumat.ic statl bar has the trapesoida.l cro&HeCtion shown
{'!ai.t.r
axis. Thus fo r any bend ing <'u rvature I p wit.hm
onding plutic
tbellelltn.! a.m ,,..N for fully pwtic bending and calculate the corresp material, the norma l fi ber stress in the wood , a t the d1Stat1 ~
, froo tbe
ac::uoo awdulua . An.a. Z, • 285 cm•.
of a steel beam of semi• neu t ral axis, is
1. ~ u I.he ~utra.l Ul6 NN for fully plastic bending
cirr:uta.r eroa-eec t.ion u aboWD in Fig. C. An,. ht - 0.4041'. " = E.y ' p
:tn><.l d -t ~ ~,
and the corresp onding elemen t of normal iorcc on the
Li
1-5-j
E.
7 dF.;:; - bydy, l.n )
I5 p
11, •
I

elt>m-e.n ti.d
where E.. is the modul us of elU.5t irity of wQ{id Sinu!art" t'. ir
i !l
N -;--...-,....,.....~..--..1- N
l
If
r
l.
N
.llt
[~'-'- "-~
area of the st.eel,
T- l
- b -.J ;.._ JG --..l dF. = E, by dy, (b )
p
F 10 A Fio. B
pure t->e_o..iwg
where E. is the mod ul us of ewtic ity of st~l In ,he..-~~ of
t. A auoply suppo ~ ~ btam ular croe,,- the.ae elemen tal forces, summed tw,•r tiw to14ll :trt>~\.:i ,u '11 ,.'{_>d !\nd s<ed,
aection 2.5cm wide by ~an deep If cbeti., • Bp&n l • I.2 m &nd a rectang
wbAt interwty -Of W 111 ...:u
· 0 , D, - 2,xio q /nu•
l bear.n
.. , A""'uoe a plaa•:
ta uniform Jy .~. i
loaded over the Cull •pan,
h.1fl&e at the rnid•po int7 The Y1·eJd
respect ively, rnUBt h• ve tl net n•11uluu \l f,m'<'
use ~
t'lf U&I to
, ,,
1,·ro,
r.hu
ar.tJ
rtl':!J4
ttki
Un.J
rWm of
m001~n t
11.resi 1' · "'c thefr rnomenta about the neutral L'U:1 111 1•, 11 11,u
· llf. tO • 2,170 lri /rn .
sT'Rl:SSES JJl BEAMS : II
14ti iHUWS OP TWO ~T'ERJA IJI B7
. Without actually maki_ng these _surnrnntio
de\'eloped by the ~ euon._I be nchanged if we write eq . (a) Ill t he en ~~, . on cqu1vaJont width b1 of the sti ffer matenal f11t--el) T he T'CVf'T'!i?
t the results -wtl u 'tUlv. into edurc will o bv1.ous Iy i... - equa IIy satisfactory
.
=
we no te •'-~
,.w<
prOC . be . f
alent form If the composite am cons1st..<1 o more t hat two materiA.1.9, t h~ t m n..'--
(ormcd section ~ a.y, in general , be obtained in term,; of any one of rhem
For exam ple, Ftg. 6.8a rep~ese nts ~he rectangular crOF..<!--1iectJon of a lam-
re rd the wood portion ~f the cross-section inated beam of three ma terials having modu li ?f elasticity £ 1 > £ , > E~.
This shows th.at we may ga f reduced width of Then 8 suit.able tra.nsformcd section of the stiffest material will be t h.al
. _1 t t 8 ,;t,eel stem o
..: .Jth b a.s equ1viuen o ·
hown in Fig. 6.8b, where bi = b, b, '"" Etb! E,, a.nd bi == EJJ 'E,
Wtu E. 8
bi= E, b (cJ If the composite beam does not have a recta ngular cro-c:s-s>r-tJon . thP
problem of finding an equivalent transformed section may b,, more com-
• . . -b Under a given loading, the composite section . licated . Consider, for example., a beam of t wo materials having a cm·ul3r
&S shown m Fig . 6._,...,
·' · · 6 7b ·11 both d
••armed section in Fig. • WI in
eve lop th ~ro~-section as shown in Fig. 6.9a and assume E1 > £ 1• Ther. lJl thP
Fig. 6.7a and t he •1 wwJ e transformed section , Fig. 6.9b, each element.al strip of width I, 1n the !own
same resisting moment. bl half of the cross-section must be reduced tn widt h b1 = ( E~ F, I) TlliS
-~ soon as we hs ve th,
"' transformed section, the pro em of. the bending
will be accomplished by making the lower half of t.he transforrr:ed ~cLo;i
of the beam of two materials is red uced t-0 that _of the bending of a s~J
beam of T-section whil'h can be treated on th~ basis of the theory previously elliptic,.I as shown .
developed in Art. 5.3. Thus for a given ben~mg moment M on th~ section, EXAMPLE 1. A timber beam having a n 20 x 30-cm rectan gular cross~rt.oo is
t he extreme tensile stress at the bottom will be rcinfo;·ced top and bottom by st(~el plau-s 20cmwide XI cm thick a., shown U) F.g
Mei 6.I0a. The moduli of elasticity are E, =2 (10) 1 ltg cm' and Eo =1l01' k_g c::: ' T l'it"
ITJ = T' allowable working streSBes are IT, =l,l20k.g •cm' and IT,. = 84 iqt 'cm' Find t~e c:u..-
mum allowable bending moment for the section .
SOLUTION. The ratio E., : E, = 1 : 20. Henct> the transformed sect 100 wLl .ie
a.nd at the t-Op the com pressive stress will be an 1-eection with web thickness b, = b/ 20 = I cm as shown m F ig. 6.!0b
MCt
= T'
r
r==~~
<11
i--20-, I 20-----j
where / is the moment of inertia. of the transformed section, Fig. 6.7b. l . . ,._
These bendi11g stresses are, of course, for the equivalent steel section.
To obtain the t rue me.ximum compressive stress in the wood, the stM
30

L
u1 must be reduced in the ratio E,./ E,.
In the above disc~n, t he less stiff material (wood) was transformed
. .22:
tz:,.ti::l ·
:'zz: '·: i=
z:zj,kzl b, -..r,~

Ei 1 (bl
d
-d
E, (o l l ( bl (a)

F10. 6.10 Flo, 6.1 l

i i
To determine the 11afe bending moment, we must tinit tl\:cid,• "'bi..:h a.llu~~ble
''
working stre1111 will govern . The allowablt: stress I.D. Ult' wt>b <>i th~ Ct?tJlraien,
beam will be"'• • 20 x 84 =- 1,680 k.g/cm•. The allowa b~ ~t~ in L~ lh ~ ~
b1 "• al,120kg/cm 1, he.nee the flange str~ go ,•e.rru.
The moment. of inertia al the tran11formed section is
(o l (b)
(o) (bl Lrl Sl•J ~
Fw. 1\.8 I •2 -.r·
[20( 1)1 t 20 )I, 1(155) ' • T - o.: ••
H ,"":,cm ·
Flo. 6.9
p
,r a JN BEAMS :
II OEAll8 or TWO w.A't'EJUAL.8 1~9
STR£5S....,
148 . t,en.si le etreas in the s teel i!! I , I 20 1t,i; lr m', ll'bat i...-t the eorrMpond ing m.&.D~
l))Jl,alJlUID[J)prfflBi ve st.rffl8 in the ca.st. iron ? A P\.t . u. - 43> i. 11 Ju n•
Then. irom '.Xl · <5.4) iJXtUlllRclorring to the circular croS&--sec t.ion of two matenaui as ahOWD in Fig. 6.9
J ~ X l , ! 20 = 830,410 kg -cm.
,11 ,., n "' : 17• "' ( 15 T I )
S, d oting th e ratio of moduli of elasticity Ev E 1 by a, calcuJ.i,,, the d1.ruu1ce 11•
IJld tenb horizon I.Al diameter to the cen troid of the t.ransfonned aa:uon A -u.
' 11 beam cut from 11 s hc('t of 7-ply (r<>ID o
. ? Th{' cro--.s-s<'ctinn ?f Fs,_s111Jl6 I ls . Altcrnatc layers of t hf' ply/ 1y.
E..X Al•IPLf ~ - . hown m ig . . Th b . I2 I . UOd 2d 1 - a
wood h.s!' thi> di me nsion~ s of thP beam . e Pam _is . m o ng , si 1n ,
ha,·e the grsm psrallcl to the lr~r:ie The modulus of e~as~1c1ty parallel to
sup:porred . a.od lo.,dt'd st the mt r . ndir ul.u to the gram 11; E 2 -.2.5 ( I O)~klJ/c ~
i~) Y• ~ 3r 1 a +
._mun is E, = (IO)bkg/cm•, t,hat _pe ~a,=84kg/cm•andtr~ =21 kg/cm•. Ca.lcu ta~ " A laminated wood beam h a.s a rectangular cr ~ n 20 on wide by 3;j cm
The oorrt:llpood mg worlunit stresses
• The layers a.re each 1.5mm thick , and alternate layer<J have t.heu gra i n s ~
the saft, ~~Jue of the load P. modulus to the larger is E 2 : E 1 "" 1 . deep -ht angles, the two outside layers being grained along the 8JW! o( the bew:l . The
11
SOLCTJO ~ . T he ratio of thr s~c;rain across the axis of the bea m should·~
~:: E, : Ea - 1 : 4, .and the corresponding work.in~ st.r~ a.re u , ~ 84 q c-n '
He!lCt' t.he width ?f lay ers ha;ng nsformed section will be as shown in Fig. 6.lJb allel to the grain and tr, -IBkg/cm •across tb.e gram. Ca.lcul.at.e ~ i!Ale ce.:uitz:g
reduced i.n the rat.lo l : 4 e.nd _ e tr\ ed section is .
The moment 'Ji inertia of this trans orm parment for the section . Ana . M = 1,364 kg- m .
mo . . --~
:----c- -
5. A prismatic rod of ~exagon.a 1 cross-sect10n 13 11~e
_ 4 (3 S)' _ [ 3 (0 .5 ) 1 ] _ 2 [ (3 X 0.5) X (0.5 X 2)1] = 11.2 cm•. up of six equilateral triangular segments a.s shown m
3
I - 12 12 Fi . A. The segments are alternatdy Redwood (E, =
3
The corresponding safe bending moment for the section is 84f10JSkg/cm•)&.nd soft Whi~ Pine. (E :=70( 10)_ lr.gj cr.1•1
lued together to form a soltd section . The duneru11on
., - { 17 - ~ x 84 = 537 kg-cm.
! -2 .5 cm . If the bar is subjected to pure ben ding m
:u m u - C I - 1.75 it.II longitudinal pla.ne of symmetry, calculate the maxi-
mum bending stress u , assuming that M = 1,000 kg-cm.
&nd the safe load P at the middle of the beam is An,. tr = 112 kg/cm•. Fm ..\
4Mmu 4 X 537 6. If the rod described in the preceding problem is support.ed at i ts ends ilS a ~T.-
p = - , - = - 120 - = 18 .kg. ple be&m and loaded a.t the middle by a transverse fo rce P = I SO l; g , o!culiu..? ti!e
ahe&r stress in the glue between the upper and lower halves of t he ~ cllo.n. _hu.
If a laminated beam of two materials has many layers, it will be justifiable to T _, 5.52 kg/c m•.
express the moment of inertia of the transformed section by the approximat-O
formula 6.3 Reinforced Concrete Beams
I = bh' _ ! (b - bi) Ir',
(d)
12 2 12 As mentioned in the preceding art.icle, co nrrete beams reinforred ,. 1th
,i.-here b, = E sb; EI is the reduced width of the softn layers. Substituting thia steel represent an important example o f ~ams of t wo different ma:t'n.!lb
value of b1 into eq . (d), we obtain A complete t.rentment of the the,1ry o f rein fo rced co n crete is bcy o~d the
scope of this book; we give here o nly a brief discussion of :bis im?')rt an::
I =bh•
- ( 1+-
Ei) · (e) problem .•
24 E,
It is well known that the strengt h o f concrete is m ur h gn>.:\ter in ~.Jm-
pression than in tension . Henre a rec tan gul ar beam of co nCie ti' '-" ill foil
PllOBLEW8ET 6.2 from the tensile stresses on the con \'e x side. The bt-3!11 ran ~~ gretLd y
l. A llimply supported beam ! I . strengthened by imbedding steel bars on the con,·ex sid~ s :: sho w11 in
middle w the cr06IHJection sh m _ong ~nd carrymg a concentrated loa<l P at the Fig. 6.12. Since concrete grips the steel s trongly , there w ill b..> 110 s lidi ng
1/20, b - 10 cm, h .,. 15 cm, and i°-:,0/n Fig. 6.7a. Taking P =- 1,000 kg, E./ E, ':'
the eteel and the m&ximum co cm, calculate the maximum tensile stre98 Ill of the steel bars with respec t to the concrt>te during bt>n d il".g crnd t he
er. -124 kg/cm• mpl'etl8lve stress in the ~ood. Am. <T, =775 kg/cm 1i methods developed in t h e previo us a rt icle c.!ln also be u$A."d h et'i.' fo r t•tt.l-
_2. A wood beam having a rectan lar . culating bending stresseR. In practice , the cross-~c ti onal area. of t he: :St&'l

r'i:o;hl~ch
.
.n:uaf~ed top and bottom by two crosa-eectt~n20 cm wide by 30 cm deep~ bars is usually such that the tensile strengt.h of tlw conc re te o n t he co n,·e."
2 cm tl11ck and is made of cut . tea _as shown ID Fig. 6.10a. The top plate IS
~ 5mm t.b.ick and ia made of 1~~~ E. -(IO}• k1/cm••and the bottom plate •For• more complete treatment of r einfor.-ed cone.rec..:-, ~ C. W Duo n.am, Th,. T~~
u c1ty for the wood ia E. • ( !OJ • q/c,n• E, -2(10)'klfcm•. The modulus of el.a,t- 11
rad Procc~e of Re,nfurced Co,1cr~u. McGraw- Hill Hook Co. , l oc., Ne;. York, 1953
. If the beam ia to be loaded so that the
r 150
sTllES-'l':8 1N 9,;A.-1(.S :

. . of t.he sreel begins, and _at larger lo


Node is overoome before y1~td111g II the tension . Hence tl is CSlabti~~ds
t.he tee! alone t.akts pracu_callY a~ in reinforced-concre te bearns o( erj
~ in ra.kulating t,endmg stre h ta.II the tension is taken by the lh~
prac ce . . 6 12 to &Siume t. a 8~1
1l
RJ:lNYOllCJ:D OONCRRJ'I) Br.A.IOI

U t.be ratio of t he area of the at.eel A, t.o that of ••=


above the center of the st.eel be denoted by
A,
p ... tx1 '
1. -
areA
of • 1. _
151

. uit:1 concn,te

type shovrn m Fig. ·. b the concrete. (a}


and all the compn-.ssion y f--ll ~
-j "c!7 I I
the solution of th e above equat ion gives
--C;:-ii ;l- T
±M,- ~ I, k = ✓2rm + (pn)1 - ;m. (8.4)
.- · m

d
_L
Instead of using the moment of inertia of the t.ranmorm.ed section ·
I
I /d the prece~ng article, it is more direct to pToceed aa follows : The av: =
I
_-_:_~ stress actmg over t~e area a~ove the neut ral axis is ",/ 2 where "• is the
::-~ :- -~r 0
JD&Ximum compres1nve stress in the concrete. The total comprem-,e force
1-+ nA1
C is (u,/ 2) bkd . . For a linear stress distribution in bending_, the center
(bl (cl
(ol of pressure of C 1s at the centroid of the tru.ngle which represen~ t.he !t.re5!
F10. 6.12 distribution; that is, at z = lkd. With no a.xia1 forces acting 00 the beam,
the tension T must equal C and these two forces consti tute a couple whose
Hooke's Jaw and a compression test diagrarn moment is the moment of resistance and is equal to the prod uct of c and
Concrete does not foll ow ' • ( .
ior this materi&l rui.s a shape similar to that for cast-iron see Fig. 2.8b). the moment arm jd, where j = I - k/ 3. Hence the bending moment
· •-~~ increases the slope of the tangent to the di&gra.rn M, based on a maximum stress "• in the concrete is
.\s the comp.ressI\'e sw = ' .h. .
decrea.ses u~t 16. em odulus of concrete decreases wit. increase
th m
I sttes.5. = ~ bkd(jd) = ~ u.kjb<P.
• .1._ ·
. M,
In calcul~ting stre58es in reinforced-eo,1crete beams, 1t 1s usua practice (6.5)
that Hooke's law holds for concrete and to compensate for the As the maximum allowable fiber stress in the steel will probably not
~=modulus by taking a lower value for t~is modulus than that be produced by the same value of bending moment as produced e1,. i,
obta.i.ned from compres.5ion tests at small compressive stresses. In specifi- is necessary to compute al.so the bending moment M, based on e1 ,. The
cations for reinforced concrete it is usually assumed that n, the ratio ol couple used for this purpose is T{jd} where T = a-.A, is the fo rce in the
the modulus of el.a.sticity of the steel to that of the concrete, is steel. Then
M, = u,A.jd. (6.6)
n = ;: = 15.
In determining the safe bending moment that may be applied. the- snwler
Transforming the area of the steel A, into an area of concrete nA., of the two values M, and M, should be used .
equivalent as far as elastic properties are concerned, the transformed If the bending moment be given rather than the maximum sJlov..-able
cl'OS8-section is that shown in Fig. 6.12c . The stress distribution follows fiber stresses, the value of the maximum fiber stresses ea~ i::,y the
a linear law, since plane cross-sections remain plane during bending and given moment can be computed in the same manner a.s above and then
since Hooke's law is a88Umed to hold for the concrete . Under these examined to see if either or both exceed permissible \·a.lues .
conditions-, the neutral axis will lie at the centroid of the shaded cro~ To protect the steel from damage by fire , the n.>inforce ment in be:i.ms,
seetion (Fig. 6.12c). This requires that the first moment of the shaded girders, and columns should not be placed ne&n'r the expt.'\j;ed su.rfac-~
a.rea above the neutral axis mm with respect to mm must equal that of than l½ in. This specification fixes a lower limit for the depth of concl't' te
the shaded area below ; that is, below the reinforcing bars designated as a in Fig. 6. 12b
kd EXAMPLE 1. If n = 15 and '1, "'4,j k_u;/cm ' , deterrrune the w e !O&J '!lt tht! m1ddk
(bkd)
2 = nA, (d - kd) . of a reinforced-concrete be&m 3 m long, fred y supportoo ai the rn~. &nd h.&\·u.g
b = 25 cm, d -= 30 cm, A, =- 7.5 sq cm. \\'hat is Ulc s ~ in tht s:tcel.' Colllilder
Thi~ _equation is quadratic in terms of k, the value of which defines th e two C&se8 : (a) if the weight 0f the concrete is neglrctl>d ; (b) if UY.i concrew \\'C.i&M
pos1tior. of the neutral axis. 2,iOJ kg/cu m.
• 152

80LUTION. (a)
The
The transformed
~

area
· l J must
neutral &Xl8
be .,..
IN &&Aid:

located
· rod ·
that 1t 18 11
II
.,
of the rcinJorcwg . ~ lB nn . - 15(7.5)
controidat
This.,...,requires t hat 'J;M 1- 1 - o, 1.e.
a . "
• Xle IOt
rtEI NYORCED CON'CJtVl'JD • ~

only be approxima~ d o n a ccoun t of dw gn conditionB, 5Ucll 118 ~ of , .


163

- . l cing rod s o.v1.u la ble, . etc .


112..5 kl cm- _1,. D in Fig. 6.13a, ' 10 or .
the shaded &re311 euOW In ideal bala nced remforceme n t, t he tenaile st.resa in the equi~ t .
( Sy) ~ _ 112.5 (30-y), concrete, wh! ch repla ces the steel a nd ':hose area is n t imea aa great a.s that
2
f the steel, 1a to be u,/ n a t t he same time a.a the compreaiv-e st:remi int.hf
0
from which 1/ .,, 12.5-4 cm. . . e roduct of the compression area and the ave tside fiber of t he a ctual concrete is u Since t he!!e st.reMeB are proportional
0•

The compressive force C 18th P


. · thereon; name '
ly
l · (30
c ..
(45/2) (25 x 12.54) ~ 7,060~-~
12 "'6 ·
':: their distan ces from t he neutral axis ,
compressive stress actmg t arm of the coup e lB - .54/3) '"' 25 82
Then since T - C, and the momen of 45 kg/cm• in the concrete is · u, q,/ n l - k a-,
ba&ed on s safe strees - = -d -kd
kd
- or -- = -
k no- ,
( 6 .7 )
cm. t.he mo ment
M• .. 7,060(25.82) = 183,000 kg-cm.
Once k is determined , the remaining proced ure is exsctJy the ~ u
. tis Pl/ 4· equating this to 183,000 kg-<:m, one finda
The IDJlXimum bending momen ' before and is illust rated in t he fo llowing numerical e.xample.
p ;:~+40/~ c _ 7,060 kg, the 8t.retl8 in the steel is T / A ,. 7,060/7 .5 • 912 EX.AMPLE 2. Design a co1krete beam wiili bala.ru:ed reinforcement to a0i&m a
bending moment of 2,000,000 kg-cm . Wid t.h of beam to be SO cm, c;, =-t , l.25 qjcm ";
q fcm• . hs 2 400 (25 )(35)/(100 x 100) = 210 k& of length of the 25x ,,, - 45 kg/cm• ; n = 15.
(b) The conc~te we1g . • 5 cm of concrete below the reinforcing rods. 'I'be
~ ~~:n~!~:1!uaed by this uniform 1~ is wl'/8 = [210(3) 1/8)100.
soLUTJON .
6.14.
The transformed area and t he stress clistnl>ution u-e !hown in Ylc-
The tensile stress in the transformed &rea of con crete equivalenL t.o ~
m&XIIDUID nH the moment aV&ilable for carrying the concentrated load Pia
original steel is one-fifteenth of tha.t in the st.eel since LU! ares w ~n ccu.ltipl»d
25,6.25 kg-cm23.625encel59 375 i.--cm Then Pl/4 = 159,375 and p,.. 2,125 kg.
I 83,000 - , - , - · by )5. The distribution of stress intensities is B.SBUmed to be a straight !me u sho,rJl
in Fig. 6.14b. The neutral axis is located by using eq. (6.7), i.e.,
From the foregoing discussion it is seen ~hat the bending mome~t which
produces the maximum permissible stress m t he conc~ete of a. re~orc~- 1 - k 1, 125
concrete beam will not necessarily stresa the steel to its safe llIIllt. Or 1t -k- = 15 x 45'

may, on the other hand, overstress the steel, and if the bending moment _ii from which
reduced so as to lower the steel stress to a safe value, then the concrete will k = 0 .375
be understressed. Both of these conditions are unfavorable from the stand, The moment arm of the couple TC is
point of economy, since the full strength of either the steel or the concrete
u, not being utilized. When a reinforced concrete beam is being designed, J.d = d - Jed = 0.875d.
3
however, it is possible to so proportion the ratio of the steel to the concre!t The compressive force in the concrt>te abo ve t he ne utral iu is is (Z!..5J 10 .3 J5d {~ }"'
in the cross-section that the maximum permissible stresses in the concrett 422d. The resisting moment is (422d) (0.8 75d) and is equal ro the gm m beDfilJIC
and the steel are realized simultaneously. The beam then is said to have moment of 2,000,000 kJt-cm . Therefore~ = 5,4 10 sq cm . from ._ hk h o - a..s =-
balanced reinforc.ement. In most practical cases, balanced reinforcement can In balanced reinforcement, T = C, and ' .tnce
1, 12s ( A .) = -.22 (73.5)
from which
A . "" 27.6 •q cm.
C
This area of steel can be supplied by six 25-rum-<lia met.er remfo.rcing ban.

I I
PROB L.lllilSET 6.3

nA•• 112.5 cm•


L..---'--,~ ' l. For a reinforced --concret.> beam b -t 5 cm . d - 23 ('1) 1, and tht> tnW aroA of tht>
~
(o) (b)
1- 1,125
15
45 reinforcing rods is 4.5 cm• . Assum~ n - 15. IT, -+~ q;,'cm•. and u , aq ,:!:>O cg 'cm•.
What is the muiroum btmding moment in k..fC-<'ro that the beam can cany f ,:ill,.
(a) (bl
Ai( - 80 ,000 kg-on.
FJq, 6.13
Flo. 6.H

I
\ ◄ \
F ~ - IN BgA.118:
11

154 b • 35 cm, d - 70 cm, &nd BEAM S OJ' ABJll TRAl lT CB.OM--s!'Cl
'lO!'I 16.5
. ~cr etA l bea"l, un.tC n c= 20, "• the Lota.I '
2.. For a. ~inf__ -1. is to ,q cm , ASS t 1
=30k g/cm • , u, ,.. 850 ~ OI
utral a.xis of the crOM -sect ion . Then t.eruu
__ ,_, rcmg l'OU" moroen ·n kg-m ? Am. 9,500 kg-m . k«1~,•- le &nd comp reaai ve st.re .es
.thethe
ne flang es . be
. •
the ~,.o maximum resisti ng b =
25 cm, d = 45 cm, total area, f . will un ifonn lY d'istn'bu•- ..l
1A:U over the widt la b, and
What II the reinforced-concrete ~ :'. be&ID is
subje cted to a bending mo~ l'et11- ~n rnal force s T and C will act at the mid- resu ltant
'·· Fo-:0: . ifi 6 sq cm, n" 15 ~:m s t ~ set up ,nte point.a of the Sa.ng,ee u shown .
in the concr ete ,n~nio1 ctear lY theae equa1 an d oppo site . f ·
orces coM t1tut
foTCIDg what will be th0 500 kg /cm• . e an i.nt.ema1 bend ing
l~T ~ :. ire so 16.25 kg/cr o•~ t~ produ the

=
8 4. ced _in th~ conc rete and steel are to
a certain beanl, the area of remforcmg rods = 45 sq cm,
1
.,_._..:ined b ., 50 cm. d = 90 ~m , 500 000 kg-cm . Ans. <r, lit
~p~ ied bendi 2
ng moment LS = 725 kg /cm2- " 71 "" 18
, •
• c .. 32.5
,_,cm' te ...,,,nm b
· f ---.:i-concre
= 45 cm, d = 70 cm, n = 15, u. "=35kg/ --=o-- -• z
d' """"'mom
"'6 ·
5. For a rem OnJCI;' ' ent of 1,000 000k
, g-cm , h ow muc h area of stet.
ell\,
If the beam is to resis t•-~~ ;,kt is the stress r
reinforcement mUBt be= set up in the steel? Ana. 11.92 1
an ; 1,325 kg/an '.
6. Solve Problem 5
if th t,end ing mom ent is 1,500 ,000 kg-cm . Am.
41.2
tq
( 0) y
.o-l J' (b)
e 5
q cin;
615 kg/qn '. . long and Flo. 6.1 5
7. A <:a?tilever ~ • ed
m
2 20 cm ~ ~de of reinforced con-
wide, is to
to carry a total unifo rmly d1Str1buted load
~rete . . It ~ to be e,,;ignht What should be of 3,000 kg
its effective dept h at the wall? 1 m fro~ mom ent abou t the y-ax is whic h is not
mcludmg its own weig · balan ced by any CO?T 'e5p01 lding
the wa.m Indicate the position of the stee l re inf
orcem t A en = 15, , "
ssum e n 11
applied exte rnal mom ent, the appl ied coup
les M bein g in the .xy-p!Aoe_
45 kg/cm •,"• = 1, 125 kg/cm•. Ana. 45.l cm; 22.6 cm .. Thus the assu med defo rmat ion (curv ature
only in the :ry-p l&ne ) is meon -
&. A reinforced-concrete beam is to carry
a bend mg mom ent of 10½,000 kg-cm. sista nt with the cond ition s of equi libri um
If d = 1.5b. what are b, d, and the total area , and t he assu mpti on is prov ed
of reinforcing steel needed? Assume inval id. Unle ss exte rnal cons train t agai
"• ==l,25u kg/cm• and "• = 40 kg/cm'; E, = nst bend ing in t he :u-pl&ne i3
21 (10) 5 kg/cm • and E. = 1.4( 10)1 kg/cm'.
:hlJ. d = 30 cm; A. = 3.11 5q cm. prov ided, we mus t conc lude that bend ing
9. A mnpl y suppo rted beam s m long carrie . coup les in the xy-pla.ne prod uce
s a cente r. load of 3,250 kg. The also some bend ing in the :u-pl ane.
economic percentage of steel is to be w,ed and d = We now proc eed to deve lop a more gene
50 cm. Fmd band A,, assuming
"• =l,125kg/cm•, uc =40kg /cm'. and n =16. Take ral theo ry of bend ing of p.rur
acco unt of the weig ht of the beam, ma.tic beam s of arbit rary crou -sect ion. In
asuming concrete to weigh 2,400 kg/cu m. Fig. 6.16, let such s. ~ be
Allow 5 cm for dept h a below the re- subje cted to pure bend ing and assu me
inforcing ba.rs. An.,. b = 36.2 cm; A, = 11.2 that plan e cross -see t.ions befor e
cm~
10 In the beam of Problem 9 three 18-:nm-dia
meter roun d bars and two 12-mm bending rema in plan e after bend ing. Then
8QUBJ"C bars are uaed for reinforceme
for the rond ition s show n, all
nt. The neutr al axis may be assumed to be long itudi nal fiber s abov e a certa in line nn
16cm below the top. Wha.t is the maximum in the cross -sect ion will be- in
shear ing stres s set up in the·concrete comp ressi on, and all fiber s below this line
and what ill the maximum bond stress between will be in tensi on . The lrne
reinf orcem ent and concrete? Ana. nn repre sents the neut ral a.xis of the secti
1.56 q/cm l; 2.13 kg/an l. on. Thro ugh any pom t O on
this line as origi n, we take coor dina te axes
z , y, z, such that Ox is ~ l c l
to the axis of the beam and the plan e .ry
is the pla:1e in wh.ich the exter nal
6.4 Bend ing of Beam s of Arbi trar y Cro ss-S ~ouples M. are sdin g. We have alrea dy
ectio n obse rved that such coupl-es will,
m gene ral, prod uce bend ing of the beam
In developing the theory of be d' both in the xy.-p iane of loadi ng and
. n mg of pnsm . . beam s Art. 5 3 it
w.as emphw,, zed that the beam was assu med to atlc ·,
7~ :try t~dl
will no~~ :
1 0
t!:
the applied loads
have an ~ial plane of
were actin g in this plan e. In general,
~o not act in a plane of i;ym mctr y bend ing
1
Art 5 3 1,~ -e Pac~ of the be~
in ~he
· · u=om es in valid . plaue of loadi ng and the' theo ry developed in
Co.nsi.der, for exam ple th
bending coupl es M e.ppl" ' d .e ca.ire of a beam . ._,1 to
of Z-se ction subJecwu 't
be assumed that the be te ~h~he xy-pl ane as show
1
n in Fig. 6.15. Let 1 (a) Y (bl
am aJ.OV bend s only in this plan e, i.e., that
Otis Fto. 6..16
II
1JJI----
IN er.,Ul8 :
91'Jlll88S8 BEAM8 01' A.BJI ITLUlT ~ CTION 157
156 Let l / P• denote the curvntu re in th
~dicu lar zz-ptane . lane. Then plane cross-se ctions ~ ~- Substit uting expre88ione (6 .8 ) fo r curvatu res into eq . (a), we obt..&in
in the d l / ,w, that in the xz-beod ing, the strain in any longit Udlti~ ~o~
plane &D • • Lane aft.er . M. (I. y - lpZ ) (6.9)
g prdi .,.s Y z, will be
....... ding rero.a.Ulmcoo tr = TJ. - 1.,,.1 '
..,.... ed b na= ,
fi~r, defin Y y z
•==;. +-·
Pi, which gives t~e bendin g st~ i~ any _fiber the location of which is defined
=0~
by the coordin ates y , z. Agam if ~ LS a _p ui:ne of ~me ~, IP
portion al to strain so that .-,.3 .
· tress
.
1S pro (6 .9) reduces to tr = M,y/ I. , which com cides with eq. (:>.5) of Art .
lastic behavior, s eqSetti ng v = O from eq . (6.9 ), we obtain, for the equatio n of t he neutral
Fore E Ez
(1-= -
Y+-·
Pr Pi,
l,,y - I •.,z = O
distribution over the cross-s ection and since t~
atressred to a couple M, about the z-axis, we have or
This defines the I.,, (6.10)
streM resultant must uce y=yz ., =zta nfl
i crdA = 0, i uydA = M,, i vzdA = 0. ~I
Thus the neutral axis of the cross-se ction makes the angie fJ
Fig. 6.16a.
= tan- 1
( / , / /11 ) with the z-axis as shown in
Substituting the value of <T from eq. (a) into these equilib rium equatioll!, 11
For any cross-s ection, it can be shown that there are always two ol""'..hog-
onal centroi dal axes in its plane for which the prod uct of inertia I.,. aa
0.
we obtain.
ix B )
E E These are called the 'J)rincipa/, au3 of the cross-section (see Append
- Ay. + - Az. = 0, eJ pl.anea
p, Pi, The corresp onding axial planes of the beAm are called the princip
of bending . From the foregoi ng discuss ion, we conclud e that fo r bending
EI,+ El,,.= M,, (c)
momen ts applied in such a princip al plane, only bendin g in th.is µls.ne
will
p, Pi,
take place and the usual theory of bendin g is valid. Thus for a be.am
EI,,.+ EI,,= 0, subject ed to bendin g momen t that .is not in a princip al pl&ne, we may
t he
p, Pi,
always resolve this bendin g momen t into compon ents coincid ing wit.h
two princip al planes of the beam. Then, by superpo sition, the tota.1 bending
wbe.e
stTe88 in any fiber will be obtaine d by adding al.gabra ie&lly the
t 1>0 stresses

I.= I
vdA, I,, - j~dA, I,,,= f yzdA, produc ed separat ely by these
the first of the exampl es following.
compo nents. This proeedu.""e is illustra ted in

All of the above di8cll88ion holds rigorou sly only for pure bending . If
&
&re momenta of inertia and produc t of inertia of the cro88-8ection aol be shearin g st.re.5ees and defor-
Ya, i., are the coordinates of the centroid of its area A. beam is bent by transve rse loads, there will
,
mation s as well as bendin g stre&;eS . In the case of beams of dOlid section
heThe first of theae three equatio ns is satisfie d by taking 11• = t, == I, it ~
u:,~ we con~lude that the neutral axis passes throug h the centroid d theee shearin g stl"e88ea do not greatly effect the bending action
~tisfactory to calcula te the bending stresses as above witho ut oonslde
and
ra-
Cl'0!!8-8ect1on. From the last two equatio ns, we obtain lled profile se.!tion , such u
tion of the shear. In the case of beams of thin-wa
the Z-eeeti on shown in Fig. 6.15, the she~ s~ a.re Qf especi& l inter-
; = E(J ~•111 • 1 M,I,,, (6.11
• II • - 1,,.1)' Pi, = E(J,,,1 - [,,[,)° est, and thi.e questio n will be cooside.red further in Art . 6.5.
Tbeee exp1'e8ftiona define the curv . . .vtl1 ~ 1. A limply aupport ed wood beam (rooi purlin) ol reewigu
.la.r ~
b
due to bend.in~ couplei5 ap lied ~tures in the xy and xz planes, re~pecti
6·16~ 88Ct.ion caniee a uniform load ol intienait y "' u sbowu in Fia- 6. 17. The plane ol
We note that when • p m the ZJl-plane as shown in Fig. 1.Ynune try ~ of the be&m ia inclined to UICI ~ .::i,,-pl:ane ol lOMlinc by an
10 - ~
while 1/p,.,. M /EI xyh~ a plane of symme try ' I "' -- 0 and 1/h * ~ G u ahown. Caloula ie the muimw n bendinc et.Nia~ .... if l • ' m.
• • " tch coin ·.a__ -. .,_ °'• l> - 15 cm. A -20 cm, and tan a - t.
Clue. with eq. (5.4) of Art. 5.3.

I ~ Df BSAMB: n
158
. L __ ..1:... moment at the middlfi of the ■P&n . BEAlfB 01' ARBlTRART CR08&-8'ZCTT 01{ 159
11,e m&JlIDll1D ~ II n
IJOl,11ff0Ji· tol' ~ X (3)' X 100 - 53,750 1.-

J,I - - - 8 --e-<:tn.
•"" 8
. ding moment in the two planes of BYDUnetry of
TbeOODJPOneDteofthila ~ 55,750 X 0.9'9 - S~,000 kg-cm ~d M 1 _ M~~
l l

are JI, - JL.,.._600l!l0670.,_· Tbw,them&n111um bending sir.. at _. .. ~ Q 10


• 33,i50 X 0.31 • ' -.~ 11 1
!2,000 10,670 x 6 _, 46_2 kg/cm• .
Ot
_j_
n
...,.., - l.~20)' + 20(15)• (o ) .Y (bl

Fw. 6. 18
8
0 __ _ _ _ _ __ ___1 For ;,omt A , y. ,. - 10 cm, z. = - 9.5 cm; for poiz...t IJ, II• = - 10 en, ,~ - + o.
Usiru; all thei,e data m eq. (tl.YJ, we obtamtam
- 50,000( - 573 X JO + 855 X 9 ..5)
205 kgfcm•,
2,293 X 573 - (855) 1

.Y
2
(b) "• =
- 50,000 (- 573X 10 - 855
2,:.!93 X 573 - (855)•
X 0 .5)
- + 527 "«i='·
·. (a)
Fla. 6.17 Thus the largest bending strese occurs at point B. This indicate! that the eeutn!
axis of the section has approximately the pO!!ition shown in F "ig . 6.18b . T o !LC'tU&Ily
EliKPLI! 2. Locate the neutral an■ of the CJ'O&lHection for the beam lo■ded 11 locate thia ax~, we use eq. (6.10) and find
■bo1n1 in Fig. 6.17.
aoLUTION. The principal moment.a of inertia of the C1'088-section with reapeet to {3 = tan _(/··)
T;I (855) = tan-
= tan- 1 573 1( U 92 ) = 5(5"1 0 .
Ul!II of aymmetry 1 and 2 are

M•
I, - _,
hi,• PllOBLJUU'ltT 6.4
lu - 0. (el
12·
11 - - 12
l . A cut-iron beam of triangular cr~tion, Fig. A. i.s to be subjec~ to pure
Fram Appendix B, we have then bending by couples applied at its ends and acting in the :ry-plane , ~ S!I to mrl'-=
compreuion at A . Find the 11&fe bending moment if the won.ing stre!IJe! in ~
l1+l1 11-/1 } &nd compre■aion ~e, respectively, u , - 950 kg/cm', er, -~ kg/cm 1 • The dirnt'D:S!OO•
I• • ~ - ~ 008 2a +l 12 sin 2a,
of the croea-eection &re b - 7.5 cm, h "" 15 cm. .4 ,u. M - H ,600 kg,-cm.
(0
1, - / 1
I,.• --r-- 11in 2a + 1 11 cos 2a.

~~t;:1' ~~~ue■ of18011., 12, 111 for b =15 cm and h =20 cm into eqs. <O, 111 '
I
r
I
e,q. (6.10) • CU• 26', I. --6,062.5 cm•,and / 11 • =1,312.5 cm•.Then from r
I ,
t&n/3 ... 1•• • 1,312.5 ~
I• ti,06:l.5 ..,, 0 .2165
and fJ • 12"' 13' m-..•--'
' .._,..,"" &1 shown · F'
y
,
.E~L£ a. A C4ntilever m 1g. 6.17a. d
.Fto.. B
~ed~ the ftee end by & f o ~ of Z-eection (Fig. 6.18) is 2.5 rn long ~dtbl Flo. A
we · ·d the ~um bend· • 200 kg which act.s in the vertical plane 0
80LUTioN. For thia bflec , Ulg llt!'e!8 u,...,._ I. A pri■ rna.tio nee! bar of ■emicircula.r c~Ucm. ~ - n. lll u9ed .. .. c:anti,.
The maximum bend· l100 : I, - 2 293 e.55 cid lever beam loaded p&r&llel t.o the v--•,tl•
at its free end. Th~ 1-enath l - 300 cm ,
mum bentf;R_ -+~- mg tnornent is (M ) ' cm•, I, - 573 cm<, I,,_~ .. h ,ni1i· r • 2 cm, and"• -1 ,260 qjcm•. C&loulat.e the -.fe load P. A!II. P - )ij, S q .
~ y occur at eittie'r q••~ • - Pl - - 50,000 kg-cKn, T e
....,. D .. . _

J>Omt A or point l:J.


Ill
I 160
STBJll!Bl'S Dl s&AMB :

of span z •2.5 m
II
. :,
carnea a unuonn1y d'
s. A aiJDplr sup~ ~ 650 q/m over the full BP!'°· The beain 4.trib~"l
- Dl:AMB or THIN-WALLED PROT!l,£ 8£C'TIONS

Fig. 6 _J9a, a nd _a ssume that the cxtc:nal load p LS applted at such di.stance t
161

veruoal load of inteOS1tY ."' as sbo1f11 in Fig. C. (a) lgn~nng the fact that th' I~~ rn t he prmc1pa l plane xy that eu mple twnding without tomon occuN
f ro
x 11 .~ .x1..2cmanP' l!«tiOD metrY, calculat.e th~ bending stress u at Poin e~ The neu t.ml a xis
·
· o f t hc r ross-sc:'t1on t he n co incid~ with the z-1u1B &11d th1!
ii°°' loaded in • plane of ~DI into oonaideration the fact that the t -i. ~)
Caloula~ the aame ~ metr'Y· ,An,. (a) 11 •l,980 kg/cm• ;(b) u ... /f"J~~ rmal 8 t re8S <1 nt nny point m t he cross-section is proportwDAl to the
}oeding i, D°' a plane syDl kcJ~, ~fstan ce y from t he neutra l _nxis. ~ ron_-i the discus.rion of Art 5.5. we kno w
that the shear stresses r w ill he d1s t n buted according to • parabolic law
p
p

v, y V, ,
A y (0 ) (bl
J'

Fro. C FIG. D Fm . 6.19

over the depths h1 and h-2 of the flanges and that the shear $L~ m the thin
'- It ia demed to make a beam of Z-eection, as shown in Fig D, of such p horizontal web will be negligible . Thus practically all of t he ~ r will be
ti<m! that the ,- and ,-uee will be ~cipal axes for the section. If all elem~
~ ll!Ction have the_ 1&me thickn~ t • l _cm. and oth?r dimensions are 1111 shown
carried by the two flanges . If we consider the two flanges a.s separ a t e ~
m the filure, what ia the proper dimenmon a to realise the required condition! having moments of inertia/, and/,, then t heir curvat ures in bending '.\"ill be
Au. • •1.45 an. equal if the load P is distributed between them in t he rat.Io l r"l 1 , antl the
shearing forces ir, and V, will then be in t.he same ratio . This rond aioo 'ft-7 U
6.5 Shearing S treaaes in Bea ma of Thin-Walled Profile Section be satisfied if the t ransverse load P nets in lhe vf' rt ica l plnne such t.bat
b, I,
I~ th~ preceding article we have eeen that pure bending couples acting in &; = ri · ta)
a J1r1na1'4! plane of a beam produces bending only in that plane. For 111ch
Thus, to obtain simple hcnding , t.he load P must be app lied in a plsne that
PUN! bendmg, there are no shearing stresses and the internal stress resultant
lies between the centroid C and the stiffe r fla nge. I n t he li.mit.ing c~<>e . Fig
on ~Y aection is a couple which equilibrates the externally applied couple
6. I 9b, where the re is only o ne flange , we may take / , = 0. and eonrlude
at either end of the beam. When bending of a beam in a principal plane il that b1 = 0. Thus, for simple b(> nd ing of a T -sectio n pt'rpead icullll to 1td
~~~~ by _transverae loads, there will also be shearing stresses to consider, plane of symmetry, t he load P must be applied in the middle pls.ne of Lh~
'" "Y °"
t.
au leC 10D W
0
millArtbea· 5force
·5 · In general, the resultant of these shear stre~
..... _ n l h
oo
_____ ..;1 flange as shown. That point O in the cross-section, m c~h ed...~ . repre-
in thia pla Thia . ~...,.e tot e plane of loading but not ne~•Y senting the point of applica tio n 0f t he sht>ar stress resul t.3.nt is called tM
t.bat ~ ti
sti uation C&U8e8 some twisting action on the beam "
IMar center of the cross-section .
atta.u:i .-impk ,:::_ro~te about the longit udinal axis during bending. To In general, the loca t ion of t he shear
the external 1~--L ~"9th,i.e., beoding without twist it is necessary to apply center for a solid cross-section of arbitrary
UIIWI m e same xial 1 ' shape (Fig. 6 .20a) is a co mplica t ed problem.
ftllUltanta act. The deter . a . P ane as that in which the shear ~tre&'
00une, ~ uire a careful e ~t•o~ of the locatiqn of this plane will, of For tunately , it is n o t so im portan t fo r
X&mination of the distribution of shear stre~ beams of such solid sectio ns because thev
over ., . the Cl'Ofl8-8ectj0 n. have considerable to rs io nu.l rigidit y a nd
A.aet US begin ·
loaded . with the caae of a be . ·on twist very little du ring bendi ng by loads
1D a Plane pen.. di .. 1_ _ a.rn of smgly symmetric cross-sect! 1n.
· ~ 0 c"'V to th ,___
e P1&11e of symmetry, as ahoWP
r 162
acting thro ugh
consider only the case o
in Fig. 6.20b.
·
8'f1U',881'.S IN BEAKS;

. 'd C Hence in our further discllBsion


the centro1

ith a beam O
· . ll d fil
f an arbitrary thin-wa e pro e section

f such cross-section loaded parallel to


. .
, we k
.U

s,~11
as sho,..~
one f .
and
BmAHB OV TBllFtvA..Ll&D ..-Onx.. 8J:Cl'ION

M

+ dM.'"'_
ck r·
163

xy 50 as to undergo simple bending in th· 0 1~


U-"
We begin w N• + dN • = I• } o 1/ d.A'
t wo princip&l plathnes, s&~ ~comes the neutral axis and the norrnallS Plan~
Fig. 6·21 · Then e z-ax1s h' h . d fi stra~ '
. d' &I fiber the location of w 1c is e ned by coo d.""<lllr that

80
in any longitu 10 r •na~
la. ydA ,
'
· sunp.
· Iv
y, z, IS M. y dN. = c1./
dM dz
T ydA = -v1.dz
- (d}
(I := y · (b) • 0 • G

where V. = dM./dx is the shear force at the C1'08IHleCticn defined by z .


Isolating an element A of the wall between cross-sections x and x di, and + Substituting expression (c) for dN .. into eq. (d), we obtain
+
of u C
length 3
,
we see that normal stress resultants
d.
N"
act on its transverse edges as shown. The ben mg moment at x
and N .. dN .
•Will
dx be'
larger than that at x, there will be & net force dN. ~n the positive directio~:~
+ -rtdz V .dz
=-/,
- i• 0
yd.A ,

the x-axis, which induces shear stress r along the mner edge of the element from which
Since the wall thickness t is assumed small, this shear stress can have no
v .. [·
T = [,t }o y dA. (6.11)
N1 +dN1
1 This expression, analogous to eq. (5.6), gives t he shear st.re8!I at &ny point.
Ttrf ~ A n
,,..
,, ,, in the wall distance a from the free edge. Since the integral therein is a
.," ,.,,, m N. function of s, we conclude that this hori~ntal shear stress bet ~ n adjacent
~"' I
fibers varies with the distance a, being a maximum &t the neatr&l plane and
zero at each free edge.
From the requirement for equality of complementary &he&l- 5treS'9e8 (!ee
p. 30), we may now conclude that there must be the same ~ l!tre.'58
A distribution in the plane of the cro...~tion as shown in Fig. 6.21b . .'\pin.
since the wall thickness t is assumed small, these shear st.res:.'lle8 in the pl&.ne
of the cross-section must act in the tangential direction at each po.int. The
----1 shearing stress per unit length of the center line AB of the ~tion i3

( b)
(a) y
rt=_.!!.
I.
1··
o
ydA,

Fro. 6.21 and this is sometimes called the ah.ear jlJJt.(l.


Considering now the equilibrium of th.at portion of the beam ~ t ween any
transverse component and m
of the element is a free rf ust act parallel to the x-axis. The outer edge !!roes-section x and the free end, it is ~n r.hst tfu> el,~ment.al shear foz-ces
· su ll.CP and · lb- Ttd& over any cross-sectio n must I"('duce to a force V . pamUel t.o t.ho y~a.mc
num of the element in the - . ~arm~s no shear stress. Thus for equi 1
x direction, we must have This shear force will act through a point O such t.lat it.a moment &bo\\t the
centroid C is equal t-O the sum of mome1,ts of the elemental forces about Uw,
Now U8ing eq (~) T t dx == dN (c)
•· same Point. Th.is require.mon t will en~ble us to locate the sb&t.r cent.er O for
. > ' we conclude that any specific cro88-8Cction. The bea.nl uudergoea &imple bending witbo-ut
twist only if the load p &t the free end acts through the shear center of the
N ,.. M . [•
• T; )0 YdA end eross-eect.ion.
STSJ:MS5 IN s&AMB : 11

164
. • thin-wai led channe 1section
·
AB shown in F'i
Br..ul8 or TlllN-WA LLED P.ftOnLJ , 8ECrlON
1- A ht-am hA\' 1';!,.nel to the web so as to produce simple
166
EXAMPLE
. ki&ded bt:•.e.~
iD • ,·ert.ical ~ e defining the location of the sheAp cent.!r dlfic ~ g,c..u&rLr: 'l. . A ~m ha~in_g th!! th.in~walled e!!mkirc ular ~ t i o . n lh,own
in
II . Find the d111tance , O of ll Fig. 6.23 is loaded m a pnncapal_pla~e :zy IIO &11 to produce 1irnple bending
thl.9 plane· in this
a,ection . ·~ e. Find the d~tance e definmg t.he location of the shear cent« o.
~1,1JTl OM . The_ab~r s t!"e88 rat eacb point!-- _ ~ &lon
g t.he middle line of ra.diw,
,,,
....

--
i s--1
r .ill be in _the direction o f the t.&ngent to thia line u aho...,n_ From
magnitu de 111

la' 1a•
eq . (8..11 ) , iu,

C ,. ,_ -V V am •·
1.t o y dA - -1.t o r roe tj,, . trdij, - _/,
Vri
~
_L
I
...l.
L- 0

.,-n
/

V V
(o l ( b) (cl ( d)

Fm. 6.22

soLOTJON . Sinoe t.he wall thickness t ~ small, the sh~ s~resses in the ftangea
will
be horuon&al, and those in the web vert1ca_l, as shown _m ~•~- 6 .~b. The
result&nl
lhear force for the web alone will be a ,·ert1c&l force comctdm g with the
middle line
o( the web, and its magnitud e must be equal to the total shear
force V on the entiit
ll!ICtion. This is shown in Fig. 6.22c. To find the resultan t shear force for either
flange, we must fin.t find the intensity of shear stress at any point ,
of a ftange. y t
Bmrring to Fig. 6.22b and using eq. (6.11) this becometi1

,. = _! [• ~uu _ F10. 6.23


VJ&._
' It}o 2 2/ We note that this shear streas i! a maximu m when~" " r / 2 and sero
Then the resultant shear force Q for either flange becomet i t0l' • - Osmi.
for~ - r. The corre5po ndinK ~lcmenta l shear force is rtda = r!nifi d
about point C is Tt:r 1d(/>. The sum of these moment s o...-er the enti.~
1t& ~ t
·• Vhlb Vhtj• 2
becomes
~_n

.f ,
Q "" } ,-,Id, • - ada - - -· (e)
0 2/ 0 4/
Tiiese homon:&1 shear stress resultan ts for the flanges act along their middle line, T '= TtT~ = - I,
Vr'Cf •sin~ 2Vr"t
M ~own m Fig. 6.22c, _
- --
a.nd constitute a counter- clockwi se couple of moment QA. Q o I,
~ cou p~ , ~t'ther with the vertical sh<'ar force l7 coincidi ng with the web, arc
The horizont al compon ents of the e leme-nt&l shear force s~ sbore ~
:tiaJly f:YU!valent to a \'ertical force V acting through a point O in the plane 01 oeutnJ
axis cancel thf' horiaont&l compont>nts of ~h~ below t.he De-UU"IU uw; ~
" ~ tion as showo in Fig. 6.22d. This point O is the required shear Cf.n!Kr . ~
00 1 shear atfeijS resultan t is a vertical forct> equal t.J th~ ~ F :u U>e ~
& nee e from the middle line of the W<'b is found from th~ relation - To
Produce the twisting moment 1', e&lculated above, t.his f ~ ml.$ ~ Uu-ough
a.
from which point O such that
Ve - Qh,
2Vrtc
ta •
Qla
-
b2h't
(fl
V(r + e) = T • 1.
The · ·
~o will undergo simple bend' .
V -;;r· from which
. . '
only if lbe plane of load ing · mg m the prmc,pa l plane parallel to the wr.b
As it "Jlf'<'tfic f!XArn pl<' ig
'fh.-n fro T bl
I m a e BA of :\pl)('nd·
core~ol Vl-d from the plane.of the web by this distanc_e ,.
' nt11, rr tht>
B CIUl(' of an IS~C 2.:.0 chnnne
1 ct1on-
~
..~- --,,
. .
cm, .. .3,828,4 cm«. 8u b6titutin ll( ' we find h =- 23.59 cm, b = 7.64 cm, !If 141 where
t ,.. .
g th t'!IC' data into eq. (()
t - (7.:.~ ) • (3_,3.:i911 ( 1.1 1) '
-I l< 3"J"ii .T - q 2.99 cm.
r
I
I 66

sui.utut.inl 1ibia e•
8'l'Jlll88r.&

. value of J, into eq.


JN s&,UIS :

1,.)
\& ,
we find

r(; - 1) • 0.272r.
II

6, 6
B E N DING STRr.881:8 lN CURVED BEA.MB

Bending Stres Bes i n C urve d Beams



Jo this section we _co~ide r brie~y t he theory of pure bending of nu
167

initially curved ~o.r w1t h m t he elastic range . Referring to Ftg. 6 .24a, con-
PBOBLJ:MSJ:T 6.5 'der a short port10n of a cu rved bar acted upon by couples of moment M in
~be plane of . i?i~ie.l curvat ure . . Such ben_d ing momen t which tends to
for a thin-walled angle section lies at th . decrease the m1t1~l curvature will be con5:1dered as P',}8iiiv€ . Each croS&-
i..t the shear cen ter .O
1. Prove •WJJO e ltl+-.
,q-, section of the bar 1s assumed to have an a Xts of symmetry which lies in the
aeciion of the two leg!. ter O for a thin-we.lled balanced Z-fiection such lane of initial curvature. The loc us of the centroids t hen is a plAne curve
%. Prove t.baU he ~8':~:°with its centroid C. &8th&1
abawn iD Fig. 6.18 CC?lDCl e from the plane of the web to the shear r.enter r ~alled the cenur line of the bar and its rad ius of curvature is denoted by R .
O
3._ Calculate_lhFie.dLS~°t it ia a standard: ISJC 150 channel section. The avo the (o) (b) ( C)
eeauonshownin. g. · "M• e - 2.12 cm. erage
thi kness IS t - 0 69 an . .11 ,.., . '
6&nge e · th sh ~nter o for the unbalanced I-section shown in Fig A
"- Locate. e_ the&r lane of the web. An.t. e = 1.02 cm.
simple bendmg
. ' 1or
in eP

r2.5-,--S--,
_j_ IZZZ2'0,'.7VZZ7222}- -
T ,
T ~:ZZ:ZZ:ZZZ~--. -
Q25
,
5 0.125
-f
-·--r --+
1.5
0
I 0

1.5

o~s
0.25
...L•Z2~~"'77~'771..-
l:i
-+-
1.0
T
Fio. A F10. B Fm. 6.24

. S. Locate the shear center O for the C«ction shown in Fig. B, for simple bending In discussing the stress distribution produced by pure bending oi such a
in the plane of the web. Am. e = UB cm. curved bar, we make the same assumptions as in t he case of straigh t. bars,
6• A simply supported
girder of span l - 6 m has ISWB 500 section and carri111 namely, that transverse cross-sections of the bar, originally pbne s.nd
a concentnt.ted v!rtical load P .. 100,000 kg at mid-span. (a) Calculate the shear
normal to the center line, remain so after bending. Let ab~ cd denote two
: : - :~he vertical junction ~lane between a half flange and the web. {b) Calcu·
web ,t_... ear( s)tre:811 on th e horizontal jun_ction plane between a full flange and tbe
neighboring cross-sections of the bar and let d~ d~note the souul angle
. •.... . a T • 2 79 kg/cm•; (b) r = 860 k • between them before bending. As a res ult of bendi ng, th e ~rocss,-section cd
7. Reeernng tot.he C- . . . g/cm . o
flanges extend out a section shown in Fig. B, assume that the lips on the t_, rotates with respect to ab. Let od~ denote t he s m8ll angle oi rota tion.
what iH commonly =:mi!he ne~tre.l axi~ ins~d of in toward it thus produc~: Due to this rotation, t he longitudinal fibers on the convex side a,! the bar
plane of the web to the shear t-tlection. 'Y1ll this change the distanc? e fr~m ! are compressed and the fibers on t he co ncave sidt• are exre.uded. If n, n
AM. e = LI S crn. center and, tf so, what is the value of e in tblB ca.¢ denotes the neutral surf a ce, the ex tensio n of any fi~ r at the distance Y
~- A ~ant.ilever beam has the . de- from this surface is y( fl.dq,) and t he corre~ponding uni t elonga tio n is
s~nbed in Example a, page lS8. ~tion sho"'.n in Fig. 6.18 and is loaded a~unc-
~1on b~tw~ the web and the low alculate the mten.'iity of shear stress on the J dcd y(6dq,) • (a)
1
Lt diP8 = (r - y)dq>
~a prmcJpal plane ; this means
~ ~ ~t:oy J>Oint
. ;i
2
g/cm ,
in the sectiona:~ ~fut.
fta.nge. Hint: Note that the beam is not 1
(e. 9) must be used to calculate the beO,411'·
t eq. (6.l'l) is not directly applic&ble,
E

where r denotes the radius of the ne ut ral surface and the denominator m
eq. (a) is the length of the fiber bet.wee n th e adjl.\\lt'nt cross-sections before
II EAMS 169
....,.,£5 rN BJ:Alf.8: DEND INO STRF. 88.F.8 TN CU RVF..l> B
ST.R ~

168 . latera l press ure between t he 1on ·


. hat there 1s n0 dista nce y from the ne u t ra l ,., git~.
t
:· "g Assuming d. t ., Utfa 0 or A
g strt>SS a .. · !I = R - _ , (6. 1-1 )
be.nd .. ·
din&! fibers,• the bcn in

" == Et ==
Ey(.o.dq,) .
(r - y)dq,
ce i~
(bJ
!~A
distri butio n is no longe r linea r ns in th R is the initia l radiu s o f. eu rvat ure of the
crnte r line of the bar.
of
Eq. (b) shows that ~ie s~~~ lows a hype rboli c law as show n in Fi;. c~ where fo r the tibcr st r ~;j in tRrm s of
1 distr ibute d
2
· 4c. Eq. (d) may be used to uht a rn a form u la
st.raight bar:-, but t aht •h: sum of t.he norm al force s O
Ver th eq. (d J Gi h rst si mplif ied a.s follow -. .
th condition t ·tat. ~ be concl uded t.hat the neut ral axis is di I e the bend ing mom e11t. The in tcgru l it1
f rom e_ .
CJ"(l69-SeCtton LS zero, i can d h f space
curv ature f
d
f(y - r_!Jj_ )d A = - !y dA + rf ryd- Ay. (e )
from the ccntrol o
b&r. •"L-
.d· f the cross -secti on tow11r s t c cent er o

f
·
e,rtarl flular cross -secti on, the shad
h
o the

ed area (Fig . 6 .24 c) in


f y2d ,'1
. r-Y
= -
- y

The fin;t intP.g ral on the right s ide of eq .


(e) re prese nts t he mom ent of the
In ux: c!lS!e o a r ne u t ral axis , an ti the :,C<'o nd, u.s i ➔
" '11 .
grea test bend ing st res.
. t equal that in comp rcssw n ; henc e t e . 1 c•ro:-;s -~ct ic1nnl area with resp<'d to t he
tenSJon mus
acts on the conca \·e side. In orde_r t.o make ~h~
stres se s m the most remote :-W.t>ll from NJ . (<'), is equa l to
zc ro . H e ncc
.:.1 ____ in tension and in comp ressw
n equa l, it is nece.ssary to use sect ·ronnl
n ut:-1" .
shapes 1,1,·hich have the centr oid neare r the conc
Eq. (I,) contains two unkn owns , the radiu s .'
ave side of the bar.
of the neut ral .surface and
displ acem ent due to bcndi ng.
I y'dA = - [A ( - y )] = .-1 y .
r - y
( i)

the angle &ftp which repre sents the angu lar. Eq . (d) then IJ(~<·omes
. t hem, we must u~ two equa tions o f stati·cs. The first eq
T o detenm ne
the norm al force s distri bu~:
tion i:, ba~ on the condi tion that the sum of
d equa tion is based on the
over _a_cros.. ,~ctio n is equal t.o zero. The secon
force s is equa l to the hendin 8u h:stituti11g this in f'q . ( b ),
rond1uon that t he mom ent of these norm al g
mome n t :l/. Thus Jl y 1g )
(]" = Ay (r - y f
~dA = E'(~q ,) ( ydA = 0 (c)
J° dq, } r - y '
ThP :-:trcs:-.p:-,; in the most r<>motf' fibers wht
e h a re t he m aX1m um ~tn>s..--es iu

~ dA = E(Ad1 p) ;·y2d A = M. (d) tl1c- liar arc


J° d,p r - y .1/ h1 .lfh~ tti 13 )
Cl,n ., a = a.11d Ucntn = - .-t yc ·
The intc~ration in both equat ions is exten dt>d
over the
• total area of the A.ya
cro88-Section .
1w1 1t r ..ll a xis to tht.• m o., c rcmo lt'
·
Eq. (c) en1:1blc.s one to deter min r_.an d , Ill
. · .
nce y (cons1drred
in whid 1 h1 u11d hi !HP thc distaHc·c:-: fro m the
e turn, the dista r r:.1di i of the O.:lr
a positi ve quan tity) f h
rom t e centro1dal axis · h fib<-r:-;, 1\11(1 a a11d c [1ft' t hc i1111f'r at:d ou te
section. Let u represent the d.ista to t e neut ral axis of the cross· tl1f' ea.,;c of pun· lx• tH.i ir,~ \\ hf'n.' t!i1.· h1r i--.
H~, far Wf' hav1\ con;-; i<lf'rl'd
nee from the cente r of curv ature to any gerw r:d 1':..i......:(' whl'l l :1 ,·ttrn:-J ba.r b
eleme ut d.4 · then y _
- r - " , and eq . (c) can be writt .cn :-;uliJC('tt>d to cnd eoupl C's 0111.v . 111 a m u rC'
'
hv f · · ·It :- p Ianc or· ~ym m f'tr_, . r h,, !·u rn •~ . \ dil· l)!
hP11t • t r1\1 t::.-\' ('f"l,;(• nr<.·c~ !l.<'tmg
· 111
·
' ,- idl' .,f :rny l'l'l >-.--S-s('dion m:1y be
red tlt't><l

from which f- -;---=o,


(r - v)dA llpou the porti on of thC' har to om
lo u t·oup le and a fon•p appl ied a t tlw et•utr oid of t he t.·ru~ diun
cxt<'rnul fo re(':- wir h n•~pe rt t 11 thl·
. T he

rnomC'nt of this coup le ct1ua ls tha t of the


cc11tmi la J I a.xis· of the cwss -sect1 0 11 . Thc :-t~~~,. prod u<'t'd by t hc ,-. >u ph~ n rl'
.

•The e.\ w:t t.h~ . .


-1~·
T - A

V
(6.13)
11
lcu oht.u ined us cxpla inerl abov e . The
·1 • I f
j"uent:-. ' a Iu11g1' t llulllU orcc N 111
fnn·c i~ n.- sul\'(-d int\l t \, •> eom µo -
· l h C l 1·lf'(' (' tl·tlll t) f t h(' (1UJgc nt t o r hC C"\' tll l' r
T he
at.anti.al effect I) I U()Wa tbat t.h . b· 111
, (' '. f the• bar u11d ll shcu ring
1 for<'(' }' i11 th,, pl:ltl l' ti( rht' (' n),-., - ,-'(_'t"CIO ll
oo !.he nre.ai, • IUld ere Ill a certai n rad 1, 1 it h&B no 911 · · · Iy 11·
~I ·,.
rrir
can be neglec ted. • prc1111ure but that ongal 1tdi1 1:.1 I f or1·t~ prn d 11<·1•s tcn - a·ll'.' or ,·or11 p r{'..,,1, ,. ,-t l"l':-.-,l'~ 11111J on n
_...,.gs 1H sSAJ'S =
e1'a.-,-,
11 > DENDHW 8TRUSZ8 l.N CURVED B.E.A.MS 171
170 al to N / A . To get the total axi I
. the ~ t i o n _•nd eqture~ is added a.lgebra.ically to tha at, (roCfl which
:ting
ted over .
. --J bv the oouple.
. 1fom1 s
in .xiY fibtu-, this;:e transverse force V produ ~es shearing str r~
tre~-s over the cro~-sect1on ca n be tak ,
e 8l

caw,.:u , · of these s """ P.n th


e
. h + hi = 10 cm, we find h 1 c: J .00 cm , h, ,.. 6-.92 an. Th~ l.ocates t he
an
d tl>e distribuuon
straight b&r-
The n1nnce'
raJ is of t,he crOM-Secwon •: an d l·U· ,..~awi
...1 ·
r - a +
t
= 8 + 3.08 =l l J ~.. a,
ne~t __ }uxs of the neutral axis is aoo defined by eq . (6. 13) wherein
same &! for. . nurneric&l va.lue of the ~ti(.. _- m.,,JO'mln for the
n,ouL.& 1. Det~ the -~tion in pure bend mg 1£ R = 15 crn and h ~ or
beam of rectanglllar c , ~ IO Cltl
Th1' rau•
Ad: _!.
• . -;-b + f •
dv dv
\.BJ + l.n {18~
(10\
• oum,d
~wrr~-
From eqs. (6.15)
h
O'a,a.a iC,
f 19
-; = b 1ln \JO) = 0.223b , + 0.588,

~ • hsa (h) "° that


. d Ii • 'l&/2
•here hi • h/ 2 - Y an '
+ v (see Fig. 6.24b). To calculate y, We,,_
....
r
=~=
f ~
zjl
0..223b,
+8
+
X 2
0.588
,.. 11 .00 cm.

eq. (6.14) in which ,

f d: .. J.< r: = b lnG) = b ln(i~ = 0 .693 b.


This gives b, = 20 cm. . .
MJ>LE 3. A semicircular curved bar 18 loaded as shoWD m Fig.. 6.2tia and ha
th~;apezoidal cross-section shown in Fig. 6.26c. Calculate the teD-•ik iStresB ,; , a.t
ThW! · t l il a = b = h = 2 c:m and P = 500 kg.
bh po:LUTION . Considering that portion of the b.:ir above \he section 1-2 (F1g.
; .. R - O.S9Sb = 15 - 14.43 = 0.57 cm. 6.26b), we see that the s~ress resultant _o n this section consi.s-.a ~f ~ f~ P ~~
at the centroid of the section and a bending moment M_= PR woere R G the rad1WJ
Then h, = 5 - 0.57 = 4.43 cm and hz = S + 0.57 = 5.57 cm. With these values of the centroidal axis. The extreme fiber stress at pomt 1 d ue to ~ mom~nt J!
of 111 &rui h,, eq. (h) becomes will be given by the first of eqs. (6.15), an d that due to the fo rce P will be SJm ply
O'mn _ 4.43 )( 20 _ l
Pf A . Thus the tot&l stress at point 1 is
59•
O'mln - 5.57 X 10 - · PRh l p
(i)
o- 1 = Aya +A
llAllPIZ 2. A curved beam with a circular center line has the inverted T-section
~ in F~. 6.2~, and is subjected to pure bending in its plane of symmetry. Find For the given cross-section (see Fi~ . 6.26c) we find
tbP dimenaion b1 m order to have equal tensile and compressive stresses in extreme
fiben. b + b/2 3
A =- -2 - X h = 4-M = 3 cm••
80LUTJow. S-moe we require I O'mu l ,. l O'm1n I , it follows from eqs. (6.15) that
h1 h1
-=-,
a C

p
R f ~1<
= -"-- =
vdA

A
vldv
_c=--"--
lM
f--2-, p
b
I ~
f dA bf"
- =- dv = -(c - a) =l cm.

I~ +
c•l8 h A V C O C

I a
0
Y=R-J":A = 28
9
3
-.-x> , >~ g
t

~
a

!i-~ t II
= ii = 9 -
:tt, I
T - I cm .
I J~ h, R - a - 2 -
f

LL._
F ro. tl.26
-1 D•8
p
(o)
Substituting these numerical valu<•;: inl-o ,-q. (i) , w~ obuiu
~ X 'l~/<J X l 500 ,, :'>00 k 1, t
u, ... 7x l /9 X :! + - r "'" ... s1~m .
.F'10. 6.26
sTRESSJ:8 IN BmAMB : II >
ITJ
,aoau»UT 6.6
·ca1 ,'&lue of the ratio ",_./ "min fo~ the C&8e
1. Determine .the nUDl:ving • 25 x 2.5 cm square cro~tion if the r or. P1111
bendiogo!. curved
n••~.ature of the ~
1::ta1 axis is R - S.75 cm. An,. <1...,./ amlr; ... l.59 &d111aor
. u•-· centerline b&S the T-section sho- : i;,,
7
.,.... '--"",..-~a ctrC uw Tb . " U 1n
1. A cun'«i ·...-u I • in iu plane of symmetry. e radius of 4•1g. ~
and is subjected tc P~ ben~ All dimensions of the cross-section ar~~1t11re Analysis of Plane Stress
of the c,onca,·e
abown except t.he
!~~; t ';·the stem. Find tho proper value of t 80 thaed. 1.1
. bending will be numerically equal. Ana. 1 _ 3 t the and Plane Strain
ut,reme fiber~ 1D ,l8c111.

• __
_L
8
__
~
~•.-:_ __ _L__L_
7.1 General Case of Plane Stre!IS
In preceding discussions of beams bent by t ransverse loads, ,,.-e have
:il'Cn hmv an element of material in the beam can be subjected to both
normal and shearing stresses on its edges n.s shown in Fig. 7.la. A gimilar
situation will occur in the case of an element of a sbaf t subjected to OJtial
Fro. A FIG. B FIG. C loads and twisting moments as shown in Fig. i .l b. Such a stat~ oi stres.,
P.
S. A curved beam with & circular centerline has the trapezoidal croSHection
ah.own in Fig. Band is subjected to pure bending in its plane of symmetry. The face
bi is the concave side of the be&m. If bi =-10 cm, h =IO cm, and a =IO cm, find the
proper value of b, to make extreme fiber etffl!se8 in tension and compression numer•
ieally equal. An,. b, = 1.62 an.
'- A cireulv ring has the V-«'JCtion shown in Fig. C, which is a 10 cmXIO cm X 3
~ ugie leetion. What ia the allowable bending moment in the ring if the compm·
11ft atff.11! &t the out.er edges of the lep is not to exceed 1,125 kg/cm• and a• 7.5
lb)
cm? A.a,. M - 9,4-40 kg<m.
FIG. 7.1
S. A steel crane hook carries a load P =- 2,00!
ti u shown in Fig. D. The cross-section mn of 011 the edge::! of n rectangular element in which there are no stresses normal
the hook ia trape10idal as shown in the figure. to its face is called plane stress. After such normal and sh(-ruing stresses
Find the tot.al stresses "• and ",. at point.CJ ~ and as those shown in Fig. 7.1 have be-en found, it is frequently n~ry to
n. Tb.e following numerical data are given: examine further the state of stress within the element to find the magnitudes
bi "'" • cm, "2 • I cm, a =- 3 cm, and c • 12 CJII.
Ans. u. •- 264- kg/cm'· f1' = 10• kg/cmZ, nnd directions of maximum stresses.
~· A circular torus of ~e&n radius R has ' Let us com1ider 110w the general ca1'C of an t>lement ur\d~r P!Me st~~
~n-walled circular cross-~ction of mean as shown in Fig. 7 .2n. The normal stress in the .r d irec~iou 1.s de~~tcd
d~met.er d. Calculate the radius r of the neutral by a,,_, that in the y direl'tion bv "•• and tension ~ ronside.red po&t.lvc.
axis for the particular case where R = d. Anl• The shear stresses on the edgrs· of tht> t•lement tha t an- 11 ormal t~ the
r • 0.866d.
7 0e · . tiO .r-axiR nre denoted hy r,,_, whilo those on the t'dgt.•ti uormt\l to the y-aus ~
• termine the numerical value of the :i·d denot«'d by r,,,_. The shear stn-se.ses r,QI , hav i11g u ci(){',kun~ ~nse of rotation
rr_-.../u,,,1,; for the ca.,e of a curved beam of 1 · • the clt>me11 t, are to U\~
· ms1de . ·d __..J. n,:1S1ttre in accordance
circular c . R • 12 cJll about a pomt 1...
ron6.I e,~-u r- • .
and the ~?8'J-aection in pure bending 1( 111. JOClfl·
F10. D A,i, •ame~r d of the cross-section with our previous rule (see p. 28 ). T he shear stresse~ Tr•• ha,•mg 11 ru u~ -
· u.,,_.;"•'• • l.97.
p 8
-rs,rB8 AND pLANJl STRAIN

174 pL,\NS . GENERAL CA.Sil o• PLAN~ 8TRU6
175
. sre negoJ,we. from the reqm.rement of e
~ sense of rot.at.Jon, (~- p. 30) , we have Tsw = - r n<lUaljt,,
bear stresses =" . . i-z , ~ J 'fo find dth~ l_~ti~n o/~}he gb.ne of maximum nonnal at.res!! ,,. we
of 00roplement&rY 5 shear stresses, 1t lS customary to ca~ ,nake the env1t1ve 11,. '.6¥' • from the first of eqs. (7.1) and obtain
• f Orthogona1 Uf!e 0
of this equality o t ~-a without regard to order of sub 11ly
• f these s re.,..,.,., SCti - (11. - u.) sin~ - 2T.., 008 ~ .., O, (c)
one notation r.. or to remember that shear stre11.-ft Pte
· · necess&l'Y
·1t 15 """" gi . ,
but in so doing1 . to be t reated as negative. "1ng from which
count.ercloekwi98 rotation are
y tan 24> = - ~-
..., y
u. - a. (7.2)

This condition.ffdefines two values of 24> differing by 180° and hen t


· b 90°
values of ♦ d 1 enng ~ . . For one of theae values, u. m& maximum ce ;wo
I /l and for the other, a mm1mum.
,
1
dA1 TdA,, Considering ~he aec~nd of eqs. (7.1) and setting the shear mea., equal
T,,dA, to zero, we ago.m obtam eq. (c) . From thil!, it may be concluded that on
those planes where a. is a maximum or a minimum the shear streae r
", Tllf , 1 dA,
vanishes. The corresponding normal stresses (u.) max and (a.). 1... an,
(al (b) called principal &tre,ae,, and the planes on which they act &re ea.Ue,d
Flo. 7.2 principal planes of stress.
Referring to the second of eqs. (7.1) and setting dT/ dii, - O, we obtain
Given the state of plane stress shown in Fig. 7.2a, the nonnal stl'e&t! (a. - u.,) cos 2q, - 2T_., sin 24) = 0, (d)
". and the shear stress r on any plane whose nonnal n makes the angle •
with the x-axis can easily be found from the equilibrium conditions of the from which
triangul&r element shown in Fig. 7 .2b. Let the area of the inclined f~
of this element be denoted by dA.; then the areas of the other two faces cot24)=~- (7.3)
a, - tr•
are dA. = dA. COi! ♦ and dA, = dA,. sin f/,. Multiplying the various
8t.r'et!8e8 by the areas of the faces on which they act, the total forces on the Comparing this with eq. (7.2), we see that the maximum shear stireSSeS
triangular element will be as shown in Fig. 7.2b. Then for equilibrium in occur on orthogonal planes bisecting the angle between princip&l pl&aes,
the n direction, we must have i.e., at 45° to the planes of principal stress a.5 already concluded in Art. 3--2..
To evaluate the maximum normal and shear stresses in t.be ele.ment.,
O'.dA. = O'.dA. coal ♦ + a,dA. sin' f#, - 2r,.wd.A,. cos f#, sin f/,. (a) it is necessary to replace q, in eqs. (7 .1) by the values defined by eqs. (7 .2)
and (7.3). Because of the transcendental character of the equstioM, this
Similarly, for equilibrium in the direction perpendicular to n, we must
ha.VP. becomes somewhat involved and, for this purpose, it will be ampler to
use Mohr's circle.
TdA .... O'.dA. COIi ♦ ain ♦ - u,dA. sin ti, cos"'+ r~A (cos'"' - sin';). The general case of plane stress is shown again in Fig. i .3a. To construct
• (b) Mohr's circle for this case, one proceeds as follows: Lay out first the
Equations (a) and (b) are readily reduced to coordinate axes a and T with origin at O a.5 shown in fig. 7.3.b Th.en to
2
Ua ""'O'sCOS "" + O'. sm· ♦ - 2T_., BID ♦ cos"'
.., 2 , f locate the point D representing t he st.ress conditions on the r-plaue. i.e.,
the plane normal to the x-axis, lay off the Yalue of "• as :i.b:.--ci.ss& OF &nd
= Ha. + O'.) + ½(q. - <111) cos 24> - r sin U (7.1)
the shear stress Tq, as a positi\'e ordinate FD. Xe.~t-, locate the point Di.
T • (a- . . 1111 ' representing the st.ate of stress on the y-pla.ne, by laying off the abecisss
• - O'r) sm • C08.; + T-.,(c082 ♦ - sin' t/,)
OF1 to represent the nonnal stress <1f and the nepti,-e ordinate FJJ1 to
• H~. - O'.) sin'> . . + r C08 ...,,
'>""
"'II J
1111 represent the shear stress - r~ . Sinc"t' t.he ~- and y-planes are orthogonal,
wbich a.re &nalogo to ' the corresponding points D and D on Mohr's circle tire 180° apart t.n.d
1.18 eqs. (3.2) in Art. 3.2. 1

~ I
r I7
6 ~ 81'Jd88
AJ1fD PLAIOI

h .
115'.l'JLAJ..N

diaroetBr. Connecting _t e&e pom ts with a et .


.....--

GEN ERAL CASI: 0 1' PLA.Nll 8TB.l'.88


177
- - n t the ends of a h -axis and the circle can be drawn DD "''«ht ~ t "• = + 150 kg /cm , T , . = - T . . = + 100 kg/cm•. Coll8truct Mohr's circle
2
•...,-~ terC on t e CT -sho
line Joe.ates the cen . . um normal stresses are represented . i.n. - ' h.' element and find :
. and rninun . . t 1n h , for a~ ~he magnitudes and directiona of principal stresses u 1 and u 1 •
The maxunum res tively. These pnnc1pa1 s resses are de r ig
~b) the normal stresa u n a nd shear_stress T on the diagonal plaM ah.
7.3b by OA and OB, pee== as shown. To locate their direct· no~ c) the aspect of the p~a~e of maxunum positive shear stress r,...,. .
and (e1.)m1n O't, • d' Ions .
by (o-,.).-s = cri . • t D 00 the circle, correspon mg to the tn ~LUTION . From the origin O of t~e UT-pla ne in Fig. 7.4 b, lay off OIY = - ~i cm=
. .,_ start with poin h Th .t-J>le.
Fig. 7..-, we bel this point ♦ = 0, 88 s own_- en fo reach .n, d D'D =+100 kg/cm 2 to locate point D, repre8eoting the gi ven state of stress on
nd
of the elem~nt, a la nding to the plane of maximum prinicpal ~ 1nt ::e .x-plane. Also from o, _lay off O_D' 1 =-+ I.SO kg/cmland lY1D 1 =- lOOkg/ cml to
the given state of stress on the y-pla.ne. Connect
.A on the circle, corresthporo· ugh the cwckwiM angle U.. Hence, the di- r~,
8
Joca te po int Di, representing
· h t 1·me to 1ocate t he center C of :\1ohr's circle and draw
it . necessary to pass l k . •<:Clio . ts D and D1 by a etra1g
.. . . 7 3a i8 found by laying out, ~lso c oc w~, the angle ♦• fro n :;ncircle as shown. Also label CD with rJ> = 0 to serve aa a referenee line . Then t.he
of ' 1 m ~ - . h
s The direction of "' 1s then at nght angles to ti·- Ill
the %-6Xl8 as own. h uat 0r
' . ·pa1 planes are located 88 s own. ( y
'1 and the pnnci

-·~
,,... ,,..
,,..
"1
~
..I
I
.
~
--x
7
0-t--r-=t,'---~'----'~:-+=-~ A (1

.,.,
(a) (b) ( C)
( bl Fm. 7.4

Fro. 7.3 meuured angle DCA = 24,.. = -135°, negative to show that it~ clocl:u::-ue. Hence
,p• .. -67°30' e.nd the direction of the principal stress 0' 1 is as shown in Fig. 7.u ;
the direction of a2 is, of course, e.t right angles to thu!. The magnitudes of cr 1 and,,
I. In general, any plane through the element whose normal n makes the are represented, respectively, by 0.4 = "i
= -191 kg/cm2 and by OB .. u , ,. - 91
I
angle • with the x-axis and the corresponding point E on the circle, repre- q/cm1 , scaled directly from the drawing . The S&me value8 can be ob.tamed from
aenting the state of stress on this plane, are related in the same way, eqs.(7.4) by substituting therein the given numerical valuee of trz , "•• and T.-, wrth
namely : the angle DCE in Fig. 7.3b is always double the angle ,p in Fig. due regard to signs.
7.3a and i!I to be measured in the same direction. The normal to the plane ab which bi!ects t.he &ngle between the ::r- and H>~
is seen to make an angle ,J, = - 45° with the ~ x~. H~nce from t he referenl-e_pomt
Expreanons for the principal stresses c,1 and "' are easily found in terms

l
D in Fig. 7.4b, we measure off the angle DCE .. 241 - -90" to locate point E.
of"•, O'_., &nd ,.,,.., from the geometry of Mohr's circle, Fig. 7.3b, as follows: representing the state of strees on this plane. Scaling the coordina~ of point 8 , we
find CT,. -+150 kg/cmland T = + 100 kg/cm 1 •
., - OA - oc + CD = •• ! ~- + ✓(•· ; ••)' + ,,..., (7.4)
Reference to Fig. 7 .4b shows that point F on Mohr's circle rep~ nta ~ plane o!
maximum positive shear stress in the element. T hen sinet- the ~gle ~" measures
~ = -45°, the plane of this m&Ximum po.,itive she~ ~ IS detined by fl "."
"
1
-= OB = OC - CD = "• +
2
"• _ ✓( o-• -2 o-11 )
2
+ Tn.
2
- ~ as shown in Fig. 7.4c. The magnit ude of this mLu~ ut~ sh~r s ~ is
T••• - CF -1-llkg/cm•,scaled from the drawing. As a check, )t wtll be noted t~t
the angle between planes of principal stress and planes of ma.um um s.hear stre.s5 118
These are the same values wh . h 1ue
of 29 from eq . (7.2) into th ~c would be found by substituting the va
e rst of eqs. (7. 1) on p. 174.
67°30' - 22°30' - 45• a11 it should be. .
EXAMPLE 2. Fig. 7 .5a repreBents one element of ~ plai.e m a _ t
a ta~ pl&ne 8 nda
and Fig. 7 .5b, the col'ffilponding Mohr's circle. Pomt D on t.h.i:, c.tn: e ~
tre.

poi
1. A aq l
ll.UQtLI!!
in Fig. 7.4a. The gi=•=t 0
~• thinPlate subjected to plane stress is ebO: to the x-plane, point D 1 to the y-plane, and poi nt.s A and _B, to e
th !:'6"~
1
on ,ta rnutu.Ily .Perpendicular facea are~. • - Planee, Point D ia vertically below D and point lY •· vertically abo '
·, '
r 178
p~
STB-8 ,uro PL&Nll 8TllAIN ►

of
.A"'•
,. For
3~0 kg/cm•;
. -
,,, the el?~ent
7 -L
i,uO~ m

O&M"&B.AL CA8~ 0~

on the diagonal plane ab &re aero. U_17•


11U:: shear stress 0;0'1 - 700 kg/cm• .
0'1 - -
PL.Urs 8Tll.'.al!I

Fi1g. 7.a, (7. -.20 .ltg1=:•and IT. - 0. What ia the


179

rr, - - 3!1() q ;cmc,wbat iJI the value


r.,.? What att the magrutudee oft.he principal nrm,ee o-1 and <rt?

. um perJDUISlble magrutude of the shear streeB Ti,y rf the larger principal 5trMI
~ o t to exceed 630 q ;r.m•? An.a. T-, _- 364 kg/cm•.
tr1 l.8 n

_ ' Principal Stresses in Beams


72
(b) Consider in Fig. 7.6 the case of a simply supported beam of recta.ngular
( 0)
Flo. 7.5 ross,-eection subjected to uniformly distributed transverse loacling .
~hen for any cross-section defined by the distance z from the left support,
the shear force will be
that the sides of the rectangle AD' ,BD' represent the directions of principal st,._
v. = -wl2 - !DX
in:e=~ce AB is a di&met.er of ~e circle, it follows that AD'B and D'B!Y
are right angles. A.JBo, since the &re AD JB equal to the arc DA, the angle A.Bf)'~ and the bending moment will be
eqtW to half the angle DCA. Thus the angle A!3D~ = 'Po, which d~fines the direc-
tion of tbe principal stress ~1, ~ shown .. ~h•s simple construction furniahl!II 1 wlx wx'
M =2- - -2,
convenient way t:o find the directions of prmcipal stresses from Mohr's circle. •
both of which vary with the distance x defining the location of the ~
section. The normal stress o-" and the shear stress T.., at a.ny distance y
PBOBLEIISET 7.1 from the neutral surface can be readily calculated from the iormula.s
l. For the rect.angular element shown in Fig. 7 .3, the following data are gino: developed in Chapter V. Thus, from eq. (5.5)
1
"•. -36flq/cm , _"• =200q/cm~,aod Taw -60kcJcm•. Using eqs. (7.4) and (7.2), deter- M,,y
mme _the ~1tudes of the two principal stres&E.3 o-1 and o-1 and the angle,;. betwet.11 u. == 1' (a)
the directiom ~"•and f!i• Au. "• -~cm•;cr, =-JBOkg/cm';q,. = -180 26'.
!leConmuddct Mo~s CllCle for the case of plane stress described in the precedin& and, from eq. (5.8),
P mu eternune therefrom the val f d .
3. Find the itud d . . ues o O'i, O'z, an ••·
in 1,1;_ 7 ? if magn • es an direetiom of principal stresses for the element, llholl"ll (b)
....... ,o tr,. .,. -350 q/ I 2
280qfcm•·tr,. _ ....,.. .. _ an.;_"•== 10 q/cm•,and T-. - 210 kg/cm'I. Am. 0'1 =
I TLU -e1CD)I • Y, ,_ 710 ~Al
· '· CoMttuet Mohr' · ' • th - .:n · ~ince M" and V., both vary with x, these stresses are seen to vary con -
problem &nd de.tennin: :ieffor e cue of plane stress described in the preoedirig
5•.For the rectangular e~r:m the val~es ~f <Ti, o-1,. and ti,•. tmuously with both x and y throughout the beam. For any pa.rticuhu-
are given: "i ""'810 kcicmt : shown Ill Fig. 7 .3, the following numerical daU cross-section, the bending stress u. varies linearly with y and ~ shear
th: normal etress tr., and, th;~ q/cmi,r.,. ""'210 .kg/cmt.Using eqs. (7.1), calculate stress TZJI varies parabolicall¥ as shown in Fig. 7 .6a. Fo.r a rectangular
30 . .Am. "•""1518 q/cm•· r ... ear stress r on the plane defined by the angle',. element such SB A or B, Fig. 7.6b, the shear stress vanishes aud the cor-
6• Conatruct Mohr' • ' 3f7 kc/cm•.
proble d
iB d ~~ determine therefrom
7 C 'Y ,; ... ~o.
t:
8 circle for th
8 ta
__,.,r
. te of plane stress de3eribed in the P""""'~
e va1ues of o-• and T on the plane whose norw--
responding tensile or compressive stress due t.o bending represenes a.
Principal stress. For an element C situated at the neutral surface, the
norma.J streSB vanishes and the element is in a state of pure sheu. For
. • onstruet Mohr's c· l
350kg/cuis.From th· lJ'C e for the elern . d ,. - •uch conditions the directious of principal st~ are inclined by ± 45°
th
angle;. between eons~ction, find th/nt_in_Fig. 7.3 jf "• ... o-., - 0 an ;"tbe
"i to the x-axis. For any intermediate element D, there a.re bot.h nonn&l and
-"1 =sso CB/cm•·• e ~ 0and the dir ti pnncipal stresses and "' and • ,.
a. Fortbeaq~; --fh • ec on of the principal stress O'J• Anl• di ebe&ring etre88e8 to consider. These st.rt)88 oonditiona for the element D
etnent ah.own in Fi rJ1 Ue shown aeparately in Fig. 7 .6o.
g, 7·4, both the normal 8U't'IS and thesb
-.c..Q
PL,OlS ::,,.,._
AND pl.A.Nil STRAIN
> PRINCIPAL BTRl:881'.8 IN B ~ 111
180
for points in the neutral surface of t he beam, the directiom of
08
~ al stress croBB the x-a xis at ± 4.5° as already antici pated .
'.
prtJlCl~culating q, from eq . (7 .6) for a. number of points defined by eA:r
~y t,es x and y, it is possible to const ruct t wo families of orthogonal
or<fJ.11& whose tangent.a at each point coincide with the directions of p rincipal
curves s at these points. Such curves for t he right half of the beam under
"i:rm load will be as ~ho_wn iu Fig. 7.6a. They _are called ~ 1!°1
I u 8 trajed,qriea . The solid line curves represent teruule strese traJec~o~
, (o)
,tru
d -the dotte d 1·me curves, compressive . s tr ess tra.Jectonea.
. . Both ae ta of

~.:y
an es cross the x-axis at 45° and always cross each other a t right angles,
terminate in the upper and lower free surfaces normal to the a.m
of the beam.
In beam design, one usually wants the numerically maximum valueB of
normal stress u,.. From eq. (7.5) it can be seen that for the most remote
fibers in tension, where the shear stress is zero, the longitudinal norm.al
stress us becomes the principal stress. For fibers nearer to the neutral
I
2 axis, the longitudinal fiber stress us is less than at the extreme fiber ; how-
( C)
ever, we now have a shear stress Tri, also and the stresses ~. ~nd Tsw acting
Fm. 7.6 together at this point may produce a principal stress., given by eq. (7 ..5),
which will be uumerico.lly larger than that at the extreme fiber. In the
The state of plane stress shown in Fig. 7.6c is simply a particular ca.5e case of beams of rectangular or circular cross-section, in which the she.!Lrin&
of plane stress as discussed in Art. 7.1, one of the normal stresses in this stress r.,11 varies continuously over the depth of the beam, this is not usually
c~ being u, = 0. Thus to find the magnitudes of principal st.ress for the case, that is, the stress (u,,)mn calculated for the most remo.te fiber
t.bis element we use eqs. (7.4 ), which, with u.,, = 0, become at the section of maximum bending moment is the maximwn stress acting
in the beam. However, in the case of an I-beam , where a sudden change
Ut.2 =~± ✓(~)2 + Tzv!• (7.5) occurs in the magnitude of shearing stress a t the junction of flange and
web (see p . 129), the maximum stress calculated
It is seen that in this case th . at this junction from eq . (7.5) may be larger than
,.,
. .
be po . . . . . · e maxunum prmc1pal stress u 1 will always
be
srtne 1 e tensmn whil tl
. ' ·. ·, '
· ·
e ie m11umum principal s t.ress u 2 will always the tensile stress (a,,)n,ox in the most remote fiber ~
; ega.t1ve, i.e ., compression.

I
and should be taken into account in design . •
- he d~l.lons To illustrate, consider the case of a simple beam
(, .2) which, with of these
_
principal
be stresses will be found by using eq.
u, - 0 , comes

t.an ~ =- 2raw. (7.6)


carrying a concentrated load P at the middle, z--"+.J ?.
auuming l = 60 cm and P = 30,000 kg. The cross-
section is an I -section as shown in Fig. 7.7; h = 30
~
~. l~
:

F ~ cm. h, = 26.25 cm, b = 12.5 cm, and b1 = 1.25 cm,.. ,


or known values of d /, -11,190 cm•. Then M mu = 450,000 q-cm, V ,nu. =
the directions of prin~~a~n t T""' this gives two values of q, whi ch locate 15,000 q. From eq. (a) , the tensile stress in the L - 1>__.J
ta A~~lying eq. (7.6) to el: ~eli.~s 171 and u~ 8..8 shown in Fig. 7.6c. rnoat remote fiber is lo'lo . 'i .7
ctf1..,.,, .,, 0, yielding ♦1 = 0 ents A or B, we have r = 0 and obtain
su 8-0ea of th '--· ' ~ "" 900 Th "" f ~ ka /cm'.
x- ""d ,.
-•
surfac ,·axes
e ~m , the directi·o
88 · .
ant,c1pated bo ns
·
of ·
us at the lower and upper rllf
·
Pnncipa.l Etress coincide wit the
· h
( o-.,. . . . = 450,000 x 15 _
11,190
:e, u. = 0, and eq. (7 6)& . ve. For the element C at the neutrs.l
· gives ta
n ~ ""' - a:, ,
4r. o.
yielding 4' == ::t: :>
PLANB~
/JlD Pl ,ANll STRAIN > COMBINEO BENDINO AM O TORSION IS3

shearing stres.9Cs : St rc 8 sc 8 r>u e t o Co mbin ed Bending and Tonion


norms! and 7,3
values of
111... 604
X 2~:7
=
5
= 529 kg/cm•; of t he m ost import ant nppli catio nl! of the t heory of com b-iucd
One 8 arises in the case of s haf t..'3 subjected to ~)th bending and torsion
f'.,, ,.,, 35-t qfcm'. st re~de of ci rc ular cross-section acted
r in Fig. 7.8, a portion of a s haft .
Const , .
(7.5), the principal stress is n.t it.1:1 e nd s by bending couples M in the xy-plane and by t wistmg
0
Then {rom eq. ( )
<1-,. max
== 706 kg /cm•. t1Po 1 5
coLIP e
r about t h e x-axis. Under 1mch loading, a n elemen t A in the t.op

( ) at the junction between flange and web is 1


lt is seen ~I t t11,.:;•in the most. remote fiber and therefore shouJadrger
than the ~ns1 e s , = be
considered in design •
PROBLJ:MSET 7.2

l. For the simply supported ~-bear:n


dbeiectussed flon p. 181d, debter'!llilne the splQ
lmgth l auch that (e1.)max at the Junction ween ange an we WI be equ&) ~
(•.)-- in an extreme fiber. Ana. l =- 99.4 cm. . I

%. A simply supported beam has an ISHB 200 (37.3 kg/m) section and Carries
coneent.rated !oed pat mid-span. Dete~ine the span length l _such that (u,.)mu a~ Fm. 7.8
r.he junction bet ween flange and web WJII be equal to (u .,,) mu m an extreme fiber
Aa. l • 188 cm. surface of the shaft will carry normal stress tT,, due to bending, &nd shearing
3. A ca.ntilever beamSOcmlong ha.s a rectangular cross-section 10cm wide by20an
derp and ani~ a load Pat the free end. Considering a cross-section at the built-in stresses r,.. due to torsion . These s tresses a.re readily compu ted from eq.
end, what is the ratio of the principal tension halfway between the neutral axis and (5.5) and from eq . (4.5) . Thus, we have
the upper surlace to that at the upper surface? Ans. 0.53. Mc 32M
4. A short cast iron beam has a span of30cm and carries a load P= 2,000 kg at the u.,, =1 = fli' (e }
middle u shown in Fig. A. The cross-section of the beam is an inverted T-tieCtion
with dimensioru; as shown, both flange and stem being5 mm thick. What is the maxi-
and
mum t.ensle stress induced in the material and at. what poiut does it occur? An,.
(I~ 448 kg/an 1 , Tr 16T (b )
= J = -r<P '
C

r.,,.
1 15 -157
p
r .,, --- --- --- ..... - - ::==-
where d is the diameter of the shaft.
The element A is seen to be in a stat~ of p lane stn>ss a.nd the principal
,

A+
Jl I

,-.10.....; ,
I / ,/ / stresses will be found from eqs. (i .4) of A.r t . 7 .1. Thus the p ri ncipal o oril.liil
streSSf>s are

Fie . .\ (7.7)
,:, R,J ' . FIG. ll
v. cernog to the ~ m
Mohr's circle for the state f 11upport,e.d d
an_ loaded a..~ shown in Fig. A, constntct

[ sing thu, cm:IP fiutl the


i
~d su~port and th" load u.s:~ess at 8 . pomt . I which il:. midway between the left·
. al~ midway bet~·een the top and bottom surfaces,
The maximum s hearing stress, equal to half the differen ~ be-c~7een prin cipal
str~&;es, is
at th.iilp_omt. A ,11 11, a, -=l;~a~1tu~.e nnd d irection of the principal tensile stress a1
6· \-\nte the differen t ·
for th ·1
cm'¢ '=- 54°20 '.
ta1 equation f h · Tm&ll = CJJ -
2 CJ1
= ± ✓(a'.J~)' + Ta
I
.
t7.8)
e cantt ever IJP,arn of rectan I or t e fa mi ly of r>rincipal stress trajectories
Ii' gu ar cross-section shown in Fig. H. 1n the d esign of a ~ha.ft to ca rry a givon lo&i1ng, allowablt.> working
.1n.,, (4 - 11•)(d11)1+ 2.z dy - (h' ) :stt"e$,es in tensiqn (or comp rcs.sion) a nd in she.u will usuall~• b6 pres.aribed .
dz y dx 4- y' "" 0.. It th en becomes necessar)' to fi nd out froro CQS. (".7) aod (7.8) which of
184 c o1omn:o Bl:NDlNO ANO TO""
design and then to select the rcqui~d ct· n,c,lON 185
the
these stresses go vems . For · l l' k
a brittle materia ·
I e ca.st-iron t h
ta
tne,.
_, rdmg1y. . ' e . ~r ed . H owever, when bending by t ransve,..... f • b.
d of the sh iu t acco Id be used, while for a d11ct1le material like
lllnit 1
tnurri regard 'd . ' "" orces 1a com med
WI
' th torsion, some co ns1 era.t1on of the shear str d . h
0 .....,.0 •
normal streAA ~ 1 shou hear stress r..,u is most commonly used . structul'>.I , . · ..._,., &MOelB.te Wit
!)ending m~Y be neceSBary. Consider, for example, the abaft loaded a5
steel, the max1mu~red s diameter d of the shaft, it is neccs.sary to b
T find the requi shown in F1~. 7.9 . I n such ~ • a surface elemen t A o n the fron t side of
·
o . ( ) d (b) into eqs. (1- .7) an d (7 .8) . D omg·
this w
su Slit
u4 he !lhaf t will sustain shearmg stress
expre...,;s1ons a on ' e obta· 1
111 , 16T
o-i = ~ (M + ✓M2 + 1'2) r rv = rdl (c )
(7.7a1
due to the torque T and also shearing stress
and ,, 4P
r '11 = 3A (d)
(7.Sa) due to the transverse load P. Thus the maximum shear stresa r.,. =
r'•r + r" sv · Even though the normal stress u. for this element v&o.i.3hes,
The n t.o select the diameter d on the ha.sis
. h . of a.n allowable working stresi
c:. in tension , we use eq. (7.7a) w h1c gives it might be in a worse condition t han an element B at the t op surface.
EXAMPLE _ l . A steel s~ft supported in bearings at A and B e.nd carrying ~ya
d = a/~ (M + ✓M 2 + '1'2). ,t C and D, JS to tralll!m1t 100 hp at 500 rpm from the drive pulley D to the offt&k.e
"\J 'l'O'.,, (7.9) pulley C as shown in Fig. 7.10. Cfhe following numerical data are given : P 1 - 2P 1,
Q1 ,.. 2Q 2, Rd =15 cm,R, = 20 cm,l- 1.2 m, a =30cm,andthe working lftnm in shear
Similarly, t.o select the required diameter d on the basis of an allowable is r,. =420 kg/cm•. Calculate the required diameter d of the shaft.
working stress r. in shear, we use eq. (7.8a) and find SOLUTION. From eq . (4.7) , p. 73, the torque required to tnuwnit 100 hp at
500 rpm is
d = a/~ ✓M 2 + Tl.
'\Jrr. (7.10) T -
71,600 X 100
500 = 14,320 kg-cm .
Then since T ,_ (P, - P,) R" we find, with the given dat.&,
If allowable stresses in both tension and shear are prescribed, we calculatt
d from both eqs. (7.9) and (7.10) and select the larger value . P1 -= 2P, "" 1,909 k.1 ,
Usually the bending of a shaft will be produced by transverse for~
rather than by bending couples as shown in Fig. 7 .8. In such case, we have
to consider also shearing stresses associated with non-uniform bending.
As_observed in Art. 7.2, these shearing stresses usually vanish at those L
pomts where the normal stresses due to bending are a maximum so thal
tbey do not influence the maximum principal stresses and can be dis-
y

I
I
I
(o l
I

p
F10. 7.10
F 10. 7.9
I

~ ,.,.... _!:!\,£,· r- - : ;1:- 1.I,"e -:.i :.~ ~::r ~ Ii.:i!::.. ~

-~ = 9 = :...:r_,Y~'"? ; ,S:, = ,,-~ ~ = rt'"' ?:?"

L :--.. . r~ tr:.:!..'.: u-: ,7 .r ~ :::. ~ ' ¥ ~ ~ E u ;.t ~ x-:-a .1 ~ ~


~~ C ~ U ~ ...:.. J .;z_ 1._ ";.-7~
=
-'"' ~--- - ~ :7 .l. " " ~ ~ '-- -' ~ ~
~ . ~-:r.;¢: :I. ~ ~.r,~-==-~ :..:r. ::.,j,,. ·~ -~ :x.; .t=. ;;;-,,,: t>F"= :n...e.- ,;I ~
~ ~ ~ .q :_,.-_ ~ ~-: ;.__:? ~ : : q -~ ..=.Rr ~ +..Cl!:. 7~ ~ ~ - c
~ ~ !2 ~ u:a::-: =
';.:e ..-_- ~ ~ C"'!: -:_. - ~~ ,-.:.;_:;n"'.l iX -,. ~
q.~ ~ - ~ - ~=-
~ !:,.. q
~ - .,.. ..:_7 . ~ ,;;:;;;:.,'
~
,T !. -i
~ -r
-,
_,., -,.... ~

---~~ ,.

~-
\
" •
/ .
W .J..
'
,-r-.. e
-
p .,..,Q AND PLANE STRAIN
PLA NE sTR.,._,..,
► COMB I NED BEND f NG AND T OBS ION
189
188 . .
orted in be&nngs carnes a , J-cm-<lia = lve t he precedi ng problem if the load Pat B is repla--' b t. ._. ;
. ter shaft supp I -L ft h . F· ,.. e~ t I So f h. h · "d ·h h . v<..-u Y a ,.,.u.., ng cou p1e
2- ~ G-c- ni - dtame h 0 ·ng end of t 1e !\na as s own in 1g. B C ll1J.U, ~be pJanc o :' ic_ c?mc1 cs v.~1t t e vertical plane through A B . What. rs t.b.e
weighi~ 250 kg •~ an over 't .'re section mn if t he horizontal belt ten~ alclllt~ f , ·tudc of th is pnnci pal stress , Ana . q, .,, 90° · (J' .,, 32 T _,,
. d I . , 1
. I t sile stress a w, one a , agrtl h . t r w.
the princ1ps en .., I 120 kg,'cm' • . . l'l! ll 111 Referring tot e torsion pen u um sho wn in Fig. 7 .1l a, we hAve for lh.P ~ha.ft
shown . An., . .,., . . d t ine the required dis.meter do( t he shaft to
c ta.~ :•50 4
cm, d -= 3 m'.11•~-&4 ( I0) kg /cm• In 8Uch CMe , what is th~ maxi mum allows.hie
3. Referring to Fig. C,ll e -~ ~.o pulley D. The power is a pplied at by 1 I ei ht lF of t he dis~ m order _that the pendulum may hAve an amphtode ~ = l2"
50 hp at 300 rpm from P~ :~rough a ve~tical belt. Each pull ey is 75 cm in~-10ri~. "urfng torsional osc1llafion without exceeding t.he allowable worlung st.Ju& T . =
tal belt and taken off at . on the taut side to that on the slack side is p ~llletei ~ kg/cm• in shear? Am. W = 26.4 kg.
an d the ratio o! belIt umston ,1•1 "' 3'
Use r. ""420 kg, m 1 . A M•
I
,..,.
d = G cm. .
_ Analysi• of Strain

I: I r 74

' T£ C
Very often t he s trains ind uced in an element of material s:u b~ted t-0
lane stress a.re of prac t ica l interest. T o completely define the slate of
! dz
ptrain in the xy-plane of such an elemen t, it is necessary t-0 s-peofy t~
s d .
linear strains f,. an f11 m two perpendicular d irections a.nd the change io
FIG.D angle be'tween these t w o d irections (the shearing strain..,.,.). In Fig. 7.1 2a,
Fro. C
let OX and O Y represen t two su ch perpendicular lines of u nit length
4• A stepped shaft of circular cro~tion is supported in a bearing at c, Fig,D coinciding with two edges of t he ele ment before deforma tion. Dunng
&Dd another at D (not shown) and cames a ~ r w~~l at an overhanging end .i'
The gear wheel has a pitch radius r =10 cm and_ m ~nvm~ another g~ (not shown) deformation, point X moves to X ' a nd point Y m oves to Y' relatit,·e to
is subjected t.o a vertie&I tangential force P at _1ts nm. Fmd the _ratio d1/d, of shalt the origin 0, X' and Y ' re maining in the plane XOY . Then t.he st.rain in
diameters such thB.t m&Ximum shear stresses will be equal at sections B and C of I.be the neighborhood of O is completely defined by t he two exterurion..s , .. = XX'
shaft. Am . di/d, = 0.83: and e., = YY' and by the change -y .. in the original right a ngle X OY
5. A 90-cm lengthof2-cm-diameter steel pipe extends horizontally from a concrete

'
-.all. A workman fits a pipe wrench to the end of the pipe and pushes downw&rd
with a foroe P on the end of the handle, which is 37 .5cm long. If the section modulw _Lr" y
for the pipe c~ tion is Z = I. l cm• and the allowable stress in shear ii
r. •l,400 kg 'cmt,wha.t is the safe value of the force P? Am. P = 31.6 kg.
6. -~ hollow steel pipe is to be used as a standard to support a highway road sign
as shown in Fig. E. The maximum wind presmire on the face of the board is warned
TTI
\
r 8

~ be 2JO kg!m~ . The standard is unsupported laterally and its outside-to-iruide


~ r ratio 1s 1.12. The allowable working stress in shear is given as r,, • fiOO
_LJOI - .._-+-x·
X
0
kg.'c.m'.Calculs.te the required outside diameter d of the pipe.Ana. d ""' 13 cm. 1.--,---.J •• C-
Tlir--~--, ( 0) (b)
90 Si;n
boord Flo . 7.12
p
as it deforms into the final angle X 'OY' . Extensio~l strains ~ considered

11
P<>eitive and compressive strains, negative. Tht> shes.r st n iu y ,.. will be
F10 . E considered positive whe n t h e right a ngle XOY is i ~- \V~t~ tb~ sign
convent ions, positive st rains will be see n to t'orrespond to positive s ~
(/
aa Previously defined . . .
7• A t.baft of diameter d be . R~ We now conside r the follo wing q uestion : W hen a stat.e of ~tnu!1 Ill. one
~uilt-i u at A and loaded at 13 b nt in the form of a semicircle AB of radius f th' plane is defined by given values of ~.,,,, and ..,_, in o~ogo~I du-ecttons
O
rmg 11.;; shown in Fig. F. Th us ~ a force P acting perp~ndicular to the pJane tiotb OX and OY, what are the corresponding str&U)l) ~tated w-i th the ~u-
beo<lm~ and torsion. Assumin ny cr~-se.ction C of the ring is subjected ~ eorJ tua)ly perpendicular directions OA. and 08 whitli ma k e th e an gle ~ wi~
of be nd mi _of !!lraight bars g ~at d 18 small compared with R so that the ~h -~
st resp a, will be n rnaxirnummo..yt UF,ed, find the v11lue of~ for which the pr1nCJ OX and OY , res pectively, as shown in Fig. 7. l 2 b? T o a.nswer thlS
. , ru. t/1 ... 120°.

\
A.ND PLANE STRA.IN .ANALYSIS OJ' STRAIN 191
~STBJ'SS
190 . ar element OXAY whoae diagonal 0,1 + t•) + ½(t. - t.) cos 24> - haw ~n 24,,·}
e. .,. \(e.
--~• n consider a rectangul . Fig 7.13. Let dx, dy, d e note the 1~ (7 .11 )
qun,wO ' X shoWD 1D
t.-be angle i; with O as I
.
of its diagOnal.
As enDI
a result of the ~\~ f6 ., J(e. + e.) + H es - e.) COS 24> + hew SlD 2q).
·ts sides and th the ength by the aD1ount e.dx, 0 Y elonga.~ bgi\'en eral expreBSion for the shearing strain -r. may be obtained in a
ol 1 ·
ri:ginal right angle YO X mcreases
stnins ta. .., ..,.., OX elongates
b Y 1l
y the a ~ A_ ge':n.nner. Consider, in Fig. 7.14, two adjacen t rectangular elem~n ts
amount t,dY, ~d tbe
O
three defonnations of_ the element a.re ::lltit siJnilar rtioned that their d iagonals OA and OB are mu t ually perpendicular
.., For clarity, tbe8e b The con-espondmg changes in the 1 ~ soProirom Fig. 7 .14a represen ti ng the influence of ts alone, we aee that the
... . - Fi""'- 7.13&, , e. . d . enlPlh
aepAZ'&tdY in -e~ ,,lb; 008 ,; +~Y SID~. an --y,.,p,x sm q, as h 'I' 'fheP in the original right angle BOA iB a.a follows:
oftJiediagons.lO.A aret+change in
The ne
OA
mthe algebraic sutn of these~t~~n
. h d'1rec t ion
' . obtained·~
OA 18
cbaDge
_ AA1 +
BB1 = e. dx sin ti> e. dx' cos tit +
• •LA 4:..,,_
m Wll'. ~ ... - _
quanuuea and the co.. ......--
...nnnding strain m t e
by LAOA, + L BOB I - OA OB th ds'
= E• COS ~ Bin q> + f~ 8ID t/> 008 ~ = l!a sin 24>.
1.. ~~-~.!~).
-,dy r ,, A y Y_.,<11 Si Similarly, from Fig. 7.14b, the change in t he right angle BOA due to the
8 Tr: //// ..,,,,, strain fv alone is
AA1 BB1 dy cos<? e. dy am ~
1
dy /.
/.'dJ ~ X - LAOA, - LBOB, =- OA - OB=
ft!
th d8'
I --- L.l
Oj__dz____..j
1--dx- --:i---f, = -ft, sin t/, cos q, - ~ 008 tp ain ii, = - i,. !ml 24,.

(o l (b) ( C) Finally, referring to Fig. 7.14c, the change in the right angle BOA ~used
by the shearing strain -y sw alone is
Fro. 7.13
_ A'A 1 _ BB 1 _ "'Y -. d:z cos ti>_ "'Y .. dz' sin 9
LA'OA, - LBOB i - OA OB - th da'
dividing this sum by the length tU. Thus with the observation that dz/ru
• ooe ♦ and dy/ rh = sin ♦, we obtain = 'Ysw coal ti> - 'Yaw sin1 (j) = -y .. cos 2ip.

fa s fs coe' t/> + e. sin' f) - 'Y-. sin ,P COS ff). (1) The algebraic s1..tm of the above angle changes gives the net change in the
angle BOA, i.e., the desired shearing strain "'YO!b as follows :
~strain~ in the direction of OB may be obtained simply by replacing
• m expre&mon (a) by 90" + ••
giving -y""' = (e. - e.) sin 2t/, + 'Ysw COB 24>,
B . f6 = ls sinl t/> + t, coel ♦ + 'Yaw sin ti, COS•· (b; which can arbitrarily be written in t he for m
Y rn&king
oor the trigono
♦ ,. ½(I + C08 ~)
tri bs · •
m~ c su t1tut1ons sin• q, = ½(I - cos 2,), ha = ½(e~ - Ew) sin 2c/> + ½, %11 COS 2•· (7 .12}

{b), they 111&y be writLn~d ewth • cos 4' = ½sin 2q) in expressions (a) and Comparing the first of eqs. (7 .11 ) and eq. (7.12) wit.h eqs. (7.1) of Art.
~ m e form 7.1, we see that there is a. complete analogy between a state of plane st.rain
and one of plane stress. The linear s trains t z, , .,. and ~ i..r, cqs . (7 .11 } and
<7-~2) correspond, respectively, t.o t he stresses ir., is,,, and 11 .. in eqs. t7.l ) ,
8 , ~ while the halfaMar strains -y,_/ 2 and "'Y,../ 2 corl"t'.spond t.o th~ shear stresses
\
~
---
tis //
T■w and T, respectively. This suggest.s that a Mohr's circle for st.rs.ins m&y
ds' ~ 1/ 1/ / f. _j_. be constructed in the same manner a.s was p rEwioush' dont.> for stl'e.:!Se.S.
I 01 ____ J r,, d' I n ~ ig. 7. 15a, le t f'., and e11 repreaent givt, n ~nt'~ r s~ns in t~e ~ and y
U'ecttons and let -Yaw represen t t.hc eorrespoudmg shearmg $tnUn, 1.e.'. _the
/_d.A'• dx__..j f
11.Jn~unt that the original right angle X VY increases due to_deform~t1on.
(b)
( c) This 1:1hearing strain is represented in Fig. 7.15a by & c.lockWL~ rot.at.ion of
Fro. 7.l{
>
, rrf7Jt. ~ -ai~ de!onr~ .;:i sod ~ ~ ..:..ch ~ ~ ~ ~ c.:.d
._ ~ reepec'J7~~y , Z IL U T !'!l":r. &c.d & ~ f ro;;... ~ - o::rl,e.
• w,
1"' t5b, we !!P-e theft: p ~ ~ ~ 1'yll::5eLJ.ed uy -;,, r• ~
',t"'.od. BThe. swee
d~
~ b --;-e
~ LCA -
JJ(IUl.!6 Ie:"!") c. ~ ~~
~ o ~,l clr't e:::t"!ie red~ a·a ~
:iO ~
Cle

y
::=:
~ : · o( YJe princrpel ~
OX- ~ilarly ~ clru ,n e ~ D£B - ~ ~~ ~ ~
€1 ~ & doe~ ~ .-i ilii7.h ~
2
::=:
. · of princip&! stn.rr. ,., ma.us the ~ ~~ ~ ~ .riih the
OY . Thll3. iM prineipal ~ vi 11:ttam 01 and <n will b e ~
~ in Fig, 7 . I .x:. The ~ d .f} and - L~ 3Bel 1,b be ,. - O.A.
:C.,.. = OB on M~s -~ . 11.ey ue,il!to ~._.. 10 ~the . :u:5::::u:= ~
~um )ioear s 4 ~ m t h e ~ of U)e ~
Analytic eXP•eSl!l,O'll.5 for the ~ ~ eci &:X! ., b rzr:::::s a ~
~oel! of e..., e.., and -,.. can eaaily be ~ . fr;c. tt. ,rr:-ev-, ,;t. M:obra
1 ~ (Fig. 7.15b) u f~ :
11 =- OA = OC - CA ""' OC - CD,
ti = OB = OC - BC = OC - CD.
Then noting th.a r. oc = Ht• • t,J .t:me cD = "' K IYr + w lY? .
CD' = ½(e.. - ~ ) and DD' = ty.., e<;!-, (e) 'i:::e£r.rrr
1 ( . ,)1
E} =
1
~ i-~ + "t.) + '\ (f•- 2
- -ts ) "l'
' .2 '
fl .13)
ft = 2 ~- + t,) -
l(
\
(t·• - a,): . (.,•·
~ 2
):· -

Tbe clockwi.!ie angle ¢,i defu..:ng :he ~ n of ~ ~ ~ "'1th


m'erence to OX is given by the eq"MtlG.D
DD' - 7 .1'2 - ') ..,. (7 )
tan z+, =- = Ht-2 - c. ) = ~
CIY - '· 14·
c,btei:red from t.be trigo no?llC u,· of the ciiot aw# CDD' m Y~ 7. 19.:t..
Er.t11ations (7 . 13) and (7. 1-ll are seen w ::ie ~- ~ w ~ (7" ~} a.od (7 .2)
cl, Art_ 7. 1 lOI the magnitudes 3-DC a:m,c!m:.::S of p r ' ~ ~ - . -n:.e,, CSD
he t.nied for the. ca.Jc~tioa oi ~ it ~ and ~ uoo of ~ ~ram
llllhout t~ oecd of co n5r-ruclmg :\fohJ-'3 cm..-¾e. tf prefend.

h
A.NI> pLA,NS STBAIN
p ~ 8'1']lJl88 THE STRAIN R08E'ITJ: 195

19f . - -o.00014, tw - -~.00050 , "Y.sw -= +0.00o36_


p,obleDl I 1f •• -•· ~ _ -22°30 ,tllt. . the point where bb, intersect s the t-a.x.is ' The line DD 1, so obtained 1
2. Solve IO)-•·,. s -575 (10) tind
the magnitudes and direction 8 of . F
l l)elllg
·ameter of the reqmred
.
Mohr's circle which can now be drawn. This
.
(7.13) and (7.14), . defined by E. - +0.00050 , e_ _ +~l'Uici~ . the d l . d'
11 - -~ (
8 I~ ill cut the m te rme 1ate vertical bb. at a point p BUch t hat CP will be
S: U11fn.g: .state of pJane35stralll(IO)_.. fl • -135 (10)--t; 4>t - +39036'. - ~. c1rc1e w D . b
st,ral.Dll or A ft ., g , dicular to D 1, smce Y construc tion the triangles CDD' and FCF'
'Y• • -0.00105, "'· perpebenen made congrue nt. Thus points D, F 1 and D 1, having , respectiv ely ,
bave .
~ tb, and Ee, and being 90° apart . in countercl ockwi!le order in
The Strain Rosette . . bsc1,,,.,.. ~.,
' oc,Q.
s
7.5 . bJ·ected to plnne stress, 1t 1s ofu-n desirabJe I{) a . 7 J6b, represen t complet ely t he strams associate d with the direction
1
Of a materiad .SU t measure ment.
s·mce stresses cann t fig. b . and oc w h 1c . 7. l 6a. The princip&l strains are
. h are 45° apart .m F 1g.
In the CS..'lfst,res.,es o be
obtain the
by ,rec
.
f .
•res the measure ment o strams or deforma r ()(J'==OOA and E1 = OB. The direction of the principal strain e, makes· t he
th ' gle ,pi = ½ L DCA with the direction oa as
1 shown in Fig. 7.16c.
measured directly,_ ~ req~~erial during loading. Such measure ments Ions
an ½ cb a
to the surface of tahre
which .take Pia.<:ehm t e ':sitive strlf.in gages attached
usu
ally made wit very se
. . 1__ ...1_,.1 so that they can recor t ll
body before 1t 1s OSUt:U •
d h
t
d . loading. The question na ura Y arises as to how l-0
e amount of strain th
• at
e
~··~· ' ••---t
t&kes placeh untn~ ,_a-es so as to get sufficien t strain . data to be able,.~
arrange t ese s ram e-..
compu te t e
h refrom the correspon ding stresses.· An obvious d ·
answer to this oL1a
c 2 (o)
I
I A
question would be to measure the l!near strams E:r: an E11 m any two per. I 0
ndicular directions at a chosen pomt on the surface of the body and the I
♦,""
I
:ange in angle 'Ysr between these two dire?tion s . . Then the pri_ncipal strains I
I

could be found as explained in the precedm g article, and having these, the I
__,+...., -
oL-_-_____ o
principal stresses would be calculate d from eqs. (3.6) of Art. (3.2). How-
ever, accurate measurement of the shearing strain 'Y.1111 is found to be very
(b)
,--, ( C)
difficult. It is easier and more accurate to measure , in the neighborhood
of a chosen point. on the surface of the body, three linear strains Ea, ~. t,, Fro. 7.16
in three different directions and then deduce from these measure ments the
magnitudes and directions of the principal strains Ei and Et- Such a group Knowing the principa l strains E1 and ti, the principal stresses u, and O'J

of strain gage!! is called a strain rosette. One standard arrangem ent of the can be calculate d from eqs. (3.6); namely
11train rosette will now be discussed in detail. •
Eb and E are measure d along ui = (t 1+ µ«i)E, ui = (ti + µ.t~)E _ (a)
r Tke0045° Strain Rosette. IC linear strains E
o, I C
1-µ1 1-.u
mes , ob, a~d oc, 45° apart as shown in Fig. 7.16a., Mohr's circle can be
constructed without ambi guity and the principa l strains t 1 and Et can be This assumes, of course, that the material has not been stressed beyond it.s
nd
fou the refrom. This Mohr's circle will be construc ted in the following elastic limit.
manner: In the. t ' "r/ 2-pla ne (F'ig. -t . lGb) draw three verticals aa bb, an d ct, ' 7, 1711) re.daa.s follo1'8 :
h .· ' a:s- EXAMPLE 1. Data taken from a 45°-strain rOBette (see Fig.
a, ing respective ly the a b . . : (If any of the me ~d ,. ,. 210 microaKtrc S/_m .
ured strains were ' . scissa.s t,., th, tc, as shown • the left
~- = 750 micromct ro /m, '• ,. - I IO micromc t~m,
negative the cor d. . principal stram.s and the cor~di ng
of the origin o) Lo t h respon mg verticals would he to
1
~d the magnitud es and direction s of
Principal stresses. Assume E=2 ( IO)• kg/cm•an d µ = 0.30. .
vert icals aa and cc ~a he L e center C of the circle midwav
ass own . If th · . ~
between the two
. h left SOLUTION . Beginnin g with the f , "r/ 2-plane, Fig. 7,l?b, construe, !oo
v~t1cals

of C, as a.s,5umed here 1 • · e intermed iate vertical bb hes to t ~ aa, bb, cc, with abscissas tea = 700 units, '• = - I 10 urut.s, and •• ."" .. JO unrta, a.s
0
shown. Locate center C on the t-axis midwsy between t.he_.vertt~ aa and cc.
D 'I:, == C'F' and on the ~-eat~ ff on the vertical aa the positive ordinllf~
r ica 1cc ' )avJ 0ff th e negative . ordinate . D' 1D • -C
-
' l'~en since the vertical bb lies to the left of C, lay off the poeit!Vc! 0 • d m&te D' D. -
Cr' aJ . · t V · s 1·n, · 1- -ty !av otl the negat1ve

potn~r•· ~tbj~~:~
.
"For additional inforrnat.i 0
. ong t. h c vertical aa which locates pom - .1~ ' ; .

:.AG1,raphka1 !\fethod of R;~;.~""' tAhenalvs1e


str~in ro11ette see K J B t d G A. BreweVr, a~~~te D' 1D 1 = CF' along the vntical cc to !oca~
.,o. , p . 1.
..
" p
'
' · · 088&r an
roe . SO(", Exp. Stress Analysi•
·. y 0 I I ,
· Vert· D, as a diameter , draw Mohr's circlt> as shO\~n .9h 1~ ~ ~li.og OA and
•cal bb at F making CF pt'rpcndicular to DD1 &8 it ou ·

\
L. I
p
PI.Al" sTRJ1lti A.ND PL.A.NE STRAIN
196

8
Deflection of Beams

S,l Differential Equation of the El-..tic Line


( b)

C 0 In discussing pure oending of a prismatic bar in Art. 5.3, it wa.s shown


tho.t the curvat.ure of the neutrtt.l surface was given by the equation
Fm. 7.17
l M
P = E{ (a.)
the diagram we find that the principal strains are e1 = 1130 (IO)'•
OB.. ~%o (IO}-'. Th; angle DCA is mwsured with asa protrac~r a_nd found to~ Thus, for pure oending, where Mis constant along the length oi the bar. it.s
~o~,. Hence,, = 32°45', clockwise fro~ oa to ol shown m Fig. 7.17c. Su!). axis bends in a circular arc . It is customary to call this curved ~ of the
st.i ting the scaled values of t 1 and e, mto eqs. (a) above, the corresponding bar the elastic line or the deflection curve.
~cipal stresses are found_to ~ a- 1 =2~371 kg/cm•and 0'2 '.'"371 kg/cm•._rt w!ll.~
noted that although the stram e, lB negatn·e, the corresponding stress 11, ts positive.
In the case of a beam bent by transverse loads acting in a plane of
symmetry, the bending moment M varies along the length of the be~. a.nd
we represent this variation by a bending moment diagram. For such ooo-
PBOBLEKBET 7.5
uniform bending it is usually assumed that eq. (a ) holds a.t each c~'ES-
L Referred to the directions shown in Fig. 7.16a, a 45°-strain rosette gives the section. Thus the curvature l / p is seen to vary along the beam s.s t he
followillg data : e. = 670 micromctres/m, f6 = 330 micl'omctrcs/m, E, = l&l bending moment varies and the elastic line
micrometrCJ/m. Construct a Mohr's circle for this state of plane strain and find the C
becomes a rather complicated curve, the shape
principal st.reaaes 0' 1 and O" , and the angle tj, 1 defining their directions with reference I
of which is defined by eq. (a) . To express the
to :he direction oa. use E =2(10)•kg/cm•and µ. = 0.30. Ana. 111 =l,590kg/crn';
l

<12 = 753 kg/cm ; 'Pl = -8°33'.


1 shape of this curve in rectangular coordinates,
2. Solve Problem 1 if the rnea.sured strains are e0 = 232, Eb = 123, t, = -80, we consider any portion of a bent beam as ._ ,' - C~
1
all values being micrometres pa metre. Aru. u 1 =468 kg/cm•· a- 2 =-34 kg/cm ;¢1 = shown in Fig. 8.1. Through any point O on ,
+8"23'. ' this elastic line, we take coordina~ axes x and Y r1-.,
0 '
3. Solve Problem 1 if the measured strains are t = 815 · t 6 = -72, f, = 165·
as shown, the x-axis coinciding with the original ...,,,';_~ .....-:::_ '
~I~~ being micromccm per metre. Am. a, =2,400 kg/cm~;a-,=400 kg/cm 1;¢1=
straight axis of the beam, positive to the right, ' ~ ' ""I' .....-~r
-~:: :- - ...1
- ""l"
th•· ~ metimes,_ instead of a <i5°-strain rosette a 60°-strain rosette is used, i~ and t he y-axis positive downwards. At the en ds ,1e
t .ree in~ sirains ~• 4 • and Ee are measured ~long three sides of an equilate of any element. ds of this elastic line, we ronstruot '
~':~~- bro~e tha t m au.ch case, the center C of Mohr'll circle for strain will atwaJ'' normals which intersect at C, thus defining the Fw. S. l
ea ~~ eq~J to (t. + 4 + t,)/3. radius of curvature p of the element. Denoting
5· When a OO°-stra1n rosette 16· • sed · btained
according to the precedi u and the c·enter of Mohr's circle 1s O . 16 i th e angle between these two normals by de, which i.::I also the angle betw~n
that one which makes e ~~blem, prove that the correct radius of the ci~1118 c 1r; th e tangents to the elastic line at tht' end:; of the elemeut, we have t he
iwociated with the threeqt· zer? the algebraic sum of the three -y/ 2-ord .,y
• rnear strain • .
t.o construct the correct circle b . 8 t., f6, t,. This condition makes it
· f111·rly e.- relationship ds = pd.8, from which
y trial and error. de l (b)
- 5" - ·
d~ P
i
198
. .
DEFLECl'lV" w -

"thin the elastic range of its materi


r EQUATION OJI' ELAsTIC

twice differentiate eq. (8.1) with


LINE
199
1
For a beam which 18 bent WI.flat curve. Limiting the discussion ~I, the
If we to eqs. (.5.1) and (5. 2) respect to x, we obtain, with
elastic line will usu.all~ be a ";.ry 1 we now introduce the appro,cun ~llch reference
conditions, and refemng to Jg. 8. ,(b) becomes at10'1t!
d3y dM
<h = dx and e = dy/dx, so th at eq. EI dr =- dx = -V,
(8.2)

If:!= f (c) a.nd d4y dV


EI dx4 = - dx = w, (8.3)
"d r only the absolute value of d2y/dx2.
indir.ating that we conSJ e h y is the shear force at any cross-section and · th · te •t
w 18 f
Combining expressions (a) and (c), we ave wb e re E . (S . e m ns1 y o
distributed _load. quation .3) wil~ be fou_nd particularly useful in those

-~=±;- (d)
in which the load has a non-umform d 18tribution.
~ e application of eq~. (8.I) an~ (8.3) to find the deflection curves for
various transverse loa.dmgs of prismatic beams will 110w be shown by
· · this expression must now be chosen such that it will be
The SJgn severe.I examples.
m . . F" d h con.
sistant with the choice of coordmate ~es m ig, 8· 1 an t e definition of x,u,PLE 1. A simply supported pris-
positive bending moment as that whwh produces ~urv~ture concave up. Et·c beam AB carries a uniformly dis-
wards (seep. 96). For the coordinate axes as shown m Fig. 8.1, we see that ma i • • ·t
t "buted load of mtens1ty w over 1 s span
l
when the curvature is concave upward~, the sl~pe dy~dx i~ algebraically ri shown in Fig. 8.2. Develop the equa-
decreasing with x and hence <fly/dx2 IS negative. Likewise, when the :n of the elastic line and ~d the maxi-
mum deflection l> at the middle of the
curvature is concave downwards (negative bending moment), the slope
span.
dy/dx is algebraically increasing with x and <fly/dx2 is positive. Thus d2y/dx' SOLUTION. Taking coordinate axes x and
is always opposite in sign to Mand we take expression (d) in the form y as shown, we have for the bending mo-
ment at any point x Fro. 8.2
wl wx 2
(8.1) M z = -x --1
2 2
and eq. (8.1) becomes
This is the differential, equation of the elastic line for a beam subjected to d2y wlx wx' (e)
bending in a plane of symmetry. Its solution y = f(x) defines the shape of EI c1x2 = -y + 2 ·
the elastic line or the deflection curve as it is frequently called. Multiplying both sides by dx and integrating, we obtain
Equation (8.1) carries with it two important limitations resulting from
El dy = - wlx2 + w6xJ + Ci, (f)
the assumptions made in its derivation. First, the moment-curvature re- dx 4
lationship (a), derived in Art. 5.3, assumes that stress is proportional to te this constan t we note from
where C I is an integration constant . To eva1ua . d't' w~ find
strain, i.e., tha.t Hooke's law applies. Thus the equation is vaHd only for symmetry that when x = l/ 2, dy/ dx = 0. From this con 1 ton,
beams that are not stressed beyond the elastic limit of their materials. wl3
Second, since expression (c) assumes that the curvature is always small, C1 = 24 .
the equation is limited to the treatment of small deflections. Most beams and eq. (f) becomes
d wlx2 wx' wl3_ (g)
encountered in engineering practice will be well within the validity of these
EI ~ = - 4 + 6 + 24
two limitations. Furthermore, eq. (8.1), derived originally for pure bending
. account for. .any deflection resulti·ng f rom shear d e f orm at1·0 n of the
does not . Again multiplying both sides by dx and integrating,
matenaJ.
. The additional deflections du t h d f
e o sue e orma 10n t· wi·11 be dis- wl.r' wx4 ~ + C,. (h)
cu~d m Art. 8.4. _It will be shown there that for beams of ordinary pro· Ely = - 12 + 24 + 24
port10ns, the deflections due to shear deform t· II 11 and can h t - O when x = 0.
a 10n are usua y sma the condition t a y -
be neglec ted . The integration constant C 2 is found from
f DEFLECl'l01' or BEAMS

uation for the elastic line becomes


EQUATION or .. ·~
--aTic LINE ~
201
Thus C, - 0 and the reguired eq .: .. .- :i • l in eq . (k), we obtain, for the deflec•·
setloU"D •lOn at B,
y •
~<l'
24EI
-
2ll:' + r).
(i) c5 '"' Pl• .
3E!
To find the maximum deflection at mid-span, we set x ,. l/ 2 in eq. (i) &nd ohta· AIIPLE3. ~ simply supported Prismatic beam AB .
5wl' ~ J;lt hown in Fig. 8.4. _Locate the point of tnaxim cam~ a concentrat.ed load
8- 384E.i' P11,8: d the value of this deflection . um deflect1on on the elastic line
and 1.,~oN. Choosing coordinate axes z &nd y as 8h
The maximum slope 8A at the left end of the beam can be found by Bett' 80 own, we have for O < z < a
Pb '
: - 0 in eq. (g), which gives Ulg Afz = -z
l '
(dxdy) •·• = 8A = 24EI
wl' .
while for a <x <l
EXAMPLE 2. A simply supported beam AB carries ~ triangularly distributed 1
Pb
as shown in Fig. 8.3. Find the equation of the deflectio~ curve referred to the c :
Mr =l x - p (x - a).
dinate axes x and y as shown. Determine also the maXImum deflection &. •
SOLUTION. In this case, we begin directly with eq. (8.3) and write SU L&tituting these expressions for bending moment into eq (8 1-1 we obtain
' • h
tort e
uo . f h d fl t' . . •• 1,
two portions o t e e ec 10n curve, the following two differential equations
Ei'y .. wx. O<x<a
dx' l a< x <l
Separating variables and integrating twice, we obtain Eldly = _Pbx d2y Pbx
a'y u,xl
dx 2 l EI dx' = - - l- + p (x - a)
Eldx 2 = 6l + C1x + C,. G) Successive integrations of these equations give
Again, 11eparating variables and performing two more integrations, we obtain dy Pbx 2 EI ~ __ Pbx: P(I - a)! ,
EI dx = -2l + C1 (I) +
(n)
• wx• x'r dx - 2l 2 T Di
~=1~+~+~+~+~ ~ Pbx 3 P(x - a)l
Ely = -
61 + 6
+ D x + D:
1
To find the four constants of integratioll, we now note that the bending moment,
represented by eq. (j), and the deflection, represented by eq. (k), both vanish when (o)
x = 0 and when x = l. From these four boundary conditions, we find where C1, C2, D 1, D 2, are constants of integration. To find these fo ur constant!,, we
wl 7wl• have the following conditions:
C1 - -
6, C, = 0, Ca =
360
, C, = 0.
Substituting these values back into eq. (k) and tearranging terms,
1. At x = 0, y = 0. I 2. At x = l, y = o.
·1 p
dy
3. At x = a, dx
dy
= dx at x = a.
4. At x = a, y = y at x = a.
8

Using condition (1) in eq. (m), we find Ci = 0. Using condition (3) in eqs: (ll :~

FIG. 8.3 y
~n), we find C 1 = D 1• Using condition (4) in eqs. (m)_autl 1
1 = D • while C = 0, we find thu tr D~ = 0. Final(Y, usmit ron wn -
1

·
(o) (~\an\~tf~ :q
y FIG. 8.4 2

we obtain and noting that a = l - . b, we find


wx c. = Pb (P - b') C D,.
y = 360lE/W - 10ltz2 + 3x'). ru ) defining the two poroons o
. ,~e
tr .
Ising the constants as lfr•termined , eqs. (m) allcl (o
To fi~d the_ maximum ~efiection 6, we first set dy/dx = 0 and find x == 0.51 91 · e astic line of the ~am become ]
Then usmg this value of z 1n the expression for Y, the maximum deflection t,ecomes
Ely""
Pbx
ill (lt - bt - x~) (p)
I El y=
Pb['
6l b (x
- a )l + (II - b')L - Ll . (q)
6 '"" Lima,. • 0,00652 wl•.
EI
oULECfJON 01' BEAIIM ~ EQUATION or EL.\snc L0a

al2 . d flectioo will occur in the left portion of the


For a> b, the maximum_ e th drrivati ve of this expression equal to /Pan, I<,
I ..
s1.1bet1wting
theee cond.ition.s into the a.bov .
e equationa, •e find
203

wh.ich eq. (p) applies. Sett.mg e rro giv.. M rJ, M ,ai


CI - M ,IJ - - - --·
~
'<I
~ . 3
\J~
X'"'
C, • 0 ,--
DM.a•
2
. ( the point having a horizontal tangent and he
which defin~ the abscdefl~-:n Substituting this value of x into eq. (p) we ~cedthe With these values, eqs. (s) and (t) for the portion AC of the e1utic line beoome
point of lll&Xlmum ec 1 • ' n
dy Mo,
El di = 67l6al - 3(a' + %1) _ 211},
Y,nu
- Pb ✓(l
-- 9-y3J.El 1
- b')3.
(r)
MoX
E/11 = 6l[6al - 3aa _ 21 _ 211). (x)
EXAMPLE 4. A simply supported prisrnat'
beam AB is acted upon by a couple M tc
plied at an intermediate point C a.s sh a~ Setting z = a in eq. (x), we find for the deflection of pomi c
in Fig. 8.5. Derive the general equation°?
the portion AC of the elastic line. Find
the deflection of point C and the slope at :
at &,
Mtjl
= 3lE1[3al - 2ai - l~. (y )

soLUTI?N- The reac~ion RA = Mol l and Setting z = 0 in eq. (w) , we find for the slope at A
the bendmg moments m the two portions of
the span become Mo
Fro. 8.5 8A = 6Elr
- ·f6al - 3a1 - ".
ru,]
·
O<x<a a< x < l
Mo:r, PROJI L l:M SET 8.1
M. = MoX M. = -l- - Mo.
l 1. With reference to the coordinate axes x and y &S shown in F\g. A, dmve Ule
equation defining the deflection curve of a uniformly loaded wiaile.eJ" besm. Prom
Substituting these expl'es.5ions into eq. (8.1), we obtain this, evaluate the deflection 6 at the free end. Ans. o = trl• &Bl .
2. Repeat the solution of the previous problem for the cs..~ of a dis::ributai losd
d2y Mo:r, the intensity of which increases uniformly from 1ero at t.he free end to m 11.t the
EI d'y = - MTor EI dx 2 = --l- + Mo. built-in end. Ana. ~ = wl'/30EI.
dx'
3. Repeat the solution of Problem 1 for the ca.,e of a c l ~ couple oi moment
M, applied at the free end of the beam. Ans. 6 "" M,Jl/ 2EI .
Two integrations of each of these equations produce -i. Repeat the solution of Problem 1 for the case of a con~tr&tai veni~ IO&d
dy MoXI 2
Pat the free end of the beam. An1. 6 = Pf.l/ 3EI.
El~= -2l + C1 (s) Eldy = _Mo:r, +Mo:r,+D1 (u)
dx 2l
MoX 1
+ Mo:r, 2 + D1x + D,. (v)
Elyx ---+C1x+C 2 (t) Ely= _ Mo:r,•
6l 6l 2

The integr~tion constants are determined fr om th e con d't'


1 ions
F10. B
(1 ) y = 0 at X = 0 FIG. A
I (2) y = 0 at x = l S • . b • aounterclookwi...~ eoupl-e of
Ill • A simply supported beam 1s acted u ~ .Y the equation of the def!eotion
ornent Ma at the end B as shown in Fig. B. vunv-6
3) at x = a, <!Ji - dy t
dx - dx' a x = a curv . • ~ at z • l/-{3.
e &nd find the maximum defteotion. A"-'· V-" 9{3 El
4) at x ,.. a, y .., Y , at x • a.
204
. .
0_.,,,.,oN o• BEAIIB

t.he eiastio Jine for th~ cant~ever beam in Fig. A. .


r .
MOKENT--A.B:E.A, lilTHO-D

lationship can now be given a very sim le . . .


205

6. Derive the eq~uon or bolically from 1 ero 1ntens1~y at the built..in e~I th& f}ll 9 reference to the ~la.stic line of t.he "-- d ~hical Lnl-t"7Pretat 101 1
- P gra
di..tributed load vanes paraWb t 1• the maximum deflection 8 at the free end . d I() tb re . . . ~m an 1ta be.ru:ii
in.tensity."' at the _r~:d~. c/a) !
~stead 0 tS_.l). An.,. 8 - l~wl'/ 1_80EJ _1n th~ -Ifi rn In Fig . 8 .6, AH 1s any portion of the 1.a.st· r ng mom ent
c&..-~! H~nt. &«in beam r.arries a d1stnbuf:OO I ~ the ~tensity of 19 . dis,gr:p0~ding bending momen t dia gram . AO :nd ~01::nd A1B 1 is the
7. A s mply supported 10 . Fi C too being the mtens1ty at mid-span Fi dhith correB intersecting at 0 , and 8 is the angle betw t b tangents at A
varie!!I sine.,oidally &!! _shtwn
d J~~e
equation of the dasuo me an 6 _ u,,/,'/ .-4E/.
therefrom the maximum deft~ti: th&
n 6, t oocl d ' small. The vertical distance B' B is the deQ~n10 esef ~gen
,
t.a, as-
u=t n o point B away
urnc A h"
the middle of the beam- Ana. .,___, 8 the tangent ath , w 1le ..4. A is the deflection of A awav from the
rrorn
nt at B . All t ese quan t1t1es ar e understood to be very ·~ma11 .
tange be I
I

• 'If X
Now let d8 = dx any e emen t of the elastic li ne at the distance :r from
Band note that the angle ~ tween tangeota at its end.s is de Then from
c:==~========~~-.r
11n7

eq. (b), we concl_ude that this ~ngle is_eq~al to the area JI dx of the 3haded
1 . - - - - -. , I ' - - - - - iMiP of the bendmg moment d1~m m Fig. 8.6 di-..'1ded by EI . lntegrat.mg
y eQ - (b) between A and H , Wf' obtain for the total anrle 1J between ta~nts r.r.c
FIG. D A and B
FIG. C

L A cantilever beam carries a sinesoidally dis_tributed l<>e:<1 as shown in Fig. D.


The int.erulity of load at any section defined by z 1s w. =- too sin(rx/2l), Wo being the
8 = f.t
s M d:r
EI
(8.-1 )

intensity at the built-in end. Derive the equation of the elastic line and find the
deflection at the free end. Am. Since this·integra l represents the total a.re:i. of the be.cdug momem diagram
A1B1 divided by El, we have the followi ng conclUEion
8- 2wol'(! - ~)·
rEI 3 r
THEOREM I. The angle 8 between tangents at any t-.ro _oorw A. end B 011 w
C). For the simply supported beam loaded as shown in Fig. 8.3, find the slope of elastic line is eq-ual to the total area of the CDrTespondt.ng ~!Mm oJ :& bald-
the elastic line at the end B, i.e., the value of dy/dx at z =- l. Ana.
ing moment diagram , divided by EI .
( dy) - -~-
dx _, 45El Let us now consider t he deflection of poin t B rel3uve t0 the ungent at .-l.
10. A c&nti.lever beam of length l carries a concentrated load at its free end. The i.e., the vertical distance B' B in Fig. 8.6. h:eeping m mmd :h!.\t the angre~
cl'Oll!HleCtional moment of inertia of the beam varies linearly from Jo at the built-in between these tangents are very small, we note from the ngun> (ha~ benriiog
end to lo/2 at the free end. Find the deflection .5 under the load P. Am. 8 = 0.386
PP/ EI..

8.2 The Moment-A rea Method


We shs:ll now discuss a semigraphic al method of dealing with the problem
of deflectw_n of ~ subjected to bending. Combining eqs. (a) and (b) of
the precedmg article, we obtain ---
"'i"s_:i.
·
d8 M ) _ _ ---,--- .r- , - - I

ds = El (a '
where d8 is the_ angle subtended by an arc element ds and M is the bending
~oment to which th.e element is sllbjected. Then as before for flat defier·
ion curves, we may take cu "" dx (see F""'1g.
' '
8.1) and write
'

d8. ~ - (b) Fw. S.7


EI Pio. ~.6
p....... .
--J:Cl'ION or BEAIIIB MOIIENT--AREA --•£DOD
~7
~
. to this deflection by the amount xd8. ·Jllilarly, for the deflection of B away fr th
of the element d, contr1~utes B'B becomes
1'hen by ti- S1 om e tangent at A, eq . (8.5)
.
summat 10 ,
n the total distance
• f" ET. Mxdx
i,ecorn",s
~= f ~ r· --gy-· Mx dz
~ =
!A
xd8 ==
A
(8.5)
.
A
D
EI
-
}0
the required deflection is obtained 88 th d'ff
(d )

. . h - a of the shaded strip of the bending moment diaar 'fhat 18 ' ··th t B e 1 erence between the
Smee M dz is t e a,.- h . ht h d .d "' ain statical ~oments \\I t rde~pec to o~ the positive and negative portions of
. . d.18ta from B we conclude that t e ng - an s1 e of eq. (8 5)
B f h l be d" nding momen 1a.gram. Th1S can also be d
and xis its nee '
~--' moment with respect to o t e tota n mg mom · the be expresse as follows:
represents th estat= h h ent
area between A and B, divided by El. Thus, we ave t e conclusion:
8 = [ a;~a]: . i1 - [ ~7]: x,, (e)
THEOREM II. The dejl,eclion of B away from the tangent at A is equal to the
where .fi and fs are the c~ntroidal distances shown in Fig. 8_7 .
,tatical moment, with rupee/, to B, of the 'bending moment area between A and
Since most of the ~ndmg moment areas with which we have to deal are
B, divukd by EI. simple recta.ngl~s, tr1an~les, and p~rabolic segments, their areas and the
This statical moment will be obtained simply as the product of the total location of their centroids are easily found . A few of these are sum-
area of the bending moment diagram bet.ween A and B multiplied by the mari~ in Fig. 8.8 for ready reference. In these diagrams, C denotes the
distance f to its centroid C (Fig. 8.6). centroid of the shaded area and V, the vertex of the bounding curve.
If there is an inflection point in the elastic line between A and Bas Applications of the moment-area method v.ill now be illustrated by
shown in Fig. 8.7, caution must be exercised in using the above theorems. several examples.
In such a case, the bending moment diagram divides itself into a positive
ELUlPLE 1. Determine the defleetion ~ and the slope 8 at the free end A of t.he
portion A 1D1 and a neitative portion D1B1, with centroids Ci and C2, re-
cantilever beam AB loaded as shown in Fig. 8.9a.
spectively. Then to find the angle 8 between tangents at A and B, we have SOLUTION. The bending moment diagram is shown in Fig. 8.9b. Sinee the tangent
from eq. (8.4) to the elastic line at B coincides with the undeflected &.ns of the beam, ~ required

8 = fAD M dx _
EI
M dx_
}D EI
r· (c)
deflection 8 will be the deflection of A away from the tangent at 8. Thus, IWllg
Theorem II, we have
(f)
In short, to obtain the angle 8 between tangents at A and B, we must take
the net area of the bending moment diagram between A and B, divided by 1

Likewise, the slope at A is the angle between tangent.a at .-l. and at B and from
Theorem I, we have
r-b/27 '-b/37 t3b/s7 Pl'
8 = 2EI°
(g)

1111 H1111
I Il.__b___.j ~1
1--b___.j
~J
1--b~
(o)
J- I '~
•_ -- -ola

:-j r ~ I
6'Ginl11ill1
, 1
Areoab/1 Areo•bll/2 1
Areo•2bh/3 :., I ----.J8 '
I I I
;1-----

~
p
b/5~- :(a) Al : P~/J,
~
7,, cubic
porobolo 1 I
I C•
I
17
i_L_
vi-- • _l V • "
_j_ A, -~
I

J,
b___.j i-=b---1-
Areo•bh/3
1---b-=-.i ( bl ( b) 3

Area• bh/4 Areo • zbhl~ Flo. 8.10


Fro. 8.8 Fm. 8.9
p
'"~ION 01' BEA.1f.8
DEFL.,..,&

IDS be&m AB c..-riea a <,ancent"'ted load P 200


2 A simply suppor«:<1 d the deflection o of point D from the cord 1·at rrorn
whirh
!in nd the tangent at A .
. Fig. 8.10a. Fm
= ✓a(l + b) ~
EX4KPLE . Lile
e a is shown in Fig. 8.10b. The ar x _
point D &S show~ i; between th.is cord
A B and It;; ~f: ~nding mo~ent d~';:centroid C from D_is }~(l
with respect to
b) ll.'l sh:~or
pomt B , we obtain thn.e
+
.
Then sin ce Om a• is the deflection of
3 - \ / ~-
A away from the tangent at E ha·
{I)
~dL?T . is Pab/ 2 and the dist.a?ce
t f this area h , we ,e
this l&gT&ffi 2
Pbx 2x Pb
Taking the statical mo~ec° ; the tangent at A. . T us
deflection B ' B of B awa.} ro b) Omu = 2lEJ · -3{cf2 _ b2) 1
= ~
9 v3LEI .
Pab ~ - . . en to agree with eq. (r) on p . 202.
B'B = 2El y 3 h IS se · · ·1
T IS LE 4. A prismatic canti ever beam AB carries a uniform ly distributed
d Aportion b of its length as shown in Fig. 8. I 2a. Find the deflection of
r,,cAllP the
Joa.dover
Then noting from the figure th.at 8A = B'B ..,_• l we have 1 O
th~ f~:T~iN. The b_ending m_oment diagram is shown in Fig: 8.12b. Its area wo~/ 6
Pab( l
-
8 A _- 6lEI + b.) (h)
~
sothe position of 1t:' centroid are foun_d _b~ reference to Fig. 8.8. From T heorem
aoclth d flection o will be obtained by d1v1d10g bv EI the statical moment of this
II ew1·t~ respect to point A 1- Thus
area ·
We see also from the figure t ba t th e required deflection of point D from the chord wb 1
line AB is i5 = 6EI (a + ¾b). (n )
0 = a8 A - o' I (i)

where
_
o' is the deflectionof D awayth
f the tangent at A. This deflection i5' can
ro~. n A D of the bending moment diagram. I
Cr 2/
by using Theorem II fo~ e pot i_~ is ~t ~be distance a/3 to the left of D.
r 1 )a
A~
be found
The &rea of this is Paib/ 21. and its cen r01
a I ~
Thus 2-
(a )
Pa'b a A J..-a I
b---'""'.I
o' = 2lEI X 3 (j)
I I I C
I I (a) I
· to eq. (i) gives
(h) and (j) 10

t.l=:~Jl:-
.
Substitution of expressiom
P.1

i5 =
Pa•b
6lEI (l +
Pa•b - Pa'b' _
b) - 6lEI - 3lEI
(k)
!<1--LLILLJ....L...C..J...L........_........_.....__ _ _ r'
EXAKPLE 3. Locate the point of max- ~- --.~ { - 1 _ ,
imum deflection on the simple beam ( b)
(bl
d.isell88ed in the preceding example and
p 8.12
evaluate this deflection by the moment- F10. e,.
area method. \
---- -- .L
8
SOLUTION. The maximum deflection
EXAMPLE 5. One of the ad•,antages of

--
OCCUJ'B at point E (Fig. 8.1 la) where .....

--- -
the moment-area method is that it per- '-<'
.... ....
the t.angent to the elastic line is h~ri- ..... .....
mits a fairly simple treatment of the
aontal, i.e., parallel to the chord lme Po A
'
(c l ..,_
deflection of beams that have abrupt
AB. Let z denote the distance of this Pl>1
..t'I
(o) 7 chan~es in cross-seetion. In Fig . 8.13a, Fie. 8.13
point from A . Then the angle between
tang,mts at A and E must be equal to ~2/,x for example, a simply supported beam . moment of inN tia I o, c· r
I ' AB carrying. a load P at the m1'ddl c h as c ross-sect1ona 1 hod ·
8 .A &8 already found in eq. (h) of the I ( bl . • • thP moment-:irea met
preceding example. Th~ the area ~f I the left half of the spo.n and 2/ over the right hul\ l sm~nts ond th e deR £>dion oc
I find th, angles of rotation 8, and •• of the en ,aog . I
that portion of the bending moment di-
agram between A and E (Fig. 8.llb) Under the load p be ·s ~hown by the tnang e
· . f th e a m 1 ~ · 'd b · two
~ ~~s
must be such that ~LOTION. The bending moment diagram or-d ·a ram , we simply d1v1 e i t oi
F10 . 8.11 :fb in Fig. 8.13b. To convert this into an .\/ 1
lhleor~inates for the right-hand half of the bee. '
obt3 ini ng th e shaded p.ar
Pab Ptn 2
8A = 6lEJ (l + b) - 2lE[ diagrarn as showu.
• 210
- ..,..,.yoN OP' BllAKB
o:s..-~··
. curve &8 shown in Fig. 8.13c. To obtn·
IIOMl:NT-AREA --

. ts two concentrated couplea of mo


"-'• n OD
211
.poin
We now consider the de:iectio~e t,&Dg1lDt at B, we have, by using the tn the l,bi rd be moment-area met h od, find the angles
menta M d
f an . 2M a.s shown in Fig c
deflection of point A away roro 8econd vsing ~ at A and B, respectively . A11-'. 8A "" Mi°; ; 0 ~tJon 8A and 88 of the ~d
~en 6 1, 9a .., o.
moment-area theorem, Pl l 2l Pl'
Pl l l + _. -·- - - -· "- 2M
L 2/ ,
6,. 18 •AA'=- 4Ef4.3 BEi 4 3 24E/ I
:i______J~
Then, from the geometry of the figure,
A.A' Pl' I~ '3 '3
e. - -r - UEI· FIG. C
Fro. D
stepped shaft supported 88 a cantilever beam •
5. • A sh · F. D
free end as own m ig. . The cro~tional cames a. concentra·-..1 1- ..1 p
In the same way, we find mom . . (.Ql >ua.u
Pl l l Pl l 2l 5Pl' • ~ of the shaft are I and 21 88 shown. Using the mo ~ of merw. of tlie t wo
a.,,. - BB' - sE(4.3 + 4EI'i°3 = ~efiection OA at the free end A. An.,. i .. = 3PP/ l 6 El.men rea method, find the
96EI
6. A simply supported ~m !1B
loaded at the middle as shown in Fi
~tional moments of mert1a I and 2[ as indicated u · th g. E ha.a
and
BB' 5Pll
~ethod,'find the deflection oat the middle of the beam . · Aru~ ~ = e35 °:;j~~
B,. = l '"'96EI°

We note that the ratfo 8../Bs =- ¾- .


To find the deflection 0c at the middle of the beam, w:e first look for the deflection
of point C away from the ta.ngent at B. By the second moment-area theorem, this
iii
Pl l l Pll
f>c,s = SE( 4. 6'"" 192EI° Fm. E Fm. F
7. A simply supported beam AB carries a. distribut-ed load t.he intensitv oi which
Then from the geometry of Fig. 8.13c, we see that varies a.c~ording to a sine law as shown in Fig. F . Using the data given in ~e !&st
AA' Pl' Pl' Pl• case of Fig. 8.8 and the moment-area method, find the deflection oat t.he mnidle of
ic .. 2 - Oc/B = 48EI - 192E/ - 64EJ° the beam. Ans. o = wJ.4/r'El.
8._ A cantilever beam of length land uniform flexural rigidity El is 5llbjecred tu
PROBLEKSET 8.2 continuously distributed externally applied moment of intensity m kg-cm per cm
I. Using the moment-area method, find the deflection c,A at the free end A of t~e : length of the beam. Using the area-moment method, show th3t the d~~t.!on ci
cantilever beam loaded u shown in Fig. A. The flexural rigidity EI of the beam 18 e free end of the beam is o = ml3/3El and explain why thi.5 is the ssm~ def~Cl)D
uniform throughout it.e length. Ana. 6A = Pa1 (3l - a)/6El. :e~:~tained for the case of a conc~ntra.ted force P = m applioo a t the end ai

t.h!·Referring to the beam shown in Fig. 8.5, find the angle oi ro ts ti~,n ~ .1 b~ u.,.ing
moment.area method . Shew that t he result agrees with th.!H ~ivt~n t-:- e~ , :1
0
n Page 203.

8.3 D fl
Fm. A e ections by Superposition
Fm. B
2. SolvAef the i~~~ing problem for the case in which a concentrated couple of lo ~he differential equation of the elastie line of :i. ~l!J1~ ~ •n t by t~s,·erse
moment D repUK-"t:21 the force pin Fi A A 2El 8
·,~ (eq. 8 I) was d • d . " t Sint<t' tht' l' qu:1ttllU 1~ li netlr rn !I s nd
in ~gA ~ant~~:erthbeam of length l ca~es.twon:~r:~; !r/.:~hir:!toin~ &11 shdow; 11,(1 de . · en ve m ..~r . 8 . 1• ... d d t 1•
· · .,...g e moment--are& method fi d h
ll\a" ber1vatives, it follows that its sohi t io n~ fo r vsrious lt)a mg con i),od1,.,,
. , n t e deflection at t h e f r ee en~, ,,,
.
The flexural rigidity of the bea .
I0Pl'/81E/. 18
a. . finin
J su · .
J>er1mposed This means tor p, amp t' , ·
I ch•1t 1f \• ~ h1we tM .t t'-
· t :i
m uniform throughout its length . An•-
th
load gp e deflection ~urve o f 11 cu:1t ile vt'f he:i.n, c,'\rr_ving ti dC:l) nuL"i~;~.;:~~~.
4. A simply supported beam AB of uniform ftexurit.l rigidity El carries at i~ at it f · i ft •t io n l• un·e un er -
8 rec end and y (.i:) detining tht' l l' ec
1
r
'
'

7
SLOPE AT
Table 8.1

I
BEAM DEFLECT ION FORMULA S

DEFLEC'I'IO N AT ANY SECTION IN TERMS


-
t-:)

t -:)

BEAM TYPE MAXIMUM DEFLECTION


FREE END OF x: y IS POSITIVE l>OWNWARD

). Cantilever Beam - Concentrat ed load P at the free end

I=;~ lsJ -
l
7 Pmo•
Pl'
2El
X I •~ y I Px•
= 6ET (3l - x) Omu
pp
= 3El

2. Cantilever Beam - Concentrat ed load Pat any point


Px'
!I = EI (3a - x) for O < x <a

ts
__t'J.__'

\
l b1 l Pa' 6 - Pa' (3l-a)
8 Omu - 6EI
=-t.:.:::::I
-
X = 2EI Pa 2
< <
--
----1T11 mo• Y = El (3x - a) for a x l
6

I
3. Cantilever Beam - Uniformly distributed load of w kg per unit length 0
z
llmox
:_ _ ; i - . - -
X 8 =wla
- 6EI
I Y ""' 24
1
~ EI (x' + 6l' - 4lx)
I Omu = 8El
wl'
~
lit

4. Cantilever Beam - Uniformly varying load; maximum intensity w "kg per unit length ~
ffi
-
~ ~
4 ;t------1-r; -mo!.
X I (J __
= 24E/wlJI wx'
Y = I20lEI (IOP - 10l2x + 5/x' - x•) j wl•
O,nu = 30El

c~ -x 8=:: 5. Cantilever Beam - Couple M applied a.t the free end

Y &m~t•
M
I I y=:e~' ) 6mu = 2E/
Ml'

6. Beam Freely Supported at Ends - Concentrat ed load P at the center

~
-{::::::
\ 61 ~ 62 = -Pl•
-_
16El
I y Px- (3l'
= -12EI -4 - x•) for O < x < 2-l /_ Om••= 48EE
Pl'


-:-able 8.1 (continue d)

BEAM TYrE
SLOPE AT
ENDS
I DEFLECTION
OP' X.' y
AT ANY SECTOIN
IS POSITIVE DOWNW ARO
IN TERMS
\
MAX1V.'3'W. AN'D
'OETLEC"T\ON
CY.N,E~

1. Beam Freely Supported at Ends - Concentrat ed load at any point


Pb(l1 - b~)m
Left End. Oma"= 9'1;3 l EI
Pb(/ 2 - b2) Y Pbx 2
=- (l - x2 - b2)(0 < x < a] ✓t2-- -
b2
X
e, "" 6/EI 6lEI atx =
3
Right End. Pb [ z ] >b
-,I, _ Pab(2l - b) Y = 61EI
h(x - a)
3
+ (l2 - 2
b )x - x
3
[a < x < l] At center, if a
82 - 6lEI o= Pb (3/2 - 4b2)
.
t::,
l'l
48El
;;1
l'l
Uniformly distributed load of w kg per unit length
B. Beam Freely Supported a.t Ends - g
5w[• 0
wP wx z
~ x
61 = 82 = 24El y = 24El (l 3 - 2lx
2
+x 3
) 6mu = 384£/ (SI

y
~- ,<
t:11

9. Beam Freely Supported a.t Ends - Couple M at the rii,;ht end rp


Af/2 c
"d

\1l cSmu = 9V3 /!:/ t-1


::,
"d

~ x
8, = 6EI
y
Allx(
= 6El I -
x
"jt
1
) at x = l / V3
At center
?fi
:::;
.Hl 0
= 1lll'
O, 3Ef
cS = 16£/
z

10. Beam Freely Supported at Endi, - Uniformly ve.ryinii; IOfld : max. intenaity w
wl•
Omax = .00652£}
11, 7wl'
8, ~ 360.E I al x = 0.5 19/

~J
X wx
y = 360lEI (7 t• - 10l'x
2
+ 3x4) At ce nter
wP
.t ~ wt•
y
o. = 45£ / 6 = .00651El
DEFLECTIONS BY SuPE
DEFLECTION OJI' BEAMS RPOSITION
215
214
tion of the deflection curve for the 8 . pLE 2. A_ ei~ply Ruppor~d priamatic bearn with
distributed load , then the ~qu\s simply lll'luJ. ~ shown 1D Fig. 8.15a. Fmd the deflecti 6 span land overhang a is
taneous action of bot h loadings ~ 1oaded asoN. Owing to the distributed load on t~ •, at th e end C of the overhang.
801,~~ the condition of a simple beam subje te~•~hang, the portion AB oft.he
y(z) := y1(x) + y,(x). (n) t,ealll 1B t the end B as shown in Fig. 8.15b. He~ee . a. couple o! moment Ms =
IJJ(J•/2 a usmg Ca.ae 9 of Table 8.1. we
. t the free end under the load P is 61 = Pl•; 3r, have
d e flect,on a . ~1
8 peeifically ' t he
. d f ;ntensity w, the deflection at the free end .· 8a = Mal = wail_
Under the uniform oa. o • 1 . d l - ..J " h d fl . lS 3EI 6EI
~ = wl4/SEI. Hence, due to the combme OIM.lmg, t e e ection at the
free end is simply this rotation of the tangent at B the overl-.a "....;_
Due to bs I tel · ·d ld ' ~iug pomon of the beam
sidered as a. o u y ngi .' wou take the poeition BC' in Fi 8..1 c;... d '
con ,
that C wou
ld have a deflection ig. ..-, &a we see
(b)
wail
61 = a8s =- ·
For ready reference, a table of deflections, slopes, and complete elastic 6EJ
line equati~s for ten primary cases of loading of c~ntilever and simply sup.
In addition to this, the . overhanging portion BC does bend due to tJle d3 t.ribut.ed
ported beams is presented in Table 8.1. These various results h~ve alr~ady loading and we have , with reference to Case 3 of Table 8.1 ,
been found either in t he examples C\r problems of the two precedmg articles.
Using t his table and t he method of superposition, it is possible to obtain 62 = _wa• ,
various required deflections and slopes for beams and loadings comprising 8El
various combinations of these primary cases. The idea of superposition can 18 the deflection C'C of point C away from the tangent at B. Thus the c.otal defiec..
he employed in a variety of ways and some of these will be illustrated by the tion of point C becomes
examples which follow .
t00~l wa'
EXAKPLE 1. A uniform cantilever beam AB is loa.ded as shown in Fig. 8.14. Find 6 = 01 + 6~• = 6El -
- + BEI
the deflection 6 of the free end B.
SOLUTION. Using Case 2 of Table 8.1, we have EXAMPLE 3. A prismatic cantilever beam of length l carnes s uruio.rm !O&d of
intensity w between x = a a.nu x = l a.s shown in Fig. 8.16s. Find me defiec.tion o
P1a 2 P.,/J 1 at the free end of the beam.
~ = 6EI (3l - a) + 6El (3l - b) . _SOLUTION. Consider the beam acted upon by just one elecien\ c d:: of the ~
tnbuted load as shown in Fig. 8.16b. T hen from Cs.::--e 2 of Tsb!e S l , -..~ hsn-
AB a particular case, if P 1 = P 2 = P, a = l/3, and b = 2l/3, this reduces to
w:rt dx
2Pl• ~ = - - (3l - x ).
6=-· 6El
9E/

•B
( b)
F10. 8.14 F to, S.17
Fm. 8.15 F10 . 8. 16
DEFLECT IONS BY BUPERPo e
oU"I,llC'l'ION or BEAIIB ITION 217
216 pi.E 5. A ai~ply supported beam havin a
distribute d load between x - a and x - l p ~ch end cames a uniformly distribute d gloadepan_ land1 overhanga of 1-..-.h
Now since each element 0 ~ the t th free ehd we obtain by summatio n l'od\lct,e
ea shown ID
,t bas · F'ig. 8 · 18a · p ·md the ratio /l 1· of mteos·tY 1D over it.a ""'-"6~
entire
a like increment of deffect1on a e . , ' O rd
le°'~ line at each end of the beam will be ho~ ~ er that the tangent,i to thP
1 ~ (31 - .i:) - WEI (314 - 4a'l + a•) . ell,ltlC...,.10N. Since both the beam and the load'mgn &re· symm tri l
l t' r
6•
1.. 6EI 24 tangent to the e ~ 1c me ~t (! remains horizontal.
B()LU •

tbd)e J>e&ID as a cantileve r bwlt mat C (Fig. 8.ISb c) Hence


e ca about point C,
we may corurider haU
If the load ext.ends over the full length of the beam, a ... 0 and this reduces to 01 the slope 8 1 at the free end'due to the uniform' 10 ~hen from ~ ase 3 of Table
g alone (Fig. 8.18b) is
8,1,
6 = sE/
wl4
81 =w (a+
6El
~Y.
which ehecb with Cue 3 of Table 8.1, as it should.
I,jkewise, due to the concentr ated force RA = tv (a + l/ 2) acting 1
EX.UlPLE 4. A simple beam AB of length l carries. a u~iformly distribute d load of at the free end (Case 2 of Table 8.1) becomes a one, the elope 8 1
intenlity w over a portion b ~f the_ span as shown m Fig. 8.17. Assµming a > b,
find t.he deAection 6, at the mid-po.Ult C of the span.
SOLUTION . We first con.sider just one element ~ of the load at
the distance t 8
2
= w(a+D GY.
from B. Then using Cue 7 of Table 8.1, 2EI

r·~
11 • } a
fx
6l~l (l' - x' - f') =
wb'x
24lEl (2l' - 2x' - b').
These slopes are of opposite sign; hence setting 91 = 9. we reafue lh
condition of the problem. This yields the quadratic equa.tion e req
uired

l'
Setting x - l/ 2 in this expression, we obtain a'+ al - -2 = 0I
wb' from which
6, = 96EI (3l' - 2b').
l ~
-2 l .
a= --2 :it:
For the particular cue where b -= l/2, this becomes
· · root as physically meaningful, we obt&in a/ l
Th Taking onlY the positive = 0.366.
6. ~ ~(a!:!1} end :,andard _meter stick in Paris is supported in this manner to insure thAt. i~ two
1'088-sections will remain parallel.
u is to be expected from a consideration of Case 8 in Table 8.1.
PROBLEM SET 8.3
"' per uni! len9!h
Uat,?!e beamth in Fig.superpos
A has uniform flexural rigidity EI throughou t its full l~ gth .
"eight
0
: i:..m.odAm.
of ition, find the deflection ~o under the !Oild P ~ ~ lec t
~D = Pa 2(l + a)/ 3El.
( 0)
:kc! ~ :=Y
pl
2
W••'
,1 ,,1111111 _1

l -=-~- 1
osr:---~,~
0
[ L a-!--- - 1 - - - --i
I

Flo, B
F10. A
a 2. A. llitnp) . I ded as sho\\'ll in Fig. B. If
1
•e"' l/2, find Y supporte d beam with overhang ~ oa under the load p equal to
ro. ,4'11 the ratio p I W to make the deffectton Jo
Fro. 8.18 · P/W • j -.
DEFLECI'ION OF BEAMS STRAIN ENERGY OF BENDING
218 219
. shown in Fig. A a.nd using the method of BUJ)e . simply suppo~ bea?IB of uniform er . .
3. Refemng to the bea.flm . of the mid-point C of the span AB. Al'J:>oa,tion 12, 'f\110 respect, each d1mens1on of one being ~tlon are geometncally sim-
calculate the upward de ect1on 6c na. 6C ._' UsJ' jp eve~ther, and they are made of the same ~~m~l th~ corresponding dimen-
Pal'/1 6EI. h bea. shown in Fig. A, assume that the load p .8 'oP of_ th e due to their own distributed weights. A na.i/
·• . ompare the mid-point
4. Referring again to ti el te tile upward deflection ov of the free end of ~htno,,.~
81 t10PB ·1 bea 3 I na. o o, = n2
deffec teel cant1 ever m m ong has an ISLB200 sect10 . . .
from D to C a.nd then ca. cu a. e o,,. 13· }. s ertical load P = 1,500 kg at its free end T n and carnes a con-
h Ana o = Pal'/16EI. l .. d. EI d er. celltrated vlf of the beam is reinforced by two 5 cm. b o revent overstressing, the
e.ng. . . D rted beam of uniform flexura. r1_g1 ity an span! carri
5· A .~?,!Y ~P~ loads pat its third-points. Usmg the method of superp ~ ~Wo
81
symmetncis.uY ~ a.c h "ddl of the beam . An&. o = 23Pl3/648E/ Os1t1on
i11board !~ extending from the wall ~ the middle of ~e ;:;tee!
c~ver plates top
a11d b<)t osition, calculate the deflection at the free end f th. UbeaSUlg the method
find the deflect.ton oat t e IIll e . , of superP o e m. AM. 6 =

,~ ,,l

.,
C
1
IT ·1
I 50 1 •

,,A,, 8
7,
75 ~ ,
2.6 cm•

8,4
Strain Energy of Bending

Consider in Fig. 8.19a, a p~matic beam subjected to pure bending within


n<>tic limit of the material. For such loading the bending
the el..., h , moment M

~
is constant along the lengt l of the beam and the elastic line is a circular
x-·-j p
arc of curvature MI EI(see p . 197) and the angle esubtended by this arc is
Fm. D Ml
Frn. C 8 = E{ (a )
6. A cantilever beam AB of length_! a.n? uniform flexur~l rigidity JFI has a bracket
AC attached to its free end a.s shown m Fig. C, and a vertical load P 1s applied to th This represents a linear relationship between Mand 8 as shown in Fig. 8.19b.
free end C of the bracket. Find the ratio a/ l required in order that the deflection 0j Hence we conclude that as the applied couples at the ends of the beam are
point A will be zero. Ans. a/l = J. gradually increased from zero to any value M , the work which they do is
7 . In Fig. D , a beam AC having uniform flexural rigidity EI = 90(10) 1 kg-cm•j 8
supported by a fulcrum at Band a vertical tie-rod at A . The tie-rod AD is a sl.ei!I represented by the shaded area OAB of the diagram in Fig. 8.19b. This
wire 3 mm in diameter and 3 m long. The beam is loaded at C with a vertical force work, equal to the strain energy stored in the beam, has the magnitude
P = 50 kg. Find the deflection oc of point C. Ans. oc = 0.367 cm.
8. A simply supported beam with overhang carries a uniformly distributed load
MB (b)
U=-'
of intensity was shown in Fig. E. If it is desired to have zero deflection at C, what 2
is the required distance a from the end A of the beam to the beginning of the which is analogous to the strain energy of ton,ion in a bar subjected to
distributed load? Ans. a = 5.4 m.
210- 90 180 twist [see eq. (d), p. 83].

r
1i:
0

7 111kg/m

''' 'B£Yl A
C
Using expression (a), the strain energy (b) may be written in either of the
following two forms:
or U - ElfF_
- Zl
(8.6)
120-l
800 200 p
FIG. E FIG. F I \
I \
9. The beam shown in Fig. F has rigidly attached to it at point Can arm A~ to ,.._8~
the f~ end A of which a vertical load P =5,000 kg is applied. Compute the vertical
deflection of the end B of the beam if E = 2(10)0 kg/cm2and I = 13,600 cm( An!.
OB = 0.27 cm, up. ,
I
I
I
I \
\
\
\
~r--l rdr (al
I

I
. IO. U~i ng the result for Orr.ax in case 2 of Table 8.1 and the method of superposi· i
~
: I, I
t1on, venf;; the result for Omn: in ca$e 4 of Table 8. 1. . ~ 1I M
II. ~ s1mP!Y supporte~ be_am having uniform flexural rigidity El and spun 1;
I I
I I ' 1
I I
~a c:ar~1es a tri~ngula.rly ~1str~buted load over the right-hand haif of the span . T d LWL.J...IJ...L~-s
00 0 8 ( bl
mt.ens1ty of -·this loo.cl vanes
h linearly from . zero
· at m1'd -span t o Wo a t the right-h
. ( 0) ( b)
of
~upport. U~mg t 11e met od of RUperposition find th d fl t· • t the UJtdd 1c FIG. 8.00
the beam. Ans. 0 = 3wcll4/BOl!.'l. • e e ec 10n o a Fxo. 8.19

L
DEFLECI'ION 01' BEAMS STRAIN l:NJ:RGY OF B•-
.....,uING
~ 221
t M we use the first fonn to calcut
If we know the bending mohmen ~ture defined by 8/l, we use the a~ th~ p
strain energy. If we kno""'. t e curv ' l!econd
l l te the stram energy. U.
form to ca cu a ss the strain energy ma bar sub·
It is somet~mes_ us:~:o 0~x~: maximum fiber stress "'max == Afc~7¼d .,l '/
to pure bending m .115 In the particular case of a beam of rectan ' as i.-----(a)
given by eq. (5.5a), p. · hh (5 5a) gives M = ¼bht gu\ar I I
arn,, and
~
cross-section of width b and dept ' eq. ·

·~~
the first of eqs. (8.6) becomes :
2

= 3I (bhl) ~
·
U 2E (c) (b)
F10. 8.21
This shows that for the same maximum.fiber stress, a_h~r in pur~ bending Fio. 8.2'l
can store only one-third as much st.ram energy as if it were m sitnple
C paring this with expression (c) above, we observe that for the same
tension. b · ted m::mum bending stress, the canti~ever beam carries only one-third 118
In discussing the strain energy in a ~am su Jee to non-uniforrn
stram energy of shear and consider much st~ain energy as the beam subJected to uniform bending.
. (F'1g. 8 .20a) , we shall
be nd mg neglect . .
only strain energy of hending. • Cons1?ermg any ~lement of the bearn of To illustrate the ?se ?f the seco~d of eqs. (8.7) , consider the simply
constant over the ported beam AB m Fig. 8.22. Th1S beam is so loaded that its defleetion
length dx, the bending moment M., will be essentially
:~~ve has the form of a half sine wave defined by the equation
length of the element. Hence eqs. (8.6) apply to the element provided we
replace M by M.,, l by d.x, and 8 by d8 = dx/p = (,Py/dx2)dx. Then
summing up such expressions for all elements of the beam, we obtain {f)
1 1
(<f!y) The mid-point deflection ~ being specified, it is desired to find the tot&I
f.
2
M 2dx EI {
U= 2.EI or U= 2 } dx2 dx. (8.7)
0 strain energy stored in the beam. Differentiating expression (f) twice with
respect to x, we find
To illustrate the use of the first of eqs. (8.7), consider, for example, a
cantilever beam loaded as shown in Fig. 8.21a. The bending moment at <Py n2 . rx
any cross-section defined by x is M., = - Px and the first of eqs. (8.7)
dx2 = - l2 sm T
becomes
Substitution of this into the second of eqs. (8.7) gives
1

1
2 P2l
P2x dx 3
(d}
U = o 2EI = 6E{ EI!.' 621r' . 2 rx dx _ El-r'f _ (g)
U =2 o T sm T - 4P
If the bea.m has a rectangular cross-section of width b and depth h,
the maximum bending stress at the built-in end is
Mc 6Pl
Umax =T = bh2'

Expressed in terms of this maximum stress, eq. (d) becomes


y
V = ! (bhl) Umaw.2. (e)
F10. 8.2-3
9 2E
. e~ergy due to sh~ar w1'11 be email ·m r.omparison with that due to bend ini; e.xcept
•s tram i:t h middle of a simply supported
pri,.....~~LE 1. Calculate the deflection 6 st t e own in Fig. 8.23.
for short thick beams. This question is discussed further in the fol11>wing article. -..iatic beam AB, under the action of a load pas sh
r 222

W LUTION .
.
Tht> bend mg mo
DEFLECflON OF BEAMS

ment at &DY cross-section defined by x is


Px
Mz = 2 ·
. g 0 n ly
r ,.k1ndeflection ,
fll ulll
..
STRAI N ENERoy O
F BENDI NG

the pos1t1ve root of this quad ra t 1·c equaf


223

ton, we obtain , for the maxi-

(n)
. th firs f f eq (8 7) we obtain for t h<' strain r nergy of bn-.nct·1ng i
Then, usmg e t orm O • • '
~~ n

U = 2j 0
rm~= 8El
ptp .
96El (h)

. th. strain energv to the work done by the applied load as it is grad Unl!y
Eq ua tin g lS . .. . .
incre.o.sed from O to P durmg deflection•
Po ptp
2 = 96Er'
pp
from which 0 = 48Er° (i) FIG. 8.24
Ffc . 8.25

EXillPLE 2. A simply supported beam as shown in Fig. 8.24 is struck at its mid- It is seen from this that the dynamic deflect ion o is always larger than the static
point C by 8 ball of weight W freely fall~ng from~ height h above_the beam. Neglect,. deflection 6,, . . Even when h = 0, I.e., _when the weight W has no free fall but is
ing the weight of the beam and assuming that 1t behaves elastically, find the total Ii = U,.. That is the sudd"-1 app.ut:1.1
,:_.,
sUd
denlv. o.pphed .to the beam, fl (n). gives
h deq. , i:my
deflection o that v.-ill be produced at point C. load produces twice as muc e cct1on as when gradually applied.
SOLUTION. The gravity force W of the ball falling through the total distance As anothPr PxtrC'nW , we may tak1' h » Ii., in which case the quantit \· 0 t undr r
h + odoes work equal to the rarlical may bP nl'!!;k•(·tpd and Ii = ll,t + ✓2ho, 1 . • "

EXAMPLE 3. The rim of a flywheel of weight II' and mean center-line radn;g r is
lV(h + o). (j)
attached to a_hub by_four ~pokes as shown in Fig. 8.25 Each spoke has a rectangu-
N ow let p denote the force exerted by the ball on the beam in the extreme position lar cross-section of d1mens1ons b X h and a length I. The spokes are built in at the
of maximum deflection o. Then for this configuration the strain energy in the hub and pinned at the rim . While t he wheel rotates with constant angu~ \'e!ocny
deflected beam is w, the hub is suddenly locked . What maximum bending stress 11,ill be indured in
each spoke where it joins the hub? ~ eglect the weight of the spok~
Po (k) SOLUTION. During free rotation, the flywheel rim has kinetic energy
2
/w2 11" rtwt
T=- = - - ·
Equating this strain energy (k) to the work (j), we obtain 2 g 2

Po= W(h + o), When the hub suddenly locks the rim continues ~o rotate through some !lng!e 8
2 befor~ coming to rest and the ~pokes bend enough to absorb the kinetir ene~v of
the rim in the form of strain energy of bending. Since each spoke 13 a_~nulen"r
from which p = 2t (h + o). bea~ of rectangular cross-section loaded at its pinned end, eq. (e) ap~ and th"
stra1n energv of bending in four spokes is

Substituting this expression for P in eq. (i) above, we obtain 4 O'mu s (p)
U = - (bhl) ~ E ·
0
2Wl3 h+ o (I) 9 -
= 48EI_o_· Equating kinetic energy (o) to strain energy (p),
Wl3 4 2
IV~.
~ oting that 01 O' m u
= 48EI 9(bhl) 2E =g 2
· d efl ect1·on o f t he bearn under the load W, eq. (l) can be written
repreS<'nts t he static (q)
in the form from Which 3 /w!}_.
a,., .. = ~ rw\ g (bhl)
(Jll)
r 224

.
I. A SJ mp 1v suppor e s ·
at mid --spa.n .. Calculate the
DEFLECTIO N OF BEAMS

PROBLEMS ET

amount of stram
8.4

t d tee! beam carries .a vertical concentrat


en~rgy s
t d· h
~e m t
. ed load i•,Y "" 10()
e beain if the kg
. g stre,,_,.,
10
. tri
BHEARINo

. nl deflection of the beam due to defonnation


110°
En'IlAJN

""os . Due to the fact that tho= 8hear str


....,.,
'buted from to p to bottom of the beam croS!H!ec
.
8.88oc1&ted with the ~
e88eg are not unifonnly
·
225
"L __ _
-

l = 3 m and the cross-section is a rectangle of width b - 5 cm and depth h"'- '2. ~Pan d18 h wn in Fig. 8.26, which shows curvatur e' of the bea t10ns be.eome warpect
. . · t11t. rto s o . S .
Ans. V = 216 kg-cm. ,..., deformat10 n a1one. mall rectangul m axis produced by
2. Calculate the strain energy in the beam of the pre?edmg problem if the hear h . I
· ar e ementa h
s e rhombuses , t e vertical edges of wh·1ch remain on. rt· t e neutral a.xis
U: = 100 kg is dist.ributed uniformly over the full span mstead of concentrat load becom .
the mjddle. An.~. U = 86.4 kg-cm. . . . . ed at t ion . Thus the s 1ope of the deflection curve d ue to "ehea.real dunng de-
forrn8
3. Two cantilever beams of the sam~ ma!enal a~e s1m1lar m every respect e ·on is simply equa1 to the shear strain ""' at th s rat &ny C1"08&-
sec t 1 by y the de flect1on
· d ue to shea r, we write ' e neutral. axis· nuenoting
_ .
sire, one hAving all of _its lines.: dimensions JUSt n tn1:1es those of t~e other. Wh~e~t 1 th
the ratio of their stram energies when. they are subJected to their own dist .b tis th eo lope of the elastic line e folloWing expression for
. nu~
weights? Ans. UJU1 = n • 5 the s
4 A thjn high-strength-steel hacksaw blade, of length l between the holes at it.s dyi _ Tinu kV
· an d t he two overIappmg · en ds f astened to · h dx - G = AGa,
ends. 1 is bent' into a complete circle (a)
v;ith a bolt. How much strain energy is sorted in the blade if its flexural ri.,.;dg~tt er
· bel
EI and the bending st.res.'>CS re_mam . ow t e e as h I ric r ·t?
_1m1 . A ns. U_ '= "'' 1
211"1E1Y isI
•here Vs! A is the average shear stress in the cross-section . .
~ b h. h h" and le 18 a
5. A so~id circular rotor of ~eight n, = 17 5 kg and rad ms r 25 ~m 1s suppor~ w
merical factor Y w 1c t Is average shear '8tress mus t
· be . .
nU multiplied to
by a bearmg at the end of a simply supported beam as shown m Fig. A. Initial! tain
.
the maximum shear stress at the neut-''"' a.xis..- F
the rotor turns at a constant angular speed of 30 rpm. If the bearing sudden[' ob ·. . _ or a rectangular
cross-sect10n, k - 3/ 2 (see eq. 5.9, p. 128) and for a circular .
freezes so that the rotor stops almost instantly ,_ what dynamical reaction will~
produced at A? Bearing and support at B are mdependent, and the beam has 8 k = 4/3 (see eq. (g), p. 132). Owing to the fact that the ,uanges
L c~ J L~~~
. I . l h 01 an ~
rectangular cros.s-section 2.5cm wide by 10cm deep. Ans. R,1. = 293 kg, down. carry very htt e vert1ca s ear, the factor k mav have a val .r ? .
· h - ue 01 _ or 3 rn
this case depen d mg on t e proportion s of the /-section .
B C

®
0 £
A •
fr
200 ~.+.J_.4--, p
FIG. A Fw. n
6. The compound beam shown in Fig. B carries a concentrated vertical force Pat
E. If the flexural rigid ity EI is the same for both portions of the beam, calculate the
total amount of strain energy stored. Using this strain energy, find the deflection y

OE under the load P. Ans. oe = 4Pa3/ 3EI. FIG. 8.26


1. A si mply supported wooden beam of rectangular cross-section and spar
l = 3 m _is struck at mid-span by a weight W = 20 kg falling from a height h = ~0 cm,
Determine the required cross-sectional area A if the maximum bending stress is not Assuming some continuou s distribution of transverse losd of intensity w.
to exceed 70 k.g /cm' and E = (10)1 kg/cm 2 • Ans. A = 735 sq cm. : t~e bea°1:, the shear force V" will be a continuous function of z whi~ ~ay
5 30 differentiated with respect to x. Then the curn 1ture of the el~ltc line
8. It is decided to give the beam in the preceding problem a cross-section 2 _)(
cm. W?en the 30-cm dimension is vertical, the falling weight produces a m_axi~urn 01
becomes
deflection 01. Wh en the 25-cm dimension i!' vertical the maximum deflection is ·
What is the ratio o, :oi? Ans. 0 :6 :::: 0.834 . ' . iJly1 k dVi _ kw (S.S)
1 2
dx'l = AG di - -.-tG.

8.5 Deflec tion s Due lo S hearing S train tnalogous to eq . (8.1) this represents the differenti31 cquatil)ll of the elastic
:;e(of the beam for shear deflection alone. Tht> deflections calcu)la ted :~~
· d rng
I n t he pr_cce .
· d 1·scus.'$1·ons of beam deflec tions, only t he def orrn atio
due to bending stresses was considered . In all cases of non-unifor m bend ;.1g
.
1
11
th · 8·8) can he added directly to t,hose rs.lculated from eq. (~. l tt ~~m
where there is a shear force at each cross-t,;ecti on there will be some acl •
th: dtotflal deflect ions of a beam· subjec ted t.o
e ect·10 ns due to s hea r will be foun d sm~
non-un.iforred
,. 11 00mpa
m
w1 •
"
be•~thdmtgh. .M>
..,.,..,
~uue [~

'
BIIEARINO 81'B..UN
,,.,.ION OF BEAMS
DEFLE<- ~ 227
226

bending and t he former ma)d


~ e sheM e
, be disregarded. H owever , in the
flections may become signifi cant case of
·
~ng,:
,r11
ti,te k
3
a particular case, a steel heam of

5wl4
t
;2, E/G = 2 .5, and rt = h2; 12 : : .an~lar r,ross-section, we
which eq. (g) becomPR
,
b
short deep earns, 'd t he case of a simple beam uniformly 1
T o illustrate, let us cons, h~r ase t he shear force a t any section d oaded 61 = 384EI ( 3 ~} (g')
as shown in Fig. 8.27. In t ,s c ' efined
.., that for a depth-span ratio h/ l = 1/ IO t h .
bv x is · sbo" 8 · , e maxunum d fl t '
wl 1
'fbJS shear is approximate y 3 per cent of that due . e ec ion
v.. == 2 - wx , due to an ratio increases, the deflection due t 8hto bendmg. As the
depth-SP -0 ear becomes more
rt;ant.
s.nd eq . (8.8) becomes irnPo In the case of an /-beam F" 8 28
gives ' ig. · 1 eq . (5.7') , p. 129,
(b)
b
= kV = !'.: [~ (~ _ h,1 ) + 8th J] ,
Integrating once
T Tmu
A It 2 4 4
1

and again
(c)

__ l
FJo. 8.28
from which

(h)

(d)
Then for an ISLB400 section (see Table B 3 of Appendix B) we find A =
cm•, / =19,306.3 cm', r = 16.33 cm, t = 0.80 cm, b = 16.5 cm, h =

1µ ~,Jw per uni! length

11, , , ~I _
72.43
40.0 cm, h1 = 37.5 cm, and eq. (h) gives k = 2.53. With this va lue of k
and with E/G = 2.5, eq. (g) becomes
4
hi ) .
5wlE/ ( 10.121i'
'

c51 = 384 (g' )


~J ~ - - - - - -.,t---~ ~ -~
T r T
This shows that for a span l = 3 m the maximum deflection due to
I

y,i shear would be approximate ly 18 per cen t of that due to bending, which is
FIG. 8. 27 by no means negligible.
= l, In the case of a simply supported beam thatcarriesac oncentratro load P
Using the end conditions: y 1 = 0 at x = 0, Yi = 0 at x we find
at mid-span (Fig. 8 .29), the shear force V = ± P/2 is constant in magnitude
C, = 0 and C, = kwl/ 2AG. Then eq . (d) becomes
but changes sign at the middle of the beam. When V is constant, we see
Yi = ~~~ ( 1 - n· (e) ~rom eq. (a) that the slope of t he deflec tion curve due to shear defo nnalion
18 also constant. Thus it may be concluded in t his case t hst the deflection

Taking x = l / 2, this gives for t he maximum deflection at the middle of ~urve due to shear is represented by t wo straight lines AC and C' B as shown
t he span in Fig. 8.29c, where the constant slope of the line AC is
k wl2 (fl dyi kV kP
-- ·
c5, = 8AG dx =AG= 2AG.
lntrod ~cing t he notation/ = Ar2 , where r is the radius of gyration of the Hence th e maximum shear deflection at c 1·s Si·mp IY
cross-sect1on , eq. (f) can be wri tt~ n in the form
l dy 1 kPl (i)

0I -_ 384E/
5wl
4
[
9.6k (r)'
l G E] .
(g) c5i = 2 dx = 4AG.

g;
DEFLSCTION or BEAMS
, 8 1BARINo 8TLuN
229
2'l8
. attain the value compa tible with th sh
Again introd ucing the notati on I = Ar, this may be writte n in t.he fo ~JJ'P 111 gb:" diseus sion, it must be concluded thaet . ear force on the section
pp [ r' E] ~ JJl t ,.., .
fro sectio n the norma1 streSB distrib ution
ID the n..:..i.. bo h
"'6U r ood of t he
·
7i G . !Jljddle entaTY theor y of bendi ng (see p. 113~n;t ~ tha~
61 = .48El 12k G) predicted by
tbe ele:i and the additi onal deflec tion due to ~ arp~ng will be partia lly
Takin g
. .
agam, as a pa
rt·1cular case a steel beam
_
,
of rectan gular cross- .
d
2 5 and r2 = h /12, an eq.
' G) becomes Bect1011 , preve;}lat predic ted by eq. 0) above . more A de~:t" be ~~w hat less
we h ave k == 3 / 2 , E/G - • , tbjll . shows that for a simpl y supPo rted steel beam mvest1 gat10n of this
of f'eetan .,.,Jft.
61
Pl' ( 3.75 h')
= 4SEI z, · U')
estJOJl
qu . with a conce ntrate d load at the middle the
'
.
m&xlD lum
" 6 =
deflect
cr088-
ion due
section be ding and shear is
t,oth n
sion that all er
It has been assum ed throug hout the forego ing discus
sectio ns of the beam are free to w~~ ~ shown i~ Fig. 8.26.. In the case~~ to - pp [
I - 48EI 1 +
"'
2.95 zi 0.02 + n- (k)

ximat ely satisfi ed. The sh


unifon nly loaded beam, this cond1t10n 1s appro Additional deflec tion of a trans ve~ly loaded beam
due to shear de-
force at the middl e of such a _ ~ is zero and there
'_ Vill be no warping~;
formation may also ~ calcul ~ted by using the strain energy of shear. To
with the shear force as
this cross-section. The warpm g mcrea ses gradu all! illustrate, let us cons1 ~er agam the case of the simply suppor ted beam of
of the middle. The
we procee d along the beam to the left or to the nght rectangular cross- sectio n loade d at the middle as shown in Fig. S.29&. In
to, the middle section
condit ion of symm etry of deform ation with refere nce such case, the shear force at any cr~se ction betwee n ..4. and C is simply
on any elemen t
is theref ore satisfi ed. y = P/2, and f1:°m eq. (5.8), p. 128, the shear stress
situated at the dIStan ce Y from the neutra l surface is

T = ~ (~ - y2} (l)

in this one
(o) P/2 Now using the first of eqs. (4.12), p. Bi, the strain energy of shear
P/2
I element of volum e bdydx is
I
2
.d _!:__ (~
= 3'lGP - y2) bd•..J· (m)

,,,i1 11111111 I111111111rp' 2


dU 4
and the total strain energ y of shear in the entire be&m becom
es
lf'H' ,

, b)

p
U = 2 (''•
Jo
f-:i
1 3'l<ll
pt i (~ -
4
11)1 bdydx = &Xll
ptf.At (n)

~ . F.quating this strain energ y to the work P6i./ 2 done by


the applied load as
h the deflection &i
C (c) it ~ual ly increa ses from zero to the final value P throug
of its point of applic ation C, we obtain
Fm. 8.29
PW (o)
ntrate d load at e
th 61 = 4001
Consi der now the case of the simple beam with a conce
the middle. cross;
mid?l e, Fig. 8.29._ From the condit ion of symm etry, Which can arbitr arily be writte n in the form
adjace nt sections \
sect10n must rem~m a plane se_ctio n. At the same time, Pl' ( 2 h' E) (p)
force equal to Pl 1. li G . .
the left or to the nght of the middl e section carry a shear th 61 = 48EI
Howe ver, from : for which E/ G = 2.5, this
and -~arpin g of ~he~e sectio ns should take place. 1'aking th l be
am
condit ion of contm u1ty of deform ation, there can
be no abrup t chang be e partic ular case of a stee
nt sectio ns. There must be a · Cornes
from a plane middl e section to warpe d adjace 1 (p')
gradu al inc~~ase in warpin g as we procee d along the beam in either directl~: Pl1 (
3.00 7i
h) .
finite di stance from the middle cant 61 = 48El
from the middle. Thus only at some
DEFLE CTION OF UEAM 8
230
( 'J. and ii') we see that there is a discr epanc y '--t
ue w
. v , . . h .
C ompa .nng eqs. p(p') obtai energ y of h~ll
. It E
t h e t.wo resu s. q . · , th
ned by cons1
q (J' ') ' o
dermg t e stram
bta' d b y assum mg th t B en.
me
. ..,
a the d '
9
. . .
gIVes a small er deflec tion an e ·
st havm g slope s equal tot:
ftection curve (Fig. 8.29c) was two raigh t Imes
shear strain nt the neutr al surfac e of
th e_ beam . To expla in this d' e
we assum ed ~hat all cro~
Statically Indeterminate Beams
c·repancy, we recall that in arrivi ng at eq. (J'),
obser ved, this 8.Ssurnption
sectio ns were free to warp . Howe ver, as alrea ~y
le of the beam . Since
violat es the condi tion of symm etry at the midd
shear , does not invol'Ve t~·
(p'), obtai ned on the basis of strain energ y of
r resul t. In fact this ca. ~
assum ption , it must be consid ered as the bette Meth od of Supe rpos ition
(j') and (p') with the mnore
seen to be the case by comp aring both eqs. 9, 1
Consi der in Fig. 9.l a beam AB suppo rted by a
hi,ige at A and rollers a t B
rigoro us result repre sente d by eq. (k) .
acting in a plane of symm e-
and C, and subje cted to appli ed ~oads P1 and P2
react ions will be induc ed
PROB LEMS ET 8.5 try of the beam . Due to the actio n of these loads,
m of the beam will be as
eq. (8.8), develop the equation at the point s of supp ort and a free-b ody diagra
I. Considering only shear deformation and using B and C will be vert ical
of the deflec tion curve for the unifor mly loaded cantil ever beam shown in Fig. A shown in Fig. 9.1 b . The react ions Y n and Y cat
that due to bending deform,,'. int. The reac tion at A ,
Plot this deflection curve and note how it differs from because the roller s can offer no horizo ntal restra
. The intens ity w of the figure , is represented by its
t ion. Find also the deflection 01 at the free end of the beam which can have any direc tion in the plane of the
, the shear modu lus G of the material ·
horizontal and verti cal comp onen ts XA and YA
load, the cross- sectional area A of the beam
and the shape factor k are all assumf',d to be given. Ans. o1 = kwl2/2AG . '
equil ibrium of this free body , we have the
Consi derin g now the static
For such a system there are
w per unit length general case of a syste m of forc~ s in one plane.
.. WO ~
three equat ions of equil ibriu m, name ly :
: 1 11 1111 1 1111 111 ~I •1
1 ~x. = o (a)

J---.-.-~------- , J ,e .I Since these three equa tions are insufficien t to determ


ine four unknowns, the
is no way , from the stand-
Fm.B beam is said to be statically indeterminate. There
Frn. A ine t he reactions ~t the
for the case where the intensity point of the static s of a rigid body , to determ
2. Repea t the solution of the preced ing problem
built- in end (x = 0) to 1ero at 8UPPorts.
of the distrib uted load varies linearly from w0 at the
the free end (x = l), as shown in Fig.
3. If the cantil ever beam in Fig. A
B.
has
Ans.
a
the
o
rectan
ratio
1 =
E/G
lcwr/,2/6AG.
gule.r cross-section 5 cm wide by
= 2.5, find the ratio oi/6 of ~
P,\
__
(i 8
' _ , . L - - - - ? . µ .8
§ _ _....;;.,:.c_ _ ( o)
15 cm deep an? is made of_steel for which
shear deflect10n to bendm g deflection at the free end. Assum e l = 45 cm. Alli,
'4 f\\ C r2 cf I
I
I
I

it
I
01/0 = 0 .139. C l I I

I,
(0 )
~
I I " ft~ fac" ::::-.::nI .; (b )
I A- I
I
~ I ,;_ I
I I
I P, : I ;:, I
x.L~ \ I

r ~ '/ ,. !' ~ ,<I


I
18

( b)
i Ye
( FIG. 9.2
Fto. 9.1
231
p •TT y JNDET ERIIIN ATE BEAK S
STATJCJU'M METHOD OF BuPER POSIT ION
232
is t.o.ken away, we are left with a sirnpl 233
If the roller_ at Cd s ·nce such suppo rts ~re suffic ient e be . ·ing reacti ons X A, YA and Y s may be found f
rnain ( )
supported a t 1ts en s: 1 ,plane • for am 1\.13
we say that the interm edia~ - corn l>le½ re 'fo I·iiustr ate t h e proce d ure more. specifically rom eqs. a .
. Of th beam m one h . .d
, we consi er the b<-am
' ~
constra.mt e h
. ed nd nt If t e ro 11 erat B instea d of that at C is remov ed suppOrt at hown · in fig. 9.3. C• oosmbe g the suppo rt C as redun dant
~ is r u a . C with .overhang, which agam is comp le~ly , we o hta.in
.
am A~ of span 2l. Unde r the action of our
. . ' We ha s rirnarY system a simp1e as
constr ai ve_a
~oiforlll load, the down ward deflec tion of point c, from
\he
simple beamThA 'ther the suppo rt at B or the one at Case 8 of Table S. l,
e C may be con ~ded II\
on planed. us e1 f b . 213, )Vj}l be
as redun ant. n ge
I neral
'
any suppo rts o a earn m exces s of th s1 el'ed
ffi •ent for . .
its comp lete const ramt m the plane ofose both P· o'. = 5w(2t)• - 5wz•
necessary an d su c1 1oad1n .,
384E I - 24E{
are sa1'd to be redundant &Up'fX)Tls · , g
the roller at C as t he re d un dant suppo rt Th
Le t us now Choose . . · Likewise from Case_ 6 of Table 8 .1, the upwa rd deflection
movm . g th ' support we consider the simpl e beam AB · en re of point C due t o a
1s , . as our . · vertical force Y c will be
,~ In Fig. 9.2 we study the deflec tion P7'l?I\Q
sys..,,,. . ' . .
curve s of this bearn und ry
separa,te conditions of loadmg. In Fig. l h . er two !t.,, _ Y c(2l) 1 Y cP
9.2a, on y t e ~pphe d forces P1 and ° C - 48E I = 6E {
p 1 are acting and 6'. representn the down ward deflec
tion of point C h
the support there is removed. In Fig. 9.2b, only a vertic Thus condi tion (b) above gives
al force y c is :\en
at point C and 6''. represents the upwa rd deflec tion 5wl• Y cP
• of this point Uc ~ng
the methods of the ~receding chapt ~r, th~ deflec tions · sing 24EI - 6El = O,
6'. and a". can be
calculated witho ut difficulty. Super impos mg the two from which
states of loading i
point C .s. = 6' ~
Fig. 9.2a and b, we conclude that the net deflec tion of
5
a•,. Now since the suppo rt at C allows no deflec tion, the trueis value Ye=
4 wl.
of a
is zero and we conclude that
' With this value of Y c, eqs. (a) give YA = YB = iwl
and the reacti ons are
6'. - 6", ·= 0. completely deter mined .
(b) The above exam ple may be looke d at in a somew
When the deflections 6', and 6", are expre ssed in terma hat differ ent way by
of P 1 , P 2 , and Yc, noting that the tange nt to the elasti c line at C remai
condition (b) defines the true value of Y c. As soon ns horizo ntal becau se
as Y c is known, the of symm etry condi tions. Thus we may regard each
half of the beam as a
cantilever built- in at C and suppo rted at its free

,+
end . Consi dering t he
w per unit length ~rtio~ CB, we have then a so-cal led "prop ped cantile
I I I I: I I I iJ I I I I 11 I 151: Fig. 9.4. When this beam deflec ts under the applie d loadin
tan~ent to the elasti c line at C is preve nted and a counte
ver" a.s shown in
g, rotat ion of t he
rclockwise mome nt
I
I
;<, ---I. . . - - -,..l ----i
I

I
Mc 18. devel oped at C as shown . Remo ving this constr
;dund a.nt reacti on, we obtai n a simpl e beam CB a.sour
aining mome nt as the
prima ry system .
I ( o) I hen, under the action of the distri buted load, the
I I th rough the tange nt at C rotate s
angle (J'. as show n in Fig. 9.4b, while under the action
;wQ=-=:" !,V
I _..... ..___I
_ ~I
red undan t rnom ent
a~tde 6" •• Fig. 9.4c. The true value of M. is found by
of the
M. at c it rotate s in the opposite direction throug h an
makin g 9' • - 9" • = O,
I c I since this realiz es the condi tion of a built- in end . From
Case 8 of Table S.l ,
I ( b) I We find
I
I c

l ------- :
I

A. v.,,..,,--~
~
a; ~
From C
>c; 8.Se 9, we find
( C)
M.l
F10. 9.3 e". = 3E{
r 234 STATICALLY INDETERMINATE BEAMS

Equating these val ues in accordance with the condition that 8' ~ _ 9,,
~~~n

M,=g·
wl'
• "" 0
,
b
.
METHOD OF SUJ>ERP081TtON

. as our primary system , a si mply su


10
obte. 're two redundant reactions M an/MI>Ort.etl beam AB. In this case
. t .
there auilt-in en d s 1s
with'der first t h e primary
.
' A
wice statically indete
e and w
.'
ha
~5

e say t t the beam


rminate To fi d 1' and Ma
system under the t' . n f .!
~

(c) c~nsJ _5 b. Then from Case 7 of Table 8 _1~~~on of the applied load
9
p:
The positive value indicates that t he assumed direction of M, in Fig 9 fig . . n of the end tangents have , for the angles of
4 rotatio
was correct ; according to our sign convention for bending mome. t . _a Pb(ll - b2)
represents a negative bending moment at t he b m·1t-m. en d C of the hea
n ' it I -
and 9' 8 = Pab (2l - b)
8A - 6lEI
Returning now t.o Fig. 9.4a and equating to zero the algebraic sum rn. 6lEJ
moments of all forces around point C, we obtain of nsidering now the primary system under the action f d
Co
Mn ass h own m · F'1g. 9 .5c, and using Case 9 f T en °
moments M A
wZS
y Bl - 2 + MC = 0 . an d
bl
°
a e 8.1, we have
,, - .M Al M Bl M l
8 ,. - 3El +
6EI and 8" a = _e + M Al _
3EI 6EI
Substituting expression (c) for M., this gives
3 Finally, since the net angle of rotation at each end of the beam be
Y B= wl, zero, we have O' A - 8" A = 0 and fJ' 8 - 8" 8 = o. Thus
8 must

as found before . Pb(l2 - b2.) M .!l M 8 l


6lEI - 3EI - 6EI = O,

A ==
M~ t =
a
f
====-
( 0)
~
C
b--l

~- Pab(2l-b )
6lEI
M8 l M4l
- 3El - 6EI = O.
These two equations yield for MA and Mn the following values :
Pab2 Pa2b
M,. = - l2- and Mo= T . ~d )

~--~l-~ A
( b)
C B
Having the end moments MA and M 8 , the bending moment d.ill.gram fo r
the beam is easily constructed by superimposing the bending momen t
diagrams for the loadings shown in Figs. 9.5b and c. This superpo8tion
18_shown in Fig. 9.5d , where the triangle abc represents the bending moment
~ 6 diagram for the luad p and the t rapezoid abde, that fo r the end moments..
~ llllli~
e·8 By plotting the negative ordinates due to 1\11 A and .\/ a on the same side
J----

~,•M,
C of t~e base line ab as the positive ordinates due t-0 P, ~c overl.ipptng
( b) th
i>Or~ions automatically cancel and the net bendi ng momen t L.S gn-e n by e
0rdinates of the shaded portions of the diagram .
M
c~
_=-----
_\~
/:; ~
4~,v beAssuming b < a, the numerically la rgest negati,:e bending moment will
b MB as given bv the last of eqs. (d) . Th is has its gre~t~st value when
.

( C)
o
(d)
b
th::: l/~ for which I.M nlmu = 4Pl/ 27 . The positi,·~
e Point of application of the load P, by referenCt to ig. ·
be1tng9~::nt at
FIG . 9.4 FIG. 9.5
.h Pab MA. + Ms ::::: ~ -
As a final illustration of the method of superposition in dealing w~t Mc= -l- - 2 21
statically indete rminate beams, consider the beam in Fig. 9 .5 ha~ing 8
1st l'his h . _ I •J so that ' Jfc10." = Pl ·
both end s built-in and carrying a conc;entrated load P . Removing the res · ''"hie ~s its greatest value whe n a = /J - -,
ance agaimst rotation at each end of the beam as a redundant constraint, wr h 18 see n to bf' slightl y less thun j.\1 ul"'·"
236 STATICALLY INDETERMINATE BEAMS
IIETBOD OP BuP:np
061TI0N
EXAIIPLE 1. Two wood beams cut from the same timber are arranged , t" ng this back into either of the defl.e t· 237
9
in Fig. 9.6a, the free end of the cantilever DC being su~ported at the mid; ho'lil\ 5111,et1tu I c ton expressions gives
simpie beam AB. Both beams are horizontal and at right angles to one .e of the 6'. = 6''e = 6 _ Pl 1
Find the vertical deflection &. at the point of contact due to a vertical load / : 0 t~er. C - mi"
to the end of the cantilever as shown. PPhect
. SOLUTION. Let X represent the magnitude of t_ he force with which the two bea umes of course, that there was contact b t
mt.ere.ct as the point of con~ct. Then the cantilever has a net load p _ X ~e 1'bis asst c ~rior to the application of the load p u no pressure between the two
8
end, Fig. 9.6b, and the deflection of this point, from Case l of Table 8 1 b at 11.s t,es.!Il a LE 2. A cantilever beam ABC is built-in · t A
·
(P - X)ll
· , ecoine8 i,;~::t
1oad .
the free end as shown in Fig. 9.7. Find athe ' propped e.t B, e.nd carries a
t B. magnitude of the redundant
6'< = 3EI ct1on a la . th
re& 1JTION- Rep cm~ e support at B by the rea t'
p ~ system, the cantilever beam AC loaded at ~tve Jo~ce Ra, we o_btain as our
P case 1 of Table 8.1, the downward deflection 0 tn . t Ba.sdshown m Fig. 9.7b.
B from pom ue to the load p is

·i
Pl 2
6' B = 6 EI [3 (I+ a) - ~-
~ A
la
,JJ,; Likewise, from Case 1, the upward deflection of point B due to the force Ra is

( 0) 6" B = Rsl1_
A

p
~A

( b)
-~- LB C
Since the net deflection 6' s - 6" s
Pl 2
= 0,

6EI (2l + Ja) - 3El = O,


3EI
we obtain
RBlJ

~ C from which Rs = P( l +: }
{ b)

8
~
~
{ C)
Using this value for Rs, we find for the positive bending moment e.t the built-in
end of the beam

MA = RBl - P (a + l) = -2
Pa

A
~ ~
( d)
R•
PBOBLEIISET 9.1
I. Find the redundant reaction Ra for the beam A. B uniformly losded as shown
in Fig. A by taking the cantilever built-in st A and free s t Bas the primsry sy-st,em.
FIG. 9.6 F10. 9.7 Ana. Rs = fwl.

At the same time the mid-point of the beam AB is subjected to a downward force X
and deflects by the amount
8~
6
,, = X(2l) 3
• 48EI'
by Case 6 of Table 8.1. Since these two deflections must be equal, we obtain FIG. B
P X X F10. A ·
2 A . he action of s couple of moment
3- a= 6 1
M • propped cantilever beam is subiected to _t t Ma will be induced st
from which
.tat the end A
the b . as sh ow IP .m F"1g . B . mhat
"
reactJVe momen
l. Utlt-in _e nd B? Ans. .'1tf8 = M A/ 2. . • am in Fig. B. find the msximu~
de~ ~efernng a.gain to the propped cantilever be 4ns o_. = .\{ ,1(1/27 El st z -
2 l/3 ection Produced by the applied couple MA· • ·
-P .
X • 3
r 238 STATICA LLY INDETE RMINAT E BEAMS
METHOD OF SUPERPOSITION

r
239
C D
A
C 8
.
l .l .-L.J FIG. C FIG. D
~ t _.\.- J ---- .J
:F10. G
p

Fto. H
Find the reaction s R.t and Ra at the two ends of th .
11 • pported and loaded . .
4. F ind the redunda nt reaction RA at th~ su~port A of the statical 5
as shown in Fig H An., R e co~tmuous pn.smatic
ly indeter . !Je&!Tl T\e continu ous frame ABC shown i~ J<' i~. I ~ built-"'.' tD l 4A 5; Ra = wl/
nate beam support ed and loaded as shown m Fig. C. Ans. R.t l 2· d 36.
= P/4. rni- by a ro 11er a t C, an d b. d
su Jecte to the m at , fredundan tly
5. F ind the redunda nt momen t MA at the built-in end A of the supp<>rte • R action of a ho · •-,
statically i d Fi d the reaction c a t .C' negl ect ·mg ax1a • l . · ruon141 orce Pat B
extensio n of the vertical
termina te bea m supported and loaded as shown in Fig. D . .4. ns . .MA
16.
= P(8a _ ;)j r:e flexural rigidity El 1s constan t throughout. Am. Re= aP/S.
mem r
be AB.
·
6. Find the redund ant reaction RA at the suppor t A of the statical
ly indeter .
na.te beam support ed a.nd loaded as shown in Fig. E. The beam
is continuous a~~
and its flexural rigid ity is constan t through out. Ans. R.t = 3Pa/2l
. p
e. D

a £ .t
II
h
j_
J
l
C A 8 ·, A

La
I
8
A
,l .l ~ ,,l
p
/ H
A
J~ ! H

F IG. E F10. I Fm. J


Fw. F
13. Solve the precedi ng problem if the concentrated force P st B.in
7. A cantilev er steel beam AB is built-in at A and support ed at Fig. I is
B by a vertical replaced by a uniform ly distribu ted load of intenEity w a.long the ,-erticsl
steel tie-rod BC as shown in Fig. F . Before the load Pis applied member
at B, the tie-rod is between A and B. Am. Re = wl/8.
just taut but withou t init ial tension. Find the tension Sin the tie-rod 14. The continu ous frame ABCD shown in Fig. J is pin-supported at th
after the load e lo~
P is applied at B . The beam has flexural rigidity El and the tie-rod
rigidity AE. A ns.
has tensile end of each leg and carrie!l,,a uniformly distributed load of iotens~y ro
zont.al member BC. Each member has length ! and flenml ngidity El.
on ~j~;
p horiiontal thrust H at each support . Ans. H = wl/ 'lfJ .
• R
l ->. d dC
S=----- e(erring to Fig 9 6a 1 assume that the cant•·1ever beam exten .tree"en~dyonf th.
1 + (3h/ / Al1 ) th . .
0

Wt an overhan g of length a and that the load P 18 app 1


• r ed to the O IS
·tteJ to the lower
8. Referrin g again to Fig. F, assume that the load Pat Bis remove ~rhang as in Fig. 9.ia. In such case , find the force X ;:::;:{ rigidjty
d and that th e El. .4.n.!.
system is free from stress at tempera ture T = 21 °c. Then if the X rn AB at its mid-po int C. Both beams have the ssme
tempera ture drops
to- 18°C, what tension Swill exist in the vertical tie-rod? The beam
has a rectang~lar '"' P(2Z + 3a) / 3l.
cross-section 7 .5 cm wide by 2.5 cm deep and the tie-rod has a circular
cross-sec!10n
of diamete r d = 0.5 cm. The coefficie nt of thermal expansi on for
the tie-rod 15 :
= 12 ( 10)- 0 cm/cm/° C, E = 2( 10) 6 kg/ cm2 , for both beam and tie-rod, 9,2 Th
and l "'h eorem of Three Mome nts
25 cm. Ans. S = 35.5 kg.
d , ·upports (F ig. 9-Sa),
9. Refe rring to the sy stem shown in Fig . 9.6a assume that the I n th . on man} :,
concentrateD e case of a uniform contin1 wus
1. ~
force P at C is replaced by a uniformly distribu ted load of int.ensit 0 ne 8\ (/(;um I h' ge while all the ot hers
y w between C . · 0 vab e 111 .
and C. In such case, find the force X transmi tted from one beam
to the other at ·
a 'PPort ts usually conside red as a n imm
re tre a ted as h inte.rmedial.e support
Both be~ms have ~he _same flexural rigidity El . ,ins . X = wl/4. r rollers. In such an arran gementh, eac·e the con t111uous
. bearo
. . bealf\ el>rese .
10. F ind t he react ion R,1. a t the support A of the continu ous
prismatic is nts Just one redund an t constnu.n t and enc .
re intermediate suppo rts .
support ed and loaded as shown in Fig. G. Ans . R,1. = p; 4 up . ~ t:n . • t as there e.
1
any times statically indet ennina ,e •
r 240 STATICALLY INDETER)IINA TE BEA.llS THEOREM OF THREE
wollEms
241
Thws the beam in Fig. 9.8& is five times statically indetermina te. In loadings shown in Fig. 9.8b produ
case we might choo.,e the five intermediate reactions R1, R-i, . . . ;\lch 'fhe l and l,.+1 and we indicate rotat~ sofme bending of the beam
llns "
the ~undants and th-en set the deflections c5i, 6t, · · ·, c5, of their'""~ 18 in sp rt n by 8',. for t he left span and by ion,, of the en d. tangents at the
. to th . 8
of application equal to zero. These five equa.t ions ge _er with the th
t'Vlllts sllPpO f rotation a.re considered to be pos·it·i " hor th e nght span. These
g\es o ve w en the . .
equations of statics for the entire beam would _be sufficient to dete~ 11° the end moments M ,.. Therefore ., d. . Y agree in direction
the eight unknown reaction components. This, however, turns out ·th . .
wi Of the uncut elastic lme over the, support a con 1t1on Or ~ · ·
0

continuity of
slope
be a cumbersome &nd involved set of equations. Instead, it is much
1
sun
to
8'"
n, we have
= -(J" ...
to cut the beam a t each intermediate support and introduce the ·bendi:r
(a)
moments Mi, M 2, . • • , M,, over the s_up~rts as the redundants. In th! This is the key to the solu_tion of the problem .
way, the primary system becomes six s~mple beams, each carrying its
own externally applied loads together with two redundant moments To express e1,. and 8",.
l
m terms of the applied loa-L and d
Wl en moments
.
its two ends. By dealing with these simple beams two at a time,
complexity of the problem will be greatly reduced.
t~! for each span separ~te Y, we use ~he moment-area method discussed in
Art. 8.2. The ~n~mg moment diagrams for the end moments are the
triangles shown in Fig. 9.8c. The bending moment diagrams for the applied
In Fig. 9.8b, we consider any two adjacent spans with SUPPorts n _
1 loads are represented by the shaded areas in the same figure. We denote
n , and n + 1. Let l,. and Z.+1 denote these two span lengths and le~
these areas by A,. and A n+1 , respectively, and their centroids by C. and
M ._ 1, M . , and M •+1 denote the three redundant bending moments at C.+1, the positions of which are defined by a,., b,., and a,.+ 1, b,._ , as shov."'!I.
these three supports. Whether these bending moments are Positive or Now considering the left span I,. and using superposition , we find
1

negative will depend upon the conditions of external loading. We will


~ e t hat they are positive as shown in the figure. Then if calculation 8, = M ,.l,. + M ,._ l,. + A,.a,. _
1
" 3El 6El l,,El (b)
produces for any one of them a negative value, t 11is will a.utom~tically
indicate a negative bending moment. Considering the right span ln+ t , we have
8" = M ,.l,.+1 + M n+llr.+l + A,.+10,...,1_
ii'! (c)
11
3EI 6El l,.+1EI
4 j jj
..... _________
. _____
,,Q.;.,, / Substituting (b) and (c) into eq . (a), we get

( 0) M,._,l,. + 2M ,.(l,. + Z..+1) + M 11+1ln +l = - BAl,.a,. - 5A t'b a+i _ (9.1)


" ~1

!his is called the three-moment equation ; it can be written once for each
intermediate support of the continuous beam . Then from this sys~em
of simultaneou s equations the bending moments at the intermediate
supports can be found.
Throughout t he foregomg • d1scuss1on,
• · 1·t has been a&,ume<l that .the two
(bl extreme en d · lY :.•upport.ed If one or
s of the continuous beam were simp · . ill
bo th of these ends should be built-in , the number of red tmdru~c;\: -
exceed the number of intermediate supports. In such r:L->t> , for eac . m ~m
end d . . . h d'tion thllt 110 rot.at 1on t,'\.n
,. ' an a ditional equation expressmg t e con I
t a.._e f r example
pl ·11 Suppo~, 0 •
M• ~
th t ace at such a support w1 be _a \,a_ilable ~ . · Then l'f'frrring to Fig.
'
M,,f1
a th e extreme left end of the beam is built-HI.
9.Sb and
t a k'mg n = 1, we have
(cl l--anf1--.;. -b11f1--4
Mol1 M1!1 +~ . (d)
t-'10. 9.8 80 = 3El + 6El l1EI
THEOREM OF THREE MOMENTS
STATICALLY INDETERMINATE BEAMS 243
242
th
where 60 is the angle of rotation of the tangent at e left support. Set ti,,~ 111 per un,1 length
this equal to zero. we obtain
1~11 11'l ',I 1111 1112111: 'l
! I

~~ I
(9,2) 3

Having found t he bending moments at all of the supports of a c0


. th · ~
i-k~~
tiuuous beam, there is no difficulty abo~t fi,.11 d mg e re~ctions. Taking (o)
again any two adjacent spans, as shown 111 Fig. 9 .Sb, let R " be the sini l
bE>.am reaction at n due to loads in t.he span ln a nd R" "' the reaction p e
n due to loads in t he span l .. +t• In addition, there _will be re~cti~ns produc:
by the end moments M ,._1, M ,., and M n+t• !~kmg th e ~ 1rect1ons of these
moments as shown in Fig. 9.8b, the total additional reaction at the support
n will be
M,._J - M,. + -M,. + Mn+l OAw.l 05~ 0~
l,. l,.+1
the first part coming from the span l,. and the second part from the span
~ I~ llrl~
l~+t • Adding to this, the reactions R'" and R" "' due to applied loads, we ~ IJfl ~ "QJ
obtain for the total reaction at any intermediate support ~
-0.6w..l
-0.5w..l -0.4w..l

R,. = R'" + R" n +M n-1 l- Mn + - Mi + M "+1. (9.3) ( C)


n n+l
FIG. 9.9
Having the reactions and the bending moments at all supports, the shear
force and bending moment diagram for the continuous beam may be From symmetry, we conclude that R2 = R1 while Ro= ~1- Then since_ Ro+
constructed . Applications of eqs. (9.1), (9.2) and (9.3) to several specific R1 + R 2 + R 3 = 3wl, Ro = R 3 = 4wl/ 10. Having the rea.ct1on.s, the shesr diagram
is constructed as shown in Fig. 9.9c. . .
cases of continuous beams will be illustrated in the following examples.
EXAMPLE 2. Construct bending moment and shearing force diagrsms tor the
EXAMPLE 1. Construct bending moment and shearing force diagrams for the continuous three-span beam loaded as shown in Fig. 9.10&. .
th e
continuous three-span beam under uniform load of intensity was shown in Fig. 9.9a. SOLUTION. For a simple beam with a concentrated Joa? P at the mtddle,
The three spans are equal and the beam is of constant cross-section. bending moment diagram is a triangle with maximum ordmate Pl/ 4. The ares of
. SOLU1'ION . For a simple beam under uniform load, the bending moment diagra~ this triangle is
IS a parabola with maximum ordinate wl1/8. The area of the parabolic segment is
Pl l Pl 2
2 wl wl2 3 A - -· -
- 4 2
= _8 ,
A= -l - = -•
3 8 12
anWdit:a _centroid is at mid-span so that a = b = l/ Z. f d ain fo r the last two
and its centroid is at mid-span, so that a = b = l/2. Considering now the first two ritmg eq. (9.1), once for the first two spans on the 1e tan ag
spans on the left and noting that M O = O, eq. (9.1) becomes 8Pans on the right, we have

0 + 2M 1 (2l) + M .J, = wl3 wl 3 (e) 0 + 2M (2l) + M ,1 = u,


I 1 (f)
4 4 3PP l
!from conditions of symmetry, it is evident that Mi = M 2 and eq. (e) g1F·v_es M 1l + 2M ,(2l) + 0 = O- S J
' as sl10wn 1·n ig.
n 1 = -wl2/ 10 - M 2· Th e eompIete be nding moment diagram
-
9 .9b can now be drawn. 1'h . tl1 bending moments Mt and
M ese Simultaneous equations are easily sol\'t>cl fc,r e
From eq. (9.3) the reaction at support l becomes , and we find
Pl
R 1 = wl + wl wl 11 wl Pl ,lf. = - -·
MI= +40' . .
2 2 + io = w· 10
r 244 STATICALLY INDETEBIIINATll BEAKS

p
THEOREM OF THREE

. e the left hand support is built-in and th


MOllENTg

fbell s111c eq, (9.2) gives Mo = -M i/2. It must ~en~re no external loads on the
6Jtte~'beam freely overhangs the last support on h~ also that M 2 = -Pl/
245

0
..K X
1
K
z r-113 ...A..
iil'ct t _e tc> eq. (g) , we find M 1 = +Pl/7 . Tht>n M ~ e nght. Substituting these2
111
y,Jue5 esPonding bending moment diagram is sh O- . -Pl/14 and M = -Pl/
'fbe cor~. ns are now found to be Ro = +3P/l
4 ;w~ 1 2
in Fig. 9.llb. From statics.
tberesc iohear diagram is as shown in Fia 9 Il c' i - -12P/ 14, R1 = +23P; a '
~,l I .,t .1. ;.e---l an d the 5 ..,. • • '

(o) n,n, A--: f 9.2


~
PROBLEM SET

j j (J_;:..>
O.OZ5P.t
t Find the bending moments Mi _and Mi at the supports 1 d
1· ntinuous beam loaded as shown
span CO
in Fig. A. A... M _ &n 2 of the three-
' "'· 1 - - 22 Pl/ 405, M 2 =
( b) 3zpl/405. .
- n A uniform three-span continuous b(.am with overhan , .,_ .
"' · 'bu ted l d · p·ig. B p·
O.OZ5P uniformly d1stri oa as shown in . md the ratio gmg a/ l inenws
ordercarms a
to make
the bending moments at the three supports all equal. Ane. a/ l = l / ../6 = 0.408.

( cl
Fm. 9.10 w '°er uni! lenqth
Z, Jt ,A. ,A. I I I T I I I I I ']:' : I I I I!£ 1 1 I
The corresponding bending moment diagram for the entirt, beam is shown in Fig.
9.10b. The maximum bending moment occurs under the load P and has the value
~ ,t-o-----c,J.--.l- 1 ~a+- J--,t-+a--\
F10. A FIG. B
Pl Pl Pl
M max -- ·-
4 --
20 -
- -5·
1 z 3. Referring again to the beam in Fig. B, find the ratio a/ l in order t-0 make the

T£i
~o
1---.L
I
I
I
:f I .Lio) I
I
' -
I
,
Using eq. (9.3), the reactions RI and R1
at the intermediate supports are found 118
follows
ret.ctions at the three supports equal. Ans. a/ l = 0.44.
4. A tw<rspan continuous beam with overhang at the right hand end is loaded a.s
shown in Fig. C. Calculate the reactions at the three points of support. Am.
Ro =416.7 kg,up ; R1 = 1,375 kg, down ; R2 =458.3 kg,up.
I P..t
I 1- I I
I
7 P P P 3P 5. A two-span continuous beam built-in at the left end is loadro as shown in Fig.
I Ri = O + O - 40 - 40 - 10
I I = -w' D. Calculate the bending moments at the three supports. Ans. :lfo = + BSi
kg-m; M1 = - 1,714 kg-m; .M 2 = 0.
p p p p 29
-~
11 4
I
R, = O + 2+ 40 + io + 10 = +:i(f
I
Then with the entire beam as a free body,
1 ( bl 1,500 kgfm
I wefindR 0 = +P/40andR 3 = +16P/40.
I
The shearing force diagram may now be ~o
I
I constructed as shown in Fig. 9.10c. soo - 4 m---l

:
I 3P
l~
+ lp
EXAMPLE 3. Construct bending
ment and shearing force diagrams fort e
m: -i--- - - 2 m-_..,-1 rn

Fm. C
kg f---4m
Flo. D

ll i\ l:+,li ll i11 H111 ~


1
continuous beam shown in Fig. 9.llat
which overhangs the right-hand suppor, u,
6, 1'h
0
, . - uniform load of int~nsity
e right-hand span of a continuous beam carn: t d' g moments at the two
( C) -14 and carries a load P at the free end, in½ver h~lf its length as shown in Fig. E. Fi nd th e ;S 1; ; 960.
F10. 9 .11
SOLUTION. Writing eq . (9.1) for the two
adjacent spans 01 and 12, we have
7.r~cd1ate supports. Ans. Mi = 7wl1/960 ; Mi = di
~ms for the continuous
bea onstruct bending moment and shear force . ag . R "" Ra = 5,355 "i{.
R1: Rhown in Fig. F. Ans. M1-= M, = + 532.Hg-m, o
(g)
Mrl + 2M 1(2l) + MJ = o. 2
"" - l ,90Sq.
246 STATICALLY lNDJo.:TERMIN.,TE BEAMS THEOREM OF
CASTI GLIANo

rivative of t his expression 247


f he de wi th respect to
becoJl1es the applieri torque T
dU Tl
dT = GJ = (p ,
. h is the angle of twist of one e d f
wh1c . t t th . n o the sh ft .
er- If we m erpre e pair of stat ical! b a with resoect to ti
_ 8. Calculate_ the _be nding moments and reactions_at each support of the otil " 1· d y alanced to . ie
the shaft as a genera ize fo rce" and th rques at th d
tu,uous beam m Fig. G. A ns. M 1 = - 193.l kg-m, M 2 = _ 467. 4 kg-m• · con. of h " . e angle Of t . e en s
- 206.5 kg-m ; Ro =- 193. l kg; R 1 = +1 ,343.5 kg ; R 1 = + 3,110.5 kg; Rs= + Ml"" two ends as t e ,,corresponding
. . displ acement ,, w wist between these
kg ; R, = - 137 .7 kg. 1,876.8 e "displacement 1s given by t he first d . ' . e concl ude again tha t
thith respect t o th e "force . " . energy
envat1ve of th e ~t rain
9. Calculate the bending moment Mo and the reaction Ro at the built-·
the continuous beam shown in Fig. H . Ans. Mo = -1,649 kg-m· R 0 in end of w ·1
Finally, for a canti ever beam bent by t
' = 1,545 kg.
end Fig. · 8 .21 a, p. 221 , th e stram d" . load P a t t he free
· energy aof bransverse
2t 41 ' en mg 1s

A
O

A
t l m~°m5G
A A
4
A
,
~<S
600 kg/m
1 11111 1 11
1,000 kg/m
2:f 11111 I Sit1]]
U = P2l3 .
6EI (c)

The derivative of this expression with respect to p is


l--1 m-4-- 1.5 m-+1 m _j_._ 1.5 m--l I---- 5 m - - - - 4 m --+2 m~
dU Pl3
FIG. G FIG. H dP = 3EI = o,
which is see~ to be the deflection of the end of the beam in
of the applied load P . the dire{'t ion
9.3 The Theorem of Castigliano
Each of the foregoing cases is si mply an
I n preceding chapters, general expressions for the strain energy stored example of a general theorem regarding strain Pi

in a n elastic bar subjected to tension, torsion, or bending have been energy which is called the theorem of Castigliano. P,

developed . This concept of elastic strain energy can be very useful in We s~all now proceed with a general deri vation P,

the study of deflections of various points of a structurP- under load . Con- of t~rn th eorem. In Fig. 9.12, let AB be anv
sider, for example, a prismatic bar under simple tension as shown in Fig.
elastic
.
bod"' t
J ors ructure completely constrained
. · 3/
s
in space and
p l
su b.Jee t e d to app ied forces Pi. / •"
2.14a, p. 40. The strain energy in this case is, from eq. (2.4) , 2
1
3
' P , • · • · If the material follows Hooke's
P2l (a) nd
U = 2AE. d~w a the deformations are small, the F 1G . 9. 12
I
us:~ceme~ts of the points of loading will _ . ..
By t.ak.ing the deriv ative of this expression with respect to the applied Will h ~ be hnear functions of the loads, i.e., the prmc1ple of superpostu~n
load P , we obt ain be O d. • In such cases the strain energy stored in the loaded sy-slem will
equal to the work d;ne by the applied forces l\nd indeJk'ndent of the
dU Pl d h f I'1rd
dP = AE = b. si er in wh ich they are applied . If, for examplt>, t r on.'t'~ art.' app
or
Inultaneo us 1_Y and gradually .mcreased 111 . t he s:\me
, pn•"''rt 10n then the
Work ,.,. ·
Thus the deri vative of the strain energy with respect to the applied Joahd done will be
·
P gives ·
t h e d e fl ect10n · pomt
of its · ·
of application in the direction of t c
(9.4)
load .
. . torsion U = l
2 (P 1b1 + p z02 + p ~ + ... .)
3 3 ·
A gam, m the case of a shaft of circular cross-section subjecwd to Where O1 t . .-., '3 in the direc t ions
as shown in Fig. 4 .12, p. 82, the strai n energy is, from eq . (4.13') ' '
0
~, 0a, are the deflections nf P0 1111 s 1' - ·' ·
'For •· 2nd . ed .,
~
., S /T t1<'/1irU,
0
~l ll 2 llltcept,· 0 d y 0 ung T n<ory i "
U =-· · 21. • nal cases, see Timo11he.nko M1 '
20.I
THEOREM OF CA8TIGLIANo
248 STATICALLY INDETERKINATE BEAMS
249
. enerUY, represented as a qu,u:lTatic function 0
of the corresponding forces P1, P2, P,, • · · · It mus~ be clearly understood 11affl of these forces, gives the correspond · 1 the forces, with re81)ect
here that 011 h:i, 031 ... may not be the total deflect ions of t he points l , to (J.n'Y ori:ent of application of that force . The 1UJ
tte C-O~~ of di8pl,acemml
31 • . - bu t 1 in each case , only •that component of the total deflecti· .2,
on 111 .t t}ie p(l' . h rms force" d "dis

o1 ,, must, be taken m t e generalized sense. If th " a~ . place-
t he direction of t he corresponding force .. From our assumption that the sponding displacement will be an an 1 f e fo~ce 1s a couple ,
me11t
displacements 01 1 ~ • . .. a re linear fun ctions of th~ forces P 1, p 21 .. corre . . g e o rotat10 · th
the the couple. If it 1s a true force the . n m e same
we conclude t hat if we substit ute for 01, 6', . • • , their expressions in ter ms
.
·' nse a.s . . , corresponding d · lace
d se linear deflection m the direction of th t f I.Sp ment
h
of p 11 p 21 • .• , eq. (9.4) becomes a omogeneous qua rat1c function of
WI
·n O
be "'
•ated that we speak only of statically
a orce I t
ind d · mus
t I80 be
a
t he forces. pprecI . . epen ent forces to any
Assume now t hat after t he loads P, have all been applied and the str . B•
O
f which we can give an arbitrary increment oP .h ' .
. . ( ) . a1n one f Th' • Wit out aff ectmg
energy in the system 1s U, as given by eq. 9 •4 , we mcrease any load p the other orces. 1s means t hat reactions dete • ed b .
anY Of • • d rnu n
·
v statics
by the amoun t dP n • This will cause a slight ch~nge in the deformatio~ cannot be considered as m ependent forces.
of t he body and t he stored strain energy will be mcrea~d slightly. This
increase may be expressed as the rate of change of U with respect to p
times the change made in P ,.. In this way, the new amount of strain ener~
x.oi1PLE 1. A simply supported beam with overhang is loaded
~- gJ3a. Using the t~eorem of Cas~igliano, find the vertical defiectio:o;
ho

SOLUTION, The bendmg moment ~J.&gram for the beam is shown in Fig. 9_13 b.
.
po:~
becomes Between A an~ B, a general expression for bending moment at any ~tance .t to
the right of A 1e
au
U + aPn · dPn,
Since t he fi nal st rain energy does not depend upon the order in which
t he forces are applied, we will now assume that dP n is applied first and
(d)

~+
A~,-------~x----~,c
I
. r
- - - - , , l - - - - - - a---l
I
afterwards the fo rces P1, P2, P3, .. . . When dP n is applied first, it produces
l (o l I
only infinitesimal deflections and the corresponding strain energy, of
second order, can be ignored . Applying now the forces P1, P2, Pa, ... , ixl r-1:
~
it must be seen t hat t heir effect on the structure will be unaltered by the
presence of dP n and t hey will store the same strain energy U as before.
However, during t he application of P1, P2, Pa, .. . , the force dP n, already
a pplied earlier, will ride through the displacement o,. produced by P,, Fm. 9.13
P 2 , P 3, • In so .Ioing, it produces additional work of the amount
dP,. · o,. and the fina l strain energy in this case will be Pax
l ·
Ms= - - (0
U + dP,. · on, (e)
Between B and C, the bending moment at any distance x to the left of C is
Equating t his expression for final strain energy to expression (d) ohtained (g)
Ms= -Pr .
before, we have
Deing the first of eqs. (8.7), p. 220, the strain energy of bending int.he bearu becomes
U -t- dP ,. o,. =
au
U + -p dP n,
a n

1
U -1'
-
0
P'aZx'
2Ell 2 dx +
lo" P2x'dx
o 2El
= Pia' ({ + a).
6El
(h)

from which
hen frorn eq. (9.5)

-au ·
on= aP,. (9.5)
0 =au= 3El
2
Pa (l+a).
aP
t,c.\lfp . . . e.t B and loaded at ~e free
This states t he theorem of Castigliano in general form . For any elastic end A f LE 2. The axis of a cantilever nng, built-md' R Fig. 9.14. Find the
the
system which obeys the law of superposition, the partial derivative of ' orrns & horizontal quarter circular a.re of ra ms '
250 STATICALLY INDETERMINATE BF,AMS
THEOREM OF C •"
"->TIGLIANo
\·erti.::al deflection ! of the free end A, assuming the ring to have a circular
nts of the total deflection oof point A Th
251
section the diameter of which is small compared with the radius R of its cente c\?SS- coJ11Po"~ng 8 cross-sectional area A and AC · e bars are of the same mate .
SOLUTION . Such a cun·ed cantilever is subjected to torsion as well as be rd!ne.
.4B hav110N , From statics, we see t hat th; atecr~lss-sectional area -41 "" 2A na1•
At any cross-section D, defined hy the angle 8, we see that the bending tnorn:~tn~. soLUT . th b AC ns1 e force . th .
IR
sive force m e ar are each equal top H m e bar AB and th
M, = PR sin 8, colTIPre~ . encp the strain energy in thc
(i)
systern is e
while the twisting moment on the same cross-section is U = p,l + _!:!:.._ _ 3P 2l
T, = PR (1 - cos 8) . 2AE 2A 1E - 4AE'
(i) or the vertical component of the deflection 0 f A
Then , f , we have
0• = au =- 3Pl .
iJP 2AE

1,0 find the horizontal


·
component of the deflection of · • t A
ta.I force Q w1·th respect to wh
.
Jom di.ff, we mtroduce at :A
fi titious horizon ic h we ca, .
a c . W'th th· f . . n erentiate the st.ram
enc rg)' expression . 1 1s orce acting m addition to p th .
, e s-tram energy
becomes

u = (Q/ -.{3 + P)2l + (Ql ../3 - P)ll


2,H: 2(2A )E .
Then from eq. (9.5)
p
0 _ au _ <Q + P/ -{a)t
h - aQ - 2AE ·
F10. 9 .14 Fm. 9.15
For the case in which we are interested , Q =0 and the horizontal component of
deflection becomes
The tot.a.I strain energy due to the combined bending and torsion of the ring is
Pl
12
_ ("' M,2Rd8 ("' 12 T,2 Rd8_ Oh= 2-..[aAE.
U - }o 2EI + }o 2GJ
Then by eq. (9.5) PROBLEMSET 9.3

- au -
!, - aP -
("''2 M'\ aP
}o EI
{aM,\D
,~ao + }o("''2 {aT,\D
T'\afi" rd8,
GJ
(k)
0
f I. ~l~ing the Castigliano theorem, calculate the vertical deflection os t. the middle
a si~ply supported beam (Fig. A) which carries a uniformly distri buted load of
m~ensity_u• over the full span. The flexural rigidity EI of the beam is constant and

'. j),
Qn! st rain ener_gy of bending is to be
consid ered . /lint ~ lntrodurt> s. fictional losd
where, from eqs. (i) and Oa~ the middle of the beam . i\ns. ~ = 5wl' / 3S4/1/. _ .
2
d, Usmg the Castigliano theorem , ca lculate the angle of rota tion /J of either
aM, R . 8 (i') ~n ·tangent for the simply supp ort ed beam . I B loaded as ,mown in Fig ..\ . .-\ssume
aP = sm ,
~n stant EI and consider only strain energy of bending. /lint: Introduce a firti on-
aT, (j')
a couple of moment M = Oat one end oft.he beam. :Ins. 8 = w£l/ Z4Et.
aP = R (l - cos8).
~

Substituting (i), (j), (i'), (j'), into eq . (k) and performing the indicated integrations, ~-
w Per unit len9th
1 1111111112.s A II I I I I I 11 ~ -
we find
1
+ (31r -
l __ - ----- ..t - - ~ -~
I l.
2
0 _ 1rPR 8) PR'
• - 4El 4GJ F A FIG. 13
3 IG. _,. h f
. h l curries 9 , U~in ical deH('(.'t.ion o at t e ree
EXAMPLE 3 . A simple truss composed of two bars each of lengt d erticsl ~nd of th g the Castiglian<>- theorem , rakulate the vcrt_ F' ,tns. 0 = 4l wl'/
vertical load P at joint .4 as shown in Fig. 9.15. Find the horizontal an v 384£/ e can lilever beam of length I loaded s..s shown m ig. 8 · ·

bi,
r 252 STATICAL LY INDETER MINATE BEAMS

4. A thin cantilever ring AB having a circular axis of mean radius R is b .


CASTIOL IANo's THEO

t"ties in the case of a continuo us bea


RE14 APPLIED
253
at B and carries a vertical load P at its free end A 88 shown in Fig. C. Us· lillt...in qua.n I essed as a function of these bendinm, then the strain energy can
Castigliano theorem and considering only strain energy of bending, find the ~:~.the be expr h rt· l d . . g momenta M M M •• . . . .
1
deflection of point A . Ans. ~. = ..-PR /4EI.
1C&l
uch case t e pa ia er1vatives aVI iJM i aV . i, i ,
C an~ using th~ same Proced In
9 ' t faMs, iJ U/aM,, .. . will
nt the relative rotations between tan gen
s. Referring again to the cantilever rin~ in Fig.
An., Ure represe s to the las . .
as in the preceding problem, find the horizontal deflection of pomt A. •des of each support. However ' from con t·mu1ty . e tic hne on the
PR / 2E/. . 0~"' two s1 . ?f the elastic. line
1
each support, we know that these relat"
6. Referring again to the ring in Fig. C, find the angle of rotation 9 of the tan e
1' e rotations are all
0 ver .
gam zero.
g nt
at A due to the applied load P. Ans. 8 = PR'/EI. }Jen ce , 8
7. A simple truss ABC with pinned joints is loaded
· l
as shown in Fig. D B
· oth au au au
bars are made of steel and have the same cross-secbona area A. Using the Casr aM1 = O, u.,.M2 = 0, iJM1
- =o
liano theor:m, fin? the vertical. de~ection of joint ~. Am . . 6. = 7 .62P!/ Al I ' .• (9.6b)

8. Refemng agam to the truss m Fig. D, find the horizonta l displacement of th · In general, to find the redunda nt forces in a stat"call
1
. d .
Y m etermmat e
joint A caused by the action of the vertical load P. Am. 6r. =- -l.73Pl/A Ee h d
9. A thin, circular steel ring of mean radius R = 7.5 cm. bas a 0.5 cm by o.s ·
square cross-section and is cut through at one point on its circumference. The
is then spread open and a 0.5-cm-thick block is inserted in the gap. What maximum
ri: system, w~ remove t e re undant ~onstrainta and replace them by the
correspondmg forces. Then expr~ssmg the strain energy of the system
in terms of the forces and applying the Castigliano theore m, we obta·m
. h h d
bending stress will be produced in the ring? Am. O'maz = 943 lr.g/cm•. eqs. (9.6) from w h 1c t e re undant forces can be calculated .
To illustrate , let us :econsid er the case of a. propped cantilever beam
under u~iform load! Fig. 9.16._ This is a system having one statically
indetermmate reaction . Choosmg the vertical reaction X at B as the
redundant force, the bending moment at any point along the beam will be
wx2
M,.=X x- - (a)
2
and the strain energy is
1
Fm. C Fm.D Mz2dx
{
U = }o 2EI '

9.4 Applica tions of Castigl iano's Theore m to Statical ly Indeter- which is a function of X, since M,. is a function of X . Thus, differentiating
minate Problem s ' under the integral sign

The theorem of Castiglia no is very useful in the treatmen t of statically 1 M aM,.dx


indeterm inate problem s. Consider , for example , the case of a continuou s au= ( "ax° =O (b}
beam having several redunda nt supports . Denotin g by X, Y, Z, • · · e
th ax }o EI I

staticall y indeterm inate reactions , the strain energy in the beam will_be wherein, from eq. (a),
a function of these forces and the displace ments of their points of apphca·
aM,. = x. (c)
tion will be obtained a s dU/dX, dU/dY, dU/dZ, .. .. However , _t~ese ax
displace ments a.re all known to be zero in accordan ce with the conditions
of constrai nt. Hence, we have
au au ae (9.6!1)
ax = o, aY = o, az = o-, · · ·
dundant
There will always be a s many of these equation s as there are re X
reaction s so that the reactions can be found. d nt
11
If the bending moment s over the support.s are taken as the redun
Flo. 9.16
r 254 STATICALLY INDErERMlNATE BEAMS

St1bstituting expressions (a) and (c) into eq. (b),

-1
EI
1o
1
( Xx - - wr) dx =
2
(x) 0.
Jtence,
CAB'l'lGLLANo's THEO

frorn the Castigliano theorem,


dU .., Xl
dX .4 E -
(P _ X)l
REM hPPLlllr,

A iE == 0,
255

Performing the indicated integration, we obtain


Xl3 wl 4
---=0
3 8 '
.., which
frou•
X -
-- 1
p
+ ~,.
,t
(g)

•atement of the problem requires that


from which X = }wl, as obtained previously on p. 233. fhe s...
Choosing the restraining couple MA at the built-in end of the beam as P-X
the redundant react ion, the bending moment at any cross-Rection becomes X = '1i.
(h)

M" = w~x _Mtx _w;2. (d) Eliminating X between eqs. (g) and ~h), "'.e find A i/.l = ~-
EXAMPLE 2. Two ~~ bea_ms of identical cross-section are supported at their

Then since the angle of rotation of t.he tangent at A is known to be zero ends and cross at their mtd-p<>mts a:i shown in Fig. 9.18. When unloaded, they are
just in contact at C. What interactive force X will exist between the two beams t
the Castigliano theorem. gives ' cwhen a·vertical load Pis applied to the upper beam e.s shown? 8

SOLUTION. The net downward load on the beam .4 B is P - X • that on the beam
iJM" d DE is X. The total strain energy in a simple beam loaded at th~ middle by a force
au _ ( 1 M "aM A x _ Qis, from eq. (h), p. 222,
iJM A - }o EI - O, (e)
Q2la
wherein, from eq. (d) , U=-·
96EI
iJM:z; X
(f)
iJMA = - T
Substituting expressions (d) and (f) into eq. (e),
X

1111 (w~x - Mt- w:2)(-ndx - 0.


A
I
I
I
I
I
8

D I
I
Performing the indicated integration, we find MA = wl2 /S, as previously .J, I
obtained on p. 234. ,I ...--2
'
I
..t 2
~
I
I
I
EXAMPLE 1. Three bars each of length l and pinned at their ends are arranged ~ I I
I
a vertical plane as shown in Fig. 9.17. The vertical bar has a cross-sectional _a~e~ C I I
I I
and each inclined bar has cross-sectional area A 1• A vertical load Pacts at l~in in I 8
and it is desired to find the ratio A ii .4 of cross-sectional areas to make the tension
DC numerically equal to the compressive forces in AC and BC. t bar,
SOLUTION. Let X represent the tensile force in DC, chosen as the redundan .
Then the compressive force in each inclined bar is (P - X)/,/2. Thus the stra•n
FIG. 9.18
energy of the system becomes F10. 9.17
l'hua th
l + (_P_-_X_)2_l e st rai11 energy in the two beams becomes
2
U _X_ =
2AE 2A 1E (i)
• . (P-X)'P X s.
+~ f

In th18 case, the end D of the vertical bar must have a displacement equa1 to zero- U == OOEl 96El
I

t
CA8T1GLIANo's TJQo
'&al( AlPLJ-.o
256 STATICALLY IND£T&RIIINAT E BEAMS
257
P RO BLll( ll:T 9
Regarding the p.cur of int{'no.ctfre forces X M a_gene!'1'lised force and observin
rnt Ix-tween tlwg t~t prismat ic beam . AB with built-in end1i

the rorresponding displact> mmt is thr relative d1spla~en~ .
two beams, which is sero bc:-cause they remam 111 cont.net 1\t C, We : 1d- l• .A.1o,.d p at the middle. Using the theo and 8J>&n l Carri
points of the the bea. ~m of Cae~ a !IUlgle con-een-
lt'ted JI" and M. at the enda of "'

111olJlCIPtl t the solution of the PJ'eeedi rn . A"'· M,. - M ' find ~ bending
ave 0
by the O&stigliano throrem
dU (P - X)/1 Xa 3 _
0 , .. ~v:ifo rmly distribu ted load of in1:: ~robJern for the~ -/l/8.the beam
48El + 48El - ' e¢1e8 • ...n;12. nsity 1D overt.be full "' ere
dX lp&n A
· q _ ,,f,. •
i
- -Wt- . .
J[, ,A. continuous pnsmat 1c beain having t 0
P/3
S, . ted Joad of intensit y w over one 8 "' equal 'P&na l earn
.
from which X = p + ai· U::b:
di 0
9 X at the middle suppor t by usingp : ~~ly, &8 shown in Fiigf>.8; unFiifonnJy
fdC"' wieorein of 0 ....: _,. · · tnd the
~--&!W lo. A111 . X ,..
EXAXPL E 3. To reduce deflection, a simply support
the middle, is trussed by steel rables AD ~d BD
both
and
the
ed wood beam A.B lo d
n post CD arranged
beam and _t~e post,
a:
fi~d
~ e.t
own
f,Pl/8,

,r/un,t ltnQlh
in Fig. 9._19. Negle_c~ing a.x~l shortening of
&\·e force X mduced m t he post . The beam has flexural r1g1ditv EI d the 8
oompres
cables ha Ye cross-sectional area A, and modulus of elasticit y E 1•
• an the ~11 11: ;: 2.
~-~ +
net downwa rd load on the middle of the lx>am is Q = p _ X
SOLUTION. The
the tension in each cable is S1 = X/ 2 sin a. I:Iencc, neglecti
compression in the bc>.am.and post, the strain energy of the system is
ng strain ener,,,~
·1:>.1
nd
of .L---l
2
1
+2( 1"JG. A
u= QP S1 l1 )·
. FIG. B
96EI 2A1E1 . th theorem o f Caatig)ia no find th ben .
at ~ rn:icldle
'- U81Dg e
= ~l sec a, this becomes 1111pport of the two-spa n continu ous t>e'am load~ 18~g mo_ment Mc
Substitu ting for Q and S1 and noting that l1
,ol'/16. own tn Fig. A. Au. JIc ,..
(P - X)2P X 2l sec a 5. A propped-eantilever beam of uniform • .
- 1-
- - + BA
u = -96EI --·
E I sin 2 a t:i~uted load of intensit y w over the right-h an~ro~ ~n cames a unif°:"111 , dis-
Uang the theorem of Castigliano find th . d e span a.s shown m Fig. B.
Now sho~n~ g of the post is t he gen?ralized dis~la_cement correspo
nding to the
i.e., that it
propped e~ of ~ beam. A""· Ji= 7u,l/ t ~
28
tu e of the reaction R u the
this to be neglected, . .
~ mpressi:ve 1orces X and we have specified that IS
6. A thin sem1c1rcular arch ring of mean rad' R .18
IS to be taken as zero. Hence, the Castigli nno theorem gives foundations at its ends A and B and . ius pin-supported an U.n)'leldi ng
shown in Fig. C. Using the theore cam~<; ~ vertical load P a~ the crown c, s.s
dU (P-X) P Xlseca ano, find the horisonts..l ~ompone n~
-= ----+----= 0 Hof the reactions at A and B Am of Cast1gh
dX 48El 4A 2
1E 1 sin a ' • na. H = P/ r.
p p
from which -----
X = - 12seca
1
+ EI C

l'sin 1 aA 1E 1
Taki~ er ""'1ff and EI I A ,E ,l1 = O.Ql, this gives X = 0.478P. In such ~ the
deflect.ion of the bea.m will be reduced by approximately .50 per cent.

FIG. C FIG. D
7• A thin · · R ,s · ..-d
· subiect
to the . circular ring of uniform cross-sec tion and mean r~ius
18
· : (Ts-
ing th:~t•on of two equal and opposite fon ·es P pulling a.long a diam~terth st
lhe end heorem of Castigl iano find the bendi.ng moments .I/ induced m e rmg
! 1B tn.s M,., 0.182.PR .
s of th d " · · · d . ·t .
8, I" E e 1ameter which is pcrpend1.cul&r · to ·· ·th ' beam an pos v. ere
n .. xarn I 3, _the strain energy of comp~i on rn ; C' ulate this com-
0 eglect'<l in P e 1
Ptes.1ive lore cal,culat mg the comp~ ive (on-e . X in the ~i~ion: 1: train energy of
F10. 9.19 e .t more C:\:actly by tak111g account or the
258 STATICALLY INDET ERIIIN ATE Blill8
LIVIT ANALYSIS

compression. Use the data given in Example 3 and assum_ d p is furthe r increa sed the 259
e, in additio n, "'-t .
post is cut from the same piece of timber u the beam Wltb AE • 4A E be loa.
f.S t to grow un t l'l l't a lso attain' s the negati
,:_ . . bend'mg moment M
ve
X - 0.427 P. ' ,. -itb~ ·nues h d th be
"-. t
coll i_ • is reac e ' e am will not resist . um1tmg val M
9. A wood cantilever beaml.5JD long and having a rectangul_ar
~roSIH!ection d1t10Il ue p. When thiss
wide byl5cm deep is built-in at B and supported: ~t A
by an
coll rnes a mec h.anism
.
wit
. h hi
nges at A D any furth .
er mcreaae in load
shown in Fig. D. The beam has modulus of elast1C1ty 14(10~ •mchned guy 'tii.~tlll !llld
t,eco h •
'thout furt er mcrea .
se m load. Th
, , and B whi h .
kg/ant , and th '~ , c will
wire bas modulus of elasticity 21 (10} 1 kgfcm• and cr~ss-s
sq cm. Find the tensile force S induced in the guy
wire due to the vertical 1:a
ectional a.re& A, _ Oe lily rreelY :~ce
IP pr?.
this condi tion_rep~esents the limi;
is repres ented m Fig. 9.2Oc.
z:u;
of the load required
L for the beam. This
collapse

applied at A. Ana. S ,.. 1.48P.


l> colld1t1onIculate the 1·1m1t. l d .
oa , 1t is not nece
9.5 Limit Analy sis of Static ally Indet ermin ate Beam
s fr~
'fo bees.ginning to end as outlin ed above
in D l ff . ·
8;ry
~ trace the behavior
e simply a.ssu ,__ .
at B and at , eac 1 o enng consta nt bendin
5 me PI.IWltic
In all the foregoing discussions of statica lly indete rmina h111 ge olve for the value of Pr.. requir ed to maintain e gil~o_
te bea'"" . ment MP , and
has been assum ed that the beam behav es e lastica . ll h .. ,s, It tbeFn.s 9.2Oc, the reacti on at A in the limit conditi quwilli_nbeum.
Y t rough out. It .. tn 1g. Referring
also of intere st to invest igate the behav ior of such on
beam s under lo ~~
which induce plastic bendin g of the beam, as discus sed
in Art. 6.1. Consi : s RA= Ptb - Mp
for examp le, a propp ed cantil ever steel beam subjec l - l.
ted to a load p e~
shown in Fig. 9.2Oa. For any value of this load less Then the bendi ng mome nt at D becomes
than the elastic lirnit
value P 8 , the bendin g mome nt diagra m will be as
shown in Fig. 9.20b.
From this diagra m, it is seen that points B and D
are points of largest
Mn= RAa = Ptlab - Mpa - M
bendin g mome nt. If P is increa sed beyon d the value l - P,
PE, a plastic hinge
will begin to fonn either at Dor at B, depen ding upon wheth from which
er the elastic
mome nt MD or Ms is greater. PL= MP(l + a),
This, of course , will be deter- a(l - a) (a)

A....,___
r _o-=_•---~ -=-b==;~1/ mined by the positi on of the since b = l - a. Takin g the safe working load P.
= P L/n, we attain
~
-~ load P on the beam, i.e., by a factor of safety n again st compl ete collapse.
Ji; 1
- -- -;._ _--. J--- --i l!i
the ratio b/a. As s. matter of
fa.ct, it can be shown that the
It will be noted from expres sion (a) that the limit load
PL is a function
of the ratio a/l defini ng the positio n of the load on the
\ elastic mome nt MD will be span. To find the
\ (a) \ most critical positi on, i.e., the ratio a/l to make PL a
numer ically greate r than the minimum, we set
\~ I dPL/da = 0. This gives
\ M II elastic mome nt Ms if b/a <
I
I
~ M
~
-{2. Assum ing this to be the case, a1 + 2la - l2 = 0,
I,~ - -
--, _ _ _ _ ___;;;:,//1 a plasti c hinge will form first at from which
\
I
I
I
I
I
I
fJ.
( b) I
I
I
8\ 1= -v'2 - 1 = 0.414.
I
I
Mp
A ~I ~--- .,-+ ~:;- ----: ;_:::~,l
.Qt~'-
o -p--l;0
o---M - -~~~~~-b- =:8,:;1bf
; :i1', Substituting this value of a/l into eq. (a), we obtain (PL)m
in. = 5-84 M1:/l.
. Another metho d for calcul ating the limit load PL -for
the mechanism
in Fig. 9.2Oc is to use the princi ple of virtual work. We the system in
the li,-..,.•t take
uui condi. tion
. . . ·th l t' hinges at
(cl 1 (blar.k · l
1.e. as a mechamsm WI pas ic . B. and D
FIG. 9.20
FIG. 9.21 · circ es) and a' simple
' h' ·
hinge at A (w ite me e - l ) .. a shown m Fig. 9.21.
1'he loM p . .
point D. After this plastic hinge is fully develo ped at 1,.. L and the plastic mome nts at
B d D we consider as active
point D, the bes~ J,,,eea on th' an. . t f the system
behav es as a statically determ inate system repres ented can be . is mova ble system . The? a v1rtua · l displacemen o
hy a c01npot1n of the hinge D. At the
beam A.DB acted upon at D by the load PL and an P.A... defined by a small vertic al displacement 6Y
intern al moment Mr· ...... 1e ti'"" h . n . bout the im-
...e t e portio AD will rotate bY the angle 6y/ a a.

_J
260 STATICALLY INDETERMINATE BEAMS LlMrr ANA.LY818

mov~ble hinge .4. and t,he portion BD will '.otate ~y t_he angl~ -~y/b abo1.1 t the supporte C and B. Af~r theee h' 261
rorf11 fi!11~:1oa.d , beha:-es as two simply 8Upported1 ~!es
fonn, the 8Y8te
the 1mmovahle hinge B, as shown . Durmg this virtual d1spla.cem t , crtle,.¢ , e forms, either at Dor at E .. 1€t
10 ~lllg AB &nd BC rn, ~n further
the load PL produces positive virtual work P L~Y while the plastic rno ent, 1&8tic h~ng e Then the right-hand span becllll &a.,urne that D will~ until another
MP at B produces negative virtual work M p~/_b_a~d those at D, neg:~~~t ~i,.s~~•';!a.~hed. At such time, we have foro~~~b~ haniatn and~~~~~
work MP (oy/ a + oy/ b) . As a condition of eqmhbrm m of the system e
th
tiJ9 M = wd2
algebraic sum of these virtual works must be zero . Hence ' e D 8 - Mp'= .\-[p,
Pdy - Mpoy / b - Mp(oy / a + oy/b) = 0, .., which
from which , on canceling oy, fr<>•"
16Mp 16 x 16,222
WL = _l_2_ = 4 x 4 "" 16,222 kg/m.
PL= 2Mp+ MP _
b a (b) 'th a plastic hinge at B, the reaction at A will be l
Wi t anY section distance x to the right of A be w I 2 - Jf p/ l a.nd the sh.ear
Substitu ting -b = l - a, this gives the same result as expressed by eq. (a) force a · comes
above . Vz=~- MP
2 l - IDX .
Limit analysis of statically indeterm inate beams as illustrate d abovP
is often much simpler than elastic analysis, since by such procedure th~ Then to find the section of maxi°:um bending moment in the •
express•· 00 equal to zero and. obta.m
5
problem is always reduced to a statically determin ate one. It is also pan AB, we set this
gaining favor amo.ng structura l engineers because it leads to greater l MP
X = - - -·
economy in design. In dealing with more complica ted statically indeter- 2 wl (c)

t ti
minate beams and frames, the question of selecting, in advance, the proper
positions of all plastic hinges is not always a simple one. Sometimes w kg perm
there will be more than one possible mode of collapse for the system and
each of these musL be investiga ted separatel y to find the one that leads
to the lowest limit load. These questions belong more properly to special
I: , : 11 1 111 I.I~ 11111 'cl
books on plastic analysis and limit design and will not be discussed here.•
{o)
EXAMPLE l. A two-span continuous beam simply supported at A and B nnd
buil~in at C carries a uniformly distributed load of intensity w over its entire length
(Fig. 9.22a). The beam has an ISMB 300 section for which the plastic section
modulus ZP = 648 .9 cm•. Calculate the limit intensity WL of the load for com-
plete collapse of the system.
~ ~

(b)
,
SOLUTION .
AITi 11 1 E,11111111{111111111 ~I11 111 i 9._'
~ I ~ I
Assuming a yield stress
moment becomes
u ) .p. = :l,500 kg/ cm!, the corresponding plastic bending ~x-\ I
~2 n----2 m_j
: (Cl
•'[ 2. '.> 00
- -1-00648.9 ~
= X .
·• P - - = l(i,222 kitm , \

_In the comple~ly e_lastic condition, th~ bending moment diagram for th_c ~ea: ~ SJM,
will be as shown m Fig. 9.22b . From this, we see that plastic hinges arc like Y
(dl
•see, for cxo.mple, Plasti:c Design in .Step/, Am . ln!.t.. of SIRel Constructi on, 1959, F10. 9.22
262 STATICALLY INDETERM INATE BEAMS LIMIT A.
NA.LYs1s

The corresponding bending moment at Eis p


d
MB= ('1- ft~p}- W;'
p
C
~ -----.r1 A
(d) 8
Substituting the value of x from eq. (c) into eq. (d), we obtain
J -1m_j
M = wl: -
E 8
J(p
2
(1 - MP)
wzt
·
V
Jril
FIG, C
2 &
2--, I

Taking w = 16,222 kg perm , l = 2 m, and Mp = 16,222 kg-m, this giv F1G . D


2,028 kg-m. Since this is less than MP = 16,222 kg-m, we conclude that : It~ t "' be beam shown in Fig. D carries a unifonnl . .
hand span does collapse first and that WL = 16,222 kg/m is the limit load ;~1&~t. 9, ieadlft f the concentrated load P, find the linu't YI distnbuted loa.d of inten.,;t
o
for the continuous beam. 10111s eding pro bl em. A ns. WL = 6,622 kg/m.va ue of w· Use all data a.s given
=Y
tls1ty ill the pr~~ntinuous frame A_Bl' built-in at A and sup
}0, A. the action of a honzontal force p a.t Bas / 0 ~ by a. roller at C is -;ub-
PROBLEM SET 9.5 jected ~ p L of this force if the uniform section or8tho~n tn Fig. E. Calculate the
litllit\l&~~ Ans.PL= 2Mp/l. e ramecande velop aplMtic
I. A two-span continuous beam carries a uniformly distributed load as sh rnoIIlent
irH•
.
Fig. A. If Mp is the plastic moment for the cross-section, find the intensitt" n l!\
the limit load. Ans. WL = 11.66Mp/ l2• Wt of
2. The continuous beam in Fig. A has a rectangular cross-section 5 cm. wid P_ _.;B
r ,===== =r, C
10 cmdeep, and l = 2 m. The material is structural steel with a yield stress u e ~
2,500 kg/cm 1 .What is the numerical value of WL? Ans. WL = 9,110 kg/m. Y.p. -
3. Calculate the limit value PL for the statically indeterminate beam shown . p
J p
Fig. B, if the plastic moment Mp for the section is given. Ans. PL= 6Mp/ l. in

l A

L . l
FIG. E F
gz
w per un it l~ngl h F10.

j£ 1I11I11 1 1i1 113L ~A 11. Referring to the continuous frame shown in Fig. E, assume that instead of
the concentrated force P at B there is a uniformly distributed horizontal bad of
l--- ,L ,t-----j intensity w acting to the right on the vertical member AB. In such case, find the
limit value of w necessary to produce complete collapse of the frame. Ans. w1 =
FIG. A FIG. B 4Mp/l1.
12. A thin steel ring of mean radius R is subjected to the action c.,f two forces P s.s
4. Calculate the limiting value PL for the beam in Fig. B if the load at the end of shown in Fig. F. The cross-section of the ring is such that the plastic moment is .\f P·
the overhang is P/ 6 instead of P/ 3. Ans. PL = 7.2Mp/l. Find the limit load PL for the ring. Ans. PL= 4Mp/ R.
5. If the statically indeterminate beam in Fig. B carries a load P at D as shown 13. If the horizontal force P in Fig. E acts at a height h abo\'e ~he ground level
and a load aP at C, what is the value of a to make the limit value of Pa maximum A instead of at Bas shown what will be its limit value? Ans. P1 == '2 .\ f p/ h.
and what is this maximum value? .4ns. PL = 8.\1 p/ l when a = ¼- 14, In Fig. 9.19 p. 256 the beam AB is a steel bsr with a 2-,m X 2-~m--square
6. A cantilever beam AB built-in at Bis supported at A by a tie-rod and loaded •
c~oss-section, the post CD• is the same, and the cable .-tD B ·is as tee I wi rehhanne: rnit
with a force P as shown in Fig. C. The beam has a rectangular cross-section s_cm diameter d =0.2 cm. Th le a = 200 and the span l =' 2 m. Cs.l~uls.te t e_ 1i •
e ang
wide by 2 cm deep, and the tie-rod has a circular cross-section of diameter d. Find va1ue of the load P if both beam and cable hav e a yiel d st ress, Or2•)00 •kg cm · · .-1 r1., .
.
the minimum diameter d of the tie-rod required to develop the full collapse l?a~ PL "' 153.8 kg.
PL for the beam. Both beam and tie-rod are made of structural steel with a yiel
stress of 2,500 kg/cm 2 .A ns. d = 0.252 cm. .
7. If the load P in Fig . C is applied at ,1 instead of at C, what will be its lirni~
value to produce collapse of the 8ystem '> The tie-rod has diameter d = 0.2 cm, an
all other data are the same as given in the preceding problem . ..1 ns. PL = 141 ~~~
_8. Calculate th~ co~lapse load PL for a beam with built-in ends and loaded a~ cm,
middle as shown m F111;. D. The beam has a depth of 20 cm, web thickness o. 5 ral
!\angcs 9.42 cm wide and 0.8 l cm thick and a span l = 3 m. The material is structU
steel with a yield stress of 2,500 kg/cm 1 . .-1 ns . p 1, = 9,933 kg.
r ECCENTRIC LOADING

f JllOillent M = Pe (Fig. 10 1 )
ie O uniform compressive
OF BTI\UT
265
• • c · The
c01JP stress p centrally appli d
..Auces • . . 171 ==
stiess diagram m Fig 10 lb
IA e 1oad
10 Pru"ho\\' 0 by the
. . · . . Lik •
over each crooo t·
.,.,..sec 10n
119 s roduces linearly vary1~~ bending stress 112 == ew1se, the end moment
MP1c Then ' by superposition, the total My/I as shown in F.
Theory of Columns .
JO, • combined bendmg and compression be
due to
compress·
comes
ive stress in any fibe
ig.
r
CT=~+ Pey
A T ' (10.1)
is the cross-sectional area and J is th
•here A . e moment of · rt·
IO.I Eccentric Loading of a Short Strut w . a.bout the z-axis. Also, since we are t me ia of the
section . mos concerned h .
ssion compressive stress is treated as posit' Th ere Wlth
Consider in Fig. 10.la a short post or atrut subjected to an eccentri co111Pre ' •• . ive. e stress diagr
is superposition of stresses is trapezoidal as h . . am
applied compressive force P at its upper end. If such a strut is co~a.Uy for th . s own m Fig. IO.la
Taking y = c1, the • distance.rrT. to the extreme fiber th . h ·
on e rig t and
atively short and stiff, the deflection due to bending action of the ecce!":· introducing the notation r = "I IA for radius of gyration of the ~ross
load will be negligible compared with the eccentricity e, and the Princi ~c section, eq. (10.1) becomes
of supperposition applies. The strut. is assumed to have a plane of sy~~
p p O' mu
ec1) ·
= AP ( 1 + r2 (10.2a.)
_... I
M•P• Similarly, taking Y = -es, for the extreme fiber on the left, eq. (10.1)
I
,--0_,'. ..-.... ---, /'o\ gives
C1 I C1
(10.2b)
ll'111i11
.,z Usually we are only interested in the maximum compressive st~ as
given by eq. (10.2a). However, it is of interest to note from eq. (10.2b)
that if the ratio ec,Jr2 becomes greater than unity, there will be tensile
stresses in some fibers on the left side of the strut. For example, if the
I
I cross-eection is a rectangle of width b in the z direction and height h in the
tx tx Ydirection, we have c1 =Ci= h/2 and r2 =// A= h2/ 12. Then from e~.
(al (l0, 2h), we conclude that no tensile stress will be produced by eccentnc
(bl (c)
compression so long as the condition

if I
I
,
is sat' fi
't _is ed.
1 Will l
(lo
•2a),
h
-6<e<6

a ways be smaller than t he compreSSlYe


h

If condition (a) is not fulfilled so _t a


stress o
btained from eq.
. h ase of such materials
(a)

h t tensile stress does occur,

F10. 10.1
as b · hut even so , it may become important· int cc
l'lck or concrete which are weak in tenS1on . -t ut under th~ action
metry (the xy-pla11e) and the load P lies in this plane at the dists.n~e ~ Let u · '
the more genera case of a :s r
1 · · I
from the centroidal axis Ox. Such loading may be replaced by its st
equivalent of a centrally applied compressive force p (Fig. IO.lb) an
att
11
of a 8 consider now . .
ither of the two pnncipa
r;:
Plan cornpressive load P which does not h~ 1 the principal axes through
ea of bending. In Fig. 10.2, let Oy and z

J
J!;CCENTR IC LOADING OF STRUT
267
ind the equation of the boundary of the core for t h e case of a n
· 1- Fded strut o r rec t angu Iar croas--section as shown
266
THEORY OF COLUMNS
•.,,i.t..
t:"" · 11)'
l
oa · t f 1· t·
. Fi 10 3
m p tg.
f the compressiv e force be
· a.
fined
the centroid of the cros...'\-section a. d , trtCII L,et the pom o a.pp ica ion o
point of applicatio n n eC~¢tO~ · ht-hand quadr9:n~ (m and n both positive). Then to make~: line:
./ b 1e t A be the ·o . of
the load P. _enotmg the coordi- Jower r g ~as through pom_, B, we set Y = -h/2 and z = -b/ 2 in eq . (10 .4) .
the •-SS p.-
nates of this pomt by m and n, the to s .... -
,e. gives mh _1.
moments of P about the a.xes 0 'fhlS _ _nu_ _
t + 2r, - 1,
2r, "
and Oz will be Pn and Pm, resPe!
o· tively. T~en, by superposi tion, the . h 2 = 2 2
h 2 /12 and ri, = b /12, becomes
compress ive stress <r for any point . . b wit "•
6n
lfbJC ' 6m
the cross-sect ion defined by coor;: T+T= 1. (c)
I n.a.tes i and y becomes
q uation of a hstraight line cd having intercepts h/ 6 and b/ 6 on the y- and
Fro. 10.2 ·s the e . F" 10 3 A l
p Pmy Pnz, ·
This 11 ctively, as s own mth· ig.1· · a.d 8 . ong as the point of applicatio n of
a-=-- +-1-+ I II (10.3) respe 1S me an pomt B , the fiber stress e.t B will be
A • z-axes, d p is situated between
the loa . n Similar argument s, dwhen the 1load P is con.fined to each o f the other
l ) Equating the right-han·d sidef of this equation
• com P
ress10 .
drants
ad
can be m e an we cone ude that the core of the se<:tion is the
analogous t.o eq • (lo · · of pomts o zero stress in the
bt • equation for the locus
- ~ ✓-- threeJ~o mb~ shown in Fig. 10.3a.
to zero we o am an cros.~secti on of diameter
cross-~cti on. With the notations r, = "'JI.IA and r" = 111/A. for radii shad .1 ~Jar
1
argument can be made for the l:!iSe of a circular
f th t· . . l .
A s m
dand we find that the core o e sec ion 1s a cue e of diameter d/ 4 as shown in Fig.
of gyration , this equation of the line of zero stress becomes
my+ nz +1 = O, (10.4) 1C.3b.
r,2 ri 10.1
PROBLEM SET
which is seen t o be the equation of a straight line, shown in Fig. 10.2
as the line ab. All longitudin al fibers situated in the unshaded portion of 1. A short strut of square cross-sect ion has a notch cut in its side as shown in Fig.
the cross-secti on v,: ill be in compress ion, while all those situated in the A. Calculate the maximum compress ive stress u at the section m n due to a centrally
applied load P. Ans. CTmax = 8P/a •
2
shaded portion will be in tension. The intercept s u and v of this line on if the cross-secti on of the strut is a circle of diame-
2. Solve the preceding problem
t he y- and z-axes are found from eq. (10.4) by set.ting first z and then y
ter a. Ans.<Tmu: = 9.1P/a 2 •
equal to zero. Thus 3. At the cross-sect ion mn, the C-clamp shown in Fig. B ha.s a rectangula r crosi:,-
V =
- r112 _ (b) section 2 X I cm. Determin e the maximum tensile stress u at n if the load P = 200
n kg and the dimension b = 6 cm. Ans. u = 1,900 kg/ cm •
1

4. A short piece of ISLC 250 channel carries a compressi ve losd the line of action
For a given cross-sect ion, it is of interest to define a region around the
of which passes through the centroid of the web. If the a.llowable ma.ximum com-
centroid "'.ithin which the load P will induce compress ion over the entire pressive stress is 1,120 kg/cm2, calculate the safe load P . A r..!. P = 2~, 770kg.
section . This region is called the core of the section. The determina tion
of this core for A. rectangul ar section is discussed in the following example.

er h
• 1
b

T b y d

_L C
I
P
·1
I

_l
I (a) ( b)
FIG. 10.3 Fro. A Flo. C
FIG. D
268 THEORY OF COLUMNS EULER'S COLU!(N
roalfUI.A
. 5. A short piece of ISA 150 150 x 1.6 cm angle iro~ carries a compressive 1 bY s. Then the hending mom 269
bne of action of which coincides with the intersection of the middle Plan °ad the ·nt B ent at the er
legs. If the maximum comprest1t.ve stress is not to exceed 1,120 kg/cm•,
safe load P? 11m. P = 11,390 kg.
wh:~ ?f the
18 the
pO' ornes
,t
i,ec
088-section defined by
M, == P(o _ y).
6. Solve the preceding problem if the angle iron has an ISA 200 too
section. Am. P = 11,220 kg . X l.6c 111 consider only a very small later 1d . (a)
· ce VI e d R. eflect
7. Show that the core of a circular cross-section is circular and find its r d. S1fl d' .g moment an curvature is (see eq 8 ion, the relation beh-
is the diameter of the section. .4.ns. d/ 8. a ius if d i-,en Jll ' · · 1, p.198) ~ween
8. A short strut loaded in compression has an equilateral triangular cro . <fly
with sides of length a (Fig. C). Prove that the core of the section is an ess~~ct1on E I d2 ==: ::i: M
X .
triangle with sides of length a/ 4 as shown in Fig. C. qui &ter&.1
. ~- A ~hort strut has a hollow circular cross-sectio~ of _outside diameter d
inside diameter d2. Prove that the core of the section 1s a circle having 1 &~d e chosen axes x and y, cPy / dxi is pos·f . .
for th . . f i ive m this case d
r, = t(d 1 + d 21/ d 1). radius •gn in this expression. Ience the diff . an we use the
plus s1 es erentlB.l equation of the elasf
line l:)ecom 1c
10.2 Long Columns; Euler's Column Formula <fly
EI dx 2 == P(o - y).
Let us consider now the case of a long slender column AB of len th (b)
1
which is built-in at its lower end A and subjected to a centrally ap\·
· upper end B as shown m
· load P at its · I'" •viding'both !'iides of eq. (b) by EI and introdu cmg
· the notation
compressive •ig. 10.4. This colp ied D1
is &S.5umed to be perfectly straight and of uniform cross-section. ~m_ n k2 = !._,
also assumed that the material is homogeneous and that it behaves elast~~ EI (c)
cally.
Experience shows that when th it may be written in the form
vertical load P is small, such a com~ <fly
p
z pressed column is laterally stable. dx2 + k2y = k20 . (d)
8
That is, if the upper end B is pushed
8 slightly to one side by a lateral force This is a linear differential equation with constant coefficients. Its solution
the column will return to its straigh; y = f(x) consists of two parts :(1) the general solution of the corresponding
form as soon as this lateral force is homogeneous equation (zero on the right-hand side) and (2) a particular
removed. However, as Pis gradually solution of the complete equation. It can easily be verified by substitution
increased, we observe that at a certain that a general solution of the homogeneous equation is
value of this load, the straight form of Yi = C, sin kx + C1 cos kx,
equilihrium becoM unstable and
the colu~ n, .if pu~ ; d lQ_one side, where Ci and C2 are arbitrary constants, while a particular solution of
="'"'~--► , stays there even after the lateral force the complete equation is
is removed. This instability phe· Yi= o.
FIG. 10.4
n~ n is called lateral bucklil'l{) and
the value of the load at which it occurs is called the critical load, denoted Thus the complete solution becomes y = Yi + Y! or
by Per· (e)
Y = C, sin kx + C, cos kx + o.
. To fin? the load Per which will cause buckling, we consider the column
T0 d C we ha\'e the boundary
m the shghtly bent configuration shown in Ji'ig. 10.4b and calculate the find the integration co11stants C1 an ~.
magnitude of the vertical load necessary to hold it there. Choosing cond·r . . end A:
1 ions at t.he bu1lt-m
coordi~ate axes thro~gh point A, as shown in Fig. 10.4b, we deno~e th~ (f)
deflection at any pomt x on the elastic line by y and the deflection ° X = 0, y = 0, ~ = Q.

J
270 THEORY 01'' COLUMNS
EULER'S COLUMN
FORMULA.
I
,

. d'fferent or neutral. In this case ·r . 27 l


One differentiation of eq. (e) with respect to x gives t:,e in I d th I ' I P<llnt B . L
to befl release ' e co umn simply rem . . is pusnt>d to the sid
dt p d . ams m th r e
dy = C1 kcosh - C2ksin k.r. all . iofl• If p > er, ~n p01~t B is pushed sli htl e s igh tly deflef'trd
dx (g) p0sit. -eases indefimtely and the verti I g . ~ to the side the drft
. nc1 Th ca equihb . , cc-
Now substituting conditions (f) into eqs. (e) and (g), we find uo11.d to1 be unstable. us the value p as d fi
er e ned b
num configuration ·
IS
~ ~irnurn load that the column can car . i . Y eq. 0 0.5) represents
C1 = 0, C, = -~, the rn load for the column. ry ' t 18 sometime,, called the
and t he general solution (e) becomes &tJler rnination of eq. (10.5) shows that the crif1 11
EiaPortional to the flexural rigidity EI a d~ oad for a given column
y = ~(1 - cos kx).
(h) · pro n inversely ·
JS square of the length l of the column . It will also proportional to
This shows that the column bends so that its elastic line takes the for the . ..,i load is independent of the compressive t hbe noted that the
a rosine curve, the amplitude ~ of which is as yet undetermined. m of critlCl'I . . .
two geometrically 1dentical slender colum
s rengt of th te . I
. e ma na .
To examine this question further , we note that when :r = l. l/ == ~ Thus h f d. ns, one of h1gh-stren th
steel and the ot er o or mary structural steel , 11f .1b ~
Substituting this condition into eq. (h), we obtain · · · IIIIoY . l th I f h 'v.1 a1 y buckling
tapproximate y e same va ue o t e load p since the mod r , l . .
~ = 6(1 - cos kl) a approximately the same. It must th~rcfore be
are . .
I~ i dor ehMt1c1 ty
rea 1.ze t at the
from which I/strength" o_f a long col~mn is m no way dependen t upon the strength of
its material m compression, but only on its geometry and the stiffness oi
~ cos kl = 0. (i) the material.
From eq. (i), we conclude that either 6 = 0 or cos kl = 0. If ~ = o, the Equation (10.5) shows also that the strength of a long column mav be
column stands in the straight vertical configuration of equilibrium and increased by increasing the moment of inertia I of its cross-section . With-
no limitation is imposed on the magnitude of the load P. On the other out increasing the cross-sectional area, this may be done by distribut ing
hand, if cos kl = 0, i.e., if the material as far as possible from the principal axes of the cross-section.
Hence tubular sections are more economical for compression members
kl = n1r, U) than solid sections. By diminishing the wall thickness of such se<.·tions
2 and increasing the transverse dimensions, the stability of thr rnlumn may
where n is an odd integer, then the column can also be in equilibrium be increased. There is a lower limit for the wall thickne~ . howc·\'rr, below
with any small deflection ~ at the top. The smallest value of P for which which the wall itself may become unstable and, instead of burk ling of the
t his can occur will be obtained by taking n = 1 in eq. (j). Then, using column as a whole, there will be buckling of its longitudinal elrmenrs
the notation (c), we obtain which brings about a corrugation of the wall , called lOC'al b11ckling.
If we do not limit our attention to the case where n == I , eq. (j) is seen
to define an infinite number of values for the cri tical load . Thus, in general ,

n2i1EI (k)
from which Per=~-
rEI (10.5) ~aking n == 1, 3, 5, . . . ' we see that the values _of ~" are in ~he ratio
Per=--·
4l2 1.9:25·· · · · . C orrespon d'mg Iy, eq. (h) for the el~ttc lme become:,
This is Euler's column formuln. for the case of a slender column built-in nirx) (I)
at its base and free at t.he top . For all value~ of p < Per, we must have y (
= 0 1 - cos Zl '
5 = 0 and the straight configuration of equilibrium shown in Fig. 10.4a and . , h2.S more and more
~
is said to be stable. If point B is pushed to the side and then released, infi We. see th at as n 1s
• mcrease
• d , 1Iie deflect10n
· curve;
f == :3 and n == J ,
ect1on · fl · · urves or "
the col_umn will return to the vertical position. If p = Per, e~uili_briu:~ are h Pomts. Two of these de cct10n c . ' theoretically po&Sible
can exist for any small value of 5 and the vertical configurat10n 15 sa s own in Fig. 10.5. Although they represent

0 J
r 272

/( p /( p
THllOBY OJ' COLUMNS

p
p
EULQ's COLu~-

lf In this case, a horizontal teactio Q


~ 10~

,o1111'l'~udkling t,.kea place. Then the ben~~~- Will be induced at th .


273

81f)lell u Q(l ~ rnornent at l!ectio e_ PUUted end


,n :s =-= - z) _ Py, n z ll!
ifferential equation of the deflection
dthe d curve becornea t·
an d'y 1
I
C Eld.x' - -Py+ Q (l - x) . p
C

fbe ge neral BOlution of this equation is


y.,. Ci cos b + C2 sin kz + i (l - z) ,
I
I
I

r r i
✓:r'
J

rJ__,
n =- 3 n•5 la) P ( b) p k =
FIG. 10.5 Fro. 10.6

modes of buck~ing for t he column_s, they are of no practical interest becau5e


:ti:n d terroining the constants CI and CI and the unkn
Q, we have the following conditions at the ends:own

the column will always buckle m the first mode, as represented in Fi .(Y)r-o = 0; (y),._, = O; (~) = o.
•~o FIG. 10.7
10.4b, as soon as P reaches the first critical value given by eq. (10.S). g.
Critical loads for long columns having other end conditions than those
Then, from the above equation for y,
of the simple cantilever column in Fig. 10.4 can easily be obtained frorn
eq. (10.5). Consider, for example, the long column with pinned ends as Ci+ ~l ,_ 0; C1 coe kl+ C1 sin kl= 0; kC, -i = o.
8hown in Fig. 10.6a. From symmetry, it may be concluded that in the
Substituting the values of CI and Ct from the first and third of the above expremons
first mode of buckling the elastic line will have a vertical tangent at i~ into the second one, there results
mid-point C. Hence, each half of the column is in the same condition
as that in Fig. 10.4 and the critical load is obtained from eq. (10.5) simply
by replacing l hy l/2 . Thus
j Gsin kl - l cos kl) = 0.
,.:iEJ As long as the critical load has not been reached, there is no buckling &nd Q is zero.
Per= -rz-· (10.6) When the critical load has been reached and buckling is present, Q is no longer zero
and hence the expression within the parentheses must be zero. This gi\-es the
Similarly, for a long column having both ends built-in, Fig. 10.6b, the following transcendental equation for determining the critical load :
first buckling mode will be a full cosine wave having inflection points at tan kl= kl.
D and E. Then each end portion behaves as a simple cantilever column
of length l/ 4 and we obtain, from eq . (10.5) The smallest value of kl, different from zero, and therefore of P, which safafies this
equation is kl = 4.49. Since k1 = PI EI ,
Por = 41r2El _ (10.7) 20.2El T 1EI
l2 (m)
P., = k1EI =- 11- = (O.il)'
The case of a column built-in at one end and pinned against lateral deflec~
tion at the other end requires some further consideration and is discusse PROBLEMSE T 10.2
in the following example. l . to be used as a pin-ended
col· A lBO cm length of ISWB 150 standard I-~m p
= S7,600 kg.
EXAMPLE 1. A slender column of length l
is built-in at its lower end A.Jj ~tnn. Calculate the critical value of the load. n.,j "
13 5
m long with the
laterally supported at its upper end Bas shown in Fig. 10.7. Find the firS t er•
value of the compressive load P.
C
lowe:n IS~ 5~0 steel beam is to be_ used fcu~~~i:e m~imum compressive
load end bu1lt-m and U,e upper end pinned. a
that the column can carry. Ans. Pm.a= 82 ,SOOkg.
274 THEORY OF COLUMNS EULER'R COLU
MN FOflt.tur,A.
3. The member AB of the simple truss in Fig. A is 1.5-m lenp;t.h of lS otion of the elai,tic line and c 275
channel section with pinned ends and the member BC is a steel rod of ci Le: 7~)~~ , I eqU h ·r fl
efltlll bee.in o.s um orm exural rigidit a1culate th
E c vertical dPfl .
section having diameter d = 2 cm. Find the largest value of the verticn{f"1nt cr ~1 !el fl,.'fhB An8. Y I and len~th l A e<' t10 ~ oat mid-
the structure can support if oy.p. = 2,000kg/cm'and E. = 2(10)9 kg/cin1 A ond P th08s. ef,4. Jfld • Ql(tan kl/2 ) . ~~urne p11,nec1 P.nd~
· I I ?l8 , p Ill
8,100 kg. "'•~ ._ • ~ ... - --1

r,- s·
2P kl/2 , where le . ~
'"' '-I P/ El .
. ply supported beam-column of u ·r
,-. s1rn p b ni orrn fl ..
9, ....,Qjve loads at oth ends and to a cou eicura\ ng1dity F;i-,. b'
mPr"~ S t th diff · Ie of
to co . Fig. F. • e up e erent1al equation of h rnornent Maat' the
P ,, su J~ te(!
end B-
s1iow~ 1n11 r:= f (x) for p < P.r, Am. I e deflection cu rve and 6 d _as
JutiOJI .,, n its
12U 80
Mo(sin kx x)
l. FIG. A FIG. B 10,3
y = P sin kl - l '

. further Discussion of Euler's Col F


where k == .../?Tii.

umn ormula
4. The pole AB in Fig. Il is a pin-ended Douglas Fir column having . 'fhe case of a column with pinned ends (Fig 10 ) .
6
cross-section of diameter d = 20 cm. Assuming that the guy wire AC~ circular ntered and is called the fundamental ca,c;e ~ .d'~ most frequently
D
.

enough, find the maximum value of the horizontal force .P that can be appli:d st rong encou . · 1v1 mg both sides of
I0.6) for this case
top of the pole without causing the structure to collapse. Am. P max == 12 the
5. For the _simple pin-connected t~uss_sho"'.n in Fig. C, t~e members AC ~nd g.
4 ~5 eq. (
notation r
_lrlA f
= ~I I A'
hy the
d'
cross-sectional
.
area A d · . od .
an mtr ucmg the
or ra ius of gyration, we obtain
are slender pm-ended steel bars havmg 1dent1cal cross-sections. Find the val BC
the angle 8, defining the direction of the applied force P, required to make the er~ 0
value of this force as large as possible. Assume buckling in the plane of the r/gica
An&. 8 = 18° 26'. ure,
l •" = m" = (~· (108)

where the ratio l/r is called the slendP.Tness ratio of the column and ,,.,, the
p
critical compressive stress. The curve ABC in Fig. 10.8, plotted from eq.
(10.8) is called Euler's curve. For a column of any give n slenderness ratio,
plotted as abscissa, this curve shows the corresponding value oi the average
compressive stress (P / A )er, plotted as ordinates, for which the colum n
becomes laterally unstable .

FIG. C FIG. D
6. In Fig. D, the ban, AB, AC, and AD are slender steel rods of circular cross-
section which all have the same flexural rigidity EI. They have pinned ends at A,
B, and D and a built-in end at C. Calculate the critical value of the vertical load P
applied at A. Assume buckling of the hars to take place in the plane of the figure.
Am. Per=2.75('11' 2El / l2).
7• A so-called beam-eolumn carries a transverse load 2Q at mid-span and ,&
compressive end load P as shown in Fig. E. For P < P or = 7r2EJ /1 2, set up the di!·
C

X
'lO
A - 0...!--------- --,.1
8
FIG. 10.8 .

F10. E
.l
2 ---;
\

J ,l. -

Fm. F
.

n1ng 0 f . h t (10. 1s v...
lateral buckling, we conclude t ~ eq. . .
gives a value of u less than the proportional hm1t cr~ .1 .
,or
\
ticallv at the begm-
. Ince it was assumed that the material beha.ves e ass) . · ol'id only if it
.
of the mater1al.
276 THEORY OF COLUMNS EULER'S COLUIO;
PolU(uLA.

Thus, by settmg aer = ap . 1., we obtain from eq. (10.8) , a limitin to<> large for strength consid . 277
et . era.t1on 8 1
l/r below which Euler's formula does not apply. This is repr g Value ()f ~IId Y ., · thin this range of slendernes . a one to 0
, g ,. 1 . s ratio . "overn Col
1,Jlill called medium column.a. Anal . s require Sl)ec.
point B on the curve in Fig. 10.8, and marks the boundary be~nted by 1 · . umns
~U~ short and long columns. Taking structural steel with a p:•Wee~ ~- ,11d ~edre cornpressive loads has been w~s1ls of their behavi.o:uncdons1deration
h d' . e Ytreated bo
JP P ticallY, but a 1scuss1on er centraU
hmit ap. l. = 2,100 kg/cm 2 and a modulus of elasticity E =21(10J' kPort1ona1 of such heh . th experi Y
example, we find l/r = ✓1000 ~ 2 = 100. Thus for l/r < 100 th g/cnit, for tbeorle and satisfactory procedure in tha.v1or will not be ai:enhtally and
'lllP e . B . y· ' ese cas . . " en ere.• A
compressive stress in a pin-ended steel column will reach the' pre av~tage si , ht )lne E m ig. 10.9 and arhitr ·1
ar1 y let 1·t is. sunply to draw a
~1g · . es,
l~it before lateral buckling can occur and eq·. (10.8) is inapplic:t~rtiona1 st . urn compressive stress for such med· s ordinates rep
-411:llll • 1urn-le h resent the
this reason, the portion AB of the Euler curve in Fig. 10.8 is sh e. For 11,,...-btain the broken hne DEBC in Fig 10 n~t columns. In th·
.pe o f · .9, which lS way
dotted line ; only the portion BC is valid. own by a ·gn for a co1umn o any length can be used as a b .
of desl . · as1s
The Euler curve for the fundament.al case of a column with . As a spe~1fic example, let us consider the case .
ends can be used for columns with other end conditions by int pdinned for which E = 21 (10)• kg/cm 2, er P.I = 2 100 kg/ of ordinary structural
· eq. ( 10.5 ) w1t.h
· eq. (10.6), we· see rothatllci ng stee I • I ' cm', and 11
a mod_ t fied le ngt h l ,_ Comparmg 'fben for all pract1ca purposes, the yield stress Y;P - == 2,800 lcg/cm1,
modified length of a cantilever column will be 11 = 21, where l is th the ngth limit of the material for very sh rt u Y.P. will represent the
stre rirnents show t hat l/ r = 60 . 0
length of the column. Similarly, from eq. (10.7), we see that the e ac_tuat . . columns · Furt hermore,
expe . 1s the upper 11m1t f
o slende .
length of a column with both ends built-in will be [1 = l/2 wtl odftfied icb a column can sustam this average . rness ratios for
wh . compressive st ·h
column bm·1t-m . at one end and pmned
. at the other, 11 = 0.7l' (seep
I e or a
buckling. Thus for slenderness ratios from Oto ress wit out
27 n . 60 ' we take 11 CI.U
In general, then, eq. (10.8) may be used in the form · 3). I
28ookg/cm ass own y
h b th 1·
e me lE m Fig. 10 10 F == (1 -
Y. P. _ -
, 100 - . or s1enderness ratios
rE <r'f'P&ter than
"•v · , CTmu - '1cr, and we use the porti' BC f h
d' l
= (li/r)2° (10.Sa) on o t e Euler
'1cr curve. For mterme iate s enderness ratios (60 < l/r < lOO) h .
· use d. Th us t he complete , t de straight
"'!e have seen above that for columns having sufficiently high slenderness line EB 1s . . line DEBC in F. IO
ig. .10 etermmes .
ratios, buckling occurs before the average compressive stress can reach the value of '1onu for any given pm-ended mild steel colum
the proportional limit of the material. In all such cases, stability of the In discussing
. working stresses for steel columns , ,,,e ,. mustn.cons1·der t he
column governs the selection of a safe working load and we use the portion stress given by the diagram in Fig. 10.10 as an ultimat€ stress. Such a
BC of the Euler curve as shown in Fig. 10.9 as a basi-3 of design. As another c.omp~ssi~e stress brings the columns t.o complete failure , either by buck-
extreme, columns having very low slenderness ratios can be expected to lmg, y1eldmg, or a combination of both, and the working stress ~hould be
fail due to some kind of weakness of
(P/A)kg/cml A
p~ the material before the critical com-
,A\ . .
,,,..-s,renoth hm,t pressive stress can be reached. This
\
2,800 O E \
o E , __/____ failure associated with the strength
""' 01 i----"""7'
,-- ·y - -------F of the material may result from
----~---
2,100 --- +t
I
crushing, as in the case of concrete, I
I
: or yielding, as in the case of struc- l ,400f--_..., 1
_ _,_4--+---+--,---
: tural steel. In any case, some max·
: imum compressive stress can be set
Short : Med Lono c as a limit of strength and the safe
---;:;t---'i ___.i..___ _ _ _ _ ..! working load chosen accordingly.
0
r This, we can represent in Fig. io.9 0 :">0 100

FIO. 10.9 by a horizontal line DF dr~~~ FIG. 10.10


through the chosen strength ltm ·
In between the ranges of short columns and long columns there will be 11 •3ee T' . . M G w-Hill Book Co., Inc.,
~ell' y imoshenko and Gere, TMory of Euulic Stability, r ra
range of slenderness ratios too small for true elastic instability to govern ork, 1961, p. 175.

j
278 THEORY OF COLU MNS EULER'S COLUl(N
JOl\lifutA
1 aken as u mu l n where n is the desired facto r of safety . The Val
Pl\OBLEM& I T 1O,a 279
chosen for this factor of gafety depends upon unforesee n or a u: to ~
inP-r~s in the load P and al~ on ~o~_ible errors in the central ap~~~P.n_ta\ 1 bar of rectangular cross-eection 2 5
~ ,tee ends. What ifl the shortest length.! fx 5 cm i8 to be U8ed
of t.h1s load , as well as possible m1tial crookedn ess of the cohirn•at10
common value for n in structura l work 1s 2.5. n. A.
11 ,!~ pinnedo • kg/cm• and 0'11 .1. ., 2,100 kg/cm., or which Euler's
~ o,,, 21(1,_~ the critical compre11Sive st~ess f. Ahn,. l • 71.a cmeq
1 : . a Column
~P .-..lcUuo . . . 120
tion app~
l or
Ex~r~ments show t hat both errors in cent ral . applicati on of the S. ~ blelll 1f 1t 18 cm ong. Ana. a., .. t e colurnn de&cnbed in th
and m1t1a.l crookedness of the column tend to mcreaS(> with i 10act ced~ proxiruded nickel steel tube having out.a·~ 75
kg/~•. e pre-
. . s. !,.JI et ::s0.3 cm~s used as a pin-ended column13~ diameter d .. 5 cm and wall
slenderness ratios. This suggests the use of a variable
. ncreas·1
factor of 11~
which is usually increased linearl y with l/ r. For example, sorne st safety tJiicincsSive streBB if E -21(10)& kg/cm'.Ana . a.,. •+4-0~ong .• Calculate the critical
~pecifications recommen d using a fad or of safety n = 2.0
for O < 1/r < 100, and n = 3.5 for 1/ r > 100.
+·o~i;~Uta\ c0IIIP~ t is the shortest length l of the column de . g/~ ·
4, W, : Euler's formula can be applied if a i :::~~~ m the P~ g problem
1/r 1or ,.hi~ late the critical compressive stress r~~ ·th kg/cm• ?_ Ana. I ,,. 71..5 c:rn.
In the who!,~ realm of structural design , there is perhaps no s·t .
t hat presents so t roublesome a quest.con . •
m regard to the proper llat1o
I
h . 11
5. C.;~rial is extruded magnesium for whic; ~1
~ the I
~';t/(:):bed
in Problem 3,
ig/cml. An,.~.. •
of a factor of safety as ,that of columns. As already indicated thi c 01ce 95.S kg/c:;1 column with built-in ends is 1.8 in long d has
8
f rom t he fact t hat .m herent maccurac
. . m
1es . centering
. t he ' ins sterns 6.,A UsingthebrokenlineDEBCinFig. 10.10an~ f ~standard ISWB150
. assumed end
load and t . eect1on, --AP •• A.,. p = 202001.-
f \oau a ac r of safetyn ==3 find
mg d. . 11 .. . l k d
con 1tions, as we as m1tia croo e ness of the ca lain·
,'l ,..,, _, a.g, ' '
1 the sa~lve the preceding problem if the column ia built-in t d .
have a pronounced effect upon its behavior under compression. 0~~n, 7, 'Ana, p • = 19,200 kg. a one en &nd pinned at
thousand s of columns have been tested over the years in an attem t Y the other. . 'la h .
B, Construct a curve s1m1 r tot e h~e DEBC in Fig. 10.l0 kl be U!ed 111! a baaia
resolve t his question . The results of such tests when plotted as sho! ~ of design for columns of extruded al~mmum alloy having E = , (lO)• kg/cm' ,ap =
Fig. 10.1 1 form a wide band and leave the question of proper facto:~~ i4()()kg/cm',and O'mu =- 2,1_00 kg/cm• Assume that l/ r,.. 30 is the upper ~rt- 1
~Jumn limit and l/r = 70 11 the lower long-eolumn limit.

lo.4 The Secant Formula


(P/A)kg/cm 2 \A
\ Referring to Fig 10.12, let us consider the case of a perfectly straight.
\8
cantilever column subjectro to a compressive load P applied with ec-

p I p

1,400:1--- -+---+- ---+---- +----+-

70011---+ --+----+ -'""""..+ ---+-


·-- I
81
I
I
l.-1
I
I
I
I I
l I
c~ I I
0 ~ -5~0:---l~0-0- ~15~0- -2~
0 0- -2~5-0 - r I
I
--11
I I
Fm. 10.11 I I

L .iL_,
I
I
I I
safety very much unsettled. One point, however, is clearly brought out by I
these test results; namely, the broken line DBC, construct ed as already I
I
explained , represent s an upper bound for the test results. This substan· I
I
tiates the theory and leaves the pronounced but indeterm inate influence of I, I

· f ections
· · \y to be adequately provided
imper s1mp for by the use of ageneroll~ (a) (bl
factor of safety.
Fto. 10.12
280 THEORY OF COLUMNS SECANT JORWlJLA

centricit,y e. This case differs from that discussed in Art. 1O. l only in th e ,_ O, we Bee that a c o for 281
fact that we now assume the colu~n to be slender so thnt the la~t e 'f•k~gp < Pc, (see eq . 10.5, p. 270) a°£ value of kl< r /2 ie
deflections cannot always be considered as small compared with ll \l~uepo the . deflection becomes inde~nn· ut wh~ kl === -.:/ 2, 'i_; _·, a..nhey
. inate • it ' w n
eccent ricity e of the applied load . As the load P is gradually incre~he p .,. "'These observations agree wit}1 those
1
' can have any BD1&ll
the ('()\umn will start to bt>nd slightly, and for any value of P t he deflecti d ~ut- case of a centrally loaded column Ha. ready reached in An 10 2
) · · owever f any Sllldll value •
curve will be as shown in Fig. 10.12b. on /ot the O eq . (10.9 gives a definite value f or I

O
Choosing coordinate axes x and y as shown and denoting by o the deft of e : that O ii1creases without limit as p or sn long as P < p"' and
tion of the upper end , the bending moment. at any cross-section definec. sho'if•t·on relationship expressed by eq. (lO 9a)~pro~hes P.r Tue load-
~x~ll~ ~ d ftec 1
e a chosen value of the eccentricity e a d .
· JSseentoL .
•~ nonlinear.
for d h. n a gJ \·en col
M = P(o. + e - y).
% represent.e grap 1cally as shown in p· umn, eq. (10.9)
(a) can be • ig. 10 13 Fo
.-i,deflection curve 1s represented by the two tra· .h . r e = 0, the
Then for small deflections within the elastic limit of the material, the loau s ig t lines OAB. Xo
differential equation of the elastic line becomes
p
EI 'Z = P(o +e- y) .
(b) P.:r. A_ ___ L~~ --- 8
c
Dividing both sides of this equation by EI and using the notation
p
k2 = E/ (c)

it may be written in the form

~ + k1y = k2 (o + e). (d)


-;o;;t-- - - - - - - - - - a
FIG. 10.13
This equation is similar to eq. (d) on p. 269 and its solution is
y = C, sin kx + Ci cos kx + o + e. (e) deflection takes place for P < P and at P = Per the defittt..'On ig in-
0

detenninate. For e > 0, but small, the load-deflection !'l'!:mon will be


Noting that when x = 0: y = 0, dy / d:c = 0, we find represented by the curve OCB . For small loads. the de.flecuon ma-eases
810
C1 = o, C2 = - (o + e) wly but as the load approaches the critics.I value, it io~re~~ r.\p.idly
and tends toward infinity .
and the solution (e) becomes For small values of kl « -,: / 2, i.e., for values of P ,<... P" 1,1. t> m:1y take
·Y = (o + e)( 1- cos kx). (0
(kl )!
cos kl = 1 - - '
This shows that the deflection curve AB has the form of a quarter cosine 2
wave . in eq. (10.9) . Then
A~ the upper end B we have y = o when x = l. Substituting these
simultaneous values into eq. (f) , we obtain

0= e (1 -coscoskl kl)· (10,9)


Ot •
'since k =: ✓P/EI ,
For a given value of e, this equation expresses the relation between th e
lg)
lateral deflection o o.nd the load P, indirectly defined by k = {Plii-
282 THEORY OF COLUMNS 81:CUfr 1 ' 0 ~

p This is seen to agree wit.h the deflection of a cantilever be , reP}aeed by ✓P/El, we have
loaded by a couple M = Pe at the free end (see Ca ll.tn ~kil p [
·j Table 8.1, p. 212) . This just~fies the _treatment of e:e~ ~f
•-=1
1
+~sec(~fu)]·
n:
4t I :
cally loaded short struts as discussed m Art. 10.1.

it
In the present case of an eccentrical
is important to note thu.t t h ere
.
1s
ly loaded long c
o1urn
no proportionalit n,
t11,
. n (10.10) represents the so-can d
f;qlled11,tJO}ender column. It gives the rn:
a
_ecantformu Jafora
O0.lOb)

I I tween the load p and the deflection othat it produces- : be. 1oad ,.~,er-age com-presttive stress p I A th ximurn compressive ~:cce?trically
I I {the ..., kl 2 If ' e eCCCntricit esa in t~rms
C I I the. method of superposition cannot be used. An axiall ence o d Euler angle . I . the colunm is sh Y ratio ec/rl and th
aH- Plied load p when acting alone
produces no lateral defley _ap, Jle sec kl/2 = ,I, and eq. (10.10) reduces ~rtt, hand the load p' ia smalle, so-
1 I
I I "th a be,n d"mg couple pction., b~re o e sa.rne f we
but when acting in company wi
{Mt, 10.1. . . orm aa eq. (10.2a)
,,
I I

./,,z
I I
I I Produces also some bending andl thel
resulting deftectio e, it 0
f e set some lumt on a-m&.c , say " rn~ == u
. one
ns ar
w_ould produce. 1' _e
I w ·
greater than the bcndmg coup e a ' :r .p. in the case of structural
lJ I I
I I
magnification is especially pronounced if th<> axial f orcehi~is
we
steCl, •
may then
d
calculate the
· ld' .
correspond
. b will first pro uce y1e mg I{l the most
tng average , .
1:ompressive stress

.~ p
near the Euler value .
All of the foregoing ~iscussdi~n can be applied
fundamental case of a pm-en e co1umn ecce_ntrically load
also to
·
the
b
YI JC
this purpose, eq.
( 10 10) .
.
Py.p.
A- =
. .
,s written m the fonn
severely stressed fi hers. For

---...:.(1.!..; Y-~P-~ - -

F10. 10.14 as shown in Fig. 10.14. From symmetry conditions t~


tangent. to the elastic line at C remains vertical and' e he
. . h l ~
1 + ~ sec (~ ffl) (10.11)

half of the column is in the same con d 1tion as t e co umn in Fig. lo.12 anJ • (
Fo r anv chosen eccentrici.ty ratio ' we may ·=
Hence , upon replacing l by l/ 2 in eq. (10.0), we ohtain, for t.he deflec~ · plot a curve which shows, as a. function \:e eq. 10 11) b ·
f l I -h Y tnal and
tion o of the mid-point C of the elastic line , error and 0 , r, t e val ue of P/ A
· h · 1..J · fi t be · ·
gm8 m the most stressed fibe A f . ,
at v.:hic yie umg rs

! =, c- c::
cos f
kl
2
) =e [sec~ - 1} (h)
curves for 8evera

.
. l
· l value!S of the eccentricity rat' rs.
. .
2,BOO kg/cm' JS s 10wn m Fig. 10.15.
We usually c<;msider. the Yalue of P/ A for which y ie
. .
as the hm1t load for a steel column Ha
;-1 :\ damily o, surh
IO ec r an fo r c

· Id - ·
· ·
Y-1> ,

tng in an extreme
d h'
=

hoer begrns
. · · vmg Joun t 1s \'alue
where Zis now the full length of the pin-ended column. for a given column from the graph in Fig. 10.15, or from eq. ( lO.ll ), thP.
The maximum btnding moment in Fig. 10.14 occurs at the mid-section C
and has the magnitude
Mmu = P(o + e). (i) (P/A /lcgfcml

Substituting expression (h) for o, this becomes


kl
Mmu = Pe sec 2 - G)

The corresponding maximum r.omprcssive stress on the concave side of the


column at C is
<Tmu =~ + ]}I muC. (k)
A I
Substituting expression (j) for M max and using the notation r = ~ for
radius of gyration of the cross-section, this becomes
c10.1oal -:d----:-1 :--~~---: ~--;-;;;;- ~- -I
--;zoo
0 40 80 120 160

F10. 10.16
SECANT FOR kuL A 285
PRO BLE MSE T 10.4
T11SORT OF coL UllN 8
nd .. .
tion stee l colu mn wit h pinn ed-e d1t ion s 18 7.5 m long . It
284 . stre ss will be obt ain ed by div
idin g the ob•-, 50 sec tral ly app lied com pres sive load s /on 8
"Ill ~ -""1sWB 5 ds cen 2-2 12 in f
1
"" ? tonnes and brac
kete d
compressive of safe ty n.
f tor l•..... ,.tit s el n_ ..1 8 p 2 = 8 tonnl es I on axis
h ' . rom a,as l I (se F '
e . 1g.
10 .16) .
safe average f -- 'ety can be inc orp ora ted dire ctly .into .ttl.,., . e o~ nt form ula, ca cu ate t e max imu
m extrem fibe -
alue by a suit able ac d _eq. r,-·. ~•., .. e r stre ss m co mpres-
o0'!1P tbe secs = 705 kg/c

v I1 fact p d not e the safe wor km g loa , and n the r
or o :sa.J mt.
If desi red, t. e 11'eli at factor of f
( 10.11) as follo
ws . Let hw com e pres sivc load whi ch . wou
.
ld brin g the equ
extre tJ,itlg ,4,ia. ~cld stre ss for stee l 2,800 kg/cm2,.wh
e aga inst fail ure due to yiel d~ e~y n does t he colu mn
t . The n t e bP- nP .,.. Sub st1 tutm g this for p . t'tle ,iO;· 1{ the Y~·ng pro ble m= hav mg m the mos t stre ssed
f )· . Id oint. will in eq_
fact.or of me . tiie pre :r; Ans . '!- 3.9 7,
.
fiber stress to t~e y1e P colu mn desc ribe d in the t
~in e 6 1• sec tion for the ::~ ~~ e ~~~le~
(l0.1 1) , we obta in
sele;:
5. f:;t or of saf ety aga inst yiel ding
m=. A m
in the mos
. ISW
.,tbs.t t 5 _Assume <Ty. p. =2, 800 kg/c l is sub ject ed to axial com
B
t
450.
str :er
- h
will
00.12) th sion as •j own
t,e,. ""' 2: ~nd ed col um n AB of leng . d while t hat at pres
~ A pin a.cl p a t A ts· t II Y app1.ie
ra ' B is applied with
.,. . A. The lo cen
ll valu es of P,p the pma, amu m ben ding moment•m th e co1um n
ill fig .. ·tY e· For sma · pres sive
tr1c1 . 1 M e. md_ the valu e of the ave rage com inte r-
eecens at Ban d is sui:ip y max "".'
occ ur at som e
.10) sho ws tha t the intr odu ct·ion of ent
ImuT? b~n dmg mom will
.son of thi·s equ atio n wit. h eq. (10 occur p/A abo ve whi ch the maX sequ ence , be grea ter than Pe . HinL:
Compari tak es pro per accou nt of ~a te poin t bet wee n A and Ba
nd will, m _con
e of pin the den om ina tor .of eq. (10..10 ) the elas tic line in this CS......'-e 4.
.
n P., in p 1ac e lme arly wit h P. 111ed wer to Pro ble m 10.2 -8 for the equ at10 n of • - na.

the fact tha t umox doe s not incr eas it ratb See ans / )2
of the sec ant for mu la mak es Pf A = 1f2E/ 4(l r .
The tran scen den tal cha ract er
pro ~ed ure s usu ally bei ng nec essa ry. sU:~
difficul t to solve, t ria l and erro r
the foll owm g exa mp le.
procedure is illus trat ed in
n stee l beam (Fig . 10.1 6a) is to
be
EXA MPL E 1. An 1-se ctio ed ends Th
hav ip.g pinn
selected for a column475cmlong and load s P 1 = 32 tonn~ : ---------..i
i . - - - - - - ~ - - - - - -.,1

,8,
lied
column is to carr y cen tral ly app er app
111
lied on the principal

(orrr
s P, =16 tonn es the latt FIG . A
bracketed load e of 25 cm from axis 1_1
axis 2-2 of the sect ion at a dist anc use d for a pin- end ed column 5 m
long whi ch is to
ion suc h tha t the coluOU: 5. An ISW B 300 stee l sec tion is
Fig. 10.16b. Select a suit able sect tral ly app lied a t the lower
= aga inst failure by yielding 50 tonnes The load P is cen
carry com pres sive loa ds P :-=
will have a factor of safe ty n 2.5 city e = l.5 cm
at the upp er end with an eccentri
P. P. (uy .p. = 2,800 kg/cm• end of the colu mn and bra cke ted fibe r stre ss indu ced in
5
from axis 2-2 on axis 1-1 . Cal
cul ate the max imu m extr eme
in Fig. 10.16b the column. Ans . O-m n = 2,103
kg/ cm•.
I
s~LU TION . The two load s app lied as sho wn applied with
I
le load P =- 48 tonnes
I
can be replaced by a l!ing in Fig . 10.16c. Then to uae
I
eccentricity e = 8.33 cm as sho wn ctin g the required aection, ent ric ity
fec tio ns by Eq ui val ent Ecc
(b)~ the secant formula as a basi s of sele IO.S Re pre sen tat ion of Int per
mn und er
we writ e it in the form out tha t t he beh avi o r of a colu
In Art . 10.3 it has bee n poi nte d
<Ty/ = ~ + ~e sec(~ ✓,:;}
ctio ns suc h as slig ht
ch affe cted by imp erfP
(c)~JJ I the mom ent of inertia of
(I)
compressive loa d is ver y mu
~rookedness of the col um n axi
s, e rro rs in cen teri ng the load
ent of the se imp erfe ctio ns var
, unc
ies
erta
suff
inty as
icie ntly
FIO. 10.16 where Z is the section mod ulus and
th e_secti?n. By tria l and erro r, we now look for an I-
eectioo r° eud con diti ons , etc . Th e ext
r to pro du ce t he rath er wid
e sca tter of test
equ al to or Jess than :ml one col um n to ano th~ · h ·
app rec iate t e m-
the left side i wt: h will mak e the righ t side of
eq . (I)
0 m 1.UIUII su ts wh en rcp re~ nte d a s s how n in Fig . 10.11 . T o.
= 2,800 + 2 .5 = 1,12 kg/c ft Uence f 1. . be obs en.:ed
s tren gt h of a colu mn , . 1t _m _u st
less than -;- .p/
11 n
Table B.2 ~f 1·p,peeqnuda.ix Bor
, we find .
that theo mp er~ ect1 ons on the mg exa ctly Wlth
For ISW,B 450 sect'ion .. R.1ght side . = 744 kg/cm2 act ion of the load not com c1d d.
the . ,y res ult m the line of ·on ·1n
F
or IS\.\ B 400 section. R ht 81'd = 934 l'l~/cm• .. axis of th e col um n, thu s intr odu cing m ore or less ben mg acti
I d
For ISWB 350 . . ~ e . adct·1 1. · l ~o~c u e
TL,_ .md1. cates that lSW section·· Rig ht. Bid e - 1,168 kg/cm•. on t O d .ir~c t com pre ssio n. On this bas is it see ms log1~a to
t the
400 ll!eCtion wsuitable ao fa r as ben din i in the prine i-1
B 1
that be sun_1l~r to
erfe ct. col um n u~d er l~d will
QIIS

Plane of load ing i.a conce plan e of smaller that of a beh avi or of a rea l imp ent rici ty e.
&a&Ume that buck.ling in the h a suit able ecc
fle-xural rigidity is preven:.::i~· We Per fec tly str aig ht ide al col um
n loa ded wit
'l'HEORY OF COLUMNS EQUIVALENT
286 ECCENT1uc1Ty

This suggests that the secant formula deriv_ed in Art. 10.4 could be
0 C
also as a basis of design for supposedly straight centrally loaded c l ~d 2,800 ,------r-.;..,....__
simply by choosing an appropriate value of the eccentricity ra:10Ufll~.
to account for the effect of imperfections. While such a proced et/tl
leaves unanswered the question of the proper value of ec/r it
• rf • I
u! 8tiU
011el'n I column~
rational means to represent the effect of unpe ections rather th •Q a kg/crna
simply allo:w for them by an overlarge factor of safety as suggested on an to 5
To select a suitable value of ec/r2 to account for imperfections P. 278.
must be had to the results of tests made on actual columns. A!3 ~ l'ecoul'se I
. l
such experiments, a value ec/r2 = 0.25 1s common Y recommended f
result _of 0
I
ended columns as used in ordinary structural work. This does n or Pin. c
~ 700
;;:,, (al ---+---~- -..,,,....,.
n£cessarily that every column is expected to be this imperfect bu~t lllean
th'3.t no column is likely to exceed this degree of imperfection.' 1'ha;a._ther A
the results for the columns tested were plotted on the diagram in Fi 18 , if F
1 0 40 80
the points would all lie a.hove the curve labeled ec/r2 = 0.3. T:· ?•15, 120
200
Values of ltr 240
cmve for ec/r2 = 0.25 and for cry.p. =2,800 kg/cm2 is shown again e_ sin~}e 0 C
1 2,800
10. l 7a.. This curve may be used a.s a basis for the design of structu ~ Fig.
columns. For such a. column of any given slenderness ratio l/r ;: s~el
resi,onding ordinate of the curve shows the value of the average co~p e C?t• "e
~2,100
stre~ P / A for which yielding of the most stressed fibers in compresstSSive .>:
.!:
be Expected to begin. The corresponding safe value would be thtn;;n
divi1led by the desired factor of safety n. Since the curve already taks A
es ac-
----~1,400
?
• •
count of the effect of unperfections, a safety factor n = 2 may be '-
· ed
Blder a.s adequate.
con- 0
.,"'::,
Si.nee lack of straightness of a column can be expected to increase with g 700 -(b)
the length, it is sometimes preferred to allow for imperfections by using an
equivalent eccentricity e which increases linearly wit.h the length l of the
column. On the ba.sis,-of test results on steel columns, a commonly recom- 0 40 80 120 160 200
mended value is e/l = 1/4:00. In such case the secant formula becomes
240
Values of ~r

F10. 10.17
(a)
... The use of such curves as those in Fig. 10.17 ba..--ed on the secant formula
•cPresents the t · .
col mos rational approach to the design of centrally loaded steel
is ~;ns. We have seen that the load which such a column can safely ~rry
II
":here the ratio c/r depends on the shape of the cross-section. For a solid inh uenced by the imperfections, the strength of the material, and the
erent. aspect 0 f .mstab'1hty.
. Each of these factors .ts rations . IIy repre-
circ~lar cro88-l!eCtion c/r = 2, for a. rectangular cross-section c/r = '13, for sented
a th1~-w~~ hollow circular cross-section c/r = ~' and for a.n /-section hy th in the secant formula. The effect of imperfections is accounted ~or
bendmg m its plane of greatest flexural rigidity c/r = 1. Two curves DE, itself \chosen equivalent eccentricity of load ; the strength of t~e ma~~ial
plotted from eq (a) for c/ r = 1 and c/r = 2 ' a.re shown in • 1017b
. Fig. by h' Y the chosen value of er . and the inherent aspect of mstab1hty,
. · · t e y .p I f 1
where the yield point 11 ' d as s blends appearance of P / A in the secant term. Thus the secant ~rmu a
\..•.:. f d . Y •P- = 2,aoo kg/cm 1 • These curves may be use
UDt1111 or es1gn of steel 1 • • d above l\n the cases of short medium and long columns into one contmuo~s
. . . co wnns m the sa.me manner as was explaine
m connection with the curve in Fig. 10.17a. Artge1and obviates
· '
the necessity to 'make these cIas.51'fi cat ion
' s as was done m
· 0.3.
r 288 THEORY OF COLUMNS

The strongest objection to the secant formu la 1s 1tR transc


character which necessitates solution by trial and error. Howeve;~denta\
.
EMPIRICAL COL

·tutes for it. Such forrnulas


UMN Foali11JL>..s

·• sllbs\,e stress (PI A)ID as a fun ct.io Usually give the 11


28
9
"" ress . d' t' h n of th l a owe.bl
coJ11P cifi ca.Ily m ica mg t e factor of safe e s enderness ta . r: avel'age
curYes as tho-~ in Fig. 10.17 are at hand , the selection of a safe 1 ' dif Ruch
. l . oa f ntlt sJ>E: ht Line Formula. One of th ty that is u.sed t 10 l/r, ?. ith-
given co umn or of a suit.able column to carry a given load becorn Or a
. 1
sim p e matter . e~ av
1-ry s1ronWforrn ulas is t,he straighl. line !or;uJrnaost cornrnonly. llon..l ..

ELUt:PLE l. The column shown in Fig. 10.7, p. 273, has length l == 2


colufll = empinca]
4
square box section IO x IO cm outside dimensions and wall thickness t == ~ rn and a
mate.rial is steel with O' y . p . = 2,800 kg/cm•. Using the curves in Fig. 10 17~ lll . 1'he
(f_)
A ,. -
- a,.+ a_,l
T
factor of sa.fety n = i_, find the safe value of the compressive load P. · and a 00. 13)
SOLUTION . The geometric properties of the cross-section are A ==
J ,, --; . 22 ,56 C 2 is an allowable working stress f h
I = 333.59 cm', r = vi I 11 = 3.85 cm, c = 5 cm. The effective length of th rn , here~~ or t e mat · 1·
,~ urnerical fa ctor. For pin-ended ena 1n comproc,,,;
is l1 = 0.7 X 240 = 168 cm. Thus l/ r = 43.6 and c/ r = 1.3. Interpolatin \ column is a n . . . , centrally IOad •--=on and
the_ curves DE in ~ - 10.17b for c/ r = 1.3, we fin~ (P/ A)y, 1, = 2,345 kgtn~tween a ns a Chicago Bu1ldmg Code recornrne d8 ed structural steel
coJum ' (l 0.13) becomes 1
n "'~ == ,120 k~/cmJ and _ .
which Py P · = 2,345 x '.22. 56 = 53,000 kg . Then with a factor of safety n '=:. ' from that eq · a - )
have P.., = 26.500 kg. 2, We 50

PROBLEMSET 10.5 1,120 - 5 i.


T (10.13a)
1. Using the design curves in Fig. 10.17b and a factor of safety n == 2 fi d
safe load P.,, for a pin~nded steel column 1.2m long and having a solid circ~la n the The use of this formula is limited to (30 < l/ r < 120 F ,
section of dis.met.er d = 5 cm . Ans. P,. = 12,4-00 kg. r cros.,. (P/,A),, ==. 980 kg /cm2 find for l/r > 120, the Euler fonnul: ( or l r_ < 30,
2. ISA 75 7 5 x 6-mm steel angle section is used as a column 3 m long with b . . be used with a factor of safety n = 2.7.
ends. Using the design curves in Fig. 10.17b with a factor of safety n == 1 1~
the allowable magnitude of the compre~sive load. Ans. P"' = 6,600 kg.
1
· ' n
tt The Parabolic Formula. In order to make greater all ,
.
eq. I0.8) should
.
o~ance tor the effeet
3. A 3.6-m length of 8tecl tubing has a circular cross-section of 10 cm outside ct· of imperfections for the more slender columns a =r-i-.11- : __1_ .
• t'- uuu c !Orm ll,wj 1.s of ten
and wall thickn~s t = 1.5 mm. Assuming pinned ends and using the curve in F~· used, such that
10.17a, find the limiting average compressive stress PI A that the column can carJ:
Am . (P/ A) 1 . p . = 1,400 kg/cm•.
4. A structural steel column 12 m long is built up of (1 0.14)
four ISA 5050 x 6-mm angles held together with diag-
onal lacing to form the square cross-section shown in
Fig. A. Assuming that the lacing is adequate to ll1llke where again a.,, is an allowable working stress in compression and p is a
the column perform as one with a solid section, and numerical factor. For centrally loaded pin~nded structural st.eel columns,
that it has pinned ends, find the safe compressive load the American Institute of Steel Construction (19-!i) recommends 11., =
P,. on the basis of the curves in Fig. 10.17b with a fact 1•190 kg/cm•and ~ = 0.034 so that
or of safety n = 2.5. Ans. P,. = 7,950 kg.
5. Using the curve EF in Fig. 10.17a and a factor of
safety n = 2, find the safe compressive load P" for a (~).,, = 1,190 - - 0.034 UY. (1 0.14a)
FIG. A pin~nded steel column if it has a standard ISLB 200
section and is 3.8 m long. Ans. P.,, = 7,450 kg. ,
06. What is the maximum length l that the pin~nded column in the precedmg Tbe use of this formula is limited to (0 < l 1r < 120}. For l/r > 120, tbe
problem can have to carry a compressive load P = 10 500 kg with a factor of safety Euler forrnula (eq. 10.8) should be used with a suitable facto r of S.'lfety.
of 2? Am. l = 3.2 m. ' .Rankine-Gordon Formula. Still another empirical formula pro!)08ed for
Ptn-eutled, centrally loaded columns is the Ranhne-Gu,rlo 11 formula
10.6 Empirical Column Formulas
(10.15)
Because of the objections to the transcendental nature of the secs~
formula, many simpler hut wholly empirical formulas have been propos
EMPlRICAL COLlJJfN
10lllcul..\g
290 THEORY OF COLUMNS
_,-1 columns of r.cctangular CY •
wher,' cr c is ~n allowable workinae stress in compression and 'Y a num"r'
~ 1~ for «ouv
rY hBB
d . I ~•on .1. .
propose a ~c1a formula ~ , '-'le Forest Proo
factor. A r-om mon form of this formula for structural Rteel colum . i,ortto 1,,....t columm, havmg an unaupnn ~d 0 n lhe results f Uct.a
na Is I> for a,..,, l d' ""rted len!rih o many
...ts, rnaUer la.tern 1mension the aJI "• not greater.~- -
(!'A.).. 1
'i"'' C)'
+ 18,000 r
Oo lSaJ tv the s
tiJ1leS ball be
' · owable ave "'1':lil 10
rage cornp~ ve
sue--"5 s
t-0 be used only for 0 < l/ r <
120. (~). = S,
For purposes of comparison , each of the foregoing empirical formulas f
steel columns is represented graphically in Fig. 10.18, together with t~:
Euler cun ·e (n = 2) and the secant formula (u~ ·P· =2,520 kg~cm 2,ec/ r2 ::::: o.
1 es of S for
.
differen t varieties
,bere v~ uA p. 343. For medium columns
pend1x ,
of timbe .
<10.I6a.)

rare gIVen in Table A..2 of


,
2
a.nd n = 2). Taking the secant curve ~s a basis of comparison, we see tha~
the straight line formula, eq. (10.13a.), 1s somewhat more conservative whil
~p (~)
A •
= S[ 1 _ ! ( l )
3 Kb
4
1
1
O0.I6b)
both the paraholic rurve and the Rankine-Gordon curve are le~ con.e
servativc. J( == ½~, E_ being the modulus of elasticity in bendin _ ..
P•tA kg /cm'
,bere table Use of thIS formula is limited to the ra (IO g ~ given
·n the . h (P/A) nge < l, b < K) .
1,400 When l/b == K, V:e see t at . • = 28/ 3. For long columns (lib > K)
Euler's ronnula 1s to be used with a factor of safety 11 = 3_ Thus '

(Ap) • = 36(l/b)'
rE
(10.16c)

Values of S, E, and K for common structural grades of timber, kept dry,


are to be found in the table mentioned above.
The empirical formulas discussed above are all intended for
centrally loaded columns, the reduction in allowable a.er-age eom- P

pressive stress with increasing l/ r being intended only to al.low for


FIG. 10.18 imperfections. In the case of columns carrying bracketed loads or,
in general, intentional bending as well as compres.5ion, the Amen--
Many empirical formulas for columns of materials other than structural can Institute of Steel Construction has proposed an empiri~
steel are available for design purposes. For example, a formula of the procedure which avoids the difficulties connected with the trial
straight line type intended for design of pin-ended aluminum alloy columns and error solution of the secant formula. To explain chi~. v.--e rcier
in the aircraft industry is to the pin-ended column in Fig. 10.19, where Pis the ~'OSI load

(~J = ~( 3,066 - 24.s1,} (10.13b)


and Mis an end moment. Neglecting the effect of the lateral
flection ~ on the bending moment at C, the mi\."<irnum compre....;;sJVe
?e-
stre~ would be Fto. 10.19
Use of this formula is limited to the range O < l/r < 83. For l/ r > 8~, the
Euler formula is to be used. p .\1
For pin-ended cast-iron columns, the New York Building Code (191 7) am.,,.= A +z'
specifies an allowable average compressive stress whe z· . he principal a."'is about
.re 1s the section modulu!- w1 th n'spe<"t to t . . b,

(f)
Wh1 hL .d f this equauon , a.,... ,
= 630 - 2.8 i, (10.13c) c •>ending takes place. Dividing both 51 es O •
A w r We obtain

f or O_< . . . • due to
l/.r < 70. 0 wmg to the wrakncss of cast-iron in tr~s1on , . (a)
bendmg, 1t should not be used as a material for columns hn.vuig l/r > 70 P/.4 + ~ == 1.
O'max <Tmu
EMPIRICAL COLUMN FO-•
•••UI.Ag
292 THEORY OF COLUMNS
king stress, the safe 10 ad p _
r 293
- 58 6
·tb tll is wo 20 x 25-cm sec t·ion, eq. (1 0.16b) . ·
l<
625 .
Now if only t he ce ntrally applied loa.ds P were ncting, i.e., if !I,~ === o, \\·e
· _11v• take ~ mU = <1 th<> allowable average compressive st rP.se
cou ld log:1c1u V C• • • •
:~- for a (.p) [ gives
===
36 700
, which is too
given by one of the empirical column formulas, say the parabolic formula A .. 61.6 1 _ ~(~ )']
eq . (10.14a). On the ot her hand , if only the end moments M_were acting' 27 3
· X 20 "" 5-i.4 kg/cm•
i.e., if p = 0. we would take <T m~x = <Tb , the ~llowable _work1_n g st~esl! ro; fe load is 54.4 x 500 = 27,200 kg. Th· . . '
t he material in pure bending. For the _combmed loading, we arbitrarily aod. th\:dequate . 18 tndicates that the 20 x 2·
~
replace <T rnu in the fi rst. te rm on the left side of eq . (a) by CT,, and CT,nnx in the sec~ l
two terms ·sh"ll
secon d t crm b.\. ub , aild then requi re that t he sum of the " not PROBLEMSET 10. 6
exceed unity. Thus
, . g the A.l .S.C. formula , eq. (10.14a) select
I, usin c. I ·f h • an I-beam t·
P IA + Al I Z < l. d-end column "m ong I t e axial load p = 60
as a fiU:<~ ,, the A.l.S.C. column formula eq (1 0 ) fi d,OOO
' sec ion to be used
,g. Ans.ISWB 300
(1 r CTh 00.17 ) 2 sJOi, dh . . ' . . 14a ' n the safe . 11 .
• 1 ipe 4.8m long an avmg pinned ends if the outside ct· ~X!a oaa P for
as~ 51Pde diameter is 30 cm. Ans. P == 98,250 kg. iam~ter15 31.Bcm . and
The use of t he lower working stress CTc in the first term is presumed to corre t the in . d" ll · Jin f u1a
Solve the
3· p _ 78 iOO kg.prece mg pro) em, usmg the straight
for t he additional bending moment Po. In calculatin~ the value of u, fro~, ,e onn , eq. (10.13aJ.
!11,!. - ' . .
t he parabolic or other empirical formula , the least radrns of gyration of the · , An ISWB 4-00 beam sect10n 1s to be used fo r a cantilever col b .
cross-section should be used regardless of the plane of the end mom<>nt.<; M.
"' f t
lower end; ree a . e P,
th to a_nd 2 m Iong. usrng · the Rankine-Gordon
um.n U1 1t-m at the
formula find
the safe load P axially applied. Ans. P = 69,500 kg. •
EXAMPLE 1. Repeat the solution of Exampl: l of A~t. 10.4'. u~~ng condition s. What is the safe load P for a 25 x 30-cm shortleaf pine column _ m long if it
75
(10.17) inster.d of the secant formula . The workmg stress m bcndmg 1s CTb = 1,400 has pinned ends? Ans. P = 25,100 kg.
kg/cm 1 • . 6, What is the s~fe axial load P for a 20 x 25-cm spruce column if it is 3.6 m
SOLUTION . The total compressive load P = 48 tonnes and the end moments long? Ans. P = 2.,100 kg.
M = 400 t-cm. For a suitable section, we try first the ISWB 400 section alreadv 7, ISWB 500 beam section serves as a pin-ended column 2.-4 m long. h C3rries
considered on p. 284 . For this section ,A =85.01 cm 2,.Z =1,lil.3cm•,and rmin = 4.0 4 an axial load P1 = 40 tonnes and an eccentric load P1 acting on &xis l-1, JO em from
cm. Then 1/ r = 475 -=- 4.04 = 117.5 and the parabolic formula, cq. (10.14a) , gives
axis 2-2. Using condition (J 0.17) , find t.he safe magnitude oi the eccentric load P1 .
Calculate <1c from eq. (10.14a) and use 11b = 1,400 kg/cm 1• Aru. P~ = 18,840 kg.
11c = l,190-0.034 (117.5) 2 = 720 kg/cm 2 • 8. Select a 30-cm I-steel section to serve &.l:i a. pin-ended column 3 m long if it is
to carry a load P == 14,00J kg with eccentricity e =7.5 cm,bending in the plane oi the
Accordingly the left side of expression (10.17 )becomes web. Use condition (10.17) with"• determined from eq. (10.1 4&) and 11, = 1,400
kg/cm1• Ana. ISWB 300.
565 342 = 1.029.
nu+ I,4UU

This being so nearly equal to unity, we many consider ISWB 400 section satis-
~~ - I
EXAMPLE 2. l;sing the Forest Products Laboratory formulas, select a Doug a~
Fir timber of r<:ctangular cross-section to be used as a pin-endPd column of lengt
l = 4.2 m if the axirtl load P = 27, 500 kg. . fi .
SO LUTION. From Table .-\.2 of Apprndix A p. 343 we find for Douglas _ rd
S = 6 1. 6 kg/cm and K = 27 .3. If the c:ol~mn wcr'e very short, the rPquir~rl
2

r
co~- sect"10naI a.rra wou id l)C A = P/ S ==27,500:GI.6 =.., 416 sq cm. 1'h.is then woutr '
require a 21 x 21-cm cross-section, which suggests that for a long column we l
first a 25 x 25-cm section. Then from eq. (10.16b)

( ~)
A "'
'=' GJ.6[ I - !(- 420 )~] = 58.6 kg /cm
3 27 .3 x 25
2

r 11
THE TENSILE TEBT

Pr - - - - - - - -

Me cha nic al Properties of Materials

11.l The Tensi le Test .23•1.c

The preceding chapters dealt with the methods of analyzing .11•1.c


the stress 0

distribution produced by various kinds of forces on structures.


Knowing
the stresses, the designer must then select the material and the
dimensions
of the structu re in such a way that it will safely withstand variou o- - - - - - . l _
s loading
conditions in service. For this purpose it is necessary to have
information FIG. 11.2
regarding the elastic properties and strength characteristics FIG, 11.l Flo. 11.3
of structural
materials under various stress conditions. The designer must know
the limits The length of the cylindrical portion of the specimen is always somewh
under which the material can be considered as perfectly elastic at
for various reater than the gage length land-is usually at least l + d. The ends
stress conditions, and also the hehavior of the material beyond of the
those limits. ~pecimen a.re generally made with a larger cros.<Hection in order to prevent
Information of this type can be obtained only by experimental investig
ations. the specimen from breaki ng in the grips of the testing machine , where
Materials-testing laboratories are equippoo. with testing machin stres5
es* which conditions arc more severe because of local irregularities in stres.5 distribu
produce certain typical deformations of test specimens, such -
as tension , tion. A cylindrical specimen with l = 10d is shown in Fig. 11.l , which
rompression , torsion, and bending. also
Experiments show that test results are sometimes affected by shows the spherical seats in the grips of the machine, used to insure central
the size application of the load.
and shape of the test specimen. Thus to make the results . .h
of tests com- Ten11ile test ma.chines a.re usually prov1·ded w1'th a deVl ce wh1c auto-
parable, certain proportions for test specimens have been establis
hed and matically draws a. tensile test diagram representing the relation between
are recognized as standa rd. The mo::;t. widely used of all mecha tbe
nical tests .of load P and the extension ~ of the specimen. Such a diagram exhibits 1
structu ral materials is undoubtedly the tension test. The standa •~-
rd ten silc Portant characteristics of the maten.a l· F.ig. 11 ··? • for e,-wnple , shows a
test specim en in the United States is cirrular, wit.h 1.25-cm . t ts of
diameter a nd .
~nes of tensile test diagrams for car bon steel with vanous con en
5-cm gage length , so t hat . ' _ the ultimate
carbon. It can be seen that as the carbon content inct~ase:.t,he elongati
t on
s rength of the steel also increases, but. at as the same 1me
fi = 4 or l = 4 .51✓A, before fracture decreases and the material h .
\e;s ductility . High-ca rbon
high value of stress
steel is relatively
brittle . It follows Hooke's law~p ~:a other hand, a mild
where A = rr(fl / 4 is the eross-se<'l ional area of the specimen. and then fractures at a very small elongati~n. n~ ,t:tchr s considerably
steel with a small carbon content is ductile a -
before fracture . ·1d struct ural st eel.
•For a desc ription of ma ttirial s- tA•sting machin es and a bibliogr , t 5t•C
aphy on the RtilJJe\ sis F' · d' m for nu . .
l• ig, 11 .3 re pre sen ts the tensile
the article by J . \farin in \L Hetcnyi ed ., Handbook
of Experimental Stress Ana Y· ' fro . .
te:sl ,agra. . , · ,, such as y-,eld poiri1'
New York, 1950. .m this diagram the important c a h ractcr1~ uc:. . ed
IIIhni t . can be obtam .
.
ate strength, and amount of plaSlic . lo11ga w 11
294 e ·
THE TENSILE ---:,
296 MECHANICAL PROPERTIES OF MATERIALS
TEST
under the tensile test diagr
re~ d am On,.,L 297
In determining the prapor tiona l limit , sensitive extensometer ftie a, . ed to pro uce fracture. Th · ~ lFig. 113
necesss.ry in order to detect t he slightest deviation from a straight
the tensile t.est diagram . Obviously the posit.ion found for this ~.e 1_n
t1
n~e
quir
i~o . tic pr ..
f h
r~ re operty o t e material and d
f h .
is qua.nf
ePends n
. ) ren
1ty ia also "resents th
U.5ed e
,teris h duct1ll ty o t e material. ot only on th as a ch4r.
depends considerably on the sensitivity of the instruments. In ord irnit 9v t e l· e str
sO oil t 'litY of a meta 1s usually con •ct ength but
0 bt.ain greater uniformity in results,
9l dti,C i s1 er1;d to be .
a specified amount of permanen:t to 'fbe . 0 of the gage length of t he specimen d . characu,nzetl
or a certain deviation from proportio nality is of ten taken as the bas· sfet 10
Jo11gfl~ •n Rrea of t he cross-section wh uring a l€nsile, .. , by the
d . . IS Ot e tJOO 1 . ere fra.ct tesl and b lh<
/ teter~ mmg thBe proportional limit. The Internat i~nal C~n~ress for Testinll' ,educ I stic elongation , from a to c in p· Ure occurr. 1 Y ~
• 1a ria1s at rnssels (190G) defined the proportional hm1t as the t . "' e of p a . l th Th . . tg. I 1.3 th ~- n the fi1ct
_ . . ens1le s~ I along its eng . is uniform 1 ' e specimen 1
:;lress at which the permanent set 1s 0.001 per cent. ·forrn Y · e ongat1on e ongates
0 p1 lateral contract10n, so that the vol . is a.ccompanted b
The yield point is a very important characteristic for structural steel
onJo:!ll))y constan t. * At point c the tens~lmef ot the specimen remay a
the yield point stress, the specimen elongates a considerable amount ,· .tAht ct1ca. •
t · prli d further extension of the specime .e orce rea.c hes a maxi ins
ou any merease in load. In the case of mild steel this elongation maVI b· Jue an . · n 1s accomp 1 d mum
more than 2 per cent. Somctimrs yielding is a.ceompanied by an aby e 1'a ' load. At this stage of plastic elongati th ~nie by a decrease
· the · b · on e detor
decrease in load , and the tensile test diagram has the Rhape shown in r;_Pt 1n . d and necking egms, the specimen t k" !TIJU1on becon1p~
1~&1izel It is difficult to determine accurately ~h ing the shape shown ~
1 ~ -~ - In such a case the upper and lower limits of the load at a and ~·
1
Fig, 1 ·4· d h b · e moment wh
d!Vlded by the initial cross-sectional area, are called the upper and I ' . g begins an t ere Y establish se parately th en
0 ecki n . d e lllagn1·1.1 , 0 f
yield points, respectively . The posit ion of the upper yield point is affe;~e: iform stretching an the magni tude of th _L ae
the un . f . e e1ongat1on due
by the s~d of testing, by the form of the specimen and by the shape of th tone e
king. It 1s there ore customary to measure the t tal .
h . o rnrro:i se
cross sect10~. The lower yield point as already discussed in Art. 2.2 i: . the gage length after t e specimen has fractured . Th """
1n d h . f . e e1onga-
usually ~onsidered a true characterist ic of the material and therefore is used
a.<s a basis for determining working stres~es.
. _()wing to the relatively large stretching of the material at the yield point
. is then. define as t e ratio o this total elongation f ·he
tion . . . . o L gage
length to its mit1al length. In practice the elongation at fmture
is usually given in ?ercentage. If l_is the original gage length and
r,
J 0

it is not necessary to use sensitive extensometers to determine this point. It 0 the total elongation, the elongation at failure in percentage i.s
ca.n be determined with the simplest instruments or can be taken directly
from the tensile test diagram. For structural carbon steel the stress at the
=r
0 FIG. 11.4
yield point is about 55-60 per cent of the ultimate strength. Structural E 1()(). (a.)
st-eel with abou t 1 per cent silicon has a yield point stress about 70--80 per
cent of the ultimate strength. The ultimate strength for the silicon steel is
about the same as for the carbon steel. Such a high value for the yield point This elongation is usually ta.ken as a measure of the ductility oi the material.
justifies the usual practice of taking higher working stresses for silicon steel. Elongation obtained in this manner depends 011 the proportions of Lhe
A sharply defined yield point is a characteristic not only of structural specimen. The increase in the gage length due to necking is a large part of
steel but also of materials such as bronze and brass. There are other the total increase and is practically the same for a short gage length as for~
materials, however, which do not have a pronounced yield point. For long gage length. Hence the elongation defined by eq. (a) becomes larger as
these materials the stress at whir,h the permanent set reaches the value 0.2 t~e gage length decreases. For steel, the elongat ion obt.sined ior specim~n.s
per cent is sometimes arbi trarily called t he yield point. It must be kept in wi th 1 == 5d is a.bout 1.22 times the elongation fo r u specimen of lhe same
mind that the yield point defined in this manner does not represent a material with l = 10d Experiments also show that the sh3pe of t~e cro65-
se f ·
definite physical characteristic of the mate rial but depends upon the e ion affects the local deformation at the neek !llld hence aft el cts, ·h
the
elon""t· arable re3n c~ wi t
arbitrarily chosen permanen t set. e.. ion of the specimen This shows that comp, · ·1
resPe t · • 1etrically s1m1 ar
The ultimate strength is usually defined as the stress obtained by dividing . c to elongation can be obtained only by tLSl!lg geon
8
t he maximum load on the specimen (point c in Fig. 11.3) by the initial cross- Pecunens.
sec:tiona.l area. This quantity also is of ten taken as a basis for determining 'The . does change csn be neglectd in
the working stresses. cornP&ri IIInall elastic deformation in which the volume •
aon With the comparatively large plastic deformation .

d
YlELo PO]Nr
298 laCRAN l CAL PROPmRTl ES or M.ATERIA LS
•th the lateral surface of th 2Q9
T he reductt:on ,,, area at t he cross-sec tion of fracture is expre~ u . n• w1
otJO PJnat,ic
d f .
e ormat1on has or.cu
e 8Jleci
d men of th'1n I
follows :
1t~
~ · h °"' · rre · h ·
·nwh10 I remain perfectly elastic By w tie the adi' aver3of ma, ri1,I .
1
i& h· . · cutti a~nt . <'
Ao - A1
q ::,: ---'---,-- ' ,i1&~rl tching, the t m plaat1c layers in th _ng the BJleci Portions of th
Ao (h)
al'°o1B v~sible, Under a microscope it is e interior of th~en ~nd '~ng :
J11,de I which have been distorte d by sl'd ~n that these laS>et1rnen can be
t.& s h h h i ing Yers .
in wh il'h .4 0 is t he initial cross-sectional a rea and A, the final cross-sec t··10 na1 crY~ rirncnts s ow t at t e values of the i. con.~ of
area at the se<-t.ion where fracture occurs. ~petrain depend upon the rate of stra· y eld Point stteS,i d
p0111t :train diagrams for mild steel for':· :he curves in F:~1the Yield
stres&'de/dt :::: 9.5 X 10-1per second to u wide range of ratg of 1.6 ahow
11. 2 'Y ield Point (u" ly the yield pomt "" 300 = es
· but also the ultim te r-r second). It ia st rain·
ot on l h a , !ltrength seen that
"· depend great y upon t e rate of strain I and the total 100
T he early portion of t he tensile test diagram in Fig. 11.3 is shown t · n general, these quan/t. ~
a tion . as the rate of strain increases.
larger scale in Fig. 11.5, as a stress-strain diagram. 1'he shape of t~·18 crease dd . i ies in-
en stretching of ste 1 1.ta · Id .
diagram at the yield point depends noticeab ly on the mechani cal arran To explain the su . e at
~ent of t he testing machine . If extensio n of the specime n is produced by~~ ggested• that the boundanes of the grain !1e !)Oi.nt, it has been
su form a r1g1 k l t h' h
. 'd s e e on w 1c prevents pl . s consll!t . Ie material
of a bntt
mcrease of distance bet ween the grips of the machine moving at a uniform and . .h ast1c deform r
speed, t he sudden plastic stretchi ng will somewh at decrease the tensile t low stress. Wit out such a skeleton the te .1e test ad111.cn-a _ion of the grains
a . d . F" ns1
force in t he specime n, and a sharp peak A in the diagram will be obtained. like that indicate m 1g. 11. 7 by the broken line f\.. .: _ "-?)•"'m would be
. v wmg t-0 the Presence
lf an elastic spring is inserted in series with the specimen, the slope of the
curve A B of the diagram can be reduced as shown by the broken line AR 1

On the other hand if the tensile load is applied directly to the specimen , the V
j,600 r---r---- r--- r--~ --
tensile force a t yielding will be affected by the inertia of the load in sudden
motion , and small vibratio ns may appear on the diagram .
" In order to study in more detail the de-
formatio ns which occur at the yield point,
specime ns with polished surfaces have been
A usedi. Such experim ents show that at the
time the tensile stress drops from point A to
8 point B (Fig. 11.5) fine, dull lines begin to
appear on the surface of the specimen .
These lines are inclined about 45° to the
C
direction of tension and are called Lueders'
lines* (see Fig. 2.2, p . 27). With further i:i
FIG. 11 .5 stretchin g, the lines increase in width and 1,4001- - t -- + - - l --+-- ---J
in number, and during stretchin g from~ to
B, t hey cover t he entire surface of the specimen. Instead of polishing ; 70()
somet imes special paints (called stress coats) are used to indicate Lueders
,
lines . The pai nts a re brittle and cannot sustain large deforma tions; hence 0
they crack during loading a nd indicate th e pa ttern of Lueders ' lines.. .
0
St udies with a microsco pe show t hat Lueders ' lines represen t the in UJ- Elon9ation ( ~' cent )
Flo. 11.7
Fro. 11.6
*These lines were first desc ribed by W. Lueders, Dinylers Polytech . J., 1854
'See P. Lud . and Scheu V J-·t Ing Ber., No. 70, 1925.
\Vtk "Werkstoff&nschu88, " er. ~"· ·
'
STRETCHING B
300 MECHANICAL PROPEilTIES OF MATERIALS EYOND YIELD
. POINT
the cross sect10ns, and after unload· 301
of t he rigid skeleton , the material remains perfectly elastic and foll
over. · amount of strain energy rem . _mg, sorne resid
Hoo~e's la~v up to point A, where the skeleton breaks down. Then °t~ 8 r~1n d' ain 1n th 1
ua stress d
ce f f ter unloa. mg we load the spe . e specimen an a
pl~'3t1c gram ma_terial suddently obtains the permanent _strain AB, o.ft,e: I a . . c1rnen a .
which t he material follows the usual curve BC for a plastic material. Th· ·ts vield pomt stress is raised Th' second time .
18 that I - t . . is characte . ' we will find
theory explains the condition of instability of the material at the up which represen s a tensile test diagr f • . ristic is sho . .
11 '9 h . t C . arn or tn1ld wn m Fig
yield point. It also acco unts for the fact that materials with small gr~r h~ bar to t e ~m , It was unloaded. Duri steel. Aft-er stretchin.
• a1n t II wed approximately a straight-line la
size u~ually show higher values for the yield stress. As a result, such ng unloading the mate . gl
fo o Wh h l w, as sho b ' na
materials undergo more stretching at the yield point, as defined by th diagram . en t e oad was applied t h wn Y the line CD
t he . f 11 d . o t e ba on
lengt~ of t he horizontal line AB in Fig. 11.7. In addition, the theo/ terial again o owe approximately H k , r a second time th
ma . t F h' h oo es law and th . ' e
~xplams the ~act t hat in high-speed tests the increase in yield point stre~ btained . At porn , w Ic corre:;;ponds t 0 h . e lme DF was
is accompamed by an increase in the amount of stretching at yieldin oh curve abruptly changed character and t·· dt ,e previous loading at c
t e <l l . ,ace the portion PG h' ,
as shown by the curves in Fig. 11.6. g, be considere a· prod ongat1on of the cut·ve BC · Th'ts represe ts, w 1ch• can
the yicl <l pomt ue to previous stretchin Of h ·. n a raising
of g t e mate I If
days arc allowed to elapse after the first unload· h ria. · several
ll.3 Stretching of Steel Beyond the Yield Point · ld · mg, t en upon reload·
still higher y1e pomt may be obtained as ind· d b mg a
-,, · 11 10 ·h h ' tcate Y the dotted line
at p . Fig. . s ows t e results of a tensile te 8·t 0 f d' .
During stretching of a steel specimen beyond the yield point, the material te-cast alummum .
hardens and the stress required for stretching the bar increases as shown ti'
by the portion BC of the stress-strai~ diagram . in Fig. 11 .8. Elongation
of the specimen is combined with uniform reduction of the cross-sectional
4,200
F;_,-
I
I
--
~ G
6,300
V

C
..,....,...
I/F
c
I)

~ ., ,,,,. . --- I

I
e
V
4,200

A
,,~V
l',...P'
C '""~ u
1 2,800
b
I
8 Q,
2,100 tlO 8
~ PL,l,400kg per sqcm

I 1 /

0 10 20 30
• I
~ I
Elonootion ( per cent)
1,400
Fxo. 11.8

area so that the volume of the specimen remains practically constan~


The work done during stretching is transform~d largely into heat, an f
the specimen becomes hot. Calorimetric measurements show that not all_ 0
the mechanical energy is transformed into heat, however; pa.rt of it rema•~s 0 - lo •
20
0
£:ongot ,on ( per cent )
1 10
in the specimen in the form of strain energy. Owing to differences ~ Elr,ngot ion ( pe r cent )
orientation of the crystals, the stresses are not uniformly distribute FIG, 11.10
F10 . 11.9
STRETCHING BEYOND -----------
302 MECHANICAL PROPERTIES OF MATERIALS YIELD POINT .,
C is the diagram obtained with the sa 303
The initial proPortional limit of the material was 390 kg/cm•.Aftcr stretch· c1.1rve heat treating at 1000c for four _me bar af ter a ""c d
ing ft,er · minute ""' on uni
the specimen 2 per cent the proportional limit upon reloading was found ~d a pletely recovered its elastic prope . s. In this case th O&Clin~
, . . , • corn f . rt1es e mat::na,1
t-0 be 1,400 kg/cm• and the yield pomt about 1,480 kg;cm • haS henomeno11 o stram hardenin d ·
More complete investigations show t.hat the time which elapse~ betw f he P . h . g ue to las •
ared 111 many tee nolog1cal proceSSes su hP tic defol"m nt· .
unloading and relm\ding has a great influence on the st.ress-strain c ee,, oun t ,, . . "'-4 ion lS
cJlC . g tubes and wires at low te-mperatu . c as rolling bars d
during reloading. If reloading begins immediately after unloading Urve . re, cutt1n h an
' ac:. dr~w111 drawing, and punc~mg holes. In all these cases~; eet metal by shears
curate measurements show that there
A C · · ' h t-line law are·
from th e stra1g 11° . h undergoes plastic deformation beco h e Part of the material
deviations 1~h1C d T 1· . . mes arder d.
at tly reduce • o e immate this undesirabl e effect of ' an its ductility
very low stress, an d the proportio · grea
15 I the material whi.ch .
• stomary to annea strain hardening
limit is greatly lowered. But if a nat ·t s
1 1 cu, . , restores th · - .
. .
l of time elapses betwcon. stram hardening of ductile te . ls _e mit1al ductility.
s1'derabl e mterva 8• ornetimes the . I . ma na is of .
. nufact.urmg. t 1s common practice to ,.ub· h . practical USe
unloading and reloading, the mate e:n r1a1 in ma h. . ~ Ject t e chams a d bl
. .. isting mac mes to a cert am amount of overst . . n ca es
recovers its elastic properties complete\ o of h h' f h . ram m. order to elimina te
Fig. 11.1 ~ sho:ws :urves obtained for miict un
desirable stretc mg o t ese parts m service. Th
· b'Jected to an initial . . . intern
d
e cy 1m ers of hydraulic
SSt1:,; are sometimes su .
steel w~1ch mdi~ate that if reloading pre d f . a1 pressure sufficient
follows m ten mmutes after overstrain, to Produce permanent e ormat1on of the walls Th t . 1..
.J__ •
• e s ram Ul:Ullerung
• the material does not follow Hooke's \aw and the residual stresses produced by this pressure preve t .
. . . _ n any perm&nent
o _______..______ but after five days it has partially re~ set in service. Overstramm~ of the met.al 1s sometimes used in the manu-
FIG. 11.11 covered its elasticity and after twenty- facture of guns. By stretching the metal in the wall of a gun beyond the
one days it has almost completely re- initial yield point and afterwards subjecting it to a mild heat treatment
covered it. the elastic properties of the material are improved and at the same tim~
Experiments also show that if the material is subjected to mild heat initial stresses are produced which combine with the stresses produced by
treatment after unloading, say in a bath of l00°C, the recovery of elastic the explosion to give a more favorable stress distribution. Turbine discs
properties occurs in a much shorter interval of time. Fig. 11.12 shows the and rotors are sometimes given an analogous treatment. By running these
results of tests made on a steel bar. The initial tensile test is represented parts at overspeed, a permanent set is obtained around the central bole,
by the curve A . Curve B represents reloading of the same bar ten minutes •~hich raises the yield point of the material and produces initial stresses
after unloading, and considerable deviation from Hooke's law is noticeable. which are in a favorable direction . Die-cast aluminum fans are sometimes
O'
subjected to overstrain at the bore to prevent any pos.sibility of their
7,000 loosening on the shaft in service. Considerable plastic flow of the metal is
C sometimes produced in pressing the hubs of locomotive wheels on~ their
5,600 axles, and this ha.s proved to have a fa vorable effect. Copper bars m tbe
E
.., Cilmmutators of electric machinery are subjected to considerable cold work
~ .
...... 4,200 by drawing in order to give them the required st,renglb.
0. In using · overstrain in this manner to raise • the yie · Id poim . and . improve
. . t keep in nund: (1) that
....,t'ic properties of a structure, 1t is necessary ·0
b0
~
the ela, .
2,800 d to annealing tempera-
"'"' the ha d • . b' t
( r en1ng disappears if the structure is SU iec .e . .on, w. h'l making
~ Utes a11 d ( .
a certamd
· d1rect1 Ie
proport.ionately
c I, 4(J(; it •t 2) that stretching the metal m t
::, , tong • . . h d' ·t'on 1 oes no ssion in the
irn er with respect to tension mt at irec spect '
to compre
·••Prove h . •h
~ · t e mechanical properties wit re
me dire ·
0 ,2 0.4 0.6 0 .8 1.0 1.2 l'he f ction. . . does not improve .
. · d1rect1011
11~m
Elongot,on ( per cent ) the act that stretching a met.al Ill a cer h ~nme proportion as it
lllecha ruca · 1 properties . 111 . compress·ion m t e .-.-
F1G. 11 .12
304
MECHANICAL PROPERTIES OJ' 11An1LU.L8 FRACTuar.a
IN TENsroN
, fypd of Fractures in Tension ~6
doe~ in tension must not be overlooked in cases in which the material .
11,"
subJ~ct~ t_o reversal of stresses. It should also be mentioned that th 18 d'scussing fractures, we distinguish be
1
&re mdicat1ons that material which has yielded in a particular rea· e': lll of cast iron or glass, and (2) sL tween (I) briute I
more sens1·t 1w· . h t,,lon is
,I, case
m t. at region to chemical action, and there is a tend d h ''«lT Ira.ct '
raaure, as 1n
ency µ.,e Jurninum an ot er metals. In th fi Ure, as in the
for ·
corrosion to enter the metal along the surf aces of sliding. This h steel, a'thout plastic deformation over a e rst c~, fracture ""case of rnild
nome~on is of particular importance in the case of boilers and o~h e- .ny w1 croBs-aecr1 ""curs practi-
CI" f the specimen. In the second case f 0n perpendicuJ
cont.amers subjected simultaneously to stress and to chemical action. er ~o d 1.__ racture ar to the
I~ constru_cting a tensile test diagram such as curve ABC in Fig. 11.8 th tic stretching an ,uws the famili.ar cu~ fo OCcurs after considerable
pl,,8 nn, ahown in Fig. 11.13.
teJ1SJ_le loa? 1s usually divided by the initial cross-sectional area Ao of the
specimen m order to obtain the conventional unit stress. But for la e
stretchi1tg there will be a considerable reduction in cross-sectional a/g~
and to obtain the true stress the actual area A, instead of Ac, should~
USed . From the constancy of the volume of the specimen we have

loAo = lA, A = Aolo = ~ .


l 1 +E (a)
and the true stress is
p p
(1 = -
A
= -(1
Ao
+ E) , (b)

To obtain the t rue stress diagram the ordinates of the conventional dia-
gram must be multiplied by 1 + E. In Fig. 11.8 such a diagram is shown by
t he broken line . It extends as far as a vertical through point C, where the Flo. 11.13
load reaches its maximum value . On further stretching of the specimen,
local reduction of the cross-section (necking) begins and E is no longer In discussing these two kinds of fracture, the theory has a.gain been ior-
constant along the specimen . Then eq. (b) is no longer applicable, since w&rded that the strength of the material cnn be described by two char-
the stresi,es over the minimum cross-section are not uniformly distribuwd. acteristics, the resistance to separation and the resistance to sliding. If the
In such a case, eq. (b) giYes only an average value of u . The average unit resistanct to sliding is greater than the resistance to separation . we M\·e a
elongation Eat the minimum section may be found from eq. (a), which gives brittle material, and fracture wil! occur as a result of o\·erroming t.he
E = ~o - 1. (c) cohesive forces without any appreciable deformation. Ii the resi.stsnce to

Using the symbol q for the unit reduction of the cross-sectional area, eq. (b), Table 11.1 ULTIMATE STRENGTH OF CYLIX DRIC:\ L .-1.\ D
p. 298 , we obtain GROOVED SPECL\lENS
A = Ao (1 - q); Strength ( kg per sq cm)
U] ti mat{>
Q Carbon Steel ~ ickel ('hrorn<' Stet'!
and eq. (c) gives
Computed Computed Com putt'd Computed irom
q (d) (film) from from
= 1- Q. from Reduced
E
Reduced OrigLOal
Original .-\res
:\rea .-\ !'1'11
From eq . (d) the unit elongation
. .
. at t he mmm1u~ sec_t'ion can be readily
This a.a
Area
ll,HJO 13,600 lo, 700
the 11.500 16,300
calculated if the reduction in aren of that section 1s measured. 1.6 12,400 12,900
11.500 10,800 li,000
quantity is called the effective elongation and is much larger th ~n gth. 3.2 11,100
JO 100 7,600 H,500
Norltlal .
elongation E = o/1determined from the total elongation oof the gage en spec,men 7,200 16,000
II"

FRACTURES IN - -
306 MECHANICAL PROPERTIES OF MATERIALS TENs10N .,

of its shape, not because of any m h . 307


separation is larger than the resistance to sliding, we have a ductile mater· cnll SC b' ted . ec an1cal
tie achine parts su Jee t~ impact all sha Property of the .
Then sliding alon g inclined planes begin~ first, and the cup-cone fract'-
Jn fll 008 and should be avoided . rp changes in crru:~~-~~nal.
occurs only s.f ter considerable uniform stretching and subsequent local Ure ~~ -~ ~ ~~
. ~
d uction of the cross-sectional area (necking) of the specimen .
The pre~eding discussion ref er_s o~ly to tensile tests of Compression Tests
11,5
standard circular specimens of cyhndncal shape. The result
rf'h compression test is commonly used f .
obtained with other shapes of specimens are quite different • e d . or testing b . le
one concrete, an cast-iron. The 8 . ntt mal,€rials h
d as illustrated by the grooved specimen shown in Fig. 11.1 4· as st ' . .h b' pec1mens u d . sue
ly rnade m e1t er cu 1c or cylindric sh se 1n the tests
During a tensile test, reduction of the cross-sectional area a~ 11s\lB l h I ape . In co . a.re
l - between t e pane surfaces of the te"t· . mpressmg the speci-
I
t he grooved section is partially prevented by the presence of lflen~ . :; mg machin · ·
: Ta the portions of larger diameter D. It is natural that this action ed that the compressive force is umfonnl . d" . e It is normally as-
surn d" . Y 1:;tributed .
I I
should increase as the width o of the groove decreases. Table tion The actua1 stress 1stnbu tion is mu h O\ er the cross--
0 sec · . f · c more comp!" d
11 . l gives the results of tests obtained with two different e surfaces arc m per ect co11tact and the load . icate , even if
th Is rentrallv appl" d O .
materials: (J) carbon steel with proportional limit 31900 1o
friction on t he su rf acos of contact between th . · ie • wmg
. h l I . e specimen and the h d f
kg/cm•, yield point 4,500 kg/cm•, ultimate strength 7,200 kg/cm•, the machme-, t e at era expans1o11 which ace ompames . e'.l. so
compr~::s:f) 1·
FIG. 11.1 4 elongation 26½ per cent, reduction in area 55 per cent; and (2) vented at these surf aces and the material in this r . . . ' n ~ pre-
. d' · A egion IS m a more favor-
_ nickel chrome steel with proportional limit, 5,600 kg/cm•, yield able stress con 1t10n . .-~s a result , the type of fr t b . .
. f b. . ac ure o tamed in a com-
pomt 6,000 kg /cm', ultimate strength 7,600 kg/cm•, elongation 27 per cent pression test
. . o a cu 1c
. specimen ·.of concre te is as sh · r·
ownm 1g. l l. Jj Tbe
reduction in area 69 per cent. These figures were obtained from ordinar; material. m couta('t with t hr machrne remains unafie cted v. h-.11i:; t1H- mat<:rnl.
.
tensile tests on normal cylindrical specimens with l.25-cm-diameter and at the ~1drs 1s aushed out.
5-cm -gage length. The original cross-sectional area was used in calculating
the stresses. The grooved specimens of the type shown in Fig. 11.14 had
d =1 ·25cmand D = 3.75 cm,
The table shows that in all cases the breaking load for the grooved speci-
mens was larger than for the corresponding cylindrical specimens. With
t he grooved specimens only a small reduction in area took place, and the
appearance of the fracture was like that of brittle materials. The true ulti-
mate strength of the cylindrical specimens was larger than for the grooved
specimens because fracture of the cylindrical specimens occurred after
considerable plastic flow . This resulted in strain hardening and increased
not only t he resistance to sliding but also the resistance to separation.
Similar conditions are sometimes encountered in engineering practice.
An effect analogous to that of the narrow groove in Fig. 11.14 may be
produced by internal cavities in large forgings, such as turborotors. Ther-
mal ·stresses and residual stresses ma.y c0mbine with the effect of the strees
concentration at the cavity to produce a crack. The resulting-fracture will
ha ve the characteristics of a brittle failure without appreciable plastic f\oW, FI G. 11.1 5
although the material may prove ductile in the .usual tensile tests.
Beca~ most of_t he grooved specimen remains elastic during a
test to failure, 1t will have a very small elongation and hence only a
te:~~
8
.
1
In <•rd
tnn , crtoohtainthetrucresist.anccto 1' L1111 pn .,~
t rete th .
or ·1 • .
,, ·ion uf a material such as
t u·tbe elimi-
f . . of contac . m "
lialrd ' e 1nfluencr of fri<·tion at t hl' ~ur !.\( (':, d thn surfaces
\ F0 pp! co,--err '"
amou nt_of work is required to produce frac t.ure.' A smal~ imp~ct
ca n easily su pply the work required for failure . The specimen 18 br
r~;: Iii cont
~re 11tl
llet
m nim1ze<l . For this purpoSt' ' ·
· h d h
\\'It paraffin aud foun t ut
the
. t stre ngth was then
ult1ma e
1
.
, different, and cubic
y redu<'ed . The type of failu re was complete}
308 lBCH ANIC AL PROHRTIE8 OJ' MATE
IUAL8 C0llPRJ:88toM 1'teTs
s~im ens failed by subd ividi ng into plate compression tests of such duct;, _
s paral lel to one of the lat 111 . d C . ~
ompress1on is &ccue Inat.e.nats aa coppe r ,~ ....
sides . Anot h~r meth od of ~limi natin g the
effect of friction forces is t. el'll ~ - obta.me . .
sped men s in the form of prism s havi ng is ae\dorl1 rnpres..i,ed cylin der ultim a~ly assornpa.me --
~
O
& leng th in the direc tion of d by lateral- ' .. 9 ' . ~
P~o n seve ral time s large r than the
porti on of the prism then appr oach es the
later al dimensions. The
cond ition of unifo rm compress· le
rn~::· &
pd a co

of Mat eria ls Under Comb' -..J


urnes the sha eXJ>lnaion
pe of a fl&t diac.
A very m~r
· •~ estm
· g method o f pr od ·
ucmg um'f orm comp ressi·on on cylind ion 11,6 'fe818 •ncu Stre a~
. ·
spec.unen s as d evelo ped .m t h e K a1ser
. -W'lh the discu ssion of simple ~nsion d
1 elm I nst1t· ut 1s
· shown in ricaJ
F'
,-
1,ea,1 011e h'
11 .16. The head pieces of the testin g ma.ch .
nsider cases m w 1ch t he ma~rials . an compr .
ine and the ends of the cylind~~ o'fl co e88Jon test.s, let ua
. . h
n We begm wit a d 1scus '
sion of ma•~~ nl-tes~d under combined
are
tre~s-
s d tatic press ure . * Suc h tests show .that IIC, UUl! tested u de .
hY ros . l d . n r Ua.i!orrn
eneou s mate na s
. un er uruior m
bom0 g can sustain enonnoua . Prertm
1•n elastic. Test s a 1so show that large presau
compreasive StJ'esae3 re,
rem& . and
changes m vo1ume. res produce only filJl&ll
Se veral attem pts have been made to prc>1iuce if h .
.
ten sion of mate rials
b
, ut up to now there has not betn aun onn vdrostauc.
· bl .... ;..: ·
to this inter estm g pro em . .,._-=actmy solutio n
Tensile tests of vario us steel s combined with
la~ral pres..mr,e ha,e shown
that the press ure has a grea t effec t on the shape
of the neck find on the
Fm. n .16 reduction in area at the mini mum cro.."-S-sectio
Fm. 11.17 n. Fig. 11 19 shows the
yoke arran geme nt by whic h rension was applie
d to specimens 1nthin the
cal spec imen are ma.chined to conical surfa
ces with the angle ex equal to the
angle of fricti on . Thus t he effect of fricti
on is comp ensa ted for by the v kg /cm•
wedg ing actio n , and unifo rm compression
results.
Com press ion tests of mate rials such as
concrete, stone, and cast-iron
show that these mate rials have a very low
prop ortio nal limit . Beyond the
prop ortio nal limit t he defo rmat ion increases
at a faste r rate relative to the
load , and the co mpre ssion test diagr am has
the shap e shown in Fig. 11.~7.
Som et imes it is desir able to have an analy
tical expression for such a du1.- Tf
gram For these cases Bach prop osed : l I

equa t ion
an expo nenti al law given by the : I
_: 11
u" ~
E = _, (a) L_J
E I

in which n is a num ber depe ndin g on the


foun d the value s n = 1.09 for pure ceme nt
prop ertie s of the mate rial. Bach Q
a nd n = 1.13 for grani te. 10 20 30
Com p ressio n tests of du ctile mate rials show 40
that the shape of t he d',ad.grani F1 0. t l.19
de pend s on t he prop ortions of the speci men. FIG . \ 1.18
As the dime nsion in the ,re:
t io11 of com press ion dec rease!:!, the eff ect of ""
friction at the ends becomes rn;or 1 he \\" Bridgman , who de-
pron ounced and the coinpres8ion test ~alor,..,, tnoat comp rehen sive tes ts of this kin d " -t> n' made
·ee
by P. · , Ph siCjof Hi gh
diagr am becomes steeper. per P I tech · h;s oooks, Tnt IJ ... w York
'~,. "' 1Hque
r"\j
for obtaining enormu~~ Pressu res • d
p~slic f l()IJ) on Fro.:tur e ., e
exam ple, F_ig. 11._1 8 slio:" s t he results 1
~~ ' B \\'iaeman and'
_of compression _tests on ec~~c i· J • "1 l~ ew York , 1"3 1 IIJlCi Studies 111 Lar,JC ''- -~ br H. ..\. ·
cylin ders wi t h vario us ratw s d/ h of tlw diam
eter to the height of t.h ~ Mn~w tria11ial •'stl'f'• , .
•teetin& mal'h 1: 1e w~ descnl)(
q 4.
'.U ·
ll'l1l, Proc . .-tm. Soc . Tut. MaJ ., \ ol. 5 ' 1. 5
310 TESTS UNDER COt,fBINE:o
MF (" IIANlf'A t. l"'"R O P EHTI ES OF MATERI A LS
STRF.saF.g
I t· 311
hg::-- . i 1.20n llnd b illus! rate fract ures o f mc>diu111 carGo, · JIY c Yli ndri cnl s pecirne11s rnuy ,
1n t)J.!'f'r,1
r
1 1 4
pl"('~ul'{' v <':--.'-d ·u& ~1
late ra l Pres.'lur~ . . - ! r_ "iT" '
, 1~1 ,.Ll ...; 5 l"'<'r 1'l'l1 l l' :t rh(111 , ill at mosplw ric pressur e llnJ at a 1ri.l 1t .21.
, n,, .l·ri ·i
dying two-<l1 m ens1ona l stress r d
0f 102.0t'lO ~ cm'. In thC' fin-t ca._~ the an:•ro.ge tnH' st.res..-; wa..-,
8,000 kg/crn•. l n fig, 1

l stll d . ,f)n 1t1un h


s a h..o l•>und 11s have been teste . By .subJert ing a tubp "· t. in-,..alled r.yl1odnc
al
the :,;c'('.i11d c~~- t h f' corrC':-p on ding ,·:1111<' was 33,300 kg /cm' . lt, wa
latl'ral prc~11r< ' the relative cxt<' nt of thC' Aat Part ,ube . tcrnal pressu re, the y1rld Po int st rf'-o f lo axial t,,.n~1r,n <:r1m1.. ed
thf\! \\ 11h an 1111·rC'::\.--<' 111 •th 1n 1 - 1>1, f) r ,·an uin
nt the b111,,m of th<' c1 ip-c,,1w fracture ' dimini s h<':-- : at a CC' rtain prC'ssu re it wl . st.res:Ws was establi:ihPd for ·~•·e l ' f) lJ, rat1r15 r,i '~e .
•-Wo
• c1pn • ra rn· p · •
0

nt in-Iv d1:-.ap'P(':1r, , flnd t h<' frart un' hecomf> i- e ntire ly sh ea r fracturp and nirke l. The . res ultsd .ohta.1np d ir, th ~t .n-.1 1' · ir c\•~d1r.;,; iron.
prin h L3 iliav WP o
pp<.'r , . ' . _r_ u: :-a·ISfactorv
co ent with t e maxim um istonirm Pne-£7,•
~~~~ .
t ';...)' t fif r1 rv • r. l.. - ,J

~~ ~
'
n11reem
~
Mt. l I. 7. , .. , ue 11sr·.L>&,j - m
1

c,< • ".~ ....• ? !v.•.-··,


~~

... . .

~a)

F10. 11.21

d In . Practica . l applica tions no t only t ht.' yie lu. P.:)Int -~r~ bur- • !USOl>rn<'d t.h?
llct11'1t ~ 111 c'3...-<t':, Ol u-ph<'.)D1
ril·:t.l
t Y and strain h a rde n ing~ of grt'a c impvrt:ll h; :,_ <)t: -,.,.,,..
S l'esse
st
8
·
u nusua.l cases o f failure. , su~'
• h - n lo,.,;ll111 • ...,.
• !l.;:, ,x~ l:l-J
1 ! : ,:\hip..,.
• :,.
h!ln
otnge .,_ k h h 11, vi Wt' l t'U L'..:l. " 0
re ""'n ·s and sudden cracks m \...t ' <"·t· u In • h . (\' !)<:'S 0 i i:,ulure,d
.
~ntly ca 11e d a tten t 1on hc,Lh t t'..--i!c rv• , treu ~th an
(b) lo to t he.&' ~u L•}Pc ~ - ) W t'cl :,.:lt\"1:iC .
IV~ark . s teel plates we re u~d w hll . ·h ,h I I • - - C

0 1 the plates
FrG . 11 .20
u .. uon .
- L • ••
·l , i'r·1-·1urt>J :,,Ur1~L"""did nu t "·hvl'-.
llctihty . ,J , hio.:;
i11 lh tn Ordma r y t e 11:--ilt• tt'st.s . But ' lt - . - ·1ure:>. oc-
hy e ex I d J . tlw d•rnu~t
\I ,~, - th<'~
cit llli
. . urf' w as w,Nt lll%t' .. p o e d pressu rf' \ ' l' s.., ,+, :u1c1 11 1 .
~
. comprr- sH1011 . and latera l p 1ess , tlnil
The c<.>m binatic, n of o.x1al
· ble. rr 1l('t,C t C•::;ts'
T ll V .K.&.rrr1a 11 ill ('(>lfl))fl• l::,.', ·lOn k c, t :;, 0 f mar sh 1
l ·t 'i
oW( c1 l).ll J l'l.lr~~~ dl'fo1•nH~t io n n.1id had bri t cle ,·h:Lr.l<·t, •r. J·, \ ·,:·cKi1ml'llSiuna1 ~tress
d rep t1...,' · In,,. n t.n\osph l•r ie tt'mp..' r:!!U rt'>
u 11.t ' - ·wd
• 1111 ' r
. .
, im·rc:.u,ing Lliu•rt1l p n ~~urc marh le \x:cornc s m o r e u.n m o roriq·.
with •lions.
r 312 MECHANICAL PROPE RTIES OF MATERIALS
1,
re,c~
0
TESTS UNDER COIQ

were circumferential, and f


ki
INEI>

or the la
B11lP.aats
313
In order to determine t he influenc.e of temperature and two-dimensiona.1 tbe _c dinal, By ma ng an additional serj rger values th
stress on t he strength and ductility of low-ca_rbon steel, . a co~siderable 1oog,tU transition from one type of fail ea of ~ta it was ey were
amoun t of expniment.al work has been done m recent times _m various tb•t tbe::= 0.76. It was found that in the~~to t~e other occu:bliahed
laboratories. Thin-walled tu bes were used to produce two-dimensional
".iue n occurred along the planes of Inaxirn of Cll'c~erentiaI crac: the
stress conditions. These tubes were subjected simultaneously to axial
tension and internal hydrostatic pressure, so that tensile stresses a, in the
,,.ct•~ of about the eame magnitude 88 in : eheanng etre,, and at t!
st~f rn the same matenal as the tui..._ I case of flat specimens Pre-
circumferential direction and a. in t he axial direction could be produced in · .l'Pli ro d ~ - n the case
any desired ration = u,/ aG• Using t ubes of medium-carbon steel (0.23 Per J)P'vk.S rupture appeare to be more brittle. Fail of the longitudinal
cent carbon) with 3.625-cm . outside diameter and 0.25-cm . wall thickness cr&C l~nes of maximum shearing stress, but owing': ~Sually started along
E . A. Davis made tests• with fi ve different values of the ration. Fig. 11.2 the P t the crack ends, it continued as brittl f g~ stress concentra-
shows the types of fractures obtained . For the small values of the ration
2 1. n a . .
t ?bout substantial plastic deformation. The maximum
e racture l.Il th .
e_ axial plane
t1t_ h the longitudinal cracks began was always h Bheanng st~ at
thiC . l ,... muc smaller than 1n . t he
f circunJerent1a cra.cM. It seems that th diff
m"' • . ·;;:_1,t· . ,
-
.'~j';·~-
,"li

- ,,,, ..- ,•'i<'),b --


-~ :;_ .,-1,'lfl',~\-
-. , • l., '17 . . -: caset;res were due largely to the shape of the speeci- erencies in the two
th .l ...,,ens. n the c~ of
, mferential cracks e ma.tena was much more free to neek down than
frac
i,: ... •, ;,~ ·~ J
c1rcu . d' k
r\:_
;; ( '. 1 -~f~\:,,:·-
. the case of long1tu ma.1 crac s and therefore the lath
1.11 •
smaller local deformation and smaller decrease in load
~ r occurred Wlth
beyond the
.
ultimate
..,; ......_ , . ·, ~ . .. . strength.
~. ~~ ~: ;\" ··.• In experiments at the University of California• teSU! were made at two
-·~ -·~-. ~
i different temperatures using thin-walled tubes of low-carbon steel. The
diameter of the tubes was 153 mm and the temperatures were 21• c and
' - 94.5°C. The tests at room temperature always gave a shear type of
r
. ·--.

fracture with considerable plastic deformation. THe tests at low tempera-
·"·
··~;
. ture (with n = I) showed brittle fracture with very small plastic deforma-
' ... tion. This brittleness was attributed to the local stresses at the welded
,:"" ~-.. ·
. ., iunctions of the tubes with the end connections.
·:.... :" ~
After these tests ~ith small tubes, large-size tubular specimens of so..an
;i ou!'
21
8ide diameter and 3-m length, made of 19 mm ship plate, were t~ted at
C and at -40° ·C. The tests at low temperature, especially wi th the
ratio n = 1, showed brittle fracture at stresses much smaller tban lhose
oh~n:-ed
14lll
. tests of ordmary
f rom tensile . . dnca
cy1m ~ imens rn&de of the
· l .:.pee
,.\,.
.. i"';.. B&!ne Inaterial.

~~
.,...,
~r;~-- - . -,.~.,..-
'',
:'.::; -:" '
,_-<
--~--
\

.. ,,·
'
··•:
; , ' ''
·· -~-'.t ·
. ll,7 St
rengtb Theories
· ls are normally deter-

-
~ 1'h
-~ J;. . .~J 4, ----
llrin~ mechanical properties of struc~ural mat.eria tively simple stress
,J
Cond' _by teats which subject the specunen to comp&ra complicated st~
ft • ~ ;
RRc:
It ,;
tJ =l () f1 ::. OO

con/!ona. The strength of materials under moretional cases, such as


Fm. 11 .22 th •t•~ns has only been investigated in a few excep
<lee discU88ed in the preceding article.
•See E . A. Davis (Westinghouee. Rea.search Laboratories), J . Appl. Mech ., Vol. 12,P· 13'
1945, and Vol. 15, p. 216, 1948.
'See lf. t . 0.via and E. R. Parker, J . Appl. MecJa ., Vol. 15, P· 20l, 1948·
314 MECHANICAL PROPERTIES OF
MATERIALS
STRENGTH 'l'b,,,, _
In orde r to dete rmi ne suit able allo . •~-.uPJ.r.s
wable s~resses ~or t.he com plic a~d
stre ss con ditio ns whi ch occu r in prac l"'ing tha t stre ss in one directio
tica l design, vari ous strength theor~ 3
hav e been develop ed. The purp ose Obi,e..,g ~rp end icul ar dire ctio ns an nd. pr?d 15
of thes e theo ries is to pred ict failu tber t.... . • . uces la~ral def
con ditio ns und er com bine d stresses ,
o · rincipal stra m com pon ents of th ua1ng orrnar .
, assu min g that the beh avio r in a simp aupel'?o8itio
tens ion or com p~i on test is kno
eith er yiel ding or act ual rup ture ,
wn . By failu re of the mat eria l is
whi chev er occu rs first.
mean:
~ three p. ons: .
~Jl)ress1
10-n 1n the
e e1emen tin Fig. 11 we find for the
. ' the folloWing
i3
The mos t gen eral stat e of streBE whic 1
can exi~t in a. bod y i~ ~lw ays com
h . t:1 = ]ila-1 - /.l(u, + era)],
pletely
dete rmm ed by specif ymg the prin 1
cipal
stre~ses a1, a2, and as (Fig . 11.2
3). In
t:2 = ]ila , - /.l(a-1 +a-1)],
the following disc ussi on, tens ion
I
is con- 1 (11.2)
side red posi tive and com pres sion nega
I
I - + -• cr2 and the axes in Fig. 11.23 a.re chos
tive ta = ]ilaa - µ(er1 +er,)].
er, I en s~
r.,,,,,,
j_ __ _ _ _ that the rela tion s betw een the alge
braic
valu es of the prin cipa l stre sses With condition (a), the first of thes
are e equations
. d th
strain and t he t h1r · · represents the rnn; _
, e mm unu m
strains '1y.p.IE m . n d ' strain. Subs•:tutmg
. tens1o .. ~u m
(a) an a- y.p./ E in comp""'"'' . the yi.eld . l,J

.
criterion of fa ilure acc or dmg point
F10. 11.23 The maximum stress theory consider
s the to the maximum......,,ion tor
,. . , 1.. ti and ti, t be
max imu m or min imu m prin cipa l oLl'al n weory beco~
as the crite rion for stre ngth . For stress
beg ins in an elem ent of a stre ssed
duc tile mat eria ls this mea ns that yield
bod y whe n eith er the max imu m stres
ing or
0'1 - + 0-1) = er
µ(a, }
reaches the yield poin t in sim ple s
tens ion or the min imu m stre ss reac
t he yiel d poin t in sim ple com pres sion hes 1.. - µ(a, + a,)I = .:: , (1 1.3)
. Thu s the con ditio ns for yielding
a.re
There are man y case s in whi ch the
maximum strain theory may iwo be
(a 1)y.p. = ay .p., or \(aa)y.p .\ = a\.. p. (11.1) ~hown to be inva lid. For exam ple , if
a plate is subjecte<l t-0 ~ual. t~nsions
in two perp end icul ar dire ctio
in whi ch a th ns, the maximum strain theory indicates
and a' are the yield poin t stre sses in simp e ~nsile stre ss at yiel ding will be high that
com pres si:~ P -respect i:~ly. The re le tension and er than the yield point in simple
, a.re man y exam ples whi ch contra.di ~nsion. Thi s resu lt is obta ined beca
use t he elongation in eatb direc tion is
the max imu m stre~s theo ry. It has
alre ady been poin ted out that m . . et l ecreased by the tens ion in the perp
tens ion slid ing occu rs alon g plan es si?1p e conclusio · endicular direc tion. However, t-his
inclined at 45° to the axis of the spec u, n is not sup por ted by experiments.
For thes e plan es neit her the tensile • nor the compressive ime~ Tests of· matena · ls d
· un er
· st resses are maxi- nifonn hyd rost atic pres sure also cont
~ond 0f radic t this theory. For this ca..~, lhe
mum and failure is eaus ed by shea eqs. (11.3) give s
r stresses inst ead. It has also bee~
poi n~d out that a homogeneous and
in sim ple com pression, may sust ain
out yiel ding . Thi s indi cate s that
isotropic mat eria l, eve n thou gh w~:
very larg e hyd rost atic pressures wit."
(1
I
y. p .
the mag nitu de of the max imu m str:' 1
~ :
I CTa Iy.p. = 1 - 2;i
I

not eufficient to dete rmin e the con


ditio ns for yielding of the mo.teria 0 in Which
its frac ture . hornoa a-, repr esen ts the hyd rostatic pressure .. Expe . riments show that
A seco nd stre ngth theo ry is the max h y . "eneous ,' thstsnd much
imum stra"in theory . In this t e?r it higher . .
mat eria ls und er uniform com press1on can wi
is assu med that a duc tile mat eria l st, reBse
1'he ... . s and . l .
begins to yield eith er whe n th~ maxim~e111 rem am e ast1c .
stra in (elongation) equ als the yield n lh. "'<Qim he ·th experiments, at
poin t stra in in sim ple tension_or ~ . reem
the min imu m stra in (shortening) pie \"'Gt for du ~m s ar_theory _gn-es
better ag_ , ent. w1 This
·
theory assumes
equ als the yield poin t stra in in s1 ltat~ield' ctile mat eria ls whi ch have
com pres sion . m ~--~ lno L_ • cT y .p . -
11
r ,P•_ th mate rial becomes
. 111\\1 i . ~ uegins whe n the maxim um h
\~ "I.I the . s ear stress m e
. . simple tensi.on
l. Si.-. tna.xunum shea r stress at the yie . Id point m a
••ot th •a1 is equal to haIf
e max imu m shea r stres · the mate ri
STRENGTH T}fllln
""-'RIJ:s
316 111:CRANICAL PROPERTIES OF MATERIALS
machin e design. • Consid . the l
.0
317
sses i . enng
. g the same reasoni ng a.s for simpl _e ement in F'
the difference between the maximu m and minimu m principa l stresae sttE' 10
e ten on, we find t~ · ll.23 and
81
and since the maximu m shear stress, in a tension test, is equal to ha~f applY per unit volume is
t the strain
the normal stress, the conditio n for yielding become s energY
u = ½(t1u1 + Ewt + ) t~a .
0'1 - as = O'y . p . • 01.4) cornpc,
t·tuting the values of the strain nenta from
subs I
This theory is in good agreeme nt with experim ents and is widely used
in IJeCoroe:,; eqs. 01.2)' this
machine design for ductile material s. 1
= 2E[u12 + u2· + 0'32
G

2u(
To compare the precedin g strength theories let us conside r the case of
1t -
,. 0'10'2 + O' +
Wi . 0'10-1)j. (11 ")
n
pure shear. For this special case of two-0im ensiona l stress the maximur mate . ,_ .J
equal A• already pointed out, we know that
tensile , compres sive, and shearing stresses are all numeric ally • h naU!. can with.st.a
· nd very 1,._
I"'
e wit out failure. Hen .
(see p. 6.1) and we have hydrostatic pressur
. . ce 1t 18 l'ea8onabl ..... ~1::
t.he tot.al stram energy mto two parts: (I) th . e to resolve
.
. -P>J of uniform
a1 = - aa = T, a2 = 0. tension or compre ssion . and (2) the strain • enel"'lnr
e st:ram
. . h 1· . . d
energy ot distortion an . ~ ,
for determmmg t e 1m1hng stress condition nl . 1.:_ la.ter P8rt f •L
. . th d' . 'O y "lll!l
Assumin g that the materia l has the same yield point in tension and corn- stram energy, 1.e., • e ~stortion energy. To accomplish this ae O •ure
pression , the conditio ns for yielding accordin g to the maximu m stress we use eqs. (11.2). Addmg these equations we obtain J)&r8tion,
theory, maximu m strain theory, and maximu m shear theory, respectively,
are
E1
+ E2 + = -1 r- (2µa 1 + + er,),
E3 O't
(c)
Ty .p . = O'y .p , r

which states that the unit volume change is proportio -UAI ~• to th .


= f th th · · e S.UllUilaLion
T
y .p. 1 +µ
O'y,p . '
(b) ;'. . ~ ree prmcip al stresses. If this summation is zero, the volume change
alnfishe~and the materia l is subjecte d only to the deformation of distort-ion .
= 2.O'y.p.
0-1 - u2 - u - . th f hy drostatic p!e$ure, we have
Ty . p. - i - P, as m , e case o

1 - 2µ
Taking µ = 0.3, we find the following results for pure shear: E1 = E2 = f3 = E = -E-p . (d)

Maximu m stress theory ... . .. . . . ...... . .. Ty .p. = CTy.p .


. h.
There will })(' no d1· s't ort·ton m d .. . .
exis•~~ alone. t 1s case an um.form tenSJon or oomp~o11
Maximu m strain theory .... .. . . .. . .. . .... Ty.p . = 0.77ay .p.
Por the
Maximu m shear theory ... . .. .... . . ... . . .. Ty . p. = 0.50ay .p. general case, we introduce the not.at.ion

It is seen that the difference between the various theories is considerable (e)
in this particul ar case . In the design of a circular shaft in torsion, for
th
example , it is first necessar y to assume an allowab le value of working 5t.ress
a11ct
be follo~/n divide each of t.he three principal stre~~,s into two parts as
in shear T,. = Tm.,. = Ty.p ./n. Then the diamete r of the shaft may
found from eq. (4.5), p. 73 . Using the three theories discusse d abo\'e,
(f)
the following ratios of the diamete rs are obtained : C) 171 = P + u'1 , a-, = P + u',, us = P + u's.
IJ\ltn . .
ll1111g Up th . ( ) . obtain
1 : 1.09 : 1.26. ese three quantities and wung eq . e ,~e
.
I n more recent . . at,.1011 of the strain eneray of de f orrn at,ioll
.
· 1
times, consider
~- . rkt 11 g o-'1 + o-'t + u's = 0.
per unit vo ume of the material has been used as a basis of selectiug wo
~ a. ll:. Pot.eraon ""'-
' ~ort.U
C ,_, .
oncen., ....wn
n.-·,.n ftJCU}r,, New York, 1953.
.V~•HII
r 318 MECHANICAL PROPERTIES OF MA TE RIALS

Since the summati on of a' 1, a',, and a' a vanishes, these stresses Produc
only distortion , and eqs. (f) provide a means f~r dividin~ the given syste;
of stresses a 1, as , and as into two systems; (l) umform tension or compression
1·bus t 1
tt1 == 2E[u1! +
~T~E°!'lr-'J'R TH

be strain energy of distortion in th

0'22 + a,2 -
-- F.OJllTi'~

2 (
e general

µ 0'10'2
case .
IS
+uw + cr,a-,)j
319

p , producin g only change of volume, and (2) the system of stresses er 1


a't , and e1's, producin g only distortion . 1 1 - 2µ
As an example of the applicati on of eqs. (f) let us consider the case of
' - 5E""(a l + Ut + a-a)2
simple t~nsion, Fig. 11.24a. · Substitut ing 0-1 = 0-1 = 0 into eqs. (e) and 1 t µ
(f) we obtain == ~[(0-1 - <12)2 + (a2 - u,)2 +(
er , - u )2]
20'1
I • (11 6)
<11
I
= 3'
1
<1 2 = 1
<1 1 = 'fbis equatio~ may n_ow be taken ns the basis for . . .
ile materials ha vmg a pronounced yield . Predicting failure oi
due t d' t h'
Simple tension in the x direction can thus be resolved into uriform tension tension. ~cc.or mg . o t .is t heory'. for the general
point stress
case :, .p. m.
simplf
(Fig. 11.24b) and a combina tion of pure shear in the xy- a.nd xz-planes d "' yieldmg begms v. hen the distortion ener of ~tresses u1, u1
(Fig. 11.24c). It ca.n be seen that the work of the stresses producin g only anlue ~f the distortio n energy at the yield pointY:f (eq: 11.6) teaches ~
va . . b . d m a Slmple ~ •
distortio n (Fig. 11.24c) on the displacem ents produced by uniform tension This latter quantity 1s o tame from eq. (11. 6) b bs . . llSl-0n test.
(Fig. 11.24b) vanishes . The strain energies of cases (b) and (c) are thus Y su trtutmg
independ ent of ea.ch other, and the total strain energy in simple tension is
u, = O'y .p., 0-2 = ua = 0,
obtained by adding together the strain energy of uniform tension and the which gives
strain energy of distortion .

(l l. 7)
Then the condition for yielding based on the distortion energy theory is

«,
(a-1 - 0-2) 2 + +
(a-2 - CT3) 2 (u1 - O"a): = 2cr1.p.~ - (11.8)
3 In the particula r case of two-dimensional stress we put a-, = 0 in eq_
(ll.8) and the conditio n for yielding becomes

( 0)
(1 l.9)
( b) ( C)
Cons'd ·
1 enng, · °f t bin
Fm. 11.24 tube8 d for. example , combined axial tension .
and tors10n
, · ·..... 1
an denotmg by <1 and -r the corresponding sire~. the pnne1.-
stresses will be
This conclusion also holds in the general case when all three principal (see eq. 7.7, p. 183)
stresses a 1, a 2, and a3 are acting. From this it follows that the strain ene~gy
of distortion is obtained by subtracti ng the strain energy of uniform tension <ll
V:-.
= 2a- + '\}4 + r, O'S = "-
2 ✓~+
4 r I
from the total strain energy . Substitu ting
and the co d' .
n 1t1on of yielding (eq. 11.9) becomes
<11 + 0'2 + a3
O'J = 0'2 = aa =- --3-- (11.10)
1 rr + 3r :; O'y .p .! •
II the Clae .
into eq. (11 .5) we obtain, for the strain energy of uniform tension a.lone of tol"Slon alone we have " = 0 and eq. (11.10) gives
th~ expression
(11.11)
_ ~ == O.577ay.p,,
Whi • Ty .p . - ~
ch la in . ta1 resul~-
gOOd agreeme nt with experunen
320 ll,ip..\en, TtsTs
MECHANICAL PROPERTIES OF MATERIALS
are other important conclu .
~he eonditi~n of yielding given by eq. (11.8) is currently accepted rl'here . <l s1on
i an be obtame on the ba . 8 321
vahd_ for ductile materials, and it is assumed that the material l _as nich c . F' sis of
'II disgram m 'ig. 11.25. Let u "
to yield when the strain energy of distortion reaches a definit )egin!\ 8
(~ eq. 11.7). e value tnc 'd r the effect of speed of load·
51
col1 e test results . It is known thing
the at
o~ 0 increase of speed the resista
11.8 Impact Tests 1th e. l'd' . nee
'ii material to s 1 mg increases wh'l =
f the . le !
~~lp&.('t tests a.re used in studying the toughness of materials, i.e., th o sistance to separation rema·ins V)
•ts re
11.b1h_ty of ~he materi~l to absorb energy during plastic deformation. e J racticallY constant. As a result of this

static tensile tests this energy is represented by the area under the t 1 .n ~e ordinates of the u, curve will in-
t.est d 1agram,
· d · be l uded that •
m order to have high ens1le
tough e and the curve will move to th
an it can cone
c~' sition A1 B 1 (v' e o
the material must have high strength and at the same time large duct'nl_ess r 1g. 11.25) while the
B. l . ,l Ity 0ew Po . .
r1tt <' materials have low toughness since they have only small pl t' · . ,. remams• stationary. Thus the
1111e vn
deformation before fracture. The use of such materials in structuras ~c . tersection pomt of the two curves is F1o. 11,
25
d • · es 1s 1
angerou~ smce fracture may occur suddenly without any noticeable ~ placed to the right, indicating that with
d1s .
deformation. • • • an Increase · h
. g the CT1t1cal temperature increases. Th'18 ~ t e speed of load-
In discussing Yarious kinds of fractures (see Art. 11.4) it was indicated JD • h' h · b · conclusion · •
impact t~sts, w 1c give nttle fractures at higher te ts verified by
that the same material may behave as a brittle or a plastic material static tests. mperaturP,g than in
depending on the external conditions. Considering, for example, such Now assume that the specimen is subjected t t . ,
0 0 00
o.n important case as mild structural steel, we find that under ordinary ·
specimen m s ear w1h ·11 be ·
gm at about the same val ™f sh. 'r. ielding 0f the
ue O eanng st~ as in
tensile tests, it may have large plastic deformation, while if t~sted at some · t b h ·
the tension tes s, ut t e corresponding value of th · .
lower temperature it may fracture entirely as a brittle mat~rial. Disastrous · h' . e maxunum normal
stress "n, equa l m t 1s case to the maximum shearing st.ress, w1·ii be about
examples of such fractures occurred during World War II in the numerous . . ·
one-half of the value of u n m a tension test Hence ·
failures of welded cargo ships.• 8ubsequent research work showed that the . . . . . .· m ronstructmg, for
torsion tests, a diagram s1m1lar to Fig · 11 ·25 , we mu,t t•k . I
- ,.. e \ a ues ot e
· th
brittleness ternp<'rature of the steel plates used in the hulls of the ships .
ordinates of the er, curve about one-half the values for tension t~sb.. As a
was in the same range us the service temperature. result the intersection point C of the curves will be displaced to the left,
To explain the transition from brittle to plastic fracture, A. F. Joffe and w'e conclude that in torsion tests the critical temperature must be
distinguished between two kinds of tensile stresses, (1) tensile stress u, lower_ than in tensile tests. This conclusion is al~ in agreement 11,i th
producing brittle fracture by separation and (2) tensile stress u, correspond- expenments.
ing to the hcginning of sliding. In Fig. 11.25 these two quantities are Considering further the influence of the state of stre~ on the value of
represented as functions of the specimen temperature t. In Joffe's exper- the ·t· l · ·
di er~ tea . temperature, let u~ assume t~at a uniform tenswn mall t ree
h
iments the m,istance to separation remained practically indepen~ent of dtch~ns Is superposed on 81mple tension , w that we obtiun_ ~ th~
tl'mperature, and in Fig. 11.25 the diagram for er,. is given by the h~nzontal rnensional st.ress condition . It is known that such a superposition d~~
line . At the same time the resistance to sliding was influenced considerably not_ affect the value of the maximum shearing stress al which yielding
by the tC'mJ)('rature of the specimen, and the ordinates of the curve for a,
decrease as the temperature .mcreascs. 'fh e pomt . of .m - tersec tion C of the
begins. The value of u increa.ses howewr, ai1d the ordinates of the t
~uhrve in Fig. 11 .25 incre~se and th; intersection point Cmoves ~o th,e ~lg ~:
two curYes definrs the critical value lcr of the temperature. If t he tempera- h , Us th ..
. e critical temperature for the assum(
,d thm -d1mcns1ona ~tres;
-d' ,· nal
·
t urp of testing is higher than t.,, the resistance to s1·d·
1 mg is· sma.lier than Forte cond1ti -1 · s·mibr
1 three 1me 11 ~10
on WI I be higher than for simplt> tension. · Such
rt>i-ista11r<' to separation and the sperimen will yield pla~tically._ foil stress e d' . h. roored specimen.
SPe . on it.ions are produced at the note 111 a g f th ,peeimens.•
temperatures lower than t., we have u,. < u,, and the specimen wi 11 . the case o smoo "
c1rne11 s I1ave higher values of lcr than 111
by a separation fracture without plastic deformation. Metall, 'F . . • \\' M. Murray (ed.),
. , Fat'igue and Fracture of
.nurra.y (ed) F or tno d . E O wan s art1de m .
•Sec the pa.per by Finn Jona.ssen in W. :\l. ,,. at~ re et.alls on stresses at grwvrs see · ro
1952. aiid Fracture nf Metals, 1952.
322 MEC'H.4-NlC .l,. L PROPCRTIES OF MATERIALS IMPAcn, T1:0,,
....-iTs
jfect of the sizr of the ~pecir
L!: _fnn dariw1 :t:1I 1df' •1:-- n- 1mroin11: t hP crit ical t€mpcra.tu re at which the ·rhr e also hcrn invrstigated . Bt ntehn on th~ i·alue f 323
tra n~t1on oec·ur::.; from brittle t.o plru-tic fracture wen: cxtC'n_ded by N. t\ hf.LS . 1 ere th . 0 the r · .
Dandenko\' and applied to rnrious kinds of st,rel. Using a dmgra.m sirnil · 111rr t gi\'e a r le:.1r int erpretation f e &1rnple ct· -ntiealu-rn
q no . o the e . Iagrarn r Pera.

to r 1g. 11 .25, he was able to predict the m . fl
uencc of :arious
. f_actors ar
on lhr
dnC· in dimen s10ns we may ex pect d Xper,rnenta1 o Fi~. l 1.,.1
• re11,sC
, alue of lhc cmical temperatu re and showed by h1~ experimental work ,nc , f.incc t he pro \)a l)1!1ty ··
nf ha .
a r"re · result
~ u1the rn.,;_ 8· Willi an
. -·
.p1rt , f v1ng c-r1t· ·-ot:jlnnep
tha.t the predictions wert> in satisfactory agreemcn~ with the experimental ffllt • Iume. Hrn cc or larger volu m l{'al imPerf . - tlJ brittle
·th , o
1 - es the h . eet1ons ·
facts.• For determining the critical temperature, impact tests were Used 11' d in Fig. 1 l.2;J , producing a d'i , <mzontal lin . 10~ ~
•err . i:;p1aeern e tor •
Sinc-e in the case of brittle f rs.cture the amount of work required to produ · 101' . !so appears that an mcrea"" 1· I ent r)f P<Jint C 7 • r,,;,U hr-:
. ·1 ure is many ti mes . f racture, t he tests t 1t. !I, ""' n vo urn to tb ·
tru smaller than for plastic showed ceat Bll rresponding lowering of thr (J c . . e reduces the val e nght.
the critical t.em peratu I'E' a sharp change in the amount of energy absorbed the<' 0 · · . ' uri. e 11'1 Fi . lleofu,and
,,..ent of C lll the opposite direction Th g. 11 .2,5 re'11lt • .
16ce,,, · . • t~ th fi .s 1n a di.S-
Fig. 11.26 represen ts the results of impact tensile tests of smooth cylindricai p elative importance of the two fact e nal result depe "-
st('el ~ imens. It can be seen that a sharp change occurs in the energ the r . . . ors. E.xpe . nu.s on
. drical specimens md1cate that the 1 • . nrnent.s \\ith .,,., h
absorbed in the interval -130°C to - l 10°C. y rY Iin . O'\lienng of tb cu10Qt
. Portant and point C moves to the right h . ,e u., lrne 13 more
By changing the process of heat treatment the grnin size of steel can be un . •• h . , s owing th . ..
rature mcrease.s wit. an increase in v • at the cnticat
,·aried considerably , and it is of practical interest to investigate the in~ tern pe 'd d o1ume 0 1 the s .
fac
tor must be cons1 ere when ,applying th fle('1men This
fluence of grain size on the magnitude of the critical temperature. It is d . e results of ~
spe cimens to the esign of large-sized structur~ Sts on snail
known that with an increase in grain size the resistance of steel to separation
In the preceding discussion we have consid -..1 n1
diminishes. Hence for coarse-grained steels the horizontal line for <Tn . . . h' e,ru O V tenme . .
(Fig. 11.25) will be lowered and the critical temperature will be higher cylindrical specimens m w ich the stress distrib . . · .. ,ests 01
. . . uuon was unuonn 1
than for fine-grained steels. To verify this conclusion, specimens of coarse. pra.ct1ce, however, notched specimens are used in im · n
• . pact t~ and stress
and fine-grained medium-carbon steel (0.23 per cent carbon) were tested concentrat10ns are present. To mvestigate the effecc· 01• • .
. . . non-um1orm stress
in impact a.nd the results are shown in Fig. 11.27. It is seen that t.hc distnbutlon
,
on the magnitude
.
of the cri tical tem~ratlil:re
r-
l ~ 1,. _ _,
, et US \.~ lll
critical temperat ure for the coarse-grained steel wa.s a.bout -95°C, while ~1th th~ case_ of ben_dmg of ~ s~ooth cylindrical specimen. faperimen t.s
in the case of t he fine-grained steel it was -160°C. mbendmg with static loads md1cate that yielding of the 5teel begins ~ta
much higher stress than in the case of uniform tension . The yield ooim
stress is first reached in the thin layer of fibel"5 at the farthe~t dis~i.nce
f:om the neutral axis, and the formation of planes of ~ielding in tho-t>
fi~rs is prevented by the presencr of the adjacent rnat<"ri.!V at lower stress.
~
:- ~b, 8
10
r I I
ine 9rained :he resulting increase in the value of the yield stress must bt> considered
111
81---+~
... - "

..
V app\yiog the diagram of Fig. 11 .25 to bending tem. The o~ina~es of
~~ ~ ... th e a, curve must
; be increased which res11lts in a di..-sµ L'lrt> mrnt or the inter-
t
..,.
1
6 1.---4---..i..444'~ ~
,. - Course 9rained
r.ection point C to the right. The critical temper:1turc, as obtained f~m
,,; 4 1-----+-~ 4 bending tests, will then be higher than the rnlue obtained from ten51le
~ l~sts'. · 'fh·is conclusion agrees wi th ex!)fnmenrn · l result,•· . ced
!
. .. .:.
2
2 ~ Similar reason .
mg can .
be apphed i o c:1-.~s o ~ re~ wf ·t ~ ,.... 11 cenc.m11on produ
.
by g t an rncrel..~ in
·
0 '----'---'------' 0 1 rooves and notches (see p. -16) and we may expec
·160 ~140 -120 · 100 -200 ·160 -120 -BO -40 0 +40 +BO ,, for n0 t 0h d
Temptroture ( •c ) Ttmperature (•cl After h' e bars · -·d tbe tYJ'lt' 0 impar ~·t
f· t te

F10. 11.26 F·10. 11 .27 l\'hioh sh t ls general. discuss!on let us rn_m,i_er The c;rrect,deierm!nacion
of 1 . ould he used tn practw<:' t0 dctrrnll,ICI,,. . · . in which the
Qr IS i . d rrous situanon
-see Davidenkov'e books, Dvnamical Teating of Mtlau, 1936,· :ind Problems_of 1 ~~~~ Ql'iti I Oiport.ant in order to a\'01d the ang , h servi('e temperat ure
l'a tem . . h same !I~ t e ' re
in Metal Study, Ed. Ac&d . Science, Moscow, H)38 (in Ru88ie.n). The results given 1 Ofth P<'rnt11re of thr m11 tpr1al 1st e - lnl ternperatul'\' 8
(' str · •t tests at r01 f
following diJ!CuBlrion, if not 11peci.6co.lly noted, are tJ,ken from the latter book . ¾t 8 \Irture. It is apparent that 11npuc . . t ,1• 0ver a range 0
Uffio. . .. 5 . series \)1 r~ ~
lent a11d in import.ant s1t11~t10Pl it - -

l
324 MECHANICAL PROPERTIES OF MATERIALS
FATIGUE or
~!ta
temperatures should be made. A transition cur:ve, similar to Fig. 11, ed the range of stress. The .
• cfl.ll • cycle 18 325
should be construrted and from it tcr determmcd . When the criti '
27 1s the maximum stress are given cornpletei ~
~pd . The avera Y denned if t
temperature has been determined, and knowing the service temperat cal u., :::: ½( ge or rnean he range
'1i of the structure, Davidenkov recommendH that the measure of
be t.aken as the ratio e ·Y
sat;e e particular case of reversed
Urr,ax +11 • )
lll•n ·
fftress is
IP th
To - Tcr .,., o. Any eyeIe of varying stre stresa u"'in == -u (h)
(a) ~,. d t ss can be b 111&1 , R - "l
To of reverse s ress on a stead o tained b - ~,aq and
eye Ie 1 f h Y average Y suPe .
in which To and Tcr are the absolute temperatures corresponding to ta a d . •mum va ues o t e varying st stress. Th . l'Jlo3lng a
mini .
ress are th .
en D'ly
e rnll. .. ,-
-.u.uum and
lcr · T~is ratio diminishes and approaches zero as To approaches Tcr• T~ formu1as. e• en by th f
e olloWJng
.
result 1s a very dangerous situation in which small external impulses m e
produce brittle_ fracture of the structure. On the other hand, the ra~~ O'max = u., +!!,,
2 11lll i~ :: 11
R
. ·-r
I

approaches unity as. Tcr approaches absolute zero. In this case brittl ~
frartures will not occur and it is only necessary to select the dimensions 0~ There are various methods of apply· h
· ing t e load ·
the structure so that it will be strong enough to carry the loads witho t The specimen can be subjected to direct te . m an endurance •~st
plastic deformation. u · ns1on d «:: ·
bending, to tors10n, or to some combinati· f an compr~on to
. on o these Th . '
In selecting a reasonable value of the ratio (a) for use in design the is by reverse d ben dmg. A common cantil f · e sunpleSt way
conditions which actually exist in the structure must be consid~red. . . 28 T ever orm of fati
shown m Fig. 11. . he cross-section of th . ,- gue test bar is
Such _stress raisers as sharp reentrant corners and imperfections in welding e specunen IS y o ~ u:u
~,J
a1ong the QJ

contribute to an increase in tcr . An increase in size of the structure ha& the

6
same effect. To have sufficient safety and to keep the ratio (a) as large as
possible, materials with low values of tcr should be used. The critical 2.70mmrod _...;;.,~= -- 92.08 -mm------.,
I
temperature can be lowered not only by changing the chemical content of m, '
the material but also by proper heat treatment. A fine-grained steel has a
1J.91
mm
I
-. 11.99 mm
- IL

lower value of t r than a coarse-grained steel. Considerable interest in


0

the brittle character of metals at low temperatures has developed recently


in this country, and we can expect an improvement in our knowledge of
this important subject.
t-----n
_,_ )07_9::mm~ ~j
tD

Flo. 11.~
ll.9 Fatigue of Metals length m · such a manner that the maximum stress occurs between cros.s-
Machine parts are frequently subjected to varying stresses and it is ~tions mn and m1n 1 and is practically constant within that region. ~he
important to know the strength of materials under such conditions. It e ect of stress concentrations is eliminated by using a large fillet radms
is well known that materials ·fail under repeated loading and unloading, and
. bY Increa.smg
· · the diameter of the bar near the fillet The load p
18
or under reversal of stress, at stresses smaller than the ultimate strength always downward and the specimen rotates at constant speed. The
stress the f · sn d the number
of- the material under static loads. The magnitude of the stress required re ore changes sign every half-revoIut.10n, . of
to produce failure decreases as the number of cycles of stresg increases. cycles of stress is equal to the number of revolutions of the machme. The
This phenomenon of the decreaia~d resi8tance of a material to repeated
stress is tre~ L,;;ug zero and
the I'll, a eompletely reversed stress, the average s •
stresses is called fatigue, and the testing of a material by the application of h ngc of stress twice u max• . p
oy taki . h at various loads , a.
such stresses is called an endurance test. d c11.... ng several specimens and testing t e~ ed H is repre-
""ve such . be btalll ere <lmu
If <Tmax and Umin are the maximum and minimum values of the repeate ~tttect, as Is · shown in Fig. 11 .29a can °
d. to produce fmcture.
stress, then the algebraic difference 1'he cuas a function of the number ~f cycles~ ret~t the beginning D"m••
(a)
R= O'max - '1min
deore tve shown was obtaintd with mild ~tee . 'Ilion cycles there is no
l\ses rapidly as n increa~s, but :if ter about 4 mi ·
326 MECHANICAL PROl'ERTIES OF MATERIALS FATIGUE
OF 1,f-.
.. ,.,..Ls
longer any appreciable change in "max, and the curve approaches asyrnptot. rrow groove (seep. 306) in wh·
ns. f f Ich th e shal'\.n f
ically the horizontal line u max = 1,890 kg/cm'. ~h~ stre8S corresp?nding to ,-~..: P,
. g a, rid there ore racture
.
occurs as !'1: o th
127
such an asymptote is called the endurance limit of the n:1atcnal. It ifl Qlid:fl ' 'fhis fracture is of the br1'ttl a result of o e8Pecirnen
· et Vere · Prcv
now the usual practice in endurance tests to plot <Tm11x against log n. In force_S· In the case of cantilever test s Y_Pe even thouorn1ng the co~n1
this manner the mBgnitude of the endurance limit is disclosed by a definite dt1ct1le, are at the outer fibers . Be Pec1rnrns (Fig llgh the lllate,.;.,'ve
sses nee the f • · 28) h · ''4 IS
stre circurnfere nee and spreads towards the at1gue crack u t e rnaxirnurn
discontinuity in the curve. An example of such a curve is shown in Fig.
11.29b. the trations due to fillets, groove center. Whe SUally startg at
ncen . hi s, or ho! re there
co most h1g y stressed portion and es, the crack are stress
11t theh cases the fracture surface sh spreads outward f Usually st.arts
2,520
O',noa
I sue . . . ows conce t . rorn this .
n. starting pomt. This is a very comrnon t n r1c rings with r P<>int.
=eu 2.380 tbJS which have been subjected to al tern t' YPe of fra cture i e8J:Ject_to
narts f f · a Ing stre n lllachtne
~
-,; r::r 2,240 \ r- t the brittle type o at1gue fracture is d sses. It is th11g .d
tba • . ue to th . evi ent
E "'
:, ..
E ~ 2,1
00 \\ of fra C '
ture not to crystalhzat1on of the
material as
. e Peculiar mecha .
ntsm.
r-.... was once thought
·•
i bO
~ 1,960 ,.__ a,4oo 1 -r,--r--r---._ ·
1,820 n
0 2 4 6 8 10 7,000
Number of cycles ( millions)
g
~e 1,750 2,ll ( 0)
1
~
5,600

EEi I.
~ u Ill)

; ~ ,.~ ~ 4,200

.,u 2,800
~ I 050
2
C
, 10• 5 7 8 ~
10 10' 10 10 ,,:, 1,400
Number of cycles C
w
( b)

Fm. 11.29 0 2,800 5,600 8,400 11,20014,00li6,800 19,600


Ultimate tensile strength /kg J>(f sq rm !
There is a great difference between the fractures of mild steel specimens
F10. 11.30
tested statically and those tested by alternating stresses. In the first
case considerable plastic flow precedes fracture, and the surfaces at ~he It is evident from the above discussion that the determination of th e
ruptured section show a silky, fibrous structure due to the great stretching endurance hmit • • 1 reqmres
· a large number
for a particular matena bl ' ofh tests
I
of the crystals. A fatigue crack, however, appears entirely different. A d · I · to esta
an considerable time Hence it is of practica Ill er~s t t . ts, reh' a-h
era.ck begins at some point in the material owing to a local defect or ~o a r . hanical propertte~w IC
stress concentration produced by an abrupt change in the cross-secti~n. ions between the endurance limit and o!her mec t of experimental
can be d . . Tl1 large amoun · '
Once formed, the crack spreads owing to the stress concentrat10ns · a t its d etermmed by static tests. . ~ . establish such a correla-
ends. This spreading progresses under the actio[! of the alternating ~tr_ess .ata accumulated has not yet made it poss1~le _tof ferrous metals under
tion A 1tm1t or .
until the cros.s-section becomes so reduced in area that the remaming ~ . s a rough estimate, the endurance to O55 times the ult1m~te
portion fractures suddenly under the load . Vel'SaJ of stresses can be taken equal to 0.40 - · test. When working
1
Two zones can usually be distinguished in a fatigue fracture, one due to ~~hngth obtained in the usuH-1 way from a _tetnscs1 :re very well k~o1bv,n,
''1 rn te .
the gradual development of the era.ck and the otlier due to sud~en frnc_t~e~
The latter zone resembles the fracture of a tensile test specimen wit

"ijeh
a rials whose mer.hanica
Q1L as earbon steels estimates of t
'lJetw·1 ' .
. I characteris J be coOSI'dered reha e.
h'15 type can
se such estimates are likely to ·
.
' . d'ng and direct er
be, mJS1ea l '
idurance
_J
PATioui o,
328 MECHANICAL PROPERTIES OF MATERIALS IO.-r.\La
the parabolas and the corre
0
tests should be used instead. S_o me re!mlts of enduran_ce tests of steels i,et~et: iwa.Ys tension or always co SPonding stra· 329
are given in Fig. 11.30 and also m Table A,4 of Appendix A. stres8 ~; bY test, are sometimes no~P~s.sion, the ~~t lines. Wh
In the majority of cases, endurance tests are carried out for complete! l)S follPl w the corresponding straight l~n Y below Ga ~es of the r en the
reversed stresses (CTmu. = -CTmin), while in many cases in machine desi/
be o Ines eruer'a ange R
~lsO · Parabo1._ b •
the stresses vary but a.re not completely reversed. It is necessary to kno ~ 141! 11t
fatigue Under Combined S
the endurance limits under these varying stresses. "'!"
ohler was the firs~ 11.10 tresses
experimenter who studied the phenomenon of fatigue systematicallv ,. t of our experimental information
He showed that the range of stress R necessary to produce fracture d~- ~os obtained under conditions of ?n ~he fatigue atr
t,een . uniaxial t engt,h f
crea.ses as the mean stress CT,.. increases. On the basis of these tests and of ~ pecimens. But m practical problem s ress, as in rot _o f!lat,e~
Ba.uschinger's work, Gerber proposed a parabolic law relating the range 1bst s d •t • . s we freq at1ngbe d.
~ bined stress, an I _is important to k uently eneounte n lng.
of st ress R and the mean stress CT,... This is illustrated by the parabolic

--
colll nditions. To obtam the fatigue st now the fatigue r c&seg of
curves in Fig. 11.31, in which the mean stress and the range of stress are ch co . rength of . strength f
su re shear, torsion tests were made . various ductil or
expressed a.s fractions of the ultimate strength. The range is a maximum · pu
when the stress is completely reversed (11,,. = O) and it approaches zero
when the mean stress approaches the ultimate strength. If the endurance
in _.d. The results of some of these te c t e angle ofetmaterials
For purpcses of companson,
In whi h h
. !h~ endurance limit
· ~ and the endurance hm1t in shear ·
~~~ ~~
.
in be w:n _m Fig. li.32.
ndmg
..
limit for reversed stress and the ultimate strength are known, the endurance abSC l • h . . 1s P1ott~ as h 18 taken .
as
1.11e
,i._

limit for any varying stress can be obtained from such curves. Other seen that the ratio of t ese hm1ts for all the materials t e o~te. It is
investigations show that there is no general law connecting the mean tested is very nearly
stress a.nd the range of stress. For instance, there are materials for wliich
t he relation between Rand CT,.. is represented more accurately by the broken ...200, - , - -i---r---. .....--
lines (Goodman law) in Fig. 11.31 than by parabolas.

R 3,500 r--,--r- --t--+.. /-__.J


~
1.0 eu
i'
.. 0 .8
l 2,soo r---1r--- +---+-~ +---1
~.. 0 .6 llO
~
0
• 0 .4' -~
0
C
0
a: 0 .2
i 2,100 t----+-----+ -~l--~--..... ---1

a,,, ..
-~
-015 -0.50 -0.25 0 25 0.50 0.75 1 trulf ! ·1,400
Mean stress •u
C

FIG. 11.31
e
:,
'O
C
1-.1
700
_T he stra.ig~t lines OA and OB in Fig. 11.31 have a slope of 2 and deter·
mme_the ~gion _AOB in which the stress changes sign during a cycle.
Outs1~e th1S region the stress always remains tension or compression.
Experunentally determined values within the region AOB usually lie 0 IL_ _ IJ,400
_ ____,2:-:,8l00~ - .
4,k200~ - 50,600~7,(7,000
Endurance timit in bendi119 (kg per sq cm)
•A. Wohler, Z. Bauwuen, Vols. 8 10 13 16 and 20 A t of this work
, 185o-70 .
0
· E lish · · · E • ' ' ' • n a.ccoun ,. FIG. 11.32
m ng 18 given m ngmeering, Vol. 11, 1871 • see a.I U • · Tht T t· ,1 M ateria,,.
of Comtruction, 3d ed., 1910. ' so nwm, ea mg o,
'See
1
ll . wrs New York, 19.53.
• E::. Peteraon, Strua Ctmemtration Design Fill '
jiP ' FATIGUE UNDER COM~T
N~T) R'J'l!
330 MECHANICAL PROP ERTIES OF MATERIALS · P./l11rs
is the endurance limit fo r u . . 33
·,h d& h .
and usmg t 1:: notati11i, r, n1ax1a] st 1
. . the value given by the maximum distortion en .
eq
ual to
·
"3 ·
h
ThJS is ' · be d.
. of the yield point stresses m P.1'~1·
n mg and shea.r ( • ,111
11 b1i;
/ (12
_
2 -
rei,g ronct· -
Qo-1 . I 1)IJQ<; ~ ..
t hcory for t e ratio flee: I t 111 ~,-,-__ 'we 'lntain i . •~ ,-,,n,
1h~ u, V1 - a + a2 ::: r, rom eq. (h) --:
eq. 11:11). !,
Fatigue t.ests un der ,."ombined stresses
.
produced hy the simultan
b eou).; of pulsating stresses in which th
. f lte
action o a rna J t·ng bending and torsion haveh a 1so een made and th e case•rnum Iue, th e correspondin
frJ
. m· Fi·g· J 1·33 · Here
· l e !
11 tb 1 va e stress .
g . . var1~ fro ....
agam t e test resu ts are in gOod
,oflle ma." substitute
results are shown . . d f or rJE •
m eq. (c). un1ax1al Pu Isating stress
. "' ze.ro to
o.greement with the maximum distortion energy theory, as might be
expected, since slip generally precedes the development of a fatigue crack. sho11ld be Iusion, it should be pointed out th 11,.,.
cone . be ff d at. the end .
Jn terials can a ecte by many extra urance hrn:t f
' us rna h. f neoUs facto F or
~sl'IO cold-stretc mg o steel has been found t rs. or example
0.8 .-----,r- ------,-- --,--,--- -, od er.ate . . H O produce so . I
JJl. ndurance hm1t. owever, when this cold-workin . me 1ncre~
Maximum distortion in118 e limit may be lowered. g 13 overdone, the
energy theory d11rance d
en t fatigue tests are ma e at room temperatur S .
Mos 1 .
.th fatigue of stee spe~1mens ma de at lower temperatures e. ome expenments
(- 20° _
wi slight increase m the endurance limit at this lo . te C) ha\e
shown a •h h wer mperature
ttoweve,r·, tests at h1g er t an room temperature (up t-0 300"C) auowed al. •

Preciable effect of temperature on the endurance limit.


no ap . . h .
Fatigue test.s on specimens m t e presence of vanous corrosive agents
has salt water have shown that the endurance limit may be greath·
:~uced by the combined action of fatigue with corrosion. There a~
many known cases of failures in servir~e which can be attributed to such
0 0.2
corrob"ion fatigue such as marine propeller shafts, turbine blades, oil-well
0.4 0 .6 0.8 1.0
Bendill!;I stress pump rods, etc. For ·this reason , special corrosion-resistant materials
Endurance limit in bending are frequently used in such cases. Protective coatin~ and rnriace_cold-
FIG. 11.33 working have also been used successfully in guarding against such failures.

To obtain an equation for calculating the endurance limit for combined 11.11 Fatigue and Stress Concentrat ions
bending and torsion, we have only to substitute into the corresponding
.
In discussing · . ns prod uced bvJ sharp variations
equation for yielding (eq. 11.10) the value of the endurance limit us for the stress concentratw • • d Lat inh
reversed bending, in place of uy .p., which gives thecro~section S) ·t was rnd1cate sue
s of bars and shafts (see Art.~ - 1 C)
.· t1mt""<=>"
8t · · th ca...-ie of vary s "'"""'~
ress concentrations are especially damagmg in e _ d to fillets,·
111g
(a) In machine parts stress concentrations are always preseth.ntt. n~:t fatigue
gl'Oo ' · e shows a
v.es, holes, keyways, etc ., and expenenc .
The correspondin g ellipse is shown in Fig. 11.33, and it is apparent that
cracks in service begin at points of stress conce~t rauofn. shaft of a large
the test results are in good agreement with the equation. F' · f 1lure o a
Other fatigue tests with biaxi~l tension or tension and compression, igure 11.34 shows the torsional fatigue a d ar resonance.• The
lllotor~generator set which unfortunately operat.e ne tration took place,
and with the ratio <11/ <12 remaining constant during a cycle, are also in c~k . h tre~~concer1
an started at the keyway, where a ~ig t a
satisfactory agreement with the maximum distortion energy theory. Thus s s:s Figure 11.35 represen 5 _
we can use for determining the fatigue limit in the case of complete reversal I0d_Ktadually developed along the helical p11.th. t r A high stress con
rsian f • . -d · , 11 genera O • •
of stresses the following equation (see eq. (11.9) p. 319): allure of the sha.f t of a Diesel me nee ~n
d at I.he Conferc h-
"tbeae E P~terson presen te Institute of Tee
(b) S~th ficurea are taken from a paper by R. · he M9S111chusett.'l
~ 'of Material Problems in Indu 9t ry, at t
'"lily 1937.
• ~32
. L l' tl () P E fiTlF. S O F MA T E RIAL
MEC HAN IC .,
S

FIG. 11.35

Fro. 11 .34 ver, experiments mowed


of the load for a spec imen with out a hole. Howe
to the ~tre:-S conrentra11on 13
helical crac ks, which when that in this case the redu ction in strength due
~-ntration at the small fi llet resulted in several small as com pare d with the calculated effect.
appe aran ce. Finally, Fig. 11.36
Joined toget her produ ced the saw- toothed To explain this disc repa ncy and to gi\'e the
nece~.;jry infonnation for
re of a heav y helical spring. The
represents a characteristic fa tigue failu made by R. E. Pmrson at
crark start ed from the · 'd h .
predicts (see p. 78) ' and agam
. designers, a very exte nsiv e series of tests were
ms1 e, as t eory • . the Westinghouse Rese arch Laboratories.• Geometnri\lly s.imi.lB.r canti-
follow ed the direct'ion of one of t hc prmc 1pal stresses. All these pictur. es O. l in. tL1 3 in ., with a fillet
· ·
clearly demonstrate the d amagmg action produced by stres s concentra.tion, lever_ test spec imen s vary ing in diamet er from
. . ent materials were tested 10
an d it 1s clear that thi8. f ac·t or must be serio .
usly considered in the design of or with a tran sver se circu lar hole and of differ
these rc~t 5 ior spec!mens
h'
maEc me p~rts subjected to alter natin g stres s. ~cial fatig ue-t estin g mac hines. The resul ts of
di1t.mcters of t~e specimens
arly fat igue tests mad 'th .
mens having shar p changes of cross with fillets are give n in Fig. l l .3i . T he small er he rntto.s k, of the
section showed t hat theree wi speci d • are tak as absc issas while t he 0rdinn tes reptt'~nt t _ d- 1.0 r the
concen tration b t th·1s rcduwa~ a re uct1on in stren gth due to thed stress f
end en
et l hr en durance. tt ·, t 1oa : . • ·ere
II c0 Uranc est oads for plai n ~perimen:- .tt t
1
, u r. tion was usua, Y smaller than expecte rorn _ Snnila r rcsu1t::- "
the magnitudP. of th ca , 1 · · · t e, ·
rrespond mg
. the ca.'><, of fl t e cu 1ated stre"
08 conccntr at.1011 facto r. For ms anc ob 1, spec imen s with '-t rr:'-s C'L)ll C' t' lltnt t1 n n:-.
m . • · d to · l 10 I ~
a stee1 specunen s ·th small C'trcu ~ ned for · h ·trc..--s conce ntra-
. ·
direct stress the tlieoret ·ica 1 factor Owi
lar holes subJ ecte 48).
. • , l'h } . spec ime ns w1 th trnn sverse e~\-alues 01. t e ~ nt of ~train at
'
If the maguitude <' f the pea k Ht . 18
f t
s ress conc entr ation 1s 3 (see p. ,e
tion IOflzontal lines in Fi g. 1t .:H gi ,·r the
rnrl:ls urt'me ~
·
r" "d, lh ress the· cont rolling factor in endu· d to
rance actors obta ined for Ni.eh fi llet !:iize hy :l di rect
testi,, it woll ld be 1:xpe . at thp· tcnsion-co mpression load require
· · ,.,.; • IQ~" and H. E. Pettlrson and
prod uce fa tiaue fa ilure rJ f a sp1:r•1. · I· d ~ 'll, R P !l und 15' • . .>.> ,
· -- men with a hole would be abou t one-t ur
"' ' M, \\~ab~te~?n, J . A ppl. Mech ., Vol. I, pp.i
'' id ., Vol. 3, p . 15, 1926.
BTRJ:8a CONct~
'l'JO!(s
~
334. MF.C' HANICAL PROPERTIF..S 1t, ~IATERIAL:,
alir,ed 0.45 per cent carbo ·
I fof po~nsiderably below the theor; .steel I are apPa
1ca .
3-1.5
.11168 "" rently
~ 2.4 r - - - , - - ~
~ ~t-
"'I

1.& r7JE!~~r~:t::±:J-

0,45•4 carbon stet! ~ ~


1.00 _......_7[5""""-.L._-;-L;:- --_J
2.5 5.0
7,5
Diameter d 1cn 11

FIG. 11.37

(3) With a decrease in the size of the specimen, the reduction in faugue
strength due to a fillet or hole becomes somewhat less· and for,.,, ·1
fill h ' ,d-V 5m3.i..
~ts or oles the reduction in fatigue strength is comparative!~: sr-...ill.
This can be clearly seen from the curves in Fig. 11.37.
FIG. 11.36 It can be appreciated that the problem of reducing the dsmagmg t>ri~1
of stress concentrations is of primary importance to designe~. ~me k111t' • ·
the points of maximum stress concentration. These values are designated mg of stre~ concentrations can be obtained bv a suit.able dwige m dt'Sl~
by k , and are called theoretical values of stress concentration in the following For example, a design can be improved considerably by eli.nunauog :harp
discussion . If the fatigue strength of the specimen depends only on the :ntrant corners and introducing fillets of gsnerolli r3d.iu~. b~· d~srung
peak stress, then k, must evidently be equal to k1 . ets of proper shape, by introd ucing relieving groove8, etc. In Ftg. 11 JS
On a basis of his tests, Peterson came to t he following conclusions: are shown methods for reducing the stres.5 concentration at a :-houldt'r uf cl
(l) I n some cases fatigue results are ·quite close to theoretical stress
concentration values. This conclusion is of great practical importance, A
.4

~L
A
since a general idea seems to exist, based on some early experiments, that
fatig ue data for stress contentration cases are always well below theoretical /
values, i.e ., on the f;8.fe sid,~ for dr.sign purposes.
(2) Fatigue results for all!Jy steels and que nched carbon steels arr I .l I

usually closer to the theoretical values than are the corresponding fa tigue ,4
i-.- -}-
·---+-
,II
results for r.arbon stf:Pls 11ot quend1ed. Jt wai-; expected in these tests t;hut
the t heorctical values of k, would he rru.chc<l for all steels prov ided thc
specimens were ma<lP large c111iuKh , l>ut Fig. 11 .:H Hh ow s t hat the curves
(o)
- ·+ ( bl
(cl

F10 . 11 .38
336 MECHANICAL PROPE RTIES OF MATERIALS

can be
shaft, while maints.ining the positio ning line AA . The stress
larger rad~~:
duced by cutting into the should er and introducing a fillet of
in Fi !l
without developing interference with t.hc fitted r_nember, as shown
11 .38b . If the shoulder height is too small, a rehef groove
may be used !~ )
shown in Fig. 11 .38c.
In Fig. 11.39 two d~ere nt_bolt-and-n~t_design_s are s~own
n. High
. In Fig. 1 l.:Jgn
stress concent
)
the nut is in compression while the bolt 1s m tens10. rn.
m the face of the nut a d
tion t.akes place at the bottom of t he thread
. , n (a)
( b)
that plan
under the action of variable forces, fatigue frac ture occurs in
d becau e. 11 .4-0
In the lip design , Fig. 11 .39b, the peak stress is somewhat relieve
F10.
bolt. Fatigu e tests show t:
the ljp is stressed in the same direct.ion as the
lip design to be about 30 per cent strong er. . t in this case. Experience shows that t'--
ffic1en h ~ · 1ie mere p fi
'l'ess t ma. bub
su --1 reduces t e. 1at1gue strength of the n-i ~e to Jess n,ft _ on
ple, the change s shown in. Filg. 1 40b Wllll half oi it..
an . trengt h while '
. 1't181s
· ~ ~ fa.~
I 1
in gth of the axle perha ps no more than 20 per

-1
st

the ree•
.
redn'tion and elimin ate fatigue failures , surface col~~ }_ unp?O\-e lhi3
~~
0

...,:~~~
...""'uflY. apl"lru.
aion of stress conce ntratio n has been suc"...._ ..J • em

U.1 2 Physi cal Prope rties of Metal s at High Temperatu1'Si

There are many cases in which parts of engineering structures are


subjected simul taneou sly to the action of stresses and oi 'rugh tem~ rures.
Such conditions are found , for instance, in power plants, eherrucal mdu.s-
tries, and in the missile indust ry. Owing to the modern tenrleney to mcra.~
g
U!mperature and pressu re in steam power plants and in the oil-refinin
tLre
(0) ( b) industries, the questi on of the strength of materials at high rempe.rat

FIG. 11.39 ha., become of practi cal importance and a considerable amount of ~ h
work has been done in this field. Experiments show that the yield prunt
~~1. ~
Sometimes theRe relieving measures are not sufficient to elimin
ate fatigue and ultimate streng th of meta.ls in tension depend wry muchmroi3ru-
th ion
failures. A.s an impor tant example let us consid er the typica l failure s which ~rnpera.ture. Fig. 11.41 shows how these as well as O er cornn
d-car axles, at the wheel or I , w·ith the temperat ure.
occur at the wheel seats of locomotive and railroa ca properties of a mediu m carbon steefI tvar) b' h t n?ratures s.s,
of long drill .d ' and st ig emr
bearing seats of automobile axles, at the pressed or fitted bits F l
or oa.ds acting over a long per10 o ,ime . ssul't' .mpo~•er nlsnr.s " ,
we
ers subjected ror ·instance the weigh t of a struct ure or st eam pre
rods in oil-well operations, etc. All these cases of fitted memb E perience shows
of fatigue d' th time effeet. x
to the action of variable stresses have been a consta nt source need add' .'
.. 1tional inform ation regar mg e • n called creep, may
a wheel hub pressed on an • us def ormauo , · . d ·gn
failures. Considering, fo r exam ple, the case of that under such condit ions a contmuo be considered m esi ·
n combin ed
axle, Fig. 11 .40a, we can see that a high stress concentratio ~e Place which is the most important fac tor tk~ . this direct-ion has_ beenf
rolling
with fri ction is produc ed at the reentr ant corners m and n. During • arch wor
~though a consid erable amount o1 rese h uestion of t e tn h behavior
'd red
o
and finally 11 d
of the axle a reversal of stress at points m and re takes place, one and . e q
gress ' t d ading canno t be cons1 e
concen tration s tn · much more is now in pro
fatigue failure over the cross-section mn, may occur. Stress
Coeta.Is Under high tempe rature and prolong e I0'
can be somewhat reduced by introducing raised seats and fillets
' l l .4 Ob . A furt h er improvement i1:; obtained by introducing '
.m F 1g. ·
as
·
shown
tht' IXlJ)Ietely cleared. senwd in svmr~t:ilr:
d b'bJiographYpre b . the A,S.T, ·
~-, ofllrt.her infonnation, see papere an ~ issued jointly
they are not 'F'or t
relief groove a. Although such changes arc an improvement,
Y
~'8.~t ·Ci"" ••T-.~ re on Propert ies of Me '
1931.
METALS AT RI
Q }{ 'l'tli(pl:11..\
~
338 MECHANICAL PROPERTIES OF MATERIALS untered in practice th 'l'\Jtr.s
enc O . . ' e Port·10
~~
ll,) 9
.~ while the entire lifetirr. f n OB..._
,tP- t:roe, . ..,e o th •1:p~se
or' I slope of the portion BC e SPeciin _nts acorn
:c. _f~~ress and temperature, is ~h~!;:senting th:\~es Within~r:tive[y
"..tAiJl the 1ife of the structure de""nd re of the Ut ~ of ex~n . range
ct),,..-~ d r- s on th lllost ston at
~ll stress is decr~ase . ' the slope of BC e rate of this Practical in~r a
tetlsiJe .dence that it will ever become h d~cre~s, but thextension. If teaht
eVI h' h th . . or1zonta1 ere i e
~,,~ . stress at w ic c specimen can . , that is s no conctu.
1jJll11,1ng persture. Hence in such cn'-'n th1ndefinitely res·'tthat there is a
h teJTl · d """CS e d 5· 18
the t
~ u·on of a certam uration of ser . e ign must be b 8 ress and
IJlP . . vice of th &sed
~ll t of distort.ion which can be con 'd e structure and on the
on h si ered . of ace ,
,;n0 sre chosen so t lt.f. the distortion f h Perrru~ible. Th rt~in
. _""9i',S d fi . . . o t e stru t e work "
s~- t exceed a e nite limit, dependin cure during i~ 11·1 _1n11
111 no d · f . g on the ty et1me
'II le in the es1gn o movmg parts su h pe of structur F
e¢JlP • c as stt e. or
ould never exceed 1 per cent in 100,000 hr (about ~m turbines, the creep
1
~ . jted to a fraction of I per cent. Creep r 1~ ~ears) and generally
,s 1iJJl • df . . al-Cs l\S high I
""" hr may be use or steam piping and b .1 as per cent in
10,vvv . k' 01 er tubes
The comparative .
wor mg stresses or creep 8t •·
rengths of ce ta·
eels over a wide range of temperature are sho 10 . F' r malloy
st wn tg. 11.43. These
ralues however , may be much affected by variations 1· . .
' d b • . n gram size, by heat
treatment, an y previous stram hardening and should be used with
csution.

0 '
20 100 200 300 400 500
Temperature in de9rffs cenTl9rode

F10 . 11.41

In most experiments of this kind, the gradual elongation of ma terials


under prolonged tension is studied . Tensile test specimens at high tem pera-
ture are subjcct.ed to a Cf'rtain const.an t load and temperat ure , and the
progressive creep under this load is investigated . The results of such an
experiment when plotted as a time-extension diagram give a curve of the
shape shown in Fig. 11.42. \Vhen the load is first applied, there is an im-
mediate elastic extension OA . The specimen then begins to stretch at a
decreasing rate as shown by the portion A. B of the curve. At B the rate of
extension reaches a value which remains substantiallv constant for sorne
t ime, that is, along the portion BC of the diagram . At.C, the rate of exten- Time
th f JG. JJ.4J
sio n begins to increase and f ract.ure finally takes place at point D. f or e F1G. 11 .42

You might also like